Pharm All Chapters

Réussis tes devoirs et examens dès maintenant avec Quizwiz!

The nurse receives a new medication order for a patient to administer 240 mg of medication per day in equally divided doses every 6 hours. How many mg of the drug should the nurse administer for each dose?

60 mg Because there are 24 hours in a day, giving a drug every 6 hours would mean giving the drug 4 times a day. Because the total daily dose is 240 mg, dividing that dose by 4 would mean each dose should be 60 mg.

The provider orders a maintenance dose of oral aminophylline, 3 mg/kg every 6 hour. Thepatient weighs 50 kg. How many mg should the nurse administer to the patient in a 24-hour period?

600 milligram The patient's weight times the number of milligram/kilogram will provide daily dosage ofmedication: 50 kg × 3 mg/kg = 150 mg per dose. The patient is to receive a dose every 6 hours. The number of hours in a day divided by the number of hours separating each dosesupplies the number of dosages the patient receives per day: 24 hours ÷ 6 hours between doses = 4 doses per day. If each dose is 150 mg and the patient receives 4 of these doses aday, the total amount of medication received is 150 mg × 4 daily doses = 600 mg

A patient is admitted with a deep vein thrombosis in his or her left calf. The physician orders Heparin, 7,500 units subcutaneously every 12 hours. The medication vial reads Heparin, 10,000 units/mL. How many milliliters does the nurse administer? A) 0.5 mL B) 0.75 mL C) 1 mL D) 1.25 mL

A

The nurse begins administering 500 mL of 5% dextrose and water solution at 01:00 to run over 4 hours. At 02:00, the nurse administers 80 mg gentamicin in 50 cc normal saline to infuse over 30 minutes. How many mL of fluid will the nurse administer to the patient between 02:00 and 03:00? A) 175 mL B) 150 mL C) 125 mL D) 100 mL

A

The nurse determines a child's body surface area is 0.4 m2 and the average adult dosage of the medication is 500 mg. The medication is supplied in liquid form with 500mg/5 mL. How many milliliter will the nurse administer? A) 3.46 mL B) 1.73 mL C) 0.5 mL D) 12 mL

A

The nurse is to infuse 100 mL of 5% dextrose and water solution containing an IV antibiotic over 30 minutes. The infusion set delivers 10 gtt/mL. How many drops per minute will the nurse administer? A) 33 gtt/min B) 30.3 gtt/min C) 30 gtt/min D) 3 gtt/min

A

The physician orders acetaminophen (Tylenol) 15 mg/kg q4 hours PRN (as needed) for pain. The drug is supplied with 160 mg/5 mL. The infant weighs 12 pounds. How many milliliter of medication will the nurse administer? A) 2.5 mL B) 5.6 mL C) 12.4 mL D) 10.7 mL

A

The nurse is caring for a patient experiencing postoperative pain. The physician orders 2.5 mg of morphine IV every two hours. Morphine is supplied in 10 mg/mL vials. How many mL will the nurse administer? A) 0.25 mL B) 0.5 mL C) 1 mL D) 2 mL

A 10 mg = 1 mL and a dose of 2.5 mg is ordered. 10 mg/1 mL: 2.5 mg/x Cross-multiply to yield 2.5 mg = 10x. Divide each side by 10 to learn the nurse should administer 0.25 mL

When a combination of drugs needs to be incorporated into the drug regimen for a hypertensive patient, what type of diuretic would the nurse expect to administer? A) Thiazide B) Loop C) Potassium-sparing D) Osmotic

A A somewhat controversial study, the Antihypertensive and Lipid-Lowering Treatment to Prevent Heart Attack Trial (ALLHAT), reported in 2002 that patients taking the less expensive, less toxic diuretics did better and had better blood pressure control than patients using other antihypertensive agents. Replications of this study have supported its findings, and the use of a thiazide diuretic is currently considered the first drug used in the stepped-care management of hypertension.

A patient with glaucoma has been prescribed a diuretic as treatment of his or her disease process. What drug does the nurse suspect that the patient will be prescribed? A) Acetazolamide (Diamox) B) Spironolactone (Aldactone) C) Chlorthalidone (Hygroton) D) Bendroflumethiazide (Naturetin)

A Acetazolamide is used to treat glaucoma. The inhibition of carbonic anhydrase results in decreased secretion of aqueous humor of the eye. Spironolactone is a potassium-sparing diuretic used to treat edema caused by congestive heart failure, liver disease, hypertension, hyperkalemia, and hyperaldosterone. Chlorthalidone is a thiazide-like diuretic and bendroflumethiazide, a thiazide diuretic, are used to treat edema caused by congestive heart failure, liver disease, kidney disease, and as adjunct treatment of hypertension.

A 64-year-old patient in hypertensive crisis is to receive furosemide (Lasix) 40 mg IV. Lasix comes in 100 mg/10 mL containers. How will the nurse administer the medication? A) Give 4 mL over 1 to 2 minutes IV. B) Give 4 mL over 10 minutes. C) Flush the line with normal saline, give 1 mL/min, flush again when finished. D) Fix 10 mL in an IV piggyback and deliver it over 30 minutes.

A Administer furosemide (Lasix) 40 mg over 1 to 2 minutes IV. The other options are not correct.

The nurse is discussing ethnic differences in response to medication with your nursing students. What group of people would the nurse tell the students may have a decreased sensitivity to pain-relieving effects of anti-inflammatory drugs and should be educated concerning signs and symptoms of gastrointestinal bleeding from use of these drugs? A) African Americans B) White Americans C) Hispanics D) Asians

A African Americans have a documented decreased sensitivity to pain-relieving effects of many anti-inflammatory drugs. They also have an increased risk of developing GI adverse effects to these drugs. In general, White Americans, Hispanics, and Asians are at lower risk for these problems.

A patient, newly diagnosed with ulcerative colitis, has been admitted to the short-stay unit. What salicylates does the nurse anticipate will be ordered for this patient? A) Balsalazide (Colazal) B) Sodium thiosalicylate (generic) C) Choline magnesium trisalicylate (Tricosal) D) Salsalate (Argesic)

A Balsalazide is delivered intact to the colon, where it delivers a local anti-inflammatory effect that is most effective for patients with ulcerative colitis. Choline salicylate and salsalate are used to treat pain, fever, and inflammation. Sodium thiosalicylate is used mainly for episodes of acute gout, for muscular pain, and to treat rheumatic fever.

A 68-year-old patient, who has type 1 diabetes, is to receive hydrochlorothiazide (HydroDIURIL). Before administration of this medication, what information is most important for the nurse to communicate to the patient? A) His or her insulin dose may need to be increased. B) His or he insulin dose may need to be decreased. C) He or she will need to have him or her urine checked for ketones four times a day. D) He or she will need to have a creatinine clearance done once a month.

A Caution should be used with the following conditions, which include systemic lupus erythematosus (SLE), which frequently causes glomerular changes and renal dysfunction that could precipitate renal failure in some cases; glucose tolerance abnormalities or diabetes mellitus, which is worsened by the glucose-elevating effects of many diuretics; and gout, which reflects an abnormality in normal tubule reabsorption and secretion. There would be no reason to check ketones four times daily or to have a creatinine clearance once a month.

What drug might the nurse administer for both analgesic and antitussive effects? A) Codeine B) Aspirin C) Ibuprofen D) Acetaminophen

A Codeine is a narcotic drug used for its analgesic and antitussive effects. Aspirin, ibuprofen, and acetaminophen do not have antitussive effects.

What statement by the 62-year-old patient indicates that the patient understand the nurse's teaching about diuretics? A) "I will weigh myself daily and report significant changes." B) "I will have to limit my high sugar foods." C) "If my leg gets swollen again, I'll take an additional pill." D) "I will take my medication before bedtime on an empty stomach."

A Daily weights and blood pressures should be monitored at home in a patient taking diuretics. Options B, C, and D would indicate that further teaching is needed.

The nurse is preparing to administer a nonsteroidal anti-inflammatory drug (NSAID) to an older patient. What NSAID is associated with increased toxicity and should be avoided if possible? A) Naproxen (Aleve) B) Ibuprofen (Motrin) C) Indomethacin (Indocin) D) Etodolac (Lodine)

A Geriatric warnings have been associated with naproxen, ketorolac, and ketoprofen because of reports of increased toxicity when they are used by older patients. These NSAIDs should be avoided in this population if possible. No such warnings exist for ibuprofen, indomethacin, or etodolac.

When caring for a patient diagnosed with rheumatoid arthritis, the patient tells the nurse that he or she has had insufficient response to nonsteroidal anti-inflammatory drugs (NSAIDs) and his or her condition continues to worsen. What drug does the nurse anticipate will be ordered next for this patient? A) Auranofin (Ridaura) B) Ibuprofen (Motrin) C) Acetaminophen (Tylenol) D) Ketorolac (Toradol)

A Gold compounds such as auranofin are prescribed when more usual anti-inflammatory therapies are ineffective and the patient's condition worsens despite weeks or months of standard pharmacological treatment. Ibuprofen and ketorolac are NSAIDs, which have been tried without good results. Acetaminophen is not an anti-inflammatory and would not be appropriate to control this patient's condition.

What drugs used to treat rheumatoid arthritis are contraindicated in a patient who has a history of toxic levels of heavy metals? A) Gold salts B) COX-2 inhibitors C) Propionic acids D) Fenamates

A Gold salts can be extremely toxic and are contraindicated in the presence of any known allergy to gold, severe diabetes mellitus, congestive heart failure, severe debilitation, renal or hepatic impairment, hypertension, blood dyscrasias, recent radiation treatment, history of toxic levels of heavy metals, and pregnancy or lactation. COX-2 inhibitors, propionic acids, and fenamates have no contraindications related to prior toxic levels of heavy metals.

A patient has been diagnosed with severe rheumatoid arthritis and hylan G-F 20 has been ordered. How is this drug given? A) Injected into the joint B) Orally C) IM D) Sub Q

A Hyaluronidase derivatives (e.g., hylan G-F 20, sodium hyaluronate) have elastic and viscous properties. These drugs are injected directly into the joints of patients with severe rheumatoid arthritis of the knee. They seem to cushion and lubricate the joint and relieve the pain associated with degenerative arthritis. They are given weekly for 3 to 5 weeks and are not given by any other route.

The nurse is caring for a patient who reports taking 800 mg of ibuprofen three times a day for relief of menstrual cramps. What lab results will the nurse find most significant in assessing this patient? A) Complete blood count B) White blood cell differential C) Arterial blood gas D) Cholesterol and triglyceride profile

A Ibuprofen, like all nonsteroidal anti-inflammatory drugs, can cause irritation to the GI mucosa and block platelet clumping, both of which can result in bleeding. Blood loss due to dysmenorrhea can exacerbate these risks so it is important to assess the complete blood count to monitor for excessive blood loss. White blood cell differential would be impacted by infection, which is not indicated here. Arterial blood gas, cholesterol, and triglyceride levels would not be impacted by ibuprofen.

The nurse is caring for a patient newly prescribed iloprost (Ventavis). What is the nurse's priority teaching point about this medication? A) "Inhale this drug six to nine times a day during waking hours." B) "Men should not touch this pill." C) "It is safe for use in pregnancy. D) "The drug causes peripheral dilation of blood vessels.

A Iloprost is an inhaled synthetic prostacyclin that directly dilates the pulmonary vascular bed, reducing pressure in the pulmonary vascular system, increasing gas exchange, and easing the signs and symptoms of pulmonary arterial hypertension. It is inhaled using a special delivery device six to nine times a day while awake. Patients report dizziness and syncope after using the drug and are therefore encouraged to change position slowly. Men and women should not ingest the drug or get it on their skin. It is a pregnancy category C drug and so is not considered safe in pregnancy.

The patient was unable to achieve an acceptable blood pressure with just lifestyle changes so in stage 2 of the stepped-care management of hypertension plan, an angiotensin-converting enzyme inhibitor was ordered. In step 3, when inadequate response was obtained from step 2, the nurse anticipates the provider will order what? A) A diuretic B) A beta-blocker C) A calcium channel blocker D) A vasodilator

A In step 3, another drug will be added for combined effect. However, fixed-combination drugs should only be used when the patient has been stabilized on each drug separately. A diuretic would be added before adding another class of medication unless the first drug was a diuretic. Vasodilators are generally used only in hypertensive emergencies.

A patient has been prescribed losartan (Cozaar) for hypertension. What patient teaching points will the nurse include about this drug include? A) "Report onset of a cough or fever to health care provider." B) "Limit fluid intake to decrease urinary output." C) "Monitor blood pressure once a week." D) "Take the drug late in the day to prevent sleepiness."

A Losartan is an angiotensin II-receptor blocker that is associated with a cough, back pain, fever, muscle weakness, and upper respiratory tract infections, so the patient should be taught to report a fever or cough to his health care provider. Fluid intake should be normal and the drug is best taken in the morning. Blood pressure should be monitored daily, especially when first starting the drug when adverse effects are not yet known.

When evaluating the effects of narcotic agonist-antagonists on a patient, what adverse effects would the nurse monitor for? A) Hypertension B) Bleeding C) Suppressed bone marrow function D) Increased pulse pressure

A Monitor for adverse effects (e.g., central nervous system changes, gastrointestinal (GI) depression, respiratory depression, arrhythmias, hypertension). Bleeding, bone marrow suppression, and increased pulse pressure are not normally seen with these drugs.

The patient in labor receives morphine every 2 hours to manage labor pain. After 22 hours of labor the woman delivers a baby boy. What is the nurse's priority action related to the newborn? A) Monitor for opioid effects. B) Administer naloxone. C) Monitor for withdrawal syndrome. D) Assess for congenital anomaly.

A Morphine, meperidine, and oxymorphone are often used for analgesia during labor. The mother should be monitored closely for adverse reactions, and, if the drug is used during a prolonged labor, the newborn infant should be monitored for opioid effects. Naloxone would only be given if the newborn displays opioid effects. Withdrawal syndrome would not be seen with less than 24 hours of use. Every newborn is assessed for congenital anomalies but this would not be related to administration of morphine to the mother and so would not be the highest priority.

The emergency department nurse is asked to prepare a nitroprusside (Nitropress) infusion for a patient being brought to the hospital in an ambulance. The nurse knows this drug is only used in what circumstances? A) Hypertensive emergencies B) Hypertension in a patient having a myocardial infarction C) Hypertension complicated by symptoms of a stroke D) Hypertension associated with diabetic ketoacidosis

A Most of the vasodilators are reserved for use in severe hypertension or hypertensive emergencies. These include hydralazine, minoxidil, and nitroprusside. The presence of absence of a comorbidity does not increase the likelihood of use. These drugs are used when blood pressure is extremely high and needs to be reduced quickly.

The patient is diagnosed with secondary hypertension. What will be the focus of nursing care to treat this patient? A) Administering epinephrine B) Administering antihypertensives C) Promoting healthy lifestyle D) Treating the cause

A Most people have essential, or primary, hypertension with no known cause that is treated with medications and lifestyle changes. Patients with secondary hypertension means there is a known cause of the blood pressure elevation. There are many different things that can result in secondary hypertension; the focus of care is treating the cause whether that requires surgery, medication, or discontinuing a medication that is causing hypertension as an adverse effect.

As the nurse settles the patient into his room after returning from the post-anesthesia care unit (PACU), the patient says he is in severe pain. The nurse checks the medical record and sees the patient has an order for morphine 4 to 8 mg every 1 to 2 hour IV as needed for pain. The nurse sees this medication has not been administered yet so the nurse administers 4 mg. After administering the drug, the PACU nurse calls to say a dose of morphine was given and not documented. What drug will the nurse be prepared to administer if the patient's respiratory rate is depressed? A) Naloxone hydrochloride tartrate (Narcan) B) Butorphanol C) Buprenorphine (Buprenex) D) Nalbuphine hydrochloride (Nubain)

A Naloxone is the drug of choice for treatment of opioid overdose. Butorphanol (INN) is a morphinan-type synthetic opioid analgesic that would not reverse the effects of an opioid. Buprenex (buprenorphine hydrochloride) is a narcotic-agonist-antagonist and would suppress respirations further. Nalbuphine is a synthetic opioid used commercially as an analgesic that would also depress respirations.

The patient has been diagnosed with rheumatoid arthritis. She also reports pain in various muscle groups secondary to a diagnosis of fibromyalgia and dysmenorrhea with painful cramping during menses. What drug would be most effective in treating all three of this patient's problems? A) Naproxen (Naprosyn) B) Acetaminophen (Tylenol) C) Etanercept (Enbrel) D) Sodium hyaluronate (Hyalgan)

A Naproxen is effective in treating muscle pain, arthritis, and dysmenorrhea. Acetaminophen has no anti-inflammatory effects and would not be helpful for treating arthritis or dysmenorrhea other than some pain relief. Etanercept is useful only for treating rheumatoid arthritis; sodium hyaluronate is used to treat rheumatoid arthritis when other traditional treatment has been ineffective and the condition continues to worsen.

The nurse is writing a plan of care for a patient who is taking a diuretic. What would be an appropriate nursing diagnosis for this patient? A) Impaired urinary elimination B) Monitor the patient response to the drug C) Imbalanced nutrition: More than body requirements D) Risk for fluid volume overload

A Nursing diagnoses related to drug therapy may include impaired urinary elimination related to drug effect. Options B, C, and D would not be appropriate nursing diagnoses.

The nurse is caring for a patient who takes ramipril (Altace) to treat hypertension. What would be an appropriate nursing diagnosis to include in this patient's care plan? A) Ineffective tissue perfusion related to changes in cardiac output B) Acute pain related to skin effects and headache C) Altered gas exchange related to unrelenting cough D) Impaired body image

A Nursing diagnoses related to drug therapy might include ineffective tissue perfusion (total body) related to changes in cardiac output because ramipril is associated with adverse effects such as reflex tachycardia, chest pain, angina, heart failure, and cardiac arrhythmias. Although dermatitis and rash may occur, headaches are not an associated adverse effect of this drug. Benazepril, enalapril, and fosinopril can cause an unrelenting cough but ramipril is not associated with this adverse effect. Impaired body image would not be associated with this drug.

The nurse receives an order for morphine sulfate 8 mg IV every 1 hour as needed for pain. For which patient would the nurse need to question this order? A) A 78-year-old with osteoarthritis B) A 45-year-old, 1-day postoperative mastectomy C) A 28-year-old with a fractured tibia D) A 17-year-old, 1-day postoperative appendectomy

A Older patients are more likely to experience the adverse effects associated with narcotics, including central nervous system, gastrointestinal, and cardiovascular effects. Furthermore, a strong narcotic analgesic would not be indicated for chronic osteoarthritis pain. For both of these reasons, the nurse would question the large dosage of a narcotic. The other patients could appropriately receive morphine 8 mg unless they were smaller than average adults.

A patient has been prescribed furosemide (Lasix). Because this drug causes potassium loss, what will the nurse instruct the patient to eat? A) Peaches B) Apples C) Pears D) Pineapple

A Peaches, as well as bananas, oranges, raisins, and other fruits, spices, and vegetables are high in potassium and consuming them should be encouraged when taking furosemide. Apples, pears, and pineapple, however, do not replace potassium in the body.

A student asks the pharmacy instructor what the difference is between the diuretics spironolactone (Aldactone) and furosemide (Lasix). What would the instructor reply? A) Potassium losses are lower with spironolactone. B) Potassium losses are greater with spironolactone. C) Water losses are greater with spironolactone. D) Sodium losses are greater with spironolactone.

A Spironolactone is a potassium sparing diuretic; therefore, it promotes retention of potassium. Furosemide promotes greater water, sodium, and potassium losses than spironolactone.

The nursing instructor asks the student nurse to explain the action of sumatriptan. What is the student's best response? A) Vasoconstrictive on cranial blood vessels B) Depresses pain response in the central nervous system C) Vasodilation of peripheral blood vessels D) Binds to acetylcholine receptors to prevent nerve transmission

A Sumatriptan binds to serotonin receptors to cause vasoconstrictive effects on cranial blood vessels. The other options are incorrect.

The nursing instructor is discussing COX-2 inhibitors with her nursing students. Where would the instructor tell her students that COX-2 inhibitors work? A) At sites of trauma and injury B) Wherever prostaglandins are present C) At the sites of blood clotting D) In the kidney

A The COX-2 inhibitors are thought to act only at sites of trauma and injury to more specifically block the inflammatory reaction. COX-1 is present in all tissues and seems to be involved in many body functions including blood clotting, protecting the stomach lining, and maintaining sodium and water balance in the kidney.

A patient who was recently prescribed spironolactone calls the clinic and complains that he is not urinating as much as he did when he first started taking this medication. What would be an appropriate question for the nurse to ask this patient? A) "Are you taking a salicylate?" B) "Are you taking acetaminophen?" C) "Are you taking ibuprofen?" D) "Are you using a lot of salt?"

A The diuretic effect decreases if potassium-sparing diuretics are combined with salicylates. Dosage adjustment may be necessary to achieve therapeutic effects. There is no decrease in effect when spironolactone is combined with acetaminophen, ibuprofen, and increased sodium intake.

The nurse recognizes blood pressure is determined by three elements including what? A) Peripheral resistance B) Pulse pressure C) Renal blood flow D) Preload

A The pressure in the cardiovascular system is determined by three elements: heart rate, stroke volume, and peripheral resistance. The small arterioles are thought to be the most important factors in determining peripheral resistance. Because they have the smallest diameter, they are able to almost stop blood flow into capillary beds when they constrict, building up tremendous pressure in the arteries behind them as they prevent the blood from flowing through.

The nurse is conducting an admission assessment of a patient who has been prescribed hydrochlorothiazide (HydroDIURIL). Which situation would contraindicate the administration of hydrochlorothiazide (HydroDIURIL)? A) Allergy to sulfa drugs B) Allergy to codeine C) BP 160/96 D) Blood glucose level of 140 mg/dL

A Thiazide and thiazide-like diuretics are contraindicated with allergy to thiazides or sulfonamides to prevent hypersensitivity reactions. The other options are not correct.

The nurse teaches a patient with rheumatic disease who is being prescribed salicylate therapy to monitor himself or herself for what? A) Tinnitus B) Visual changes C) Stomatitis D) Hirsutism

A Tinnitus is associated with salicylates. The disease-modifying antirheumatic drug (DMARD) category of antimalarials may cause visual changes, GI upset, skin rash, headaches, photosensitivity, and bleaching of hair. Eighth cranial nerve damage and stomatitis are associated with gold therapy. Hirsutism is associated with corticosteroid therapy.

The pediatric nurse examines a child with mildly elevated blood pressure who is 10% above the upper weight limits for his or her age. What is the nurse's priority teaching point? A) Encourage activity and begin weight loss diet. B) Explain how to administer diuretics to reduce adverse effects. C) Involve social services in monitoring this child's diet. D) Explain the most common adverse effects of calcium channel blockers.

A Treatment of childhood hypertension should be done very cautiously because the long-term effects of the antihypertensive agents are not known. Lifestyle changes should be instituted before drug therapy is started if at all possible. Weight loss and increased activity may bring an elevated blood pressure back to normal in many children. As a result, the priority teaching point is to help parents understand how to adapt the child's diet to reduce weight and introduce family activities to increase exercise. Drug teaching would only be required if lifestyle changes is inadequate to lower blood pressure. There is no need to involve social services.

When evaluating an 82-year-old patient receiving hydrochlorothiazide (HydroDIURIL), what laboratory value deviations may be related to the medication? A) Elevated uric acid levels B) Reduced blood urea nitrogen (BUN) levels C) A serum potassium level of 4.7 mEq/L D) A hemoglobin A1C of 4.8

A Uric acid excretion is decreased because thiazides interfere with its secretory mechanism. High levels of uric acid can result in a condition called gout. Hydrochlorothiazide does not reduce BUN levels. Thiazide diuretics may lead to hypokalemia and increase blood glucose levels. The potassium and hemoglobin A1C levels are normal and not affected by the medication.

A female patient has a history of frequent bladder infections. Which classification of diuretic would not be recommended for this patient? A) Thiazide and thiazide-like diuretics B) Loop diuretics C) potassium-sparing diuretics D) Osmotic diuretics

A Urine is slightly alkalinized when the thiazides or thiazide-like diuretics are used because they block reabsorption of bicarbonate. This effect can cause problems for patients who are susceptible to bladder infections. Loop, potassium sparing, and osmotic diuretics do not have this effect.

A physician orders 500 mL of IV solution be administered over 8 hours. If the IV infusion set delivers 15 drops per mL, how many drops per minute should the nurse administer to the patient? A) 15 drops/min B) 20 drops/min C) 32 drops/min D) 64 drops/min

A If a patient was to receive 500 mL in 8 hours, dividing 500 by 8 would mean that the patient would receive 62.5 mL in 1 hour, or 60 min. Setting up the equation, 15 drops/mL/X equals 62.5 mL/60 min; cross-multiplying, the answer will be 15 drops/min.

The nurse is calculating the patient's intake and output record and converts ounces to milliliters. What systems is this nurse converting from and to? A) From household to metric B) From metric to household C) From household to apothecary D) From apothecary to Avoirdupois

A Ounces can be either household or Avoirdupois, although they are not equal measurements and milliliters is a metric measurement. Since Avoirdupois to metric is not an available choice, the correct answer is household to metric

The nurse recognizes that what system is being used when seeing a medication ordered in minims? A) Apothecary B) Metric C) Household D) Avoirdupois

A The apothecary system used minims as the basic measure of liquid and is rarely used today. The metric system uses liters as the basic unit of measurement for fluid, while the household system uses ounces and the avoirdupois system uses ounces with a different conversion amount.

The nurse is calculating a drug dosage and converting from milligrams to grams. What measurement system is the nurse using? A) Metric system B) Apothecary system C) Household system D) Avoirdupois system

A The metric system is the most widely used system of measurement in the world; it is based on the decimal system. The gram is the basic unit of solid measure, and the liter unit of liquid measure. The apothecary system uses the grain as the basic unit of solid measure. The household system uses the pound as the basic unit of measure. The avoirdupois system uses ounces and grains, but it is mostly used by drug manufacturers for bulk medications

What is the most accurate method for the nurse to use when determining a pediatric dosage? A) A nomogram using body surface area B) Young's rule C) Fried's rule D) Clark's rule

A The most accurate means of determining a pediatric drug dosage is a nomogram using body surface area because the weight and body surface area of two children who are of the same age can be significantly different. Young's rule, Fried's rule, and Clark's rule are based on the child's age and the usual adult dosage and are rarely used today unless no other method will suffice for a specific drug.

The pediatric nurse practitioner is caring for a child who is diagnosed with cystic fibrosis (CF). The parents ask what the treatment is for because CF is considered a terminal illness. What would be the nurse's best response? A) "Treatment is aimed at maintaining airway patency as much as possible." B) "Treatment is aimed at lowering high levels of carbon dioxide in the blood." C) "Treatment is aimed at raising oxygen levels to the extremities." D) "Treatment is aimed at maintaining the child until lung transplantation can occur."

A) "Treatment is aimed at maintaining airway patency as much as possible." Feedback: Treatment is aimed at keeping the secretions fluid and moving and maintaining airway patency as much as possible. Treatment for CF is not aimed at lowering hypercapnia or raising oxygen levels to the extremities. Treatment for CF is not aimed at maintaining the child until lung transplantation can occur.

The nurse is caring for a 6-year-old patient with cystic fibrosis. The parents ask how the cystic fibrosis started. The nurse explains that the key feature in the presentation of cystic fibrosis includes what? A) Airway obstruction B) Obstructed bowel C) Sweet-tasting sweat D) Clubbing of the extremities

A) Airway obstruction Feedback: CF results in the accumulation of copious amounts of very thick secretions in the lungs. Eventually, the secretions obstruct the airways, leading to destruction of the lung tissue. Parents will often comment that their child tastes salty, not sweet. Newborns with CF often present with meconium ileus but not a bowel obstruction. Clubbing of the extremities occurs after many years of inadequate oxygenation.

A nurse is caring for a 73-year-old woman who just arrived on the medical surgical floor following a radical mastectomy for breast cancer. The nurse will encourage the patient to cough and breathe deeply to help prevent what? A) Atelectasis B) Asthma C) Bronchiectasis D) Sinusitis

A) Atelectasis Feedback: Atelectasis most commonly occurs as a result of airway blockage, which prevents air from entering the alveoli, keeping the lung expanded. This occurs when a mucous plug, edema of the bronchioles, or a collection of pus or secretions occludes the airway and prevents the movement of air. Patients may experience atelectasis after surgery, when the effects of anesthesia, pain, and decreased coughing reflexes can lead to a decreased tidal volume and accumulation of secretions in the lower airways. Asthma, bronchiectasis, and sinusitis are not directly related to surgery but instead result from other conditions.

A nursing instructor is discussing ways that the respiratory tract protects itself from bacteria and shares with the students that it is done in which ways? (Select all that apply.) A) Cough and sneeze reflex B) Surfactant in the alveoli C) Goblet cells D) Gas exchange in the alveoli E) Nasal cilia

A) Cough and sneeze reflex C) Goblet cells E) Nasal cilia Feedback: Nasal hairs, mucus-producing goblet cells, cilia, the superficial blood supply of the upper respiratory tract, and the cough and sneeze reflexes all work to keep foreign substances from entering the lower respiratory tract. Surfactant and gas exchange in the alveoli are involved in the oxygenation process but are not involved in protecting the respiratory tract.

The nurse is caring for a patient with pneumonia. The nurse would expect to see which symptoms? (Select all that apply.) A) Difficulty breathing B) Urinary retention C) Rash D) Fever E) Oxygen saturation of 88%

A) Difficulty breathing D) Fever E) Oxygen saturation of 88% Feedback: Symptoms of patients with pneumonia include fever, difficulty breathing, fatigue, noisy breath sounds, and poor oxygenation. Urinary retention and rash are not usual symptoms of pneumonia.

The anatomy and physiology teacher is discussing ventilation with the nursing students. What would the instructor say causes an increased respiratory rate? A) Increased stimulation of the respiratory center B) Decreased stimulation of the respiratory center C) Increased O2 D) Decreased pH

A) Increased stimulation of the respiratory center Feedback: Respiration, or the act of breathing to allow gas exchange, is controlled by the central nervous system. The inspiratory musclesdiaphragm, external intercostal muscles, and abdominal musclesare stimulated to contract by the respiratory center in the medulla. The medulla receives input from chemoreceptors (neuroreceptors sensitive to carbon dioxide and acid levels) to increase the rate and/or depth of respiration to maintain homeostasis in the body.

The nursing instructor is discussing the oxygenation process and explains unoxygenated blood is received from the right ventricle by the alveoli. The delivery of this blood is referred to as what? A) Perfusion B) Oxygenation C) Expiration D) Inhalation

A) Perfusion Feedback: The lung tissue receives its blood supply from the bronchial artery, which branches directly off the aorta. The alveoli receive unoxygenated blood from the right ventricle via the pulmonary artery. The delivery of this blood to the alveoli is referred to as pulmonary perfusion. Oxygenation is the process where unoxygenated blood receives oxygen in the lungs. Expiration is the act of exhaling to rid the body of excess carbon dioxide. Inhalation is the act of inhaling or taking in air to replace oxygen from the air.

A nurse is caring for a patient with chronic bronchiectasis. The nurse should assess the patient for what clinical manifestations? A) Purulent cough B) Angina C) Pigeon chest D) Pulmonary hypertension

A) Purulent cough Feedback: Patients present with the signs and symptoms of acute infection, including fever, malaise, myalgia, arthralgia, and a purulent, productive cough. A patient with bronchiectasis would not present with pulmonary hypertension, chest deformity, or chest pain related to lack of oxygen to the heart.

The nurse is aware that patients with bronchiectasis often have an underlying medical condition that increases the chance for infection. What are some of these medical conditions? (Select all that apply.) A) Rheumatoid arthritis B) AIDS C) Diabetes mellitus D) Hydrocephalus E) Organ transplant patient

A) Rheumatoid arthritis B) AIDS E) Organ transplant patient Feedback: Patients with bronchiectasis often have an underlying medical condition that makes them more susceptible to infections. These underlying medical conditions include immune suppression, which would include patients who have had organ transplants and are receiving antirejection medication. Other conditions include acquired immune deficiency syndrome (AIDS) and chronic inflammatory conditions, such as rheumatoid arthritis. Diabetes mellitus and hydrocephalus do not fit into these categories.

The nurse cares for a diabetic patient with uncontrolled hypertension who has been prescribed losartan (Cozaar). The health care provider changes this patient's prescription to losartan with hydrochlorothiazide (Hyzaar). What benefits does the nurse anticipate this patient will receive from this drug therapy? (Select all that apply.) A) Slows progression of diabetic nephropathy in type 2 diabetes B) Increases excretion of fluid and sodium resulting in lower blood volume C) Alters electrolyte and acid-base balance D) Improves control of blood pressure due to combination therapy E) Produces far fewer adverse effects resulting from the combination

A, B Many patients require more than one type of antihypertensive to achieve good control of their blood pressure. There are now many fixed-combination drugs available for treating hypertension. This allows for fewer tablets or capsules each day, making it easier for the patient to comply with drug therapy. Losartan slows the progression of diabetic nephropathy in patients with hypertension and type 2 diabetes. Hydrochlorothiazide is a diuretic that will increase excretion of fluid and sodium resulting in a lower circulating blood volume that will help to reduce blood pressure, preload, and afterload. The two drugs will work together to better control the patient's blood pressure. Alteration in electrolytes and acid-base is a reasonable expectation but it is an adverse effect and not a benefit. There are likely to be more adverse effects when taking drugs that are not fixed combinations.

The nurse assesses laboratory results related to blood clotting when the assigned patient takes what drug regularly? (Select all that apply.) A) Salicylates B) Nonsteroidal anti-inflammatory drugs (NSAIDs) C) Gold compounds D) Acetaminophen E) Disease-modifying antirheumatic drugs (DMARDs)

A, B Salicylates and NSAIDs can both inhibit blood clotting resulting in bleeding if not monitored. Gold compounds, acetaminophen, and DMARDs do not have a known impact on blood clotting.

The nurse is caring for a patient who has been noncompliant with treatment for hypertension. The nurse explains that untreated hypertension increases the risk of what? (Select all that apply.) A) Renal disease B) Cerebral infarction C) Heart failure D) Cholecystitis E) Migraine headache

A, B, C Hypertension is a common chronic disorder. It is estimated that at least 20% of the people in the United States have hypertension. Hypertension increases risks of myocardial infarction, heart failure, cerebral infarction and hemorrhage, and renal disease. It does not increase the risk of cholecystitis or migraine headache.

A patient with glaucoma has been prescribed acetazolamide (Diamox). What adverse effects would the nurse caution the patient about? (Select all that apply.) A) Paresthesia B) Confusion C) Drowsiness D) Vomiting E) Hyperkalemia

A, B, C Metabolic acidosis is a relatively common and potentially dangerous adverse effect that occurs when bicarbonate is lost. Hypokalemia is also common. Patients also complain of paresthesias of the extremities, confusion, and drowsiness. Vomiting and hyperkalemia are not common adverse effects

What is the nurse's priority assessment when administering narcotics to older adults? (Select all that apply.) A) Central nervous system (CNS) effects B) Gastrointestinal effects C) Cardiovascular effects D) Urinary effects E) Developmental effects

A, B, C Older patients are more likely to experience the adverse effects associated with these drugs, including central nervous system, gastrointestinal (GI), and cardiovascular effects. Urinary and developmental effects are not areas of high concern.

The patient is brought to the emergency department in respiratory arrest after overdosing on heroin. The person accompanying the patient says he has been using heroin for years. After being administered one dose of a narcotic antagonist, the patient begins to breathe spontaneously but remains nonresponsive to stimuli so another dose of narcotic antagonist is ordered. What symptoms would indicate the patient is experiencing acute narcotic abstinence syndrome? (Select all that apply.) A) Tachycardia B) Hypertension C) Vomiting D) Confusion E) Sedation

A, B, C The most common adverse effect is an acute narcotic abstinence syndrome that is characterized by nausea, vomiting, sweating, tachycardia, hypertension, tremulousness, and feelings of anxiety. Confusion and sedation are not associated with acute narcotic abstinence syndrome.

The nurse works in a clinic that has many African American patients. What would the nurse need to consider when caring for patients with hypertension? (Select all that apply.) A) African American men are at highest risk for hypertension. B) African Americans respond best to single-drug therapy. C) African Americans are most responsive to angiotensin-converting enzyme inhibitors. D) Increased adverse effects occur when using thiazide and thiazide-like diuretics. E) First-line use of a calcium channel blocker with changes to lifestyle is best in African Americans.

A, B, D In the United States, African Americans are at highest risk for developing hypertension, with men more likely than women to develop the disease. African Americans are most responsive to single-drug therapy (as opposed to combination drug regimens). African Americans are less responsive to angiotensin-converting enzyme inhibitors and beta-blockers. Increased adverse effects (e.g., depression, fatigue, drowsiness) often occur when using thiazide and thiazide-like diuretics. Because African Americans are more responsive to diuretics, the treatment approach should include the first-line use of a diuretic in combination with diet and other lifestyle changes.

The nurse is caring for a patient with edema who has just begun taking a diuretic. What will the nurse use to evaluate the effectiveness of this medication? (Select all that apply.) A) Daily weight B) Decrease in edema C) Increase in blood pressure D) Increase in urinary output E) Increase in pulse

A, B, D Responsiveness to the use of a diuretic can be measured by daily weights, increased urinary output, decrease in edema, decrease in blood pressure and pulse rate. Options C and E are not correct.

While studying the antihypertensive drugs, the nursing students learn that the pressure in the cardiovascular system is determined by three elements. What are they? (Select all that apply.) A) Heart rate B) Stroke volume C) Preload D) Total peripheral resistance E) Pulse pressure

A, B, D The pressure in the cardiovascular system is determined by three elements: heart rate, stroke volume, or the amount of blood that is pumped out of the ventricle with each heartbeat (primarily determined by the volume of blood in the system), and total peripheral resistance, or the resistance of the muscular arteries to the blood being pumped through. The preload and the pulse pressure are not factors that regulate pressure in the cardiovascular system.

A patient with rheumatoid arthritis is taking gold salts. What drugs should the nurse teach this patient that are contraindicated when taking gold salts? (Select all that apply.) A) Antimalarials B) Cytotoxic drugs C) Salicylates D) Penicillamine E) Anticoagulants

A, B, D These drugs should not be combined with penicillamine, antimalarials, cytotoxic drugs, or immunosuppressive agents other than low-dose corticosteroids because of the potential for severe toxicity. No contraindication exists for therapy involving gold salts and salicylates or anticoagulants.

A patient has just begun therapy with furosemide (Lasix), and the nurse is instructing the patient about the need to include foods high in potassium in the diet. Which foods would be appropriate for this patient to choose? (Select all that apply.) A) Prunes B) Apples C) Watermelon D) Lima beans E) Rice

A, C, D Foods high in potassium include avocados, bananas, broccoli, cantaloupe, dried fruits, grapefruit, lima beans, nuts, navy beans, oranges, peaches, potatoes, prunes, rhubarb, Sanka coffee, sunflower seeds, spinach, tomatoes, and watermelon. Apples and rice are not potassium-rich foods.

Narcotic agonists-antagonists have what function? (Select all that apply.) A) Relief of moderate-to-severe pain B) Adjunctive therapies to nonsteroidal anti-inflammatory drugs (NSAIDs) C) Relief of pain during labor and delivery D) Relief of orthopedic pain E) Adjuncts to general anesthesia

A, C, E These drugs have three functions: (1) relief of moderate-to-severe pain, (2) adjuncts to general anesthesia, and (3) relief of pain during labor and delivery. Adjunctive therapies to NSAIDs or specificity for orthopedic pain are not functions of this classification of medication.

The clinic nurse is caring for a patient who is taking a COX-2 inhibitor and knows that this patient needs to be assessed for what? (Select all that apply.) A) Bleeding time B) Liver function C) Altered hearing D) Gastrointestinal (GI) effects E) Water retention

A, D, E COX-2 inhibitors have an impact on many body functions and patients receiving this therapy should be assessed for GI effects, changes in bleeding time, and water retention. Patients taking COX-2 inhibitors do not need to be evaluated for liver function or altered hearing because these are not common adverse effects.

The patient does not want to take medication to treat his or her hypertension if he or she can avoid it and asks the nurse if there is anything else he or she can do? What lifestyle changes will the nurse recommend? (Select all that apply.) A) "Increase time spent exercising." B) "Eliminate all salt from your diet." C) "Reduce intake of fluids." D) Lose some weight." E) "Try meditation."

A, D, E Exercising, losing weight, and meditation to reduce stress are all effective changes the patient can make in her lifestyle. However, if these do not bring blood pressure to within acceptable limits, the next step in the stepped-care management approach to treat hypertension would be to introduce a diuretic, beta-blocker, angiotensin-converting enzyme inhibitor, calcium channel blocker, or angiotensin II-receptor blocker in addition to the lifestyle changes. It is not necessary, or even possible, to eliminate all sodium from the diet but cutting back will reduce water retention. Reducing intake of fluids is not a healthy option and would not be suggested unless comorbidities, such as possibility of heart failure, were present.

The anatomy and physiology instructor is talking about pain sensations. What produces pain sensations when stimulated by generating nerve impulses? (Select all that apply) A) A-delta fibers B) D-delta sensory nerves C) Mu receptors D) Sigma-receptors E) C fibers

A, E Two small-diameter sensory nerves, A-delta and C fibers, respond to stimulation by generating nerve impulses that produce pain sensations. Large-diameter sensory nerves (i.e., A fibers) transmit sensations associated with touch and temperature. Mu-receptors are primarily pain-blocking receptors; sigma-receptors cause papillary dilation and may be responsible for the hallucinations, dysphoria, and psychoses that can occur with narcotic use.

After administering a medication, for what would the nurse assess the patient? A)Drug effects B)Allergies C)Pregnancy D)Preexisting conditions

Ans: A Feedback: After the medication is administered, the nurse assesses the patient for drug affects, both therapeutic and adverse. The nurse would assess the patient for allergies, preexisting conditions, and pregnancy before administering a medication.

A patient asks the nurse, What is a Drug Enforcement Agency (DEA) number? What is the nurse's best response? A)DEA Numbers are given to physicians and pharmacists when they register with the DEA to prescribe and dispense controlled substances. B)Physicians must have a DEA number in order to prescribe any type of medication for patients. C)DEA numbers are case numbers given when someone breaks the law involving a controlled substance. D)DEA numbers are contact numbers to talk with someone at the DEA when questions arise about controlled substances.

Ans: A Feedback: All pharmacists and physicians must register with the DEA. They are given numbers that are required before they can dispense or prescribe controlled substances. DEA numbers are only needed when prescribing controlled substances. A DEA number is neither a case number nor a phone number.

The nursing instructor is discussing drug therapy in the older adult. What would the instructor tell the students about what could affect therapeutic dosing in an older adult? A)Changes in the gastrointestinal (GI) system can reduce drug absorption. B)In older adults, drugs enter into circulation more quickly. C)In older adults, drugs are distributed to a smaller portion of the tissues. D)In older adults, drugs have an increased action.

Ans: A Feedback: As patients age, the body undergoes many normal changes that can affect drug therapy, such as a decreased blood volume, decreased GI absorption, reduced blood flow to muscles or skin, and changes in receptor site responsiveness. They are not released more quickly into circulation; distributed to a smaller portion of tissue; nor do they have an increased action.

A nurse is caring for a 77-year-old patient. The nurse plans care for this patient based on the knowledge that the aging process impacts drug therapy in what important way? A)Blood volume decreases B)Subcutaneous tissue increases C)Total body water increases D)Muscle mass increases

Ans: A Feedback: As patients age, the body undergoes many normal changes that can affect drug therapy, such as a decreased blood volume, decreased gastrointestinal (GI) absorption, reduced blood flow to muscles or skin, and changes in receptor site responsiveness. As a person ages, subcutaneous tissue decreases, total body water decreases, and muscle mass decreases.

What concept is considered when generic drugs are substituted for brand name drugs? A)Bioavailability B)Critical concentration C)Distribution D)Half-life

Ans: A Feedback: Bioavailability is the portion of a dose of a drug that reaches the systemic circulation and is available to act on body cells. Binders used in a generic drug may not be the same as those used in the brand name drug. Therefore, the way the body breaks down and uses the drug may differ, which may eliminate a generic drug substitution. Critical concentration is the amount of a drug that is needed to cause a therapeutic effect and should not differ between generic and brand name medications. Distribution is the phase of pharmacokinetics, which involves the movement of a drug to the body's tissues and is the same in generic and brand

Student nurses are learning to weigh patients and do vital signs. How does a correct weight impact administering medication? A)Proper dosage calculation B)Assessing changes in fluid balance C)Assessing changes in nutritional status D)Caloric needs

Ans: A Feedback: Dosage of medication is often calculated based on the patient's weight, so getting patients' weight wrong could cause a medication error. The patient's weight gives information into fluid balance, nutritional status, and caloric needs but this is not associated with drug therapy. However, a patient's weight is most important in determining the appropriateness of drug dosage.

While collecting a medication history, the patient admits to doubling the recommended dosage of an over-the-counter (OTC) medication, saying It's harmless or they would require a prescription. What is the nurse's best response? A)OTC drugs are serious medications and carry serious risks if not taken as directed. B)Taking medications like that is careless and you could kill yourself doing it. C)Sometimes you need to take more than the package directs to treat the symptoms. D)Did you notify your doctor of the increased dosage you were taking?

Ans: A Feedback: OTC drugs are no less a medication than prescription drugs and carry the same types of risks for overdosage and toxicity if directions are not followed. Although increasing the dosage is careless and dangerous, it is important to use the information as a teaching opportunity rather than scolding the patient. Agreeing with the patient or asking her if she talked to the doctor misses the teaching opportunity, which could be harmful for the patient.

After admitting a patient to the unit, the nurse is organizing times to administer ordered medications. What important consideration will guide the nurse's timing of each medication? A)Comfort B)Ethnicity of patient C)Gender D)Age

Ans: A Feedback: Organizing the day and the drug regimen to make it the least intrusive on a patient's comfort can help to prevent errors and improve compliance. Ethnicity, gender, and age should be a consideration when ordering the drug, but after the drug is chosen it should be administered in a way that will maintain the patient's quality-of-life.

The nurse, assisting with Phase I drug studies, is talking with a woman who asks, Why can't I participate in this study? What would be the nurse's best response? A)Drugs pose a greater risk to women of reproductive age. B)Drugs are only tested on men because they are stronger. C)Women are more prone to adverse effects from medications. D)Drugs affect women differently than they affect women.

Ans: A Feedback: Phase I drug trials usually involve healthy male volunteers because chemicals may exert an unknown and harmful effect on ova in women which could result in fetal damage when the woman becomes pregnant. Drugs are tested on both men and women, but women must be fully informed of risks and sign a consent stating they understand the potential for birth defects. Women are not more prone to adverse effects of medications. Although some drugs may affect women differently than men, this is a rationale for why drugs need to be tested on women, not an explanation of why women are not included in a phase I study.

The nurse explains the Drug Enforcement Agency's (DEA's) schedule of controlled substances to the nursing assistant who asks, Do you ever get a prescription for Schedule I medications? What is the nurse's best response? A)Schedule I medications have no medical use so they are not prescribed. B)Schedule I medications have the lowest risk for abuse and do not require a prescription. C)Schedule I medications are only prescribed in monitored units for patient safety. D)Schedule I medications are found in antitussives and antidiarrheals sold over the counter.

Ans: A Feedback: Schedule I medications have no medical use and are never prescribed. Schedule V medications have the lowest risk for abuse and are found mostly in antitussives and antidiarrheals but they are not sold over the counter.

In response to the patient's question about how to know whether drugs are safe, the nurse explains that all medications undergo rigorous scientific testing controlled by what organization? A)Food and Drug Administration (FDA) B)Drug Enforcement Agency (DEA) C)Centers for Disease Control and Prevention (CDC) D)Joint Commission on Accreditation of Healthcare Organizations (JCAHO)

Ans: A Feedback: The FDA is responsible for controlling and regulating the development and sale of drugs in the United States, allowing new drugs to enter the market only after being subjected to rigorous scientific testing. The DEA regulates and controls the use of controlled substances. The CDC monitors and responds to infectious diseases. The JCAHO is an accrediting body that inspects acute care facilities to ensure minimum standards are met.

The nurse is preparing a medication that is new to the market and cannot be found in the nurse's drug guide. Where can the nurse get the most reliable information about this medication? A)Package insert B)Another nurse C)Drug manufacturer D)Physician

Ans: A Feedback: The most reliable information about the drug can be found on the package insert supplied by the manufacturer because it was prepared according to strict Food and Drug Administration (FDA) regulations. Asking another nurse or the physician is not reliable and cannot be verified as accurate. It would not be realistic to call the drug manufacturer for information.

The nurse receives an order to administer an unfamiliar medication and obtains a nurse's drug guide published four years earlier. What is the nurse's most prudent action? A)Find a more recent reference source. B)Use the guide if the drug is listed. C)Ask another nurse for drug information. D)Verify the information in the guide with the pharmacist.

Ans: A Feedback: The nurse is responsible for all medications administered and must find a recent reference source to ensure the information learned about the medication is correct and current. Using an older drug guide could be dangerous because it would not contain the most up-to-date information. Asking another nurse or the pharmacist does not guarantee accurate information will be obtained and could harm the patient if the information is wrong.

When moving to another state, what is the nurse responsible for becoming familiar with? A)Local policies and procedures for controlled substance administration B)Local provider's Drug Enforcement Agency (DEA) number for prescribing controlled substances C)The agency monitoring controlled substances in the new state D)Board of Nursing regulations of controlled substances in the new state

Ans: A Feedback: The nurse needs to learn local policies and procedures for controlled substance administration because they can vary with some local governments more rigorous than others. Nurses do not memorize a provider's DEA numbers. The DEA is a federal agency that monitors controlled substances in all states. State boards of nursing do not regulate controlled substances but may regulate how controlled substances are administered by nurses.

The nurse is reviewing the patient's admission medication orders. What order would the nurse to question? A)Digoxin .5 mg orally nowgive one dose only B)Lasix 20 mg. IV every 4 hours times 3 C)Gentamicin 80 mg IV to infuse over 1 hour every 12 hours D)Acetaminophen 650 mg PO every 4 hours as needed for pain

Ans: A Feedback: The nurse should question the order for Digoxin.5 mg to make sure 0.5 mg is meant versus 5 mg, or what should have been ordered which is 0.05 mg. There should always be a 0 before a decimal point if no other number is present to make sure the decimal point is seen. The remaining orders could be administered as written.

A nurse is caring for a child on the pediatric unit. A drug is ordered for the child, but no pediatric dose is listed for the drug. To make sure that the right dose has been ordered, what will the nurse use to calculate the correct dose? A)Surface area B)Height C)Birth date D)Adult dosage

Ans: A Feedback: The surface area of a child is calculated using height and weight. It is the most accurate way to determine an appropriate dosage for that child. Age does not take into consideration variations in growth. Height alone does not take into account the mass of the child. Gestational age is simply a distracter.

The patient is prescribed a medication that was just placed in Phase IV study. The patient tells the nurse, This medication is too expensive. Could the doctor order a generic form of this medication? What is the nurse's most accurate response? A)Medications are not produced in generic form until the patent expires, which normally takes several years. B)You can request the generic form but the binder used may make the drug less effective for this medication. C)The generic form of the medication would not be any less expensive because this is a relatively new medication. D)Generic medications are lower quality drugs and that would mean you would not be getting the best treatment available.

Ans: A Feedback: When a new drug enters the market, it is given a time-limited patent; generic forms of the medication cannot be produced until the patent expires. Because no generic version of this drug will exist because it is so new, it is impossible to predict what binder will be used or what the cost would be.

The nurse is preparing to administer a medication from a multidose bottle. The label is torn and soiled but the name of the medication is still readable. What is the nurse's priority action? A)Discard the entire bottle and contents and obtain a new bottle. B)Find the drug information and create a new label for the bottle. C)Ask another nurse to verify the contents of the bottle. D)Administer the medication if the name of the drug can be clearly read.

Ans: A Feedback: When the drug label is soiled obscuring some information the safest action by the nurse is to discard the bottle and contents because drug labels contain a great deal of important information, far more than just the name of the drug. Concentration of the drug, expiration date, administration directions, and precautions may be missing from the label and so put the patient at risk. Looking up drug information in a drug handbook or consulting with another nurse will not supply the expiration date or concentration of medication. Be safe and discard the bottle and its contents.

What is the responsibility of the nurse related to the patient's drug therapy? (Select all that apply.) A)Teaching the patient how to cope with therapy to ensure the best outcome B)Providing therapy as well as medications C)Evaluating the effectiveness of therapy D)Altering the drug regimen to optimize outcome E)Recommending appropriate over-the-counter medications to treat adverse effects of prescription drug therapy

Ans: A, B, C Feedback: A nurse is, therefore, a key health care provider who is in a position to assess the whole patient, to administer therapy as well as medications, to teach the patient how best to cope with the therapy to ensure the most favorable outcome, and to evaluate the effectiveness of the therapy. Nurses do not alter drug therapy or recommend over-the-counter medications because prescribing is outside the nurse's scope of practice.

It is often necessary to obtain baseline data prior to initiating many forms of drug therapy. These baseline data include what? (Select all that apply.) A)Education level B)Allergies C)Drug use D)Number of members in family E)Father's occupation

Ans: A, B, C Feedback: Assessing educational level allows the nurse to plan an effective teaching plan. Allergies must be fully assessed before administering any medication to prevent allergic responses. Understanding the patient's current drug use informs the nurse about drugs that may interact or be impacted. Knowing the number of family members and father's occupation would not promote safer drug administration.

The nurse learns that a drug needed by the patient is classified as an orphan drug and recognizes what as a reason for this classification? (Select all that apply.) A)The drug is rarely prescribed. B)The drug has dangerous adverse effects. C)The drug treats a rare disease. D)The patent on the medication is still effective. E)Production by a company that only manufactures drugs.

Ans: A, B, C Feedback: Drugs are classified as orphan drugs when they are not financially viable for a drug company to produce either because of risk for lawsuits about adverse effects or because the drug is not prescribed, which is often seen in rare diagnoses. Generic drugs are not produced until the patent expires, but this has no impact on classifying a particular drug as an orphan drug. Generic drugs are often produced by companies that only manufacture drugs without conducting research, but this has no bearing on the classification of orphan drugs.

The patient asks the nurse, Is it safe to take over-the-counter (OTC) medications with prescription medications? What is the nurse's best response? (Select all that apply.) A)OTC medications can interact with prescription medications. B)It is important to tell your doctor all medications you take, including OTC. C)OTC medications could mask or hide signs and symptoms of a disease. D)You should avoid taking any OTC medication when taking prescription drugs. E)Taking OTC medications can make your prescription medication more effective.

Ans: A, B, C Feedback: OTC medications can interact with prescription medications or other OTC so it is always important to consult your pharmacist and provider for advice. To provide the most accurate instruction, the health care provider must know all medications taken including dietary supplements, OTC, and prescription. OTC medications could mask or hide symptoms of a disease so it is always important to consult a physician if symptoms persist. OTC medications are not prohibited when taking prescription drugs as long as no drug interaction occurs. How an OTC will impact a prescription medication varies depending on the medications involved, so it is incorrect to say it will make the prescription drug more effective.

Before administering a prescription medication, what information does the nurse find on the drug label? (Select all that apply.) A)Brand name B)Generic name C)Drug concentration D)Expiration date E)Adverse effects

Ans: A, B, C, D Feedback: Prescription drug labels will contain the brand name, generic name, drug concentration, and expiration date. Adverse effects will not be listed on drug labels.

What action does the nurse take during the intervention state of the nursing process related to drug therapy? (Select all that apply.) A)Administer the medication. B)Determine medication effectiveness. C)Document the medication. D)Analyze the data collected. E)Collect a nursing history.

Ans: A, C Feedback: During the implementation phase, the nurse administers and documents the medication. Effectiveness of the medication is determined during the evaluation phase. Analyzing data occurs when assigning appropriate nursing diagnoses. Collecting a nursing history is part of the assessment stage of the nursing process.

Teaching the patient/caregiver about her or his medications is an important step in reducing the risk of medication errors. What is an important teaching point about medications? (Select all that apply.) A)Speak up and ask questions. B)Store medications in a warm humid place. C)Adjust your medication according to how you feel. D)Keep a list of your prescribed medications. E)Take all medications together in the morning.

Ans: A, D Feedback: Appropriate patient teaching will reduce the risk of medication errors and complications. Nurses teach patients to speak up, ask questions, and act as his or her own advocate when medications are being prescribed. He should keep a complete list of medications and have a copy available at all times in case of accident. Store drugs in a dry, cool place away from children and pets that could be harmed. Take medications as they have been prescribed and do not adjust dosage without authorization from the prescriber. Take medications at the time they are prescribed to be taken, always being aware that some drugs cannot be taken together.

What would be the best source of drug information for a nurse? A)Drug Facts and Comparisons B)A nurse's drug guide C)A drug package insert D)The Physicians' Drug Reference (PDR)

Ans: B Feedback: A nurse's drug guide provides nursing implications and patient teaching points that are most useful to nurses in addition to need-to-know drug information in a very user friendly organizational style.Lippincott's Nursing Drug Guide (LNDG) has drug monographs organized alphabetically and includes nursing implications and patient teaching points. Numerous other drug handbooks are also on the market and readily available for nurses to use. Although other drug reference books such as Drug Facts and Comparisons, PDR, and drug package inserts can all provide essential drug information, they will not contain nursing implications and teaching points and can be more difficult to use than nurse's drug guides.

Which statement best describes drug efficacy/toxicity in pediatric patients? A)Drug requirements for infants have been extensively studied. B)Drug dosage is altered by age and weight in children. C)Children always need smaller doses of medication than adults. D)Infants and children are not at risk for drug toxicity if the dosage is correct.

Ans: B Feedback: All aspects of pediatric drug therapy must be guided by the child's age, weight, and level of growth and development. Drugs are generally studied using healthy young men and are never studied in infants because they are not old enough to give consent for themselves. Drug dosages are very specific and a big child may weigh more than an adult and require a larger dose. Even when medications are given accurately, adverse effects can occur.

When taking a medication history on a patient why should the nurse ask about the use of complementary or alternative therapies? A)Patients starting on new drugs are usually not compliant with medical regimens. B)Many drug-alternative therapy interactions can cause serious problems. C)Natural products may be more effective and the prescribed drug may not be needed. D)The cost of the drug and the alternative therapy may be too expensive for the patient to handle.

Ans: B Feedback: Alternative therapies often involve the use of herbal products, which contain natural chemicals that affect the body. Many drug-alternative therapy interactions have been reported that could cause serious adverse effects, but patients often don't think to mention these therapies when asked about the medications they are taking. The health care provider needs to be alert to these possible interactions and to adjust treatment appropriately. Cost and effectiveness may be factors, but the balancing of these therapies in the drug regimen to prevent interactions is the main concern of the nurse.

A nurse is instructing a pregnant patient concerning the potential risk to her fetus from a Pregnancy Category B drug. What would the nurse inform the patient? A)Adequate studies in pregnant women have demonstrated there is no risk to the fetus. B)Animal studies have not demonstrated a risk to the fetus, but there have been no adequate studies in pregnant women. C)Animal studies have shown an adverse effect on the fetus, but there are no adequate studies in pregnant women. D)There is evidence of human fetal risk, but the potential benefits from use of the drug may be acceptable despite potential risks.

Ans: B Feedback: Category B indicates that animal studies have not demonstrated a risk to the fetus. However, there have not been adequate studies in pregnant women to demonstrate risk to a fetus during the first trimester of pregnancy and no evidence of risk in later trimesters. Category A indicates that adequate studies in pregnant women have not demonstrated a risk to the fetus in the first trimester or in later trimesters. Category C indicates that animal studies have shown an adverse effect on the fetus, but no adequate studies in humans. Category D reveals evidence of human fetal risk, but the potential benefits from the use of the drugs in pregnant women may outweigh potential risks.

The patient looks at the prescription provided by the doctor and asks the nurse whether he can request a generic substitution. The nurse answers No when noting what on the prescription? A)No refills B)DAW C)Brand name used on prescription D)Patient older than 65 years of age

Ans: B Feedback: DAW stands for dispense as written and means that the doctor does not want a generic substituted for the prescribed medication. Requesting no refills does not preclude the substitution of a generic medication. Even when the brand name is ordered, the pharmacist can substitute a generic equivalent so long as the prescriber does not write DAW. Generic substitutions are not impacted by the patient's age.

The nurse is reviewing the patient's medication orders and finds an order stating amoxicillin 250 mg every 8 hours. What would the nurse question regarding this order? A)Dose B)Route C)Frequency of administration D)Allowance for generic substitution

Ans: B Feedback: For the nurse to administer a medication, all essential components of a medication order must be written by the prescriber including drug name, dosage, route, frequency, and patient name. This order is missing the route and the drug could be given IV, IM, or PO. The nurse should call the ordering health care provider and clarify what route the medication is to be administered.

A physician has ordered intramuscular (IM) injections of morphine, a narcotic, every 4 hours as needed for pain in a motor vehicle accident victim. The nurse is aware this drug has a high abuse potential. Under what category would morphine be classified? A) Schedule I B) Schedule II C) Schedule III D) Schedule IV

Ans: B Feedback: Narcotics with a high abuse potential are classified as Schedule II drugs because of severe dependence liability. Schedule I drugs have high abuse potential and no accepted medical use. Schedule III drugs have a lesser abuse potential than II and an accepted medical use. Schedule IV drugs have low abuse potential and limited dependence liability.

The home health nurse is caring for a 77-year-old patient with congestive heart failure. What would the nurse consider a priority to assess to develop the most effective plan of care related to medication administration? A)Description of the patient's living environment B)Required lifestyle changes C)Family members in the community D)Compliance with therapy to reduce risk of skin breakdown

Ans: B Feedback: Nurses must consider how a person responds to disease and its treatment, including the changes in lifestyle that may be required. By recognizing required lifestyles during the home visit the nurse can teach the patient how to make healthy choices and support the process of changing to new choices. Although the environment would impact the physical plan of care, it would not be a factor in the administration of medications. Assessment of family members in the community would not be a necessary part of the assessment in relation to the patient's drug therapy. Nothing suggests the patient is at risk for skin breakdown so this would not factor into the medication regimen.

The nurse applies the nursing process in medication therapy to ensure what? A)That medications are given at the right time B)That care is efficient and effective C)That the right dose of the drug is given to the patient D)That the right drug is given to the right patient at the right time

Ans: B Feedback: Nurses use the nursing process as a decision-making, problem-solving process to improve the efficiency and effectiveness of care. Options A, C, and D are incorrect and do not describe why the nursing process is important as much as they explain how to give medications safely.

A nurse working in radiology administers iodine to a patient who is having a computed tomography (CT) scan. The nurse working on the oncology unit administers chemotherapy to patients who have cancer. At the Public Health Department, a nurse administers a measles-mumps-rubella (MMR) vaccine to a 14-month-old child as a routine immunization. Which branch of pharmacology best describes the actions of all three nurses? A) Pharmacoeconomics B) Pharmacotherapeutics C) Pharmacodynamics D) Pharmacokinetics

Ans: B Feedback: Pharmacology is the study of the biologic effects of chemicals. Nurses are involved with clinical pharmacology or pharmacotherapeutics, which is a branch of pharmacology that deals with the uses of drugs to treat, prevent, and diagnose disease. The radiology nurse is administering a drug to help diagnose a disease. The oncology nurse is administering a drug to help treat a disease. Pharmacoeconomics includes any costs involved in drug therapy. Pharmacodynamics involves how a drug affects the body and pharmacokinetics is how the body acts on the body.

The nurse explains that what drug resource book is compiled from package inserts? A)Nurses Drug Guide B)Physicias's Desk Reference (PDR) C)Drug Facts and Comparisons D)AMA Drug Evaluations

Ans: B Feedback: The PDR is a compilation of information found on package inserts. The Nurses Drug Guide uses more easily understood language and incorporates nursing considerations and patient teaching points. Drug Facts and Comparisons includes cost comparison, often not found in other drug resource guides. The AMA Drug Evaluations is far less biased than the PDR and includes drugs still in the research stage of development.

What would the nurse provide when preparing a patient for discharge and home medication self-administration? A)Personal contact information to use if the patient has questions B)Thorough medication teaching about drugs and the drug regimen C)Over-the-counter medications to use to treat potential adverse effects D) A sample size package of medication to take home until prescription is filled

Ans: B Feedback: The nurse is responsible for providing thorough medication teaching about drugs and the drug regimen to ensure the patient knows how to take the medication and when to notify the provider. The nurse never provides personal contact information to a patient. If adverse effects arise, the patient is taught to call the health care provider and should not self-medicate with over-the-counter drugs, which could mask serious symptoms. The nurse never dispenses medication because it must be properly labeled for home use; this is done by the pharmacy.

A 35-year-old male patient is admitted to the hospital with pneumonia. He was originally being treated at home, but became worse when he quit taking his antibiotic prematurely. What is an appropriate nursing diagnosis for this patient? A)Deficient knowledge: monitoring temperature B)Noncompliance C)Risk for injury related to hypoxia D)Non-adherence: overuse

Ans: B Feedback: This patient did not take his antibiotic as directed. He quit taking his medication too soon, probably when he began to feel better so he was noncompliant with care. Why he quit taking the medication is unknown; it could have been a knowledge deficit or inability to pay for the full prescription. This will require further assessment to determine. Further data would need to be collected to determine whether the patient is experiencing hypoxia. He did not overuse his medication so option D is inaccurate.

The nursing instructor observes several nursing students administer medications to their assigned patients. The instructor would stop what student from administering the medication until the error was corrected? A)The student checks the label on the medication against the administration record three times. B)The student enters the patient's room and asks the patient Are you Mr. Jones? C)The student checks the drug's brand name and generic name before taking it to the patient. D)The student documents the medication upon exiting the patient's room.

Ans: B Feedback: When the student asks the patient, Are you Mr. Jones the patient may nod in agreement, even if that is not his name. Perhaps he misheard, or maybe he wasn't paying attention to the name, or he may just want to be agreeable but this manner of ensuring the right patient receives a medication often results in errors. The student is correct to check the medication name against the medication administration record (MAR) three times to ensure the right drug is administered. The student who checks brand name and generic name is accurate in making sure the correct drug is prepared. Medication should be documented as soon as they are given so this student is also correct.

The nurse is conducting an admission assessment on a patient. When collecting data related to medications the nurse asks What medications are you currently taking? After collecting this information, what other questions should the nurse ask? (Select all that apply.) A)Do you take any medications? B)What over-the-counter (OTC) medications do you take? C)Do you take an herbs, vitamins, or supplements? D)Do you take medications safely when you take them? E)Why do you take this medication?

Ans: B, C Feedback: Patients often neglect to mention OTC drugs or alternative therapies (e.g., herbals) because they do not consider them to be actual drugs or they may be unwilling to admit their use to the health care provider. Ask patients specifically about OTC drug and alternative therapy use. The question What medications are you taking? has already been asked so there is no need to ask if they take any medications. Often, patients may take medications unsafely, so do not ask whether the patient takes drugs safely but instead assess exactly how they take medications to determine whether they are being taken safely. Patients should always know why they are taking each medication to understand whether they are getting the desired effect.

The nurse is caring for a patient who had a severe, acute, previously unseen adverse effect of a drug in Phase III testing. The patient asks, After all the testing done on this drug, didn't they know this adverse effect could occur? What is the nurse's best response? (Select all that apply.) A)Pharmaceutical companies sometimes underreport problems to make more money. B)Your response to this medication will be reported to the drug company and the Food and Drug Administration (FDA). C)When a drug begins to be used by a large clinical market, new adverse effects may be found. D)The pharmaceutical company weighs the benefits of the drug with the severity of adverse effects. E)After a drug reaches phase III testing it is considered an accepted drug and will not be recalled.

Ans: B, C Feedback: When a new and unexpected adverse effect occurs, especially one of a serious nature, it is reported to the drug company who reports it to the FDA immediately. When a large number of people begin using the drug in phase III studies, it is not unusual to identify adverse effects not previously noted. It would be both unprofessional and inaccurate to imply that pharmaceutical companies put profit ahead of patient concern because lawsuits would remove any potential profit if a drug proves harmful. The FDA is responsible for weighing risk versus benefit in deciding whether to allow the drug to move to the next phase of testing. Drugs found to have serious adverse effects can be removed from the market at any time.

The nurse, providing patient teaching about home medication use to an older adult, explains that even when drugs are taken properly they can produce negative or unexpected effects. What are these negative or unexpected effects called? A)Teratogenic effects B)Toxic effects C)Adverse effects D)Therapeutic effects

Ans: C Feedback: Negative or unexpected effects are known as adverse or side effects. Teratogenic effects are adverse effects on the fetus and not a likely concern for an older adult. Toxic effects occur when medication is taken in larger than recommended dosages caused by an increase in serum drug levels. Therapeutic effects are the desired actions for which the medication is prescribed.

After completing a course on pharmacology for nurses, what will the nurse know? A)Everything necessary for safe and effective medication administration B)Current pharmacologic therapy; the nurse will not require ongoing education for 5 years. C)General drug information; the nurse can consult a drug guide for specific drug information. D)The drug actions that are associated with each classification of medication

Ans: C Feedback: After completing a pharmacology course nurses will have general drug information needed for safe and effective medication administration but will need to consult a drug guide for specific drug information before administering any medication. Pharmacology is constantly changing, with new drugs entering the market and new uses for existing drugs identified. Continuing education in pharmacology is essential to safe practice. Nurses tend to become familiar with the medications they administer most often, but there will always be a need to research new drugs and also those the nurse is not familiar with because no nurse knows all medications.

A 70-year-old patient has just received a drug that can cause sedation. What would be the priority nursing diagnosis for this patient? A)Noncompliance: Cost of the drug B)Deficient knowledge: Unfamiliar with drug therapy C)Risk for injury: Related to adverse effects of the drug D)Ineffective health maintenance: Need for medication

Ans: C Feedback: Because of the patient's age and that the medication causes sedation, the highest priority nursing diagnosis is related to maintaining the patient's safety. Safety for the patient is the nurse's number one concern. There is nothing indicated related to the cost of the drug or the risk of noncompliance for this patient. Deficient knowledge will need to be addressed but it is not the priority when compared with patient safety. There is no indication the patient's need for this medication is related to an ineffective health maintenance issue.

In today's health care environment there is often more contact between the patient and the nurse than between the patient and the physician. How does this increased patient contact impact drug therapy? A)Choosing the best medication to treat the patient's condition B)Assessing the patient's preferred communication strategies C)Assessing the therapeutic success of the drug therapy D)Reducing dosage quickly when adverse effects arise

Ans: C Feedback: Because the nurse has the greatest direct and continued contact with the patient, the nurse is in the best position to detect minute changes that ultimately determine the course of drug therapytherapeutic success or discontinuation because of adverse or unacceptable responses. The nurse does not choose the medication or reduce dosage because it is outside the scope of practice of the nurse. Communication strategies used by patients are not related to drug therapy.

When the nurse reads in the drug handbook the section related to recommended dosage, it is important to remember that this suggested dosage is based on what? A)A 40-year-old man B)An average-sized adult C)A 150-pound adult male D)A healthy young adult

Ans: C Feedback: Drug studies base the therapeutic dosage, or that dose needed to reach a critical concentration, on the physiology of a 150-pound healthy adult male. Testing is not routinely done in women because of the potential for unknown effects on the ova. Testing would not be done on an obese adult or older adult because of the potential for underlying disease, altered metabolism, or reactions to the drug. Children and adolescents are never used for testing due to ethical concerns.

The patient tells the nurse about a new drug being tested to treat the disease she was diagnosed with and asks the nurse whether the doctor can prescribe a medication still in the preclinical phase of testing. What is the nurse's best response? A)The doctor would have to complete a great deal of paperwork to get approval to prescribe that drug. B)Sometimes pharmaceutical companies are looking for volunteers to test a new drug and the doctor could give them your name. C)Drugs in the preclinical phase of testing are only tested on animals and so would not be available to you. D)Drugs in the preclinical phase of testing are given only to healthy young men and so would not be available to you.

Ans: C Feedback: During the preclinical phase of testing drugs are tested on animals and are not available to patients. In phase I, the drug is tested on volunteers who are usually healthy young men. It is only in phase III studies that the drug is made available to prescribers who agree to closely monitor patients getting the medication.

The nurse is gathering assessment data from a medication history of a 38-year-old man with four children. What assessment information would be most important in providing care for this patient? A)The medication history of the patient's mother and/or father B)The name of the patient's pharmacy C)Insurance, financial support, and stability for the patient and his family D)The last time the patient was hospitalized

Ans: C Feedback: In this situation, insurance, financial support, and stability would be the most important data and may determine compliance with future drug therapy. The medication history of the patient's parents could indicate a pattern of overall attitude about drug therapy but is not the priority concern. The last time the patient was hospitalized could indicate whether the patient seeks medical care when appropriate or if he self-medicates, contributing to the nurse's knowledge of this individual but this is not the priority concern. The name of the pharmacy would be unnecessary unless the nurse anticipates having to call a prescription in to the pharmacy for the prescriber.

The nurse admits a patient to the unit and learns the patient has recently been diagnosed with chronic renal failure but has not informed the primary care provider of this diagnosis. What is the nurse's first priority? A)Administer medications ordered immediately. B)Maintain the patient's confidentiality. C)Call the admitting physician immediately. D)Provide teaching about chronic renal failure.

Ans: C Feedback: Knowledge of the patient's diagnosed renal failure is essential to proper medication ordering because some dosages will need to be decreased whereas other medications may be contraindicated in this patient. The nurse does not breach confidentiality when sharing information that impacts needed care to the primary care provider. Teaching about renal failure may be provided at some point but it is not the priority in this situation. Medications should not be administered until they are appropriately adjusted by a health care provider who is aware of the renal failure diagnosis.

The patient asks the nurse why generic drugs would be used and voices concerns that only the brand name product will be safe. What is the nurse's best response? A)Generic drugs are often less expensive. B)Some quality control problems have been found with generic drugs. C)Most generic drugs are very safe and can be cost effective as well. D)Although initial cost is higher for a brand name it may cost less in the long run.

Ans: C Feedback: Most generic medications are completely safe and may be identical to the brand name drug except generic medications are often less expensive, but this does not address the patient's concern about safety. Although some quality control issues have occurred in the past, this does not address the patient's concerns regarding safety or explain why generic drugs are prescribed and used. Although some doctors believe initial cost is higher but will cost less over time, this response also does not address the patient's concerns.

When assessing a patient before starting a drug regimen, why would the nurse consider it important to assess baseline kidney function? A)To anticipate adverse effects of drugs B)To determine patient's baseline electrolyte levels C)To determine patient's ability to excrete the drug D)To determine patient's ability to metabolize the drug

Ans: C Feedback: Patients with kidney or liver disease require very cautious medication administration, often needing dosages to be decreased and contraindicating some drugs altogether. The patient's renal status will indicate the ability to excrete the drug. Liver function is needed to assess metabolism. Electrolyte levels would be indicated by serum blood test results, not by studies of kidney function. The nurse's goal is to prevent or minimize adverse effects of drugs, not to anticipate them.

When involved in phase III drug evaluation studies, what responsibilities would the nurse have? A)Working with animals who are given experimental drugs B)Choosing appropriate patients to be involved in the drug study C)Monitoring and observing patients closely for adverse effects D) Conducting research to determine effectiveness of the drug

Ans: C Feedback: Phase III studies involve use of a drug in a vast clinical population in which patients are asked to record any symptoms they experience while taking the drugs. Nurses may be responsible for helping collect and analyze the information to be shared with the Food and Drug Administration (FDA) but would not conduct research independently because nurses do not prescribe medications. Use of animals in drug testing is done in the preclinical trials. Select patients who are involved in phase II studies to participate in studies where the participants have the disease the drug is intended to treat. These patients are monitored closely for drug

While studying for the test, the nursing student encounters the following drug: papaverine (Pavabid). What does the nursing student identify the name Pavabid as? A)The generic name B)The chemical name C)The brand name D)The chemical and generic name

Ans: C Feedback: Several clues indicate the brand name including capitalization of the first letter in the name and in parentheses. Generic names are not capitalized; chemical names are descriptions of the chemistry of the medication resulting in complicated names.

The nurse, working on the maternity unit, receives a call from a pregnant woman asking how she can know whether a medication is safe to take while pregnant. What is the nurse's best response? A)You can take any drug indicated as a Category A. B)No medications should be taken during pregnancy. C)Never take medication until you receive approval from your health care provider. D)Most medications are safe but you need to weigh benefit against risk.

Ans: C Feedback: The best response to a pregnant woman asking about medication usage is to talk with her obstetric practitioner because the best advice will come from someone who knows their health and pregnancy history. While Category A drugs have no known risk, they may be contraindicated by the woman's health condition or pregnancy issues and many pregnant women would not know what it means to be a Category A drug. Medications can be helpful during pregnancy if taken safely and appropriately. Although risk benefit needs to be weighed, it should occur with advice from the obstetric practitioner.

A 7-year-old boy fell off a wood pile while playing. He has been admitted to the intensive care unit with multiple broken bones and internal bleeding. What should the nurse know about drug therapy in this type of patient? A)Adverse effects may be decreased. B)Therapeutic effects may be increased. C)Pharmacodynamics may be altered. D)Pharmacokinetics remain the same.

Ans: C Feedback: The child's developmental age will influence pharmacokinetics and pharmacodynamics; the immature liver may not metabolize drugs in the same way and the kidneys may not work as efficiently as those of an adult. Adverse effects might be increased and therapeutic effects may be decreased.

A 73-year-old male patient is being discharged home today. The discharge order reads: Take 10 mL of guaifenesin (Robitussin) PO q4h. This over-the-counter pharmaceutical comes in bottles with plastic measuring caps. How much should the nurse teach the patient to take at home? A)1/2 tsp B)1 tsp C)2 tsp D)1 tbsp

Ans: C Feedback: The nurse is responsible for teaching the patient the correct drug dosage. You should teach the patient that 5 mL = 1 tsp, so 10 mL (5 × 2) = 2 tsp; therefore Options A, B, and D are incorrect. It is important that patients be taught how to take their medication using measurement systems they are familiar with and know how to use.

You are evaluating the discharge teaching you have done with your patient concerning drug therapy. What statement from the patient would indicate that teaching had been effective? A)I have to take three pills each day and I can take them at the time that fits my schedule. B)I should take the white pill in the morning because the doctor wants me to take it. C)I will add the names and dosages of these new drugs to my medication list in my wallet. D)I have prescriptions at different pharmacies. I shop around for the best price for each drug.

Ans: C Feedback: The patient needs to recognize the importance of keeping an updated list of all current medications and the need to share this list with all health care providers to avoid drug-drug interactions. The patient should understand exactly when to take medications, why that medication is being taken, and how to take it safely. Patients should be encouraged to use a single pharmacy because this will add another layer of safety because the pharmacy will know all drugs being prescribed to this patient.

The nurse admits an older adult patient to the emergency room with reports of shortness of breath on exertion and a productive cough. The nurse reviews the patient's current medications and the patient says, I take one pink pill every morning. The nurse asks the name of the drug and the patient says she doesn't know. The patient cannot supply the name of the drug or the purpose in taking it either. This happens with four other medications the patient says she takes. What is an appropriate nursing diagnosis for this patient? A)Acute confusion B)Risk-prone health behavior C)Ineffective health maintenance D)Noncompliance

Ans: C Feedback: This patient is demonstrating ineffective health maintenance because knowledge of what medications are being taken and why they are being taken is essential to her health. Patients should be instructed to carry an up-to-date list of current medications to share with all health care providers. She is taking her medication as ordered according to what she said so that would eliminate risk-prone health behaviors and noncompliance. There is no indication the patient is confused.

What aspect of pharmacology does a nurse study? (Select all that apply.) A)Chemical pharmacology B)Molecular pharmacology C)Impact of drugs on the body D)The body's response to a drug E)Adverse and anticipated drug effects

Ans: C, D, E Feedback: Nurses study pharmacology from a pharmacotherapeutic level, which includes the effect of drugs on the body, the body's response to drugs, and both expected and unexpected drug effects. Chemical and molecular pharmacology (Options A and B) are not included in nursing pharmacology courses.

The nurse would expect to see an adjusted dosage in what patients? (Select all that apply.) A)Young adult women B)Middle-aged men C)Infants D)Neonates E)Older adults

Ans: C, D, E Feedback: Patients at the extremes of the age spectrumchildren and older adultsoften require dosage adjustments based on the functional level of the liver and kidneys and the responsiveness of other organs. The child's age and developmental level will also alert the nurse to possible problems with drug delivery, such as an inability to swallow pills or follow directions related to other delivery methods. The adult, whether male or female, would not require altered dosage unless a secondary condition such as renal or hepatic alterations existed.

A 27-year-old man is admitted to the emergency department (ED) after a serious motorcycle accident. The patient has a head injury, abrasions covering the left side of his body, a broken left femur, and internal injuries that are not fully assessed at this time. He is transferred from the ED to the intensive care unit (ICU). The nurse who is going to care for this patient in the ICU knows that a priority responsibility in regard to drug therapy is what? A)Support vital functions B)Continue curative treatment C)Institute life-saving treatment D)Monitor patient's response

Ans: D Feedback: Because the nurse has the greatest direct and continued contact with the patient, the nurse is in the best position to detect minute changes that ultimately determine the course of drug therapytherapeutic success or discontinuation because of adverse or unacceptable responses. The nurse would support vital functions, continue curative treatment, and institute life-saving treatment, but these actions occur regardless of drug therapy.

The telephone triage nurse receives a call from a patient asking for a prescription for a narcotic to manage his surgical pain. The nurse explains that narcotic prescriptions must be written and cannot be called in to the pharmacy. The patient says, Why are narcotics so difficult to get a prescription for? What is the nurse's best response? A)The Drug Enforcement Agency (DEA) determines the risk for addiction and the Food and Drug Administration (FDA) enforces their control. B)The increase in the number of drug addicts has made the rules stronger. C)The Centers for Disease Control and Prevention (CDC) regulates use of controlled substances to reduce the risk of injury. D)Controlled substances like narcotics are controlled by the FDA and the DEA.

Ans: D Feedback: Controlled substances are controlled by the FDA and the DEA: the DEA enforces control while the FDA determines abuse potential. Regulations related to controlled substances have remained strict and specific and have not been significantly impacted by substance abusers. The CDC is not involved in control of narcotics and other controlled substances.

It is important for the nurse to evaluate the efficacy of what parameter when evaluating the drug therapy of a patient? A)Appropriateness of drug dosages B)Compliance C)Caregivers' knowledge level D)Nursing interventions

Ans: D Feedback: During the evaluation phase of care, nurses evaluate how effective care has been in meeting outcome goals. Appropriateness of drug dosages should be determined before administering the medication and not when evaluating their effects. Often, compliance cannot be evaluated until the nurse evaluates the effectiveness of therapy and finds the drug is not performing as expected, at which time the nurse may question the patient about whether medications are being taken as ordered. Caregivers' knowledge level is an assessment providing data that will determine the teaching plan.

During what phase of the nursing process would the nurse be required to consider the efficacy of nursing interventions related to drug therapy? A)Assessment B)Nursing diagnosis C)Interventions D)Evaluation

Ans: D Feedback: Evaluation allows the nurse to review what has changed since intervening to determine whether the nursing care has had a positive, therapeutic effect moving the patient toward a more healthful life. If outcomes have not improved, the nurse begins again at the assessment phase of the nursing process with the goal of changing the plan of care to improve outcomes. The patient's response to the drug and occurrence of adverse drug effects indicate the effectiveness of the nursing interventions related to drug therapy. Assessment involves a systematic, organized collection of data concerning a patient. A nursing diagnosis indicates actual or potential alterations in patient function based on the assessment of the clinical situation. Interventions are actions taken to meet the patient's needs, such as administration of drugs.

A 32-year-old woman is admitted to the unit with a diagnosis of hypovolemia. The nurse is developing a care plan for this patient. What is an appropriate nursing diagnosis to help prevent medication errors? A)Dysfunctional gastrointestinal motility B)Ineffective self-health maintenance C)Risk for injury D)Deficient fluid volume

Ans: D Feedback: Hypovolemia is condition involving fluid volume in the body that is less than required. This would affect drug therapy for this patient and would be an important inclusion in the plan of care. The situation described does not indicate dysfunctional GI motility or ineffective self-health maintenance. A patient who is severely fluid volume deficient might be at risk for falls and injury but more information would be needed to make this determination.

A nurse is assessing the patient's home medication use. After listening to the patient list current medications, the nurse asks what priority question? A)Do you take any generic medications? B)Are any of these medications orphan drugs? C)Are these medications safe to take during pregnancy? D)Do you take any over-the-counter medications?

Ans: D Feedback: It is important for the nurse to specifically question use of over-the-counter medications because patients may not consider them important. The patient is unlikely to know the meaning of orphan drugs unless they too are health care providers. Safety during pregnancy, use of a generic medication, or classification of orphan drugs are things the patient would be unable to answer but could be found in reference books if the nurse wishes to research them.

Discharge planning for patients leaving the hospital should include instructions on the use of over-the-counter (OTC) drugs. Which comment by the patient would demonstrate a good understanding of OTC drugs? A)OTC drugs are safe and do not cause adverse effects if taken properly. B)OTC drugs have been around for years and have not been tested by the Food and Drug Administration (FDA). C)OTC drugs are different from any drugs available by prescription and cost less. D)OTC drugs could cause serious harm if not taken according to directions.

Ans: D Feedback: It is important to follow package directions because OTCs are medications that can cause serious harm if not taken properly. OTCs are drugs that have been determined to be safe when taken as directed; however, all drugs can produce adverse effects even when taken properly. They may have originally been prescription drugs that were tested by the FDA or they may have been grandfathered in when the FDA laws changed. OTC education should always be included as a part of the hospital discharge instructions.

The nurse is caring for a patient who takes several drugs. What patient would the nurse monitor most closely because of an increased risk for adverse effects of medications? A)The school-aged child B)The obese middle-aged man C)The adolescent D)The newborn infant

Ans: D Feedback: Patients most likely to have adverse drug reactions include the very young or very old due to physiologic characteristics peculiar to these age groups. Therefore, the newborn infant would be at greater risk than the school-aged child who is at greater risk than the adolescent or middle-aged man.

The nurse is caring for a patient scheduled for surgery this morning who is not to be given anything orally. The nurse reviews the medication administration record and finds the patient has an important medication due but it is supposed to be given orally. What is the nurse's best action? A)Give the medication with a small sip of water. B)Give the medication via a different route. C)Hold the medication and put a note on the front of the chart for the surgeon. D)Call the ordering health care provider and clarify administration.

Ans: D Feedback: The nurse would consult with the ordering provider to determine whether the medication should be held, given by another route, or taken with a sip of water. Administering the medication with a small sip of water could cause the cancellation of the procedure, either because of the sip of water or because the medication may interfere with anesthesia. The nurse cannot change the route of administration without an order. Holding the medication would constitute a drug error because the medication was not given on time.

The nurse teaches the patient that a common adverse effect of loperamide (Imodium) is what? A) Fatigue B) Flatulence C) Disorientation D) Tremors

Ans: A Feedback: Adverse effects associated with antidiarrheal drugs, such as constipation, distention, abdominal discomfort, nausea, vomiting, dry mouth, and even toxic megacolon, are related to their effects on the gastrointestinal (GI) tract. Other adverse effects that have been reported include fatigue (option A), weakness, dizziness, and rash. options B, C, and D are not correct.

The nurse administers ranitidine (Zantac) cautiously to patients with evidence of what conditions? A) Renal disease B) Diabetes mellitus C) Pulmonary disease D) Migraine headaches

Ans: A Feedback: All histamine-2 antagonists are eliminated through the kidneys; dosages need to be reduced in patients with renal impairment. No caution is necessary with Zantac therapy in people with diabetes, pulmonary disease, or migraine headaches.

A man with irritable bowel syndrome reports ongoing diarrhea and asks for a prescription for alosetron (Lotronex), which was helpful for his coworker who recently started taking the drug. What is the nurses best response? A) This drug is only approved for use in women. B) This drug is used as a laxative. C) This drug is contraindicated with irritable bowel syndrome. D) This drug is no longer on the market for prescription use.

Ans: A Feedback: Alosetron (Lotronex) is approved for use in women with irritable bowel syndrome with diarrhea being the predominant complaint and should be discontinued immediately if the patient develops constipation or symptoms of ischemic colitis. Patients must read and sign a patientphysician agreement before it can be prescribed. Options B, C, and D are not correct.

What drug does the nurse recognize as being classified as a chemical stimulant? A) Bisacodyl (Dulcolax) B) Polycarbophil (FiberCon) C) Magnesium hydroxide (Milk of Magnesia) D) Docusate (Colace)

Ans: A Feedback: Bisacodyl is a chemical stimulant. Polycarbophil and magnesium hydroxide are bulk laxatives. Docusate is a lubricant laxative.

The nurse, teaching a patient to be discharged with an order to use chemical stimulant laxatives PRN, instructs the patient that one of the most common adverse effects of this type of laxative is what? A) Abdominal cramping B) Rectal bleeding C) Confusion D) Iron deficiency anemia

Ans: A Feedback: Common adverse effects of laxatives are diarrhea, abdominal cramping, and nausea. Central nervous system (CNS) adverse effects such as dizziness, headache, and weakness can occur. Rectal bleeding, confusion, and iron deficiency anemia are not associated with appropriate use of laxatives but may occur when laxatives are abused.

The nurse questions an order for a proton pump inhibitor when the patient is known to take what other medication? A) Theophylline B) Penicillin C) Digoxin D) Heparin

Ans: A Feedback: Decreased levels of ketoconazole and theophylline have been reported when combined with these drugs, leading to loss of effectiveness. There are no drugdrug interactions with penicillin, digoxin, or heparin.

The patient had surgery 2 days ago and bowel motility has not returned. What drug might the nurse administer to stimulate the gastrointestinal (GI) tract? A) Dexpanthenol B) Docusate C) Psyllium (Metamucil) D) Senna

Ans: A Feedback: Dexpanthenol is indicated for the prevention of intestinal atony or loss of intestinal muscle tone in postoperative adults. Docusate, psyllium, and senna are laxatives that would not be indicated for the postoperative patient with no bowel activity.

The nurse collects a stool culture from a patient diagnosed with travelers diarrhea. What bacterium does the nurse expect the culture to grow? A) Escherichia coli B) Staphylococcus aureus C) Streptococcus type B D) Pseudomonas

Ans: A Feedback: Escherichia coli is the most common cause of travelers diarrhea. Staphylococcus, Streptococcus, and Pseudomonas would be highly unlikely to grow in the stool of a patient with travelers diarrhea.

A patient presents at the walk-in clinic complaining of stomach pain that is relieved by eating which the nurse suspects is caused by a peptic ulcer. How would the nurse explain the ulceration in the stomach to the patient? A) A mucosal lining protects the stomach from hydrochloric acid and a reduction in the mucosal lining or increase in acid production allows peptic ulcers to develop. B) After digesting food, the stomach secretes a neutralizing solution to make the pH of the stomach neutral and if this doesnt occur peptic ulcers can develop. C) Our body makes hydrochloric acid for the digestion of the food we eat and that can eat right through the stomach causing ulceration when you eat a diet high in spicy food. D) Acid is secreted by cells in the first part of the small intestine and when we eat the acid flows into the stomach so people who eat often have more exposure to acid.

Ans: A Feedback: Gastrin and the parasympathetic system stimulate histamine-2 receptors near the parietal cells, causing the cells to release hydrochloric acid into the lumen of the stomach. Other digestive enzymes are released appropriately, in response to proteins and carbohydrates, to begin digestion. Peptic ulcers can develop when there is a decrease in the protective mucosal layer or an increase in acid production. Research indicates that people who eat a spicy diet everyday are less prone to stomach ulceration; the stomach does not neutralize the hydrochloric acid but the highly alkaline duodenum neutralizes the pH when chyme arrives from the stomach. Acid does not flow from the small intestine to the stomach.

The nurse administers metoclopramide to the patient with what condition? A) Chronic diabetic gastroparesis B) Impaction C) Encopresis D) Patients requiring diagnostic procedures

Ans: A Feedback: Indications for metoclopramide include relief of acute and chronic diabetic gastroparesis, short-term treatment of gastroesophageal reflux disorder in adults who cannot tolerate standard therapy, prevention of postoperative or chemotherapy-induced nausea and vomiting, facilitation of small-bowel intubation, stimulation of gastric emptying, and promotion of intestinal transit of barium. It would not be used for treatment of impaction, encopresis, and in patients requiring diagnostic procedures.

The nurse administers lubiprostone (Amitiza) to the patient with irritable bowel syndrome and anticipates what therapeutic action from the drug? A) Secretion of chloride-rich intestinal fluid leading to increased motility B) Adding bulk to the fecal matter to ease the process of stooling C) Irritation of the inner lining of the bowel to increase bowel motility D) Stimulate the bowel by increasing innervation

Ans: A Feedback: Lubiprostone is a locally acting chloride channel activator that increases the secretion of a chloride-rich intestinal fluid without changing sodium or potassium levels. Increasing the intestinal fluid leads to increased motility. It does not add bulk, irritate the inner lining, or innervate the bowel.

The hospice nurse is caring for a patient diagnosed with bone cancer who is receiving large doses of opioid medications to relieve pain. The patient has used other laxatives in the past to treat opioid-induced constipation but nothing is working now. What drug would the nurse request the family doctor to order for this patient? A) Methylnaltrexone (Relistor) B) Castor oil C) Paregoric D) Mineral oil

Ans: A Feedback: Methylnaltrexone (Relistor) was approved in 2008 for the treatment of opioid-induced constipation in patients with advanced disease who are receiving palliative care and are no longer responsive to traditional laxatives. Castor oil, mineral oil, and paregoric would likely be ineffective in this patient.

The nurse is caring for a 27-year-old female patient who has just been prescribed misoprostol. What is a priority teaching point for this patient? A) You will need to use a barrier-type contraceptive B) Do not take NSAIDs with this drug C) Adverse effects include nausea and diarrhea D) It protects the lining of the stomach

Ans: A Feedback: Misoprostol is contraindicated during pregnancy because it is an abortifacient. Women of childbearing age who use misoprostol should be advised to use barrier-type contraceptives. All other options are correct but are not a priority for this patient.

The family brings a patient to the emergency department saying he has been hallucinating and falls so deeply asleep he stops breathing when not stimulated. The nurse learns the patient has been self-treating diarrhea and suspects the patient was taking what medication? A) Paregoric B) Bismuth subsalicylate C) Loperamide D) Colace

Ans: A Feedback: Opium derivatives, like paregoric, are associated with light-headedness, sedation, euphoria, hallucinations, and respiratory depression related to their effect on opioid receptors. Nonopioids such as bismuth subsalicylate and loperamide would not cause respiratory depression. Colace is a stool softener, not an antidiarrheal.

A patient presents to the walk-in clinic complaining of vomiting and burning in his or her mid-epigastria. The nurse suspects peptic ulcer disease and knows that to confirm peptic ulcer disease, the physician is likely to order a diagnostic test to detect the possible presence of what? A) Infection with Helicobacter pylori B) Decreased stomach acid secretion C) Gastric irritation caused by nonsteroidal anti-inflammatory drugs (NSAIDs) D) Overconsumption of spicy foods

Ans: A Feedback: Peptic ulcers may result from increased acid production, decrease in the protective mucous lining of the stomach, infection with Helicobacter pylori bacteria, or a combination of these. Peptic ulcers do not result from decreased acid secretion or overconsumption of spicy foods. While gastric irritation can be caused by NSAIDs gastric irritation can result from many different causes so this would not be specifically assessed for unless the patient was found to use NSAIDs frequently, which is not indicated by the question.

What is the most basic type of movement that occurs in the esophagus? A)Peristalsis B) Mass movement C) Churning D) Segmentation

Ans: A Feedback: Peristalsis is the basic movement of the esophagus. The large intestine uses mass movement, the stomach uses churning, and the small intestine uses segmentation.

A patient comes to the clinic complaining of acid indigestion and tells the nurse he is tired of buying over-the-counter (OTC) antacids and wants a prescription drug to cure the problem. What would the nurse specifically assess for? A) Alkalosis B) Hypocalcemia C) Hypercholesterolemia D) Rebound tenderness at McBurneys point

Ans: A Feedback: Prolonged or excessive use of OTC antacids can lead to the development of metabolic alkalosis. Many antacids contain calcium so that low calcium levels would be unlikely. Because metabolic alkalosis is a concern, metabolic acidosis is unlikely. High cholesterol levels are not associated with OTC antacid use. Rebound tenderness at McBurneys point is related to appendicitis and not antacid use.

The nurse admits a patient who reports having taken a proton pump inhibitor for more than a decade. What assessment question will the nurse ask this patient? A) Are you experiencing diarrhea? B) Do you experience much abdominal pain? C) Have you noticed any blood in your urine? D) When is the last time you checked your blood pressure?

Ans: A Feedback: Research indicates that patients taking proton pump inhibitors or histamine-2 antagonists demonstrate a significant increase in cases of Clostridium difficile infections leading to diarrhea so the nurse should assess this patient for diarrhea. There would be no indications to ask about abdominal pain, blood in the urine, or blood pressure.

The nurse expects to find that the major activities of the GI tract decrease in a patient experiencing what? A) A stress reaction B) A medication regimen including cholinergic drugs C) A local irritation of the small intestine D) Thoughts of food

Ans: A Feedback: Stimulation of the sympathetic nervous system will decrease all activity in the GI tract, and stress is a trigger for stimulating the sympathetic nervous system. Parasympathetic stimulation caused by cholinergic drugs, local irritation of the intestine, and the thought of food will all increase GI activity.

The nurse administers loperamide (Imodium) to decrease the number and liquidity of stool by what mechanism? A) Decreasing intestinal motility B) Absorbing toxins C) Binding with fecal material to increase bulk D) Blocking the chemoreceptor trigger zone (CTZ)

Ans: A Feedback: Systemic antidiarrheal agents slow the motility of the gastrointestinal (GI) tract through direct action on the lining of the GI tract to inhibit local reflexes (bismuth subsalicylate), through direct action on the muscles of the GI tract to slow activity (loperamide), or through action on central nervous system (CNS) centers that cause GI spasm and slowing (opium derivatives). Options B, C, and D are not correct.

The 59-year-old patient has peptic ulcer disease and is started on sucralfate (Carafate). What is an appropriate nursing diagnosis related to this medication? A) Risk for constipation related to GI effects B) Risk for injury: bleeding C) Imbalanced nutrition related to nausea D) Deficient fluid volume

Ans: A Feedback: The adverse effects associated with sucralfate are primarily related to its GI effects. Constipation is the most frequently seen adverse effect. Imbalanced nutrition, if seen, would be related to diarrhea or constipation and not nausea. Fluid volume deficit and bleeding are not common adverse effects of this drug.

The patient prescribed nizatidine (Axid) chooses to take the medication once a day at bedtime. What dosage will the nurse administer? A) 150 to 300 mg B) 100 to 125 mg C) 20 to 40 mg D) 5 to 7.5 mg

Ans: A Feedback: The dosage for nizatidine, when taken PO only at bedtime, is 150 to 300 mg. Options B, C, and D are not correct.

When the nurse administers a medication that stimulates the parasympathetic nervous system the drug will have what impact on the GI system? A) Decrease sphincter tone B) Decrease muscle tone C) Decrease contractions D) Decrease secretions

Ans: A Feedback: The effect of the parasympathetic nervous system is that of rest and digest. It stimulates the GI tract causing increased muscle tone, secretions and contractions, and decreased sphincter tone allowing for easy movement.

After providing teaching to a patient prescribed lansoprazole (Prevacid), the nurse evaluates the patient understands the action of this medication when the patient makes what statement? A) The medication inhibits acid secretions. B) The medication is an antibiotic. C) The medication is an analgesic. D) The medication will promote healing of my ulcer.

Ans: A Feedback: The gastric acid pump or proton pump inhibitors suppress gastric acid secretion by specifically inhibiting the hydrogenpotassium adenosine triphosphatase (H+,K+-ATPase) enzyme system on the secretory surface of the gastric parietal cells. The statement, The medication inhibits acid secretions, indicates that the patient understands that the medication inhibits acid secretion. This medication does not act as an antibiotic or analgesic, nor will it repair the ulcer.

What actions are required by the patient to protect the lungs when vomiting? A) Closing of the glottis B) Deep respirations C) Production of extra saliva in the mouth D) Reduction in acid production in the stomach

Ans: A Feedback: The glottis must close and seal off the trachea in order to prevent entry of stomach contents into the lungs. The patient normally takes a deep breath just before the glottis closes but should not breath during emesis. Production of extra saliva and reduction in stomach acid do not protect the lungs.

The nurse is caring for a patient who has developed severe constipation. What rationale does the nurse have for why the patient has also lost his or her appetite? A) Ileogastric reflex B) Intestinalintestinal reflex C) Peritoneointestinal reflex D) Renointestinal reflex

Ans: A Feedback: The introduction of chyme or stretch to the large intestine slows stomach activity, known as the ileogastric reflex which explains why patients who are constipated have no appetite. The intestinalintestinal reflex is triggered by irritation of a section of the small intestine causing cessation of activity above that section. Peritoneointestinal reflex results from irritation of the peritoneum while renointestinal reflex results from irritation or swelling of the renal capsule.

What does the liver produce that is important in the digestive process? A) Bile B) Glucagon C) Ammonia D) Bilirubin

Ans: A Feedback: The liver produces bile, which is stored in the gallbladder. The bile is very important in the digestion of fats and is deposited into the small intestine when the gallbladder is stimulated to contract by the presence of fats. Glucagon is a hormone secreted in response to the presence of food in the bowel. Ammonia is a byproduct of the breakdown of protein and bilirubin is metabolized by the liver and stored to produce more red blood cells.

The nurse collects a nursing history from a patient who says she keeps so busy she often ignores the need to defecate. The nurse explains the danger of this behavior is what? A) External sphincter will require more distention to stimulate reflex B) Incontinence of stool due to loss of sphincter control C) Diarrhea D) Malabsorption of nutrients from the colon

Ans: A Feedback: The receptors in the external sphincter adapt relatively quickly and will stretch and require more and more distention to stimulate the reflex if the reflex is ignored. This will not lead to bowel incontinence and is more likely to result in constipation from prolonged water absorption than diarrhea. Nutrients are not absorbed from the colon. 23.

What symptoms would the nurse find to be consistent with a diagnosis of a peptic ulcer? A) Pain a few hours after eating B) Pain described as sharp and stabbing C) Explosive diarrhea within 1 hour of eating D) Unable to stand comfortably due to abdominal pain

Ans: A Feedback: Ulcer patients present with a predictable description of gnawing, burning pain often occurring a few hours after meals. The pain is not described as sharp and stabbing. Diarrhea is not usually associated with peptic ulcer disease and patients do not find it difficult to stand upright.

What does the nurse anticipate will happen first when the chemoreceptor trigger zone (CTZ) is stimulated? A) Salivation increases B) Gastric acid production decreases C) Sweating increases D) Heart rate increases

Ans: A Feedback: When the CTZ is stimulated, salivation is stimulated first, which leads to a decrease in gastric acid production and then an increase in sweating and heart rate.

The nurse recognizes the potential for stimulation of the patients CTZ in what circumstances? (Select all that apply.) A) Stimulation of the back of the throat when examining the oral cavity B) Excessive stomach distention C) Increased intracranial pressure D) Stimulation of stretch receptors in the large bowel E) When the patient sits up abruptly

Ans: A, B, C Feedback: Tactile stimulation of the back of the throat, excessive stomach distention, increased intracranial pressure, stimulation of vestibular receptors in the inner ear, and stimulation of stretch receptors in the uterus and bladder are types of stimuli that can activate the CTZ. Stimulating stretch receptors in the large bowel and sitting up abruptly are not activators of the CTZ.

What would the nurse consider an indication for the use of antacids? (Select all that apply.) A) Gastric hyperacidity B) Gastritis C) Peptic esophagitis D) Hiatal hernia E) Duodenal ulcer

Ans: A, B, C, D Feedback: Antacids neutralize stomach acid by direct chemical reaction. They are recommended for the symptomatic relief of upset stomach associated with hyperacidity, as well as the hyperacidity associated with peptic ulcer, gastritis, peptic esophagitis, gastric hyperacidity, and hiatal hernia. Duodenal ulcer is not an indication for the use of an antacid.

What organs are considered accessory digestive organs because they add in the digestive process? (Select all that apply.) A) The liver B) The pancreas C) The spleen D) The gallbladder E) The appendix

Ans: A, B, D Feedback: The pancreas, liver, and gallbladder are accessory organs that support the functions of the GI system. The spleen is not involved in digestion and the appendix is a part of the bowel, not an accessory organ.

What nursing interventions are included in the plan of care for a patient receiving antacids to relieve GI discomfort? A) Administer this drug with other drugs or food. B) Administer the antacid 1 hour before or 2 hours after other oral medications. C) Limit fluid intake to decrease dilution of the medication in the stomach. D) Have the patient swallow the antacid whole and do not crush or chew the tablet.

Ans: B Feedback: A patient taking antacids should be advised to take the antacid 1 hour before or 2 hours after other oral medications. These tablets are often chewed to increase effectiveness. Limiting fluid intake can result in rebound fluid retention so that patients should be encouraged to maintain hydration. It is not necessary to take an antacid with other drugs, nor with food.

What drug does the nurse administer that inhibits intestinal peristalsis through direct effects on the longitudinal and circular muscles of the intestinal wall? A) Bismuth subsalicylate B) Loperamide C) Paregoric D) Magnesium citrate

Ans: B Feedback: Actions of loperamide include that it inhibits intestinal peristalsis through direct effects on the longitudinal and circular muscles of the intestinal wall, slowing motility and movement of water and electrolytes. Bismuth subsalicylate inhibits local reflexes. Paregoric works through action on CNS centers that cause GI spasm and slowing. Magnesium citrate is a laxative.

A patient taking a chemical stimulant laxative and medications for heart failure and osteoarthritis calls the clinic and reports to the nurse that she is not feeling right. What is the priority question the nurse should ask this patient? A) Effectiveness of laxatives B) Timing of medication administration C) The amount of fiber in her diet D) Amount of fluid ingested

Ans: B Feedback: Because laxatives increase the motility of the gastrointestinal (GI) tract and some laxatives interfere with the timing or process of absorption, it is not advisable to take laxatives with other prescribed medications. The administration of laxatives and other medications should be separated by at least 30 minutes, so the nurse should question when the patient is taking the laxatives and other medication. Other options may be questions the nurse would eventually ask, but the priority is timing of medication administration.

Which substance is required for digestion and absorption of fats and fat-soluble vitamins? A) Hydrochloric acid B) Bile C) Pancreatic enzymes D) Gastric juice

Ans: B Feedback: Bile is an alkaline secretion that is formed continuously in the liver, carried to the gallbladder by the bile ducts, and stored there. Bile salts are required for digestion and absorption of fats, including fat-soluble vitamins. Hydrochloric acid is produced in the stomach and begins the process of digestion. Pancreatic enzymes include chymotrypsin and trypsin that break down proteins, lipases to break down fat, and amylases to break down sugars. Gastric juice includes hydrochloric acid.

The nurse, providing patient teaching, explains that difenoxin and diphenoxylate are chemically related to what medication? A) Morphine B) Demerol C) Diphenhydramine D) Diflucan

Ans: B Feedback: Difenoxin and diphenoxylate are chemically related to meperidine and are used at doses that decrease gastrointestinal activity without having analgesic or respiratory effects.

What is the stimulus for the Duodenalcolic reflex? A) Cephalic stimulation B) Presence of food C) Over extension of stomach D) Swallowing

Ans: B Feedback: Duodenalcolic reflex is stimulated by the presence of food or stretching in the duodenum that stimulates colon activity and mass movement, again to empty the colon for the new chyme.

What histamine-2 antagonist might the nurse administer to a child? A) Cimetidine B) Famotidine C) Nizatidine D) Ranitidine

Ans: B Feedback: Famotidine is the only histamine2 antagonist approved for use in children if a histamine-2 antagonist is needed. Therefore, other options are not correct.

For treatment of a gastric ulcer, what would the recommended dosing schedule of famotidine (Pepcid) be? A) 10 mg b.i.d B) 20 mg b.i.d C) 60 mg at bedtime D) 40 mg q am

Ans: B Feedback: Famotidine should be administered 40 mg every day at bedtime or 20 mg b.i.d for treatment of a gastric ulcer. Options A, C, and D are not correct.

What is the priority nursing diagnosis for an 89-year-old patient with frequent liquid stools? A) Imbalanced nutrition: less than body requirements B) Deficient fluid volume C) Deficient knowledge related to medications D) Pain related to abdominal cramping

Ans: B Feedback: Fluid volume deficits may develop rapidly in older adults with diarrhea. Older adults are more likely to develop adverse effects associated with the use of these drugs, including sedation, confusion, dizziness, electrolyte disturbances, fluid imbalance, and cardiovascular effects. Although nutrition, pain, and knowledge deficit may be concerns for this patient, they are not the priority concern.

The patient will receive ranitidine (Zantac) 150 mg PO at bedtime. Prior to administration, the nurse will inform the patient that common adverse effects related to this medication include what? A) Tremors B) Headache C) Visual disturbances D) Anxiety

Ans: B Feedback: Headache, dizziness, somnolence, and mental confusion may occur with H2 antagonists. Visual disturbances, tremors, and anxiety are not normally associated with ranitidine.

A clinic patient with a history of heart failure requires a laxative for treatment of chronic constipation. What medication would be most appropriate for this patient? A) Senna (Senokot) B) Lactulose (Chronulac) C) Magnesium sulfate (Milk of Magnesia) D) Castor oil (Neoloid)

Ans: B Feedback: Lactulose is often the drug of choice when a patient with cardiovascular problems requires a laxative. It is salt free, an important consideration in patients with heart failure; it acts by exerting a gentle osmotic pull of fluid into the intestinal lumen. Senna, magnesium sulfate, and castor oil are more aggressive laxatives and might not be the best choice for a patient with cardiovascular problems.

The nurse develops a teaching plan for a 77-year-old patient who has been prescribed loperamide PRN. The nurses priority teaching point is what? A) May cause hallucinations or respiratory depression B) Take drug after each loose stool C) Drug remains in the bowel without being absorbed into the bloodstream D) Avoid pregnancy and breast-feeding while taking drug.

Ans: B Feedback: Loperamide is taken repeatedly after each loose stool. Teaching the patient when to take the drug is the priority teaching point. Paregoric, and not loperamide, can cause hallucinations and respiratory depression. The drug is absorbed systemically. It is unlikely a 77-year-old patient will get pregnant or breast-feed so this is not the highest priority.

What is the antidiarrheal of choice the nurse will administer to children older than 2 years of age with diarrhea? A) Bismuth salts (Pepto-Bismol) B) Loperamide (Imodium) C) Paregoric (generic) D) Difenoxin (Motofen)

Ans: B Feedback: Loperamide may be the antidiarrheal of choice in children older than 2 years of age if such a drug is needed. Special precautions need to be taken to monitor for electrolyte and fluid disturbances and supportive measures should be taken as needed. Serious fluid volume deficits may rapidly develop in children with diarrhea. Appropriate fluid replacement should include oral rehydration solutions. Although bismuth salts and paregoric may be given to children, they are not the drugs of choice. Difenoxin is not for use in children under 12 years of age.

A patient who is taking metoclopramide (Reglan) has come to the clinic for a follow-up visit. The nurse will be most concerned about a drug-drug interaction when learning the patient is taking what other medication? A) Albuterol (Proventil) B) Digoxin (Lanoxin) C) Furosemide (Lasix) D) Acetylsalicylic acid (ASA)

Ans: B Feedback: Metoclopramide has been associated with decreased absorption of digoxin from the gastrointestinal (GI) tract. The nurse should monitor patients taking this combination carefully. The other options do not pose a concern about drugdrug interactions.

The nurse is caring for a patient who does not produce adequate mucus in the stomach resulting in what problem? A) Diarrhea B) Ulceration C) Constipation D) Distention

Ans: B Feedback: Mucus functions to protect the lining of the digestive tract from digestive juices and protects the gastrointestinal mucosa from injury. Lack of mucous does not cause diarrhea, constipation, or distention.

What H2 antagonist would the nurse consider the drug of choice for a patient with advanced liver failure? A) Cimetidine B) Nizatidine C) Ranitidine D) Famotidine

Ans: B Feedback: Nizatidine differs from the other three drugs in that it is eliminated by the kidneys, with no first-pass metabolism in the liver. It is the drug of choice for patients with liver disease or dysfunction.

The nurse provides drug teaching to the patient who will begin taking polycarbophil (FiberCon). What is the nurses priority teaching point? A) Do not eat or drink anything for 2 hours after taking the medication. B) Drink lots of water when taking the drug. C) Take at night before bedtime. D) The drug can be taken up to six times per day.

Ans: B Feedback: Patients must take plenty of water with polycarbophil. If only a little water is consumed, the medication may absorb enough fluid in the esophagus to swell the food into a gelatin-like mass that can cause obstruction and other severe problems. The drug should be taken no more than four times a day and should not be taken at night.

The nurse will question an order for bismuth salts (Pepto-Bismol) in a patient with what condition? A) Rheumatoid arthritis B) Allergy to aspirin C) Hypertension D) Viral gastroenteritis

Ans: B Feedback: Pepto-Bismol has aspirin in it and should not be given to a patient with an allergy to aspirin. There is no contraindication for a patient with rheumatoid arthritis, hypertension, or viral gastroenteritis.

A nurse in a rehab facility recognizes the patient with damage to what area of the brain will affect the patients ability to swallow? A) Temporal lobe B) Medulla oblongata C) Cerebellum D) Pons

Ans: B Feedback: Swallowing is a voluntary act that is regulated by a swallowing center in the medulla oblongata of the central nervous system. Therefore, other options are incorrect.

The nurse teaches the patient to best maintain optimal GI function by including what in the daily routine? A) Exercise, adequate sleep, and avoiding caffeine B) Proper diet, fluid intake, and exercise C) Proper diet, avoiding alcohol, and cautious use of laxatives D) Avoiding prescription medications, increased fluid intake, and vigorous exercise

Ans: B Feedback: The best way to maintain gastrointestinal (GI) function is through proper diet including optimizing fiber intake, adequate fluid intake, and exercise, which will stimulate GI activity. Drugs should only be used when normal function cannot be maintained. Options A, C, and D are not correct.

What reflex causes an increase in small intestine activity when the stomach is stimulated by stretching, the presence of food, or cephalic stimulation? A) Duodenalcolic B) Gastroenteric C) Gastrocolic D) Ileogastric

Ans: B Feedback: The gastroenteric reflex is stimulation of the stomach, which causes an increase in activity in the small intestine. The duodenalcolic reflex stimulates colon activity and mass movement. The gastrocolic reflex is stimulation in the stomach, which increases activity in the colon. The ileogastric reflex is the introduction of chyme or stretch to the large intestine and slow stomach activity.

The nurse admits a woman to the clinic for an annual exam. The patient denies any health issues but reports frequent constipation despite increasing fiber and water intake in her diet. What assessment questions will the nurse ask? A) How many calories do you eat per day? B) Do you often wear Spanx or other tight constraining clothing? C) Do you often ignore the need to urinate and allow your bladder to fill? D) Have you been taking narcotics every day?

Ans: B Feedback: The nurse would assess the women to determine if tight constraining clothing such as Spanx are worn because taut stretching of skin and muscles over the abdomen irritates the nerve plexus and causes a slowing or cessation of GI activity to prevent further irritation. Calories per day would not contribute to constipation. It is unlikely the urge to void could be ignored long enough to cause constipation. Asking about the use of narcotics would only be appropriate if there are other manifestations that indicate the likelihood of this problem as constipation can be caused by many other things.

What is the purpose of the peritoneum A) Help propel the GI contents down the tract B) Help keep the GI tract in place and prevent friction with movement C)Decrease the muscle tone to slow the GI tract from moving too fast D) Serve as a supportive layer and helps the tube maintain its shape

Ans: B Feedback: The peritoneum lines the abdominal wall and viscera and helps keep the GI tract in place and prevents a buildup of friction with movement. The muscularis mucosa layer helps propel the GI contents downward. The nerve plexus helps to slow the GI tract when digestion is not a priority or stimulates it for digestion. The adventitia serves as a supportive layer and helps the tube maintain its shape.

The patient who experiences nausea produces less stomach acid. The nurse recognizes this as having what effect? A) Increases the feeling of nausea B) Protects the lining of the upper GI tract C) Stimulates the parasympathetic nervous system D) Directs fluid to mucus production in the small bowel

Ans: B Feedback: There is a large increase in the production of mucus in the upper GI tract, which is accompanied by a decrease in gastric acid production. This action protects the lining of the GI tract from potential damage by the acidic stomach contents when vomiting occurs. It does not increase the feeling of nausea, stimulate the parasympathetic nervous system, nor does it direct fluid to mucus production.

The nurse, caring for a patient with cystic fibrosis, administers pancreatic enzymes on what schedule? A) 1 hour after meals and snacks B) With meals and snacks C) 2 hours before meals and snacks D) In the morning and before bed

Ans: B Feedback: To be sure the pancreatic enzymes are in place when food enters the small bowel the enzymes are administered with every meal and snack. Administering them before or after meals would render them ineffective. 35.

The nurse is caring for a patient who has had impacted stools twice in the past month. What is an appropriate laxative for this patient? A) Milk of Magnesia B) Agoral Plain C) Colace D) Dulcolax

Ans: B Feedback: Mineral oil (Agoral Plain) is not absorbed and forms a slippery coat on the contents of the intestinal tract. When the intestinal bolus is coated with mineral oil, less water is drawn out of the bolus and the bolus is less likely to become hard or impacted. Other options shown do not have this same effect of reducing the risk of another impaction as well as helping to eliminate stool.

The nurse is caring for a patient admitted with a suspected malabsorption disorder as the result of pancreatic damage. What digestive enzymes does the pancreas secrete? (Select all that apply.) A) Pepsin B) Lipase C) Amylase D) Trypsin E) Ptyalin

Ans: B, C, D Feedback: Digestive enzymes secreted by the pancreas include trypsin, which aids in digesting protein; amylase, which aids in digesting starch; and lipase, which aids in digesting fats. Pepsin is secreted by the stomach and ptyalin is secreted in the saliva.

The nurse would assess patients with what condition to determine whether there is a need for a saliva substitute? (Select all that apply.) A) Cystic fibrosis B) Stroke C) Cancer receiving chemotherapy or radiation therapy D) Myocardial infarction E) Chronic dry mouth

Ans: B, C, E Feedback: A saliva substitute helps in conditions that result in dry mouthstroke, radiation therapy, chemotherapy, and other illnesses. Patients with myocardial infarction do not require this drug. Patients with cystic fibrosis receive pancreatic enzymes rather than saliva substitute.

A patient has been prescribed esomeprazole (Nexium). What statement by the patient does the nurse evaluate as indicating that he or she has a good understanding of his newly prescribed drug? A) I should open the capsules and crush the drug into applesauce. B) It is important that I take the drug after each meal. C) I need to swallow the drug whole and not chew the capsules. D) I should always take the drug with an antacid.

Ans: C Feedback: Esomeprazole must be swallowed whole, not cut, crushed, or chewed, which would interfere with its effectiveness. The drug should not be taken with an antacid, which could interfere with absorption. The drug is taken once a day, not with each meal.

The nurse is preparing a patient for discharge with a prescription for sucralfate (Carafate) and teaches the patient to take the medication when? A) With meals B) With an antacid before breakfast C) 1 hour before or 2 hours after meals and at bedtime D) After each meal

Ans: C Feedback: Administer drug on an empty stomach, 1 hour before or 2 hours after meals and at bedtime, to ensure therapeutic effectiveness of the drug. Administer antacids, if ordered, between doses of sucralfate and not within 30 minutes of taking the drug. Options A, B, and D are not correct.

The nurse assesses the postoperative patient who had abdominal surgery and finds a complete absence of bowel sounds caused by what reflex? A) Gastrocolic reflex B) Renointestinal reflex C) Intestinalintestinal reflex D) Somatointestinal reflex

Ans: C Feedback: After the intestinal irritation that occurs with handling the bowel during abdominal surgery, the risk of paralytic ileus is high because of the intestinalintestinal reflex. The gastrocolic reflex increases GI movement when food is in the stomach. The renointestinal reflex slows GI activity in response to renal capsule swelling. The somatointestinal reflex slows GI activity in response to tight stretching of skin across the abdominal wall.

When comparing the histamine-2 antagonists to each other the nurse recognizes that cimetidine (Tagamet) is more likely to cause which adverse effect? A) Dizziness B) Headache C) Gynecomastia D) Somnolence

Ans: C Feedback: Cimetidine was the first drug in this class to be developed. It has been associated with antiandrogenic effects, including gynecomastia and galactorrhea. Dizziness, headache, and somnolence are associated with all histamine-2 antagonists.

The home health nurse is caring for a 72-year-old man in his home. He complains about almost daily diarrhea. The nurse assesses for what common cause of diarrhea in older adults? A) Diabetes B) Hypertensive medications C) Laxative overuse D) Glaucoma

Ans: C Feedback: Diarrhea in older adults may result from laxative overuse so the nurse should assess what over-the-counter (OTC) products the patient is using. Diabetes, hypertension medications, and glaucoma are not common causes of diarrhea in patients.

A patient complains of frequent acid indigestion. The nurse instructs the patient to increase intake of what to reduce stimulation of acid production? A) Protein B) Calcium C) Fiber D) Alcohol

Ans: C Feedback: Fiber does not promote acid production so it would be a safe food for patients to eat to avoid stimulating more acid in the stomach. Acid production in the stomach is stimulated by gastrin which increases in the presence of protein, calcium, caffeine, or alcohol in the stomach.

The nurse is caring for a patient who is having difficulty swallowing tablets. What action will the nurse implement to help the patient swallow their medication? A) Turn the patients head to one side B) Place a warm cloth on the back of the patients neck C) Have the patient suck on a Popsicle D) Give the patient a copious amount of water to drink with the tablet

Ans: C Feedback: Have the patient suck on a Popsicle, which will block external nerve impulses and allow the basic reflex to respond. The patients head should be straight to allow the muscle pairs to work together. A cold cloth can be placed on the back of the neck or the sternum to stimulate the reflex, but a warm cloth is not indicated. Textured food, not liquids, can also help with swallowing a tablet.

A patient with a duodenal ulcer is receiving sucralfate for short-term treatment. What will the nurse advise the patient to avoid? A) Milk of Magnesia B) Tums C) Aluminum salts D) Proton pump inhibitors

Ans: C Feedback: If aluminum salts (AlternaGEL) are taken concurrently with sucralfate, a risk of high aluminum levels and related aluminum toxicity exists. The combination of sucralfate and aluminum salts should be avoided or used with extreme caution. Adverse reactions with sucralfate are not associated with Milk of magnesia, Tums, or proton pump inhibitors.

When would it be appropriate for the nurse to administer castor oil as a laxative? A) To ease the passage of stool in the patient who recently had a baby B) To remove ingested poisons from the lower gastrointestinal (GI) tract C) To evacuate the bowel for diagnostic procedures D) To treat chronic constipation

Ans: C Feedback: Indications include evacuating the bowel for diagnostic procedures and for short-term treatment of constipation. Castor oil is not indicated to remove ingested poisons nor to ease the passage of stool after having a baby. This drug should only be used on a short-term basis and is not for the treatment of chronic constipation because repeated use will cause GI tract exhaustion.

The home health nurse is caring for a patient with encopresis who was started on mineral oil therapy. The nurse teaches the patient and family that a common adverse effect is what? A) Nausea B) Vomiting C) Leakage D) Vitamin C deficiency

Ans: C Feedback: Leakage and staining may be a problem when mineral oil is used and the stool cannot be retained by the external sphincter. Mineral oil does not cause nausea, vomiting, or vitamin C deficiency.

The nurse is providing education for a patient with peptic ulcer disease resulting from chronic nonsteroidal anti-inflammatory drug (NSAID) use who will begin a prescription of misoprostol (Cytotec). The nurse evaluates that the patient understands the actions of this drug when he or she says it does what? A) Reduces the stomachs volume of hydrochloric acid B) Increases the speed of gastric emptying C) Protects the stomachs lining D) Increases lower esophageal sphincter pressure

Ans: C Feedback: Misoprostol is a synthetic prostaglandin that, like physiologic prostaglandin, protects the gastric mucosa. NSAIDs decrease prostaglandin production and predispose the patient to peptic ulceration. Misoprostol does not reduce gastric acidity (option A), improve emptying of the stomach (option B), or increase lower esophageal sphincter pressure (option D).

While bathing the noncommunicative patient, the nurse notices the patient is swallowing repeatedly. What should the nurse do in response? A) Offer the patient a drink B) Put the patient in a supine position C) Ready the emesis basin D) Continue the bath

Ans: C Feedback: Nauseated patients who start swallowing repeatedly or complain about secretions in their throat are in the process of preparing for vomiting. The nurse should reach for the emesis basin. Offering water will increase vomiting. Placing the patient in a supine position risks the danger of aspiration of stomach contents into the lungs. Continuing the bath does not prepare the nurse if the patient begins to vomit.

The nurse develops a discharge teaching plan for a patient who was prescribed pancreatic enzyme replacement and includes what important teaching point? A) Take the enzymes on an empty stomach. B) Crush the capsules and take with food. C) Avoid spilling the powder on the skin because it may be irritating. D) Pancreatin and pancrelipase are interchangeable.

Ans: C Feedback: Patients receiving pancreatic enzymes should be taught to avoid spilling the powder on the skin because it can be very irritating. The enzymes should be taken with food and are often in a powder form. Pancreatin and pancrelipase are not interchangeable.

A patient receiving loperamide (Imodium) should be alerted by the nurse to what possible adverse effect? A) Anxiety B) Bradycardia C) Fatigue D) Urinary retention

Ans: C Feedback: Patients should be aware that they should not drive or operate machinery while taking loperamide (Imodium) because it can cause fatigue. Anxiety, bradycardia, and urinary retention are not commonly associated with loperamide.

The nurse is caring for a patient with cystic fibrosis who has recently not been taking her pancreatic enzymes recently. What is the primary assessment finding the nurse will see as a result of this noncompliance? A) Constipation B) Diarrhea C) Malnutrition D) Abdominal pain

Ans: C Feedback: Patients with cystic fibrosis take pancreatic enzymes to promote digestion and absorption of nutrients so failure to take the drug would result in malnutrition and weight loss. Constipation, diarrhea, and abdominal pain would not be in the primary assessment finding.

What classification of drugs does the nurse administer to treat peptic ulcers by suppressing the secretion of hydrochloric acid into the lumen of the stomach? A) Antipeptic agents B) Histamine-2 antagonists C) Proton pump inhibitors D) Prostaglandins

Ans: C Feedback: Proton pump inhibitors suppress the secretion of hydrochloric acid into the lumen of the stomach. Antipeptic agents coat any injured area in the stomach to prevent further injury. H2 antagonists block the release of hydrochloric acid in response to gastrin. Prostaglandins inhibit secretion of gastrin and increase secretion of the mucous lining of the stomach.

The steps of the swallowing reflex are 1. Soft palate elevates. 2. Respirations cease. 3. Larynx rises and the glottis closes. 4. Pharyngeal constrictor muscles contract. In what order do these actions occur in order to allow a person to swallow? A) 2, 4, 1, 3 B) 4, 3, 2, 1 C) 1, 2, 3, 4 D) 3, 1, 4, 2

Ans: C Feedback: Receptors send impulses to the medulla, which stimulates a series of nerves that cause the following actions, which are the soft palate elevates and seals off the nasal cavity, respirations cease in order to protect the lungs, the larynx rises and the glottis closes to seal off the airway, and the pharyngeal constrictor muscles contract and force the food bolus into the top of the esophagus, where pairs of muscles contract in turn to move the bolus down the esophagus into the stomach.

What priority teaching point does the nurse include when instructing patients about the use of antacids? A) Reduce calorie intake to reduce acid production. B) Take before each meal and before bed. C) Be aware of risk of acid rebound with long-term use. D) Consider liquid diet if diarrhea occurs.

Ans: C Feedback: Repeated use of antacids can result in rebound acid production because more gastrin is produced when pH of acid level decreases. Patients should be taught that long-term use of antacids requires follow-up care. Calorie and fluid intake does not need to be reduced because it is important to maintain nutrition, especially if diarrhea occurs. Antacids are taken at least 1 hour before or 2 hours after any other drug or meal.

What should the nurse tell the patient who will begin taking rifaximin (Xifaxan) for travelers diarrhea? A) Do not start taking the drug until the diarrhea has persisted for at least 24 hours. B) Taking the antibiotic will allow you to enjoy the local water and food without concern. C) Do not take the drug if you have bloody diarrhea. D) Start the drug 3 days before leaving for your trip and continue until you return.

Ans: C Feedback: Rifaximin should not be taken if a patient has bloody diarrhea or diarrhea that persists for more than 48 hours. It is started after the signs and symptoms of the disease develop and is taken for 3 days. The traveler should still avoid exposure to foreign bacteria by not drinking tap water and by avoiding fruits and vegetables washed in tap water. Options A, B, and D are not correct.

What is a unique characteristic of the gastrointestinal (GI) system? A) It contains multiple organs that work together. B) It is lined with epithelial tissue. C) It is the only system open to the external environment. D) The GI tract is sterile and contains no bacteria.

Ans: C Feedback: The GI system is the only system that is open to the external environment. The definition of a system is multiple organs working together to perform a function or functions. Many systems are lined with epithelial tissue. The GI tract is not sterile and does contain bacteria.

The nurse is caring for a child diagnosed with cystic fibrosis whose body does not produce pancreatic enzymes appropriately which results in what? A) Secretion of insulin B) Maintenance of serum glucose levels C) Digestion of carbohydrates D) Regulation of glucose metabolism

Ans: C Feedback: The absence of the pancreatic enzymes results in inadequate digestion and absorption of carbohydrates, fats, and proteins. The pancreas also secretes insulin and glucagon, hormones that regulate glucose metabolism and regulate blood sugar levels but this is not related to enzymes but rather to hormone secretion.

The patient learns there are healthy bacteria in the bowel that aids in digestion and asks the nurse why bacterium doesnt get into the bloodstream. The nurse described what barrier to answer the patients question? A) The mononuclear phagocyte system B) The portal system C) The omenta D) The vomiting reflex

Ans: C Feedback: The greater and lesser omenta hang from the stomach over the lower GI tract and are full of lymph nodes, lymphocytes, monocytes, and other components of the mononuclear phagocyte system. This barrier provides rapid protection for the rest of the body if any of the bacteria or other foreign agents in the GI tract should be absorbed into the body. Other options do not serve as barriers.

What occurs when the emetic zone is stimulated? A) Regurgitation B) Nothing due to overriding by mature centers C) Projectile vomiting D) Gagging

Ans: C Feedback: The more primitive center is called the emetic zone. When stimulated, it initiates projectile vomiting. This type of intense reaction is seen in young children and whenever increased pressure in the brain or brain damage allows the more primitive center to override the more mature chemoreceptor trigger zone (CTZ). Regurgitation is vomiting. In adults the CTZ does not always override the emetic zone (patient with a severe head injury), and gagging is an adult precursor to vomiting.

The nurse is caring for a patient requiring digestive enzyme replacement therapy and establishes what appropriate nursing diagnosis for this patient? A) Acute pain B) Risk for constipation C) Risk for imbalanced nutrition D) Bowel incontinence

Ans: C Feedback: The nurse would be concerned about the patients nutritional status because lack of digestive enzymes results in malnutrition. Replacement digestive enzymes help the patient improve digestion and absorption of essential nutrients. Effectiveness of the therapy is determined by the patients ability to maintain balanced nutrition. The other three options are not applicable to the use of replacement digestive enzymes.

The nurse, caring for a patient about to undergo gastric bypass surgery, explains that the majority of nutrients are absorbed where? A) Sigmoid colon B) Stomach C) Small intestine D) Ascending colon

Ans: C Feedback: The stomach empties into the small intestine where absorption of nutrients occurs. Therefore, very little absorption occurs in the stomach, mostly limited to alcohol and water. The colon allows for water to be absorbed from the feces but does not absorb nutrients.

The clinic nurse is caring for a 78-year-old male patient who is taking a proton pump inhibitor. What condition is this patient at increased risk for developing? A) Sickle cell anemia B) Megaloblastic anemia C) Pernicious anemia D) Iron deficiency anemia

Ans: C Feedback: The use of proton pump inhibitors and H2 blockers in older adults has been associated with decreased absorption of vitamin B12 and the development of pernicious anemia. The use of proton pump inhibitors does not increase the risk of sickle cell, megaloblastic, or iron deficiency anemias.

When would it be appropriate for the nurse to administer a cathartic laxative to the patient? (Select all that apply.) A) Partial small-bowel obstruction B) Appendicitis C) After having a baby D) After a myocardial infarction (MI) E) After anthelmintic therapy

Ans: C, D, E Feedback: Laxative, or cathartic, drugs are indicated for the short-term relief of constipation; to prevent straining when it is clinically undesirable (such as after surgery, myocardial infarction, or obstetric delivery); to evacuate the bowel for diagnostic procedures; to remove ingested poisons from the lower gastrointestinal (GI) tract; and as an adjunct in anthelmintic therapy when it is desirable to flush helminths from the GI tract. They are not indicated when a patient has an appendicitis or a partial small-bowel obstruction.

An adult patient is prescribed cimetidine (Tagamet). A nurse will instruct the patient that an appropriate dosage and frequency of cimetidine is what? A) 20 mg PO b.i.d B) 150 mg PO b.i.d C) 300 mg PO at bedtime D) 800 mg PO at bedtime

Ans: D Feedback: An appropriate dosage and frequency for cimetidine is 800 mg PO at bedtime. Also, 300 mg can be taken q.i.d at meals and at bedtime. Ranitidine is taken 150 mg daily or b.i.d. Famotidine is taken 20 mg PO b.i.d. Nizatidine can be taken 150 to 300 mg PO at bedtime.

What drug combination will the nurse normally administer most often to treat a gastric ulcer? A) Antibiotics and histamine-2 antagonists B) H2 antagonists, antibiotics, and bicarbonate salts C) Bicarbonate salts, antibiotics, and ZES D) Antibiotics and proton pump inhibitors

Ans: D Feedback: Currently, the most commonly used therapy for gastric ulcers is a combination of antibiotics and proton pump inhibitors that suppress or eradicate Helicobacter pylori. H2 receptor antagonists are used to treat duodenal ulcers. Bicarbonate salts are not used. ZES is the abbreviation for ZollingerEllison syndrome and not a drug.

A new mother required an episiotomy during the birth of her baby. Two days after delivery, the patient is in need of a laxative. What will be the most effective drug for the nurse to administer? A) Bisacodyl (Dulcolax) B) Castor oil (Neolid) C) Magnesium sulfate (epsom salts) D) Docusate (Colace)

Ans: D Feedback: Docusate is a stool softener that will make expulsion of stool easier in a traumatized body area following an episiotomy. Care must be taken to choose a mild laxative that will not enter breast milk and not affect the newborn if the mother is nursing. Docusate is the drug of choice from this list because it is mild and will produce a soft stool and decrease the need to strain. The other options would not be appropriate because they do not soften the stool and are harsher laxatives that can enter breast milk.

When caring for a patient diagnosed with a peptic ulcer, the nurse administers omeprazole (Prilosec) along with what antibiotic to eradicate Helicobacter pylori? A) Gentamicin B) Ketoconazole C) Tetracycline D) Amoxicillin

Ans: D Feedback: Gastric acid pump or proton pump inhibitors are recommended for the short-term treatment of active duodenal ulcers, gastroesophageal reflux disease, erosive esophagitis, and benign active gastric ulcer; for the long-term treatment of pathologic hypersecretory conditions; as maintenance therapy for healing of erosive esophagitis and ulcers; and in combination with amoxicillin and clarithromycin for the treatment of H. pylori infection. The other options are not antibiotics used to eradicate H. pylori.

What action does the histamine-2 antagonist administered by the nurse have on the human body that will help to prevent peptic ulcer disease? A) Destroys Helicobacter pylori B) Coats and protects the stomach lining C) Increases the pH of the secreted hydrochloric acid D) Reduces the amount of hydrochloric acid secreted

Ans: D Feedback: Histamine-2 antagonists are administered to reduce the amount of hydrochloric acid secreted in the stomach, which helps to prevent peptic ulcer disease. H2 antagonists do not act as an antibiotic to kill bacteria (i.e., H. pylori) coat and protect the stomach lining like sucralfate (Carafate), or increase the pH of the secreted hydrochloric acid.

The nurse is caring for a young child who fell and hit his or her head and is brought to the emergency room with projectile vomiting. How does the nurse interpret this behavior? A) The child is in intense pain B) The child probably ate too much for dinner C) The child likely has a full bladder D) The child has increased intracranial pressure

Ans: D Feedback: Increased intracranial pressure applies direct pressure to the CTZ resulting in projectile vomiting. This child requires a thorough neurological assessment. Intense pain would be indicated by crying. A full bladder or eating too much for dinner would not cause projectile vomiting in a healthy child.

What antidiarrheal would the nurse administer to reduce the volume of discharge from the patients ileostomy? A) Diphenoxylate (Lomotil) B) Octreotide (Sandostatin) C) Psyllium (Metamucil) D) Loperamide (Imodium)

Ans: D Feedback: Loperamide (Imodium), bismuth subsalicylate (Pepto-Bismol), and opium derivatives (paregoric) are indicated for the reduction of volume of discharge from ileostomies. The other options would not serve this purpose.

For what action would the nurse administer magnesium citrate? A) Block absorption of fats B) Directly stimulate the nerve plexus in the intestinal wall C) Form a slippery coat on the contents of intestine D) Increase motility, increase fluid, and enlarge bulk of fecal matter

Ans: D Feedback: Magnesium citrate is a rapid-acting, aggressive laxative that causes fecal matter to increase in bulk. It increases the motility of the gastrointestinal (GI) tract by increasing the fluid in the intestinal contents, which enlarges bulk, stimulates local stretch receptors, and activates local activity. It does not block absorption of fats, stimulate the nerve plexus, or form a slippery coat.

The nurse administers psyllium hydrophilic mucilloid (Metamucil) expecting it to have what action? A) Slows peristalsis B) Promotes reabsorption of water into bowel C) Has antibacterial properties D) Adds bulk to the stool

Ans: D Feedback: Metamucil is a natural substance that forms a gelatin-like bulk of the intestinal contents. This agent stimulates local activity. It is considered milder and less irritating than many other bulk stimulants. Patients must use caution and take it with plenty of water because Metamucil absorbs large amounts of water and produces stools of gelatin-like consistency. It does not slow peristalsis, promote water reabsorption, or have antibacterial properties.

The nurse teaches the patient who was prescribed a chemical stimulant laxative that this medication may have either a slow, steady effect or may cause severe cramping and rapid evacuation of the contents of the large intestine. What drug is the nurse teaching this patient about? A) Bismuth salts B) Bisacodyl C) Castor oil D) Cascara

Ans: D Feedback: Specifically related to chemical stimulants, cascara, although a reliable agent, may have a either slow, steady action or may cause severe cramping and rapid evacuation of the contents of the large intestine. Bisacodyl and castor oil do not have potential adverse effect. Bismuth salts are an antidiarrheal.

Which of these patients would the nurse expect to be the best candidate for misoprostol (Cytotec)? A) A 12-year-old with obsessive-compulsive disorder B) A 22-year-old pregnant patient C) A 46-year-old trial lawyer with hypertension D) An 83-year-old man with rheumatoid arthritis

Ans: D Feedback: The 83-year-old man with rheumatoid arthritis is most likely to be taking nonsteroidal anti-inflammatory drugs (NSAIDs). Misoprostol is indicated for prevention of NSAID induced ulcers in adults at high risk for development of gastric ulcers. The other three patients would not be candidates for this drug.

Water is absorbed in which portion of the intestine? A) Duodenum B) Jejunum C) Ileum D) Colon

Ans: D Feedback: The colon absorbs water and sodium. The duodenum, ileum, and jejunum are parts of the small intestine where nutrients are absorbed.

What process is used by the large intestine to evacuate waste products from the body? A) Rectal stimulation B) Basic electrical rhythm C) Segmentation D) Mass movement

Ans: D Feedback: The large intestine uses a process of mass movement with an occasional peristaltic wave. When the beginning segment of the large intestine is stimulated, it contracts and sends a massive peristaltic movement throughout the entire large intestine. The end result of the mass movement is usually excretion of waste products. Segmentation occurs in the small intestine. The nerve plexus maintains a basic electrical rhythm (BER), much like the pacemaker rhythm in the heart. Rectal stimulation is not normally required for a bowel movement.

What is a layer of the nerve plexus? A) Mesenteric B) Mucosal C) Sympathetic D) Myenteric

Ans: D Feedback: The nerve plexus has two layers of nerves, one submucosal layer and one myenteric layer. The other options are not part of the nerve plexus.

Nutrients absorbed from the small intestine travel directly to the liver via what system? A) Mesenteric system B) Splenic system C) Arterial system D) Portal system

Ans: D Feedback: The portal system drains the entire lower GI tract, where absorption occurs, and delivers what is absorbed into the venous system directly to the liver. The other options are distractors for this question.

The nurse would question an order for misoprostol if the patient was diagnosed with what condition? A) Diabetes B) Hypertension C) Arthritis D) Pregnancy

Ans: D Feedback: This drug is contraindicated during pregnancy because it is an abortifacient. The other options are not correct.

17. The patient is a 34-year-old man who recently started taking theophylline. The nurse knows that medication teaching has been successful when he agrees to what activity? A) Avoiding caffeine B) Eating foods high in potassium C) Limiting fluid intake to 1,000 mL a day D) Taking the medicine on an empty stomach

Ans: A Feedback: Both theophylline and caffeine are xanthenes. Theophylline increases cardiac output and heart rate. Caffeine also stimulates heart rate. This can have an additive effect. Eating foods high in potassium, limiting fluid intake, or taking the medicine on an empty stomach are not indications that the patient has understood the nurse's teaching.

26. A patient is prescribed salmeterol with dosage on a 4 to 6 hour schedule for treatment of exercise-induced asthma. What is the recommended dosing schedule of asthma experts regarding this drug? A) 30 minutes before exercise to prevent dyspnea during exercise B) Every 15 minutes during exercise to prevent dyspnea C) As needed to treat or prevent dyspnea during exercise D) Every 1 to 2 hours to treat or prevent dyspnea during exercise

Ans: A Feedback: Salmeterol (Serevent) adult and pediatric (12-year-old and older): two puffs every 12 hours; or two puffs 30 to 60 minutes before exercise. Therefore, options B, C, and D are incorrect.

9. A patient with chronic bronchial asthma is prescribed montelukast (Singulair). What will the nurse instruct the patient to avoid taking? A) Aspirin B) Penicillin C) Sertraline (Zoloft) D) Nifedipine (Procardia)

Ans: A Feedback: The nurse would instruct the patient to avoid aspirin, which might cause an increased montelukast level and toxicity. The other options do not cause drug-drug interactions with montelukast.

28. Why are inhaled steroids used to treat asthma and chronic obstructive pulmonary disease (COPD)? A) They act locally to decrease release of inflammatory mediators. B) They act locally to improve mobilization of edema. C) They act locally to increase histamine release. D) They act locally to decrease histamine release.

Ans: A Feedback: When administered into the lungs by inhalation, steroids decrease the effectiveness of the inflammatory cells. This has two effects, which are decreased swelling associated with inflammation and promotion of beta-adrenergic receptor activity, which may promote smooth muscle relaxation and inhibit bronchoconstriction. Options B, C, and D are incorrect.

The nurse is caring for a patient treated with flumazenil (Anexate) for benzodiazepine toxicity. After administering flumazenil what will the nurse carefully assess for? A) Agitation, confusion, and seizures B) Cerebral hemorrhage and dystonia C) Hypertension and renal insufficiency D) Hypotension, dysrhythmias, and cardiac arrest

Ans: A Feedback: Administration of flumazenil blocks the action of benzodiazepines. If the patient has been taking these medications for an extended period of time, the blockage of the drug's effects could precipitate an acute benzodiazepine withdrawal syndrome with symptoms including agitation, confusion, and seizures. Anexate does not cause cerebral hemorrhage and dystonia, hypertension, renal insufficiency, hypotension, dysrhythmias, and cardiac arrest.

The nurse is caring for a patient who is receiving an adrenergic blocking agent. While writing the care plan for this patient what nursing diagnoses would be most appropriate concerning comfort? A) Acute pain related to cardiovascular and systemic effects B) Decreased cardiac output related to cardiovascular effects C) Ineffective airway clearance related to lack of bronchodilating effects D) Deficient knowledge regarding drug therapy

Ans: A Feedback: All four options would be appropriate nursing diagnoses for a patient receiving an adrenergic blocking agent. However, acute pain would be the only nursing diagnosis related to the patient's comfort level.

The nurse administers promethazine (Phenergan) to the patient before sending the patient to the preoperative holding area. What is the rationale for administration of this drug? A) Sedation B) Oral secretions C) Hypotension and bradycardia D) Confusion

Ans: A Feedback: Antihistamines (promethazine, diphenhydramine [Benadryl]) can be very sedating in some people. They are used as preoperative medications and postoperatively to decrease the need for narcotics. Promethazine is not given for hypotension, bradycardia, confusion, or oral secretions.

A patient is admitted to the cardiac unit with a diagnosis of a myocardial infarction (MI). The nurse notes that the patient is having regular premature ventricular contractions (PVCs). Why would the nurse be concerned? A) Blood is not efficiently pumped from the heart with PVCs. B) Healing of the myocardium will be disrupted. C) PVCs usually cause severe pain. D) PVCs make it difficult to make a definitive diagnosis.

Ans: A Feedback: Arrhythmias can alter cardiac output that could affect every cell in the body. Arrhythmias usually do not cause pain, disrupt healing, or interfere with the diagnosis of an MI.

The nurse is caring for a patient with hypertension and recognizes this will have what impact on afterload? A) Increased afterload B) Reduced afterload C) Unchanged afterload D) Initial increase and then decrease in afterload

Ans: A Feedback: Blood pressure is a measure of afterload so the higher the patient's blood pressure the higher the afterload. The higher the resistance in the system, the harder the heart will have to contract to force open the valves and pump the blood along. Options B, C, and D are not correct.

A 75-year-old patient is brought to the emergency department by his family. The family relates that the patient is complaining of confusion, seizures, and abnormal perception of movement. The nurse reviews all of the medication bottles found in the house and suspects the patient overdosed on what medication? A) Benzodiazepine B) Antihypertensive C) Sedative D) Analgesic

Ans: A Feedback: Common manifestations of benzodiazepine toxicity include increased anxiety, psychomotor agitation, insomnia, irritability, headache, tremor, and palpitations. Less common but more serious manifestations include confusion, abnormal perception of movement, depersonalization, psychosis, and seizures. These symptoms are not found in association with options B, C, or D.

The nurse is caring for a science professor with hypertension who asks for a more complete explanation of blood pressure. The nurse begins by explaining that the phase when the ventricle contracts and ejects blood out into the aorta is known as what? A) Systole B) Diastole C) Hypertension D) Ejection fraction

Ans: A Feedback: Contraction of the ventricles is called systole. Each period of systole is followed by a period of diastole when the heart rests. Ejection fraction is a measurement of cardiac output, or the blood ejected with each ventricular contraction. Hypertension is defined as elevated blood pressure and may include either systole or diastole or both.

A 75-year-old male patient was admitted to the unit with angina. He was started on nadolol (Corgard). The patient asks why he is taking this medication because he does not have high blood pressure. What is the nurse's best response? A) Some beta-blockers have been approved as antianginal agents. B) This medication will prevent blood pressure problems later on. C) This drug will prevent you from developing an arrhythmia. D) This medication will reduce benign prostatic hypertrophy (BPH) as well as treat heart failure.

Ans: A Feedback: Decreased heart rate, contractility, and excitability, as well as a membrane-stabilizing effect, lead to a decrease in arrhythmias, a decreased cardiac workload, and decreased oxygen consumption. The juxtaglomerular cells are not stimulated to release renin, which further decreases the blood pressure. These effects are useful in treating hypertension and chronic angina and can help to prevent reinfarction after a myocardial infarction by decreasing cardiac workload and oxygen consumption. Corgard will not prevent blood pressure problems, arrhythmias, or glaucoma in the future. Corgard is not used to treat BPH.

A patient, diagnosed with heart failure, would like the nurse to explain what the diagnosis means. How will the nurse explain heart failure? A) The heart muscle cannot pump effectively causing a backup of blood. B) The hydrostatic pressure pushing fluid out of the capillaries is lower than the oncotic pressure. C) The decrease in venous pressure from the backup of blood increases hydrostatic pressure. D) Increased protein leads to reduced oncotic pressure and inability to pull fluid into the system.

Ans: A Feedback: Heart failure occurs when the heart muscle fails to do its job of effectively pumping blood through the system and blood backs up and the system becomes congested. The rise in venous pressure that results from the backup of blood increases hydrostatic pressure on the venous end of the capillaries. The hydrostatic pressure pushing fluid out of the capillaries becomes greater than the oncotic pressure that is trying to pull the fluid back into the vessel, thus causing fluid to be lost into the body tissues. Protein loss can lead to a fall in oncotic pressure and an inability to pull fluid back into the vascular system.

The nurse is caring for a patient in a state of hypnosis, which means the patient is in what state? A) A state of extreme sedation in which the person no longer senses or reacts to incoming stimuli. B) A state of tranquility in which the person can be made to do whatever is suggested by others. C) A feeling of tension, nervousness, apprehension, or fear with high levels of awareness. D) A state in which the brain is no longer sending out signals to the body.

Ans: A Feedback: Hypnosis is an extreme state of sedation in which the person no longer senses or reacts to incoming stimuli. A state of tranquility is produced through minor tranquilizers by decreasing anxiety. Anxiety is a feeling of tension, nervousness, apprehension, or fear. Sedation is the loss of awareness and reaction to environmental stimuli, which may lead to drowsiness. The state of suggestibility often seen in television programs is not an appropriate definition of hypnosis. If the brain stopped sending signals, the patient would stop breathing and death would follow.

The nurse is discharging a 35-year-old patient with diabetes who has been prescribed an adrenergic blocking agent. What is the priority teaching point for the nurse to discuss with this patient? A) Monitor blood glucose levels closely and report any instability B) Document signs and symptoms of hyperglycemia and hypoglycemia C) Reduce carbohydrate intake more than usual while taking the new drug D) Increase insulin dosage to compensate for the drug's effect in increasing blood sugar

Ans: A Feedback: It is important for the patient to be instructed to monitor blood sugar levels more frequently because adrenergic blocking agents mask the normal hypo- and hyperglycemic manifestations that normally alert patients such as sweating, feeling tense, increased heart rate, and rapid breathing. There is no need to change the diet or the diabetic medications. There may be no signs and symptoms to record because they are blocked by the adrenergic blocker.

Before administering a nonselective adrenergic blocker, what should the nurse assess? A) Pulse and blood pressure B) Bowel sounds and appetite C) Serum albumin level D) Serum sodium and potassium levels

Ans: A Feedback: Monitor vital signs and assess cardiovascular status including pulse, blood pressure, and cardiac output to evaluate for possible cardiac effects. Although assessment of bowel sounds, appetite, serum albumin level, or serum sodium and potassium levels may be important to patient care, they are not related to administration of a nonselective adrenergic blocking agent.

The nurse is caring for a newborn who was delivered from a woman who took benzodiazepines for anxiety during the last 2 months of her pregnancy after her husband was killed in war. What will the nurse assess for in this newborn? A) Newborn withdrawal syndrome B) Hepatic dysfunction C) Failure to thrive D) Learning deficiencies

Ans: A Feedback: Neonatal withdrawal syndrome may result in a baby born to a mother who was taking benzodiazepines in the final weeks of pregnancy. The neonate may be given very small doses of benzodiazepines that are withdrawn gradually to prevent symptoms. Hepatic dysfunction in the neonate is not associated with use of benzodiazepines. Failure to thrive and learning deficiencies would be long-term problems and are not assessed during the neonatal period.

Ans: A Feedback: Contraction of the ventricles is called systole. Each period of systole is followed by a period of diastole when the heart rests. Ejection fraction is a measurement of cardiac output, or the blood ejected with each ventricular contraction. Hypertension is defined as elevated blood pressure and may include either systole or diastole or both.

Ans: A Feedback: Phase 0 occurs when the cell reaches a point of stimulation. The sodium gates open along the cell membrane, and sodium rushes into the cell, resulting in a positive flow of electrons into the cell—an electrical potential. This is called depolarization. The membrane no longer has a positive side or pole and a negative side; instead it is depolarized, or, in other words, electrically the same on both sides. During depolarization, the cells cannot be forced to contract. Repolarization is when the sodium returns to the outside of the cell and potassium returns to the inside of the cell and the muscle is ready to contract again. Action potential is the ability of the heart to respond to an electrical stimulus. Polarity is the electrical charge.

The patient's blood pressure is low due to shock and vasodilation of the blood vessels. The nurse recognizes this will have what impact on preload? A) Preload will increase. B) Preload will decrease. C) Preload will depend on afterload. D) Preload will vary with activity of the patient.

Ans: A Feedback: Preload is the amount of blood brought back to the heart to be pumped through the body. Vasodilation and shock will reduce preload because the pressure in the system is reduced, pushing less fluid through the vessels.

A 23-year-old female patient presents at the clinic with a migraine headache. What beta-adrenergic blocking agent might the physician prescribe for the prophylactic prevention of future migraine headaches? A) Propranolol (Inderal) B) Nadolol (Corgard) C) Timolol (Blocadren) D) Sotalol (Betapace)

Ans: A Feedback: Propranolol is indicated for the treatment of hypertension, angina pectoris, idiopathic hypertrophic subaortic stenosis, supraventricular tachycardia, tremor; prevention of reinfarction after myocardial infarction; adjunctive therapy in pheochromocytoma; prophylaxis of migraine headache; and management of situational anxiety. The other options do not treat or prevent migraine headaches.

The patient has an excessive production of antidiuretic hormone (ADH). The nurse expects what change in the patient's blood pressure? A) Increase in blood volume will cause increase in blood pressure. B) Increase in water and sodium excretion will cause a decrease in blood volume. C) Increase in ADH production will cause an increase in renin production and hypertension. D) Increase in ADH production will decrease angiotensin production and hypotension.

Ans: A Feedback: Release of ADH causes the kidney to retain water and increase blood volume. Increasing blood volume increases blood pressure.

How does the nurse describe the cardiac action potential to a new coworker? A) The cycle of depolarization and repolarization B) The time it takes from the firing of the sinuatrial (SA) node to the contraction of the ventricles C) The time between the contraction of the atria and the contraction of the ventricles D) The cycle of the firing of the atrioventricular (AV) node and the contraction of the myocardium

Ans: A Feedback: The action potential of the cardiac muscle cell consists of five phases: Phase 0 occurs when the cell reaches a point of stimulation. This is called depolarization. Phase 2, or the plateau stage, is a process called repolarization. Phase 4 is when spontaneous depolarization begins again. The action potential involves electrolytes and polarization and does not involve timing of the cardiac cycle.

The nurse explains how the myocardium receives oxygen to the new graduate nurse beginning work in the constant care unit; she tells the new nurse the coronary arteries receive blood when? A) During diastole B) During systole C) When the heart is refractory D) When the aortic valve is open

Ans: A Feedback: The coronary arteries receive blood during diastole, when the muscle is at rest and relaxed so that blood can flow freely into the muscle. When the ventricle contracts, it forces the aortic valve open, which in turn causes the leaflets of the valve to cover the openings of the coronary arteries.

The nurse evaluates an improvement in the patient's heart failure (HF) status based on what assessment finding? A) Using fewer pillows to sleep B) Increased skin turgor C) Heart rate regular D) Improved mental status

Ans: A Feedback: The degree of HF is often calculated by the number of pillows required to get relief (e.g., one-pillow, two-pillow, or three-pillow orthopnea). Fluid overload is associated with HF so skin turgor is not an indicator of improvement. Regular heart rate and normal cognition can be found with acute flare-ups of HF so these findings would not indicate improvement.

The nurse explains that the reason the left ventricle is so much larger than the right ventricle is what? A) The left ventricle needs to pump blood through the entire body. B) The left ventricle needs to pump blood through both lungs. C) The right ventricle pumps blood through the entire body. D) The right ventricle pulls blood back into the heart from the lungs.

Ans: A Feedback: The left ventricle is much larger because it has to pump strongly enough to circulate blood through the entire body. The right ventricle pumps blood only to the lungs, which are nearby.

The nurse is caring for an older adult in the long-term care facility who has begun to display signs of anxiety and insomnia. What is the priority nursing action? A) Assess the patient for physical problems. B) Call the provider and request an antianxiety drug order. C) Increase the patient's social time, encouraging interaction with others. D) Suggest the family visit more often to reduce the resident's stress level.

Ans: A Feedback: The patient should be screened for physical problems, neurological deterioration, or depression, which could contribute to the insomnia or anxiety. Only after physical problems are ruled out would the nurse consider nondrug measures such as increased socialization with other residents or family members. If nothing else is effective, pharmacological intervention may be necessary.

What are the therapeutic and adverse effects associated with the adrenergic blocking agents related to? A) Receptor-site specificity B) Sympathetic nervous system manifestations C) Norepinephrine release D) Function of the nerve terminal

Ans: A Feedback: The therapeutic and adverse effects associated with these drugs are related to their adrenergic-receptor- site specificity; that is, the ability to react with specific adrenergic receptor sites without activating them, thus preventing the typical manifestations of sympathetic nervous system (SNS) activation. By occupying the adrenergic receptor site, they prevent norepinephrine released from the nerve terminal or from the adrenal medulla from activating the receptor, thus blocking the SNS effects.

The nurse is caring for a patient whose damaged ventricle is reducing left ventricular filling and causing a backup of blood into the left atrium. What valve is damaged in this patient? A) Mitral B) Pulmonic C) Aortic D) Tricuspid

Ans: A Feedback: The valve between the atrium and ventricle on the left side of the heart, called the mitral or bicuspid valve, is composed of two leaflets or cusps that allow the left ventricle to fill with blood and then close to prevent backflow of blood into the left atrium. If this valve is damaged, contraction of the ventricle will push blood back into the left atrium and result in inadequate cardiac output. The pulmonic valve is located between the right ventricle and pulmonary artery. The aortic valve allows blood to flow from the left ventricle into the aorta. The tricuspid valve is the valve between the right atria and ventricle.

The patient takes labetalol and is scheduled for surgery. The anesthesiologist plans to use halothane as one of the anesthetic agents. The nurse consults with the anesthesiologist to ensure awareness the patient's medication history knowing that the combination of labetalol and halothane will have what effect? A) Excessive hypotension B) Hypoglycemia C) Conduction system disturbances D) Vomiting

Ans: A Feedback: There is increased risk of excessive hypotension if any of these drugs is combined with volatile liquid general anesthetics such as enflurane, halothane, or isoflurane. The effectiveness of diabetic agents is increased, leading to hypoglycemia when such agents are used with these drugs. Carvedilol has been associated with potentially dangerous conduction system disturbances when combined with verapamil or diltiazem. Vomiting is not associated with this combination of drugs.

The specificity of the adrenergic blocking agents allows the clinician to select a drug to do what? A) Have the desired effect B) Multiply undesired effects C) Increase specificity with higher serum blood levels D) Improving concentration in the body

Ans: A Feedback: This specificity allows the clinician to select a drug that will have the desired therapeutic effects without the undesired effects that occur when the entire sympathetic nervous system is blocked. In general, however, the specificity of adrenergic blocking agents depends on the concentration of drug in the body. Most specificity is lost with higher serum drug levels.

The patient taking digoxin (Lanoxin) has developed an infection. What antibiotic can the nurse safely administer to this patient? A) Zithromax B) Erythromycin C) Tetracycline D) Cyclosporine

Ans: A Feedback: Zithromax may be given without impacting the effects of digoxin. There is a risk of increased therapeutic effects and toxic effects of digoxin if it is taken with verapamil, amiodarone, quinidine, quinine, erythromycin, tetracycline, or cyclosporine. If digoxin is combined with any of these drugs, it may be necessary to decrease the digoxin dose to prevent toxicity. If one of these drugs has been part of a medical regimen with digoxin and is discontinued, the digoxin dose may need to be increased.

Nonselective adrenergic blocking agents have a variety of therapeutic uses. Which agent is used for the treatment of heart failure? A) Carvedilol (Coreg) B) Sotalol (Betapace) C) Propranolol (Inderal) D) Tamsulosin (Flomax)

Ans: A Feedback: available orally and is used to treat hypertension as well as congestive heart failure and left ventricular dysfunction after a myocardial infarction. Sotalol is a nonselective beta-adrenergic blocking agent used to treat potentially life-threatening ventricular arrhythmias and to maintain normal sinus rhythm in patients with atrial fibrillation or flutter. Propranolol is a nonselective beta-adrenergic blocking agent used for treatment of hypertension, angina, idiopathic hypertrophic subaortic stenosis (IHSS)induced palpitations, angina and syncope, some cardiac arrhythmias induced by catecholamines or digoxin, pheochromocytoma; prevention of reinfarction after myocardial infarction; prophylaxis for migraine headache (which may be caused by vasodilation and is relieved by vasoconstriction, although the exact action is not clearly understood); prevention of stage fright (which is a sympathetic stress reaction to a particular situation); and treatment of essential tremors. Tamsulosin is used to treat benign prostatic hyperplasia and is analpha1-selective adrenergic blocking agent.

The nurse is working on the telemetry unit and has noted that many postmyocardial- infarction (MI) patients experience depression. What medication would the nurse question if ordered for one of these patients? A) Amitriptyline B) Escitalopram C) Fluoxetine D) Fluvoxamine

Ans: A Feedback: Amitriptyline is a tricyclic antidepressant (TCA). One contraindication to the use of TCAs includes recent myocardial infarction because of the potential occurrence of reinfarction or extension of the infarct with the cardiac effects of the drug. Selective serotonin reuptake inhibitors would be the safest antidepressant to give so there would be no need to question an order for escitalopram, fluoxetine, or fluvoxamine.

The nurse is caring for a young female patient who is 5 weeks pregnant. What statement made by the nurse about the use of antidepressants during pregnancy is most accurate? A) Antidepressants are used very cautiously during pregnancy and only when benefit outweighs risk. B) Antidepressants are contraindicated and must be discontinued if pregnancy occurs. C) Antidepressants must be chosen carefully because only a few are safe during pregnancy. D) Most antidepressants are safe during pregnancy but those that are contraindicated should be avoided.

Ans: A Feedback: Antidepressants should be used very cautiously during pregnancy and lactation because of the potential for adverse effects on the fetus and possible neurological effects on the baby. Use should be reserved for situations in which the benefits to the mother far outweigh the potential risks to the neonate.

A patient explains to a nurse that he had been taking amitriptyline (Elavil) for depression and that his physician changed his medication to clomipramine (Anafranil). Page 1 The patient is confused and does not understand why his medication was changed. The nurse's best response to the patient would be what? A) These drugs are similar but some patients respond better to one drug than another. B) Did you take the amitriptyline like you should have? C) Maybe the old medicine wasn't working anymore. D) Clomipramine is newer and will be much better for you.

Ans: A Feedback: Because all tricyclic antidepressants (TCAs) are similarly effective, the choice of which TCA depends on individual response to the drug and tolerance of adverse effects. A patient who does not respond to one TCA may respond to another drug from this class. In addition, the nurse might inform the physician of the patient's question so the physician can explain his or her rationale for changing medications. By asking the patient if he took the medication as prescribed, the nurse is insinuating that he may not have and could damage the trusting nursepatient relationship. The nurse has no basis for commenting that the medication might not be working or that another drug would work better

Obsessive-compulsive disorder (OCD) is a disorder that remains under investigation as to its actual neurophysiology. What tricyclic antidepressant is now approved by the Food and Drug Administration to treat OCD? A) Clomipramine B) Imipramine C) Nortriptyline D) Amitriptyline

Ans: A Feedback: Clomipramine is now also approved for use in the treatment of OCD. Imipramine, nortriptyline, and amitriptyline are not approved for use in treating OCD.

A patient is admitted to the unit with obsessive-compulsive disorder (OCD). What drug might the nurse administer that has been found to be effective for treating OCD? A) Fluvoxamine B) Phenelzine C) Desipramine D) Amitriptyline

Ans: A Feedback: Fluvoxamine is indicated for the treatment of OCD and is classified as a selective serotonin reuptake inhibitor (SSRI). SSRIs are indicated for the treatment of depression, OCD, panic attacks, bulimia, premenstrual dysphoria disorder, posttraumatic stress disorder, social phobias, and social anxiety disorders. Phenelzine is indicated for depression not responsive to other agents. Desipramine and amitriptyline are tricyclic antidepressants indicated for treatment of depression especially if accompanied by anxiety or sleep disturbances.

A patient diagnosed with type 1 diabetes mellitus is receiving insulin. The physician has prescribed a monoamine oxidase inhibitor (MAOI) to treat this patient's depression. What interaction will the nurse assess for with this drug combination? A) Increased risk of hypoglycemia B) Increased risk of hyperglycemia C) Increase in appetite D) Increased total cholesterol

Ans: A Feedback: MAOIs can cause an additive hypoglycemic effect if taken with insulin or oral diabetic agents. This patient would have to be monitored closely and appropriate dosage adjustments made; he should be taught the importance of more frequent blood sugar monitoring. The drug combination in this question would not cause an increase in appetite or increased total cholesterol.

The patient presents to the emergency department with a headache in the back of the head, palpitations, neck stiffness, nausea, vomiting, sweating, dilated pupils, tachycardia, and chest pain. Blood pressure measures 180/124 and heart rate is 168 beats per minute. The spouse says the only medication he takes is something for depression but she does not know the name of the drug and the patient is also unable to supply the name. What classification of antidepressant does the nurse suspect this patient is taking? A) Monoamine oxidase inhibitors (MAOIs) B) Selective serotonin reuptake inhibitors (SSRIs) C) Tricyclic antidepressants (TCAs) D) Antianxiety antidepressants

Ans: A Feedback: MAOIs have several serious adverse effects that can be fatal. This patient's symptoms indicate fatal hypertensive crisis characterized by occipital headache, palpitations, neck stiffness, nausea, vomiting, sweating, dilated pupils, photophobia, tachycardia, and chest pain. It may progress to intracranial bleeding and fatal stroke. SSRIs and TCAs Page 4 are not associated with these particular symptoms. Antianxiety antidepressants are not a classification of antidepressants

A 75-year-old male patient is brought to the emergency department by his family because he is talking to people who aren't there. During the initial admission assessment, his daughter mentions that her mother died 4 months ago and Dad just hasn't been the same. The doctor has even put him on antidepressant medication. I go by the house every day to make sure he takes his medication. What would the nurse suspect is happening to this patient? A) The patient is having hallucinations as an adverse effect of his antidepressant medication. B) The mental status change is due to the patient's age. C) The patient probably has a urinary tract infection. D) The patient is having delusions because of his depression over the loss of his wife.

Ans: A Feedback: Older patients may be more susceptible to the adverse effects of antidepressants from unanticipated central nervous system effects to increased sedation, dizziness, and even hallucinations. Although an infection cannot be ruled out without further testing, the history would lead the nurse to the antidepressants as the most likely cause.

The nurse is caring for an 8-year-old clinic patient who takes imipramine. The nurse assesses the patient who does not have a history for or signs of depression, so the nurse suspects this drug was prescribed as treatment for what? A) Enuresis B) Obsessive compulsive disorder (OCD) C) Peripheral neuropathy D) Panic disorder

Ans: A Feedback: One of the indications for use of the drug imipramine is enuresis in children older than 6 years. Imipramine is not indicated for the treatment of OCD, peripheral neuropathy, or panic disorder.

A patient has been taking Prozac (fluoxetine) for the past 3 years for depression. She is seeing her gynecologist for premenopausal symptoms and during the interview with the nurse she says that she would like to try Sarafem because her friend is taking it and she says it works great. The nurse's best response is what? A) Sarafem and Prozac are different brand names for the same generic medication. B) Before changing drugs it is important to consider how well you responded to Prozac. C) You cannot take both drugs at the same time so it will be important to decide which is best. D) When taking both of these drugs, it is best to take one in the morning and one at night.

Ans: A Feedback: Prozac and Sarafem are different brand names for fluoxetine, so there is no benefit in changing the patient's medication regimen and, if taken together, would result in a drug overdose. The other three responses are incorrect or inappropriate because they do not recognize that both drugs are the same.

What drugs would the nurse consider contraindicated for the patient taking a monoamine oxidase inhibitor (MAOI)? A) Selective serotonin reuptake inhibitor (SSRI) B) Insulin C) Acetaminophen D) Docusate (Colace)

Ans: A Feedback: SSRIs are contraindicated because of a life-threatening serotonin syndrome that could occur. If a patient requires insulin the benefit outweighs the risk but careful monitoring of glucose levels is needed because effects of insulin may be additive with an MAOI. There is no known contraindication for acetaminophen or docusate.

The nurse assesses the patient who had an abrupt withdrawal of benzodiazepines for withdrawal syndrome and would recognize what symptoms as part of the syndrome? (Select all that apply.) A) Headache B) Nightmares C) Malaise D) Bradycardia E) Hypotension

Ans: A, B, C Feedback: Abrupt cessation of benzodiazepines may lead to a withdrawal syndrome characterized by nausea, headache, vertigo, malaise, and nightmares. Withdrawal symptoms may be caused by the abrupt separation of benzodiazepine molecules from their receptor sites and the resulting acute decrease in gamma-aminobutyric acid (GABA) neurotransmission. Because GABA is an inhibitory neurotransmitter, less GABA may produce a less inhibited central nervous system (CNS) and therefore symptoms of hyperarousal or CNS stimulation. The nurse would not categorize hypotension or bradycardia as indicating benzodiazepine withdrawal.

29. The labor and delivery nurse assists with the delivery of a newborn to a woman taking an adrenergic blocker for a congenital heart defect. What organ systems may be affected in the newborn by these drugs? (Select all that apply.) A) Cardiovascular B) Respiratory C) Central nervous system (CNS) D) Gastrointestinal (GI) E) Genitourinary (GU)

Ans: A, B, C Feedback: Adrenergic blockers can affect labor, and babies born to mothers taking these drugs may exhibit adverse cardiovascular, respiratory, and CNS effects. Problems with the GI and GU systems have not been reported.

The nurse is caring for a patient who received a new diagnosis of cancer. The patient exhibits signs of a sympathetic stress reaction. What signs and symptoms will the nurse assess in this patient consistent with an acute reaction to stress? (Select all that apply.) A) Profuse sweating B) Fast heart rate C) Rapid breathing D) Hypotension E) Inability to interact with others

Ans: A, B, C Feedback: Anxiety is often accompanied by signs and symptoms of the sympathetic stress reaction that may include sweating, fast heart rate, rapid breathing, and elevated blood pressure. Chronically anxious people may be afraid to interact with other people but this is not usually seen in an acute stress reaction.

The nurse suspects the patient may have toxic levels of digoxin in the bloodstream when what is assessed? (Select all that apply.) A) Irregular heart rhythms B) Nausea C) Anorexia D) Headache E) Peripheral edema

Ans: A, B, C Feedback: Digoxin toxicity is a serious syndrome that can occur when digoxin levels are too high. The patient may present with anorexia, nausea, vomiting, malaise, depression, irregular heart rhythms including heart block, atrial arrhythmias, and ventricular tachycardia. Peripheral edema is indicative of heart failure, not digoxin toxicity. Headache is not usually associated with digoxin toxicity.

The nurse is preparing to administer a digitalizing dosage of digoxin to a geriatric patient. What factors will the nurse assess for first to avoid digoxin toxicity? (Select all that apply.) A) Renal function B) Low body mass C) Hydration D) Assessment of pulse E) Cognitive function

Ans: A, B, C Feedback: Factors that may contribute to elevated digoxin levels include impaired renal function, low body mass, and dehydration. Assessment of pulse and cognitive function are always important when caring for a geriatric patient, but they will not contribute to elevated digoxin levels.

The home care nurse is caring for a patient newly prescribed a nonselective beta-blocking agent. What would the nurse include in the teaching plan related to this drug? (Select all that apply.) A) Take with meals. B) Change position slowly. C) Avoid driving or operating hazardous machinery. D) Warn of possible increase in libido. E) Increase activity levels as much as possible.

Ans: A, B, C Feedback: Patients should be taught to change position slowly, avoid driving or operating hazardous machinery, and to pace activities as a result of potential dizziness from orthostatic hypotension in order to avoid injury. Patients should take medicine with meals when possible. Drug is more likely to decrease libido than increase it. Activity levels should be paced and care should be taken not to overdo.

The nurse works in a mental health clinic. When a new patient arrives reporting feelings of depression, what conditions are screened for before prescribing antidepressants? (Select all that apply.) A) Thyroid disease B) Hormonal imbalance C) Cardiovascular disorders D) Parkinson's disease E) Diabetes mellitus

Ans: A, B, C Feedback: Adults using these drugs should have physical causes for their depression ruled out before therapy is begun. Thyroid disease, hormonal imbalance, and cardiovascular disorders can all lead to the signs and symptoms of depression. There is no indication that Parkinson's disease or diabetes is manifested by depression.

The nurse interviews the family of a patient hospitalized with severe depression who is prescribed a tricyclic antidepressant. What assessment data are important in planning this patient's plan of care? (Select all that apply.) A) Recent suicide attempts B) Gastrointestinal (GI) obstruction C) Affect D) Physical pain E) Personal responsibilities

Ans: A, B, C Feedback: When caring for a patient with a diagnosis of depression it is always important for the nurse to assess for recent suicide attempts, suicidal ideation, and any suicidal plans. After starting the medication, as the patient begins to feel better, risk of suicide increases, so ongoing assessment is essential to the patient's safety. Other assessments include allergies, liver and kidney function, glaucoma, benign prostatic hypertrophy, cardiac dysfunction, GI obstruction, surgery, or recent myocardial infarction, all of which could be exacerbated by the effects of the drug. Assess history of psychiatric problems, or myelography within the past 24 hours or in the next 48 hours, or is taking a monoamine oxidase inhibitor to avoid potentially serious adverse reactions. Physical pain and personal responsibilities may be assessed but are not priority assessments unless indicated by other diagnoses.

The nurse is teaching a class for nurses working in prenatal clinics about the danger associated with use of benzodiazepines during pregnancy and explains that what fetal anomalies result from maternal use of benzodiazepines during the first trimester of pregnancy? (Select all that apply.) A) Cleft lip or palate B) Inguinal hernia C) Cardiac defects D) Microencephaly E) Gastroschises

Ans: A, B, C, D Feedback: Benzodiazepines are contraindicated in pregnancy because a predictable syndrome of cleft lip or palate, inguinal hernia, cardiac defects, microcephaly, or pyloric stenosis occurs when they are taken in the first trimester. Gastroschises, when the abdominal organs are found outside the abdominal cavity, is not associated with use of benzodiazepine use in the first trimester.

What reasons can the nurse give for why barbiturates are no longer considered the mainstay for treatment of anxiety? (Select all that apply.) A) Adverse effects are more severe. B) There is an increased risk of physical tolerance. C) There is an increased risk of psychological dependence. D) The most common adverse effects are related to cardiac arrhythmias. E) Hypersensitivity reactions can sometimes be fatal.

Ans: A, B, C, E Feedback: The adverse effects caused by barbiturates are more severe than those associated with other, newer sedatives/hypnotics. For this reason, barbiturates are no longer considered the mainstay for the treatment of anxiety. In addition, the development of physical tolerance and psychological dependence is more likely with the barbiturates than with other anxiolytics. The most common adverse effects are related to central nervous system (CNS) depression. Hypersensitivity reactions to barbiturates are sometimes fatal.

34. A patient with asthma is going to begin taking an inhaled steroid. The nurse teaching the patient that what adverse effects may occur when using this drug? (Select all that apply.) A) Headache B) Rebound congestion C) Sepsis D) Epistaxis E) Depression

Ans: A, B, D Feedback: Adverse effects associated with the use of inhaled steroids include irritability, not depression, headache, rebound congestion, local infection, not sepsis and epistaxis.

33. The nurse has just admitted a patient with asthma and the emergency department doctor has ordered the patient to begin taking zafirlukast (Accolate). The nurse would hold the medication and contact the physician if the patient reported taking which medication at home? (Select all that apply.) A) Propranolol B) Warfarin C) Acetaminophen D) Ampicillin E) Terfenadine

Ans: A, B, E Feedback: Use zafirlukast with caution if propranolol, theophylline, terfenadine, or warfarin is taken at the same time because increased toxicity can occur. Toxicity may also occur if these drugs are combined with calcium channel blockers, cyclosporine, or aspirin. No reported change of toxicity occurs when the patient is currently taking acetaminophen or ampicillin.

What medication, if ordered for an 8-year-old patient, should the nurse question? (Select all that apply.) A) Amiodarone (Cordarone) 400 mg orally per 24 hours B) Labetalol (Normodyne) 100 mg orally b.i.d. C) Phentolamine (Regitine) 1 mg intramuscularly 1 to 2 hours before surgery D) Prazosin (Minipress) 3 mg orally t.i.d. E) Carvedilol (Coreg) 6.25 mg orally b.i.d.

Ans: A, B, E Feedback: Amiodarone, labetalol, and carvedilol are not indicated for pediatric use and do not have established pediatric dosages. Phentolamine and prazosin have established pediatric dosages and would not need to be questioned.

32. The nursing instructor is talking to a group of nursing students about the treatment regimen for children with asthma. The students indicate they understand the information when they identify which class of drugs that comprise this regimen? (Select all that apply.) A) Long-acting inhaled steroids B) Xanthines C) Leukotriene-receptor antagonists D) Topical steroid nasal decongestants E) Beta-agonists

Ans: A, C, E Feedback: Antiasthmatics are frequently used in children. The leukotriene-receptor antagonists have been found to be especially effective for long-term prophylaxis in children. Acute episodes are best treated with a beta-agonist and then a long-acting inhaled steroid or a mast cell stabilizer. Xanthines (e.g., theophylline) have been used in children, but because of their many adverse effects and the better control afforded by newer agents, its use is reserved for patients who do not respond to other therapies. Topical steroid nasal decongestants may be used for symptom relief for nasal congestion but are not a regular part of asthma therapy in children.

It has been postulated that depression may be a syndrome that reflects either activity or lack of activity in what areas of the brain? (Select all that apply.) A) Limbic system B) Corpus callosum C) Reticular activating system (RAS) D) Substantia nigra E) Basal ganglia

Ans: A, C, E Feedback: Depression also may occur as a result of other, yet unknown causes. This condition may be a syndrome that reflects either activity or lack of activity in a number of sites in the brain, including the arousal center (i.e., RAS), the limbic system, and basal ganglia. It is not theorized that depression is associated with the corpus callosum or the substantia nigra.

A patient is prescribed sertraline (Zoloft). What adverse effects should the nurse warn of when developing a medication teaching plan? (Select all that apply.) A) Agitation B) Agglutination C) Insomnia D) Intermittent tachycardia E) Dry mouth

Ans: A, C, E Feedback: The adverse effects associated with selective serotonin reuptake inhibitors, which are related to the effects of increased 5-hydroxytriptamine levels, include central nervous system effects (e.g., headache, drowsiness, dizziness, insomnia, anxiety, tremor, agitation, seizures). Gastrointestinal effects (e.g., nausea, vomiting, diarrhea, dry mouth, anorexia, constipation, changes in taste) often occur, as do genitourinary effects (e.g., painful menstruation, cystitis, sexual dysfunction, urgency, impotence). Adverse effects of sertraline do not include agglutination of blood cells or intermittent tachycardia

What agents are used primarily to treat cardiac-related conditions? (Select all that apply.) A) Nonselective adrenergic blocking agents B) Nonselective alpha-adrenergic blocking agents C) Alpha1 -selective adrenergic blocking agents D) Nonselective beta-adrenergic blocking agents E) Beta1 -selective adrenergic blocking agents

Ans: A, D, E Feedback: Drugs that block both alpha- and beta-adrenergic receptors are primarily used to treat cardiac-related conditions. Phentolamine, a nonselective alpha-adrenergic blocking agent, is used to treat extravasation of IV norepinephrine or dopamine and hypertension related to a pheochromocytoma. Alpha1 -selective adrenergic blocking agents are used for treatment of hypertension and benign prostatic hyperplasia. The beta-adrenergic blocking agents are used to treat cardiovascular problems (hypertension, angina, migraine headaches) and to prevent reinfarction after myocardial infarction. Beta1 -selective adrenergic blocking agents are used for treating hypertension, angina, and some cardiac arrhythmias.

18. The nurse caring for a 38-year-old patient started on albuterol (Proventil) should advise the patient that he or she may experience what adverse effect? A) Polydipsia B) Tachycardia C) Hypotension D) Diarrhea

Ans: B Feedback: Adrenergic agents stimulate beta1-adrenergic receptors in the heart as well as beta2-adrenergic receptors in the lungs. Adrenergic agents do not cause polydipsia, hypotension, or diarrhea.

8. A 76-year-old man with asthma is being treated with an anticholinergic. What will the nurse be careful to assess for? A) Cardiac arrhythmias B) Prostatic hypertrophy C) Thyroid conditions D) Parkinsonism

Ans: B Feedback: Anticholinergics can produce urinary hesitancy and urinary retention, conditions that would aggravate the signs and symptoms of prostatic hypertrophy. Older patients given anti-cholinergics should be encouraged to empty the bladder before taking the drug. These drugs are used to treat parkinsonism. Thyroid conditions and cardiac arrhythmias are not cautions or contraindications to the use of these drugs.

29. The nurse is caring for a patient with chronic obstructive pulmonary disease. The plan of care will focus on what patient problem? A) Pain B) Obstructed airway C) Activity intolerance D) Adverse effects of medication therapy

Ans: B Feedback: Asthma, emphysema, chronic obstructive pulmonary disease (COPD), and respiratory distress syndrome (RDS) are pulmonary obstructive diseases. All but RDS involve obstruction of the major airways. RDS obstructs the alveoli. Pain, activity intolerance, and adverse effects of medication therapy are conditions identified to detect, manage, and minimize the unexpected outcomes the nurse should be especially aware of the potential for an obstructed airway in these patients.

19. The nursing instructor is discussing bronchodilators with a group of nursing students. The students understand the instruction when they identify what drug is most effective in treating acute bronchospasm? A) Ipratropium bromide (Atrovent) B) Epinephrine (Adrenalin) C) Cromolyn (Intal) D) Ephedrine

Ans: B Feedback: Epinephrine may be injected subcutaneously in an acute attack of bronchoconstriction, with therapeutic effects in 5 minutes that last 4 hours. It is considered the drug of choice for the treatment of acute bronchospasm. Ipratropium bromide has an onset of action of 15 minutes when inhaled with a duration of 3 to 4 hours. Cromolyn is not for use during acute times of bronchospasm but is used to help prevent bronchospasm. Ephedrine can be used in acute bronchospasm but epinephrine remains the drug of choice.

24. Epinephrine, formerly the drug of choice for acute attacks of bronchoconstriction, has been replaced by what? A) Short-acting bronchodilators are the drug of choice in this situation. B) Nothing has replaced epinephrine as drug of choice in this situation. C) Self-administered metered-dose inhalers (MDIs) D) Long-acting beta2-adrenergic agonists (LABAs)

Ans: B Feedback: Epinephrine, the prototype drug, is the drug of choice in adults and children for the treatment of acute bronchospasm, including that caused by anaphylaxis; it is also available for inhalation therapy. Because epinephrine is associated with systemic sympathomimetic effects, it is not the drug of choice for patients with cardiac conditions. Options A, C, and D are not correct.

14. What action by the patient would indicate that the patient understands how to use an inhaler? A) The patient inhales as soon as the inhaler enters his or her mouth. B) The patient holds his or her breath for several seconds after releasing the medication. C) The patient administers three doses of medication within a 1-minute time frame. D) The patient exhales as soon as he or she compresses the inhaler.

Ans: B Feedback: Holding the breath prevents exhalation of medication still remaining in the mouth. The patient should inhale when the canister is compressed, not as soon as the inhaler enters his or her mouth. The patient should only administer one dose of medication at a time and the patient should wait to exhale until after the breath has been held as long as possible.

21. The nurse has provided health teaching for a 15-year-old boy newly diagnosed with asthma. What statement, made by the patient, indicates that he has a good understanding of the teaching the nurse has done regarding inhalers? A) "I should hold my breath when administering a puff." B) "The aerosol canister should be shaken well before using." C) "I need to take three short quick breaths when I administer the inhaler." D) "A second aerosol medication cannot be administered until 30 minutes after the first aerosol medication."

Ans: B Feedback: Inhalers should be shaken well, immediately before each use. It would not be appropriate to teach the patient to hold his breath when administering a puff, to take three short quick puffs when administering the inhaler, or that a second aerosol medication should not be administered until 30 minutes after the first dose of aerosol medication.

11. The nurse has admitted a patient (who takes ipratropium) to the respiratory unit with an acute exacerbation of chronic obstructive pulmonary disease (COPD). While writing a plan of care for this patient, what would be the most appropriate nursing diagnosis to use? A) Deficient knowledge regarding alternative therapy B) Imbalanced nutrition: Less than body requirements C) Acute pain related to renal effects of the drug D) Disturbed thought processes related to central nervous system (CNS) effects

Ans: B Feedback: Nursing diagnoses related to drug therapy might include acute pain related to CNS, gastrointestinal (GI), or respiratory effects of the drug; imbalanced nutrition: Less than body requirements, related to dry mouth and GI upset; and deficient knowledge regarding drug therapy. Options A, C, and D are not correct.

25. A patient has an acute asthma attack. A bronchodilator is used to bring the exacerbation under control. What drug would be used to prevent acute bronchoconstriction? A) Salbutamol (Ventolin) B) Salmeterol (Serevent) C) Fenoterol (Berotec) D) Terbutaline (Bricanyl)

Ans: B Feedback: Salmeterol (Serevent) and formoterol (Foradil) and are long-acting beta2-adrenergic agonists used only for prophylaxis of acute bronchoconstriction. Options A, C, and D are all short-acting beta2-adrenergic agonists and not used for prophylaxis.

30. The clinic nurse is caring for a patient who has just been diagnosed with chronic obstructive pulmonary disease (COPD). The patient asks the nurse what COPD means. What would be the nurse's best response? A) "It is an umbrella term for diseases like acute bronchitis." B) "It means that the lungs have been damaged in such a way that there airflow is limited in and out of the lungs." C) "It means your lungs can't expand and contract like they are supposed to, which makes it hard for you to breathe." D) "It is a term that covers so many lung diseases I can't list them all."

Ans: B Feedback: The obstruction of asthma, emphysema, and COPD can be related to inflammation that results in narrowing of the interior of the airway and to muscular constriction that results in narrowing of the conducting tube. With chronic inflammation, muscular and cilial action is lost, and complications related to the loss of these protective processes can occur, such as infections, pneumonia, and movement of inhaled substances deep into the respiratory system. In severe COPD, air is trapped in the lower respiratory tract, the alveoli degenerate and fuse together, and the exchange of gases is greatly impaired. Options A, C, and D are not incorrect, but option B is the best response.

3. A patient is in the clinic to have blood drawn to assess his or her theophylline level. The patient is taking theophylline (Theo-Dur) and appears to being doing well on the drug. He or she reports no problems. What serum level will the nurse expect the patient to have? A) Between 0.5 to 5 mcg/mL B) Between 10 to 20 mcg/mL C) Between 25 to 35 mcg/mL D) Between 40 to 50 mcg/mL

Ans: B Feedback: Therapeutic theophylline levels should be between 10 and 20 mcg/mL. A level between 0.5 and 5 mcg/mL would be low and would not produce a therapeutic effect. Levels between 25 and 50 mcg/mL would be too high and could cause serious adverse effects.

The nurse administers an adrenergic blocking agent in order to prevent release of what neurotransmitter? A) Epinephrine B) Norepinephrine C) Serotonin D) Gamma-aminobutyric acid (GABA)

Ans: B Feedback: Adrenergic blocking agents prevent norepinephrine from being released from the adrenal medulla or from the nerve terminal from activating the receptor, which blocks sympathetic

The nurse is caring for a patient in the ICU; hypotension may be anticipated when the patient's body produces insufficient quantities of what hormone? A) Calcitonin hormone B) Antidiuretic hormone (ADH) C) Growth hormone D) Melanocyte-stimulating hormone

Ans: B Feedback: Antidiuretic hormone is released from the pituitary gland resulting from reduced plasma volume or increased plasma osmotic pressure, so it serves to help maintain blood pressure. Angiotensin II also releases aldosterone from the cortex of the suprarenal gland, which causes retention of sodium and water, leading to the release of ADH to retain water and increase blood volume. Calcitonin maintains a dense, hard bone matrix and lowers blood calcium levels. Growth hormone regulates cell division and protein synthesis necessary for normal growth. Melanocyte-stimulating hormone causes skin pigmentation in humans.

After administering digoxin, what assessment finding would indicate to the nurse that the drug was having the desired effect? A) Increased heart size B) Increased urinary output C) Decreased respiratory rate D) Increased heart rate

Ans: B Feedback: As cardiac output improves, so does urinary output due to increased renal perfusion. Respiratory rate and heart size would not be impacted by the drug, although ventricular hypertrophy is a common finding in patients with heart failure. Heart rate would decrease as the force of contraction increases, ejecting more blood with each contraction.

The nurse is caring for a patient who has a sedative hypnotic ordered. The nurse would consider this drug contraindicated if the patient had what disorder? A) Neurological diseases B) Liver failure C) Endocrine disorders D) Heart disease

Ans: B Feedback: Benzodiazepines undergo extensive hepatic metabolism. In the presence of liver disease, the metabolism of most benzodiazepines is slowed, with resultant accumulation and increased risk of adverse effects. Neurological disorders, endocrine disorders, and heart disease are not contraindications for the use of benzodiazepines.

In what patient is propranolol (Inderal) contraindicated? A) 26-year-old man with viral myocarditis B) 45-year-old woman with heart failure who suffered a myocardial infarction C) 42-year-old man with hypertension D) 65-year-old woman with persistent migraines

Ans: B Feedback: Beta-adrenergic blocking agents are contraindicated in patients with bradycardia, heart failure, and heart block. The drug would not be contraindicated in the other patients.

The home care nurse is providing teaching for a 59-year-old patient taking a nonselective beta-blocker. The nurse teaches the patient the importance of notifying the prescribing physician when what occurs related to this medication? A) If the patient's pulse stays above 100 bpm for 3 or more days B) If the patient has a sudden onset of a cough C) If the patient falls D) If the patient's pulse falls below 60 bpm for 3 or more days

Ans: B Feedback: Bronchospasm, cough, rhinitis, and bronchial obstruction are related to loss of bronchodilation of the respiratory tract and vasodilation of mucous membrane vessels so a sudden onset of a cough or difficulty breathing should be immediately reported to the health care provider. Other options may need to be reported but not in relation to the nonselective beta-blocking medication.

What anxiolytic drugs would be given to a premenopausal patient who is a registered nurse planning to return to work at the hospital after anxiety is controlled? A) Alprazolam (Xanax) B) Buspirone (BuSpar) C) Diazepam (Valium) D) Clorazepate (Tranxene)

Ans: B Feedback: Buspirone is a newer anxiolytic drug that does not cause sedation or muscle relaxation. It is preferred when the patient needs to be alert such as when driving or working. Alprazolam, diazepam, and clorazepate are benzodiazepines, which cause drowsiness, sedation, depression, lethargy, confusion, and decreased mental alertness. It would be unsafe for a nurse to function in her role while taking one of these drugs.

When compared with benzodiazepines, buspirone (BuSpar) stands out as unique among antianxiety drugs because of what factor? A) Increases the central nervous system (CNS) depression of alcohol and other drugs. B) Lacks muscle relaxant and anticonvulsant effects. C) Causes significant physical and psychological dependence. D) Rapidly absorbed from the gastrointestinal (GI) tract and metabolized in the liver.

Ans: B Feedback: Buspirone, a newer antianxiety agent, has no sedative, anticonvulsant, or muscle-relaxant properties, and its mechanism of action is unknown. However, it reduces the signs and symptoms of anxiety without many of the central nervous system effects and severe adverse effects associated with other anxiolytic drugs. Most of the antianxiety drugs are rapidly absorbed from the GI tract, metabolized in the liver, have a significant drug​drug interaction with alcohol and other drugs, and can result in psychological dependence.

What electrolyte inactivates troponin and allows actin and myosin to form a bridge enabling the muscle fibers to contract? A) Magnesium B) Calcium C) Potassium D) Sodium

Ans: B Feedback: Calcium must be present to deactivate troponin so that actin and myosin can react to form actinomycin bridges. Potassium, sodium, and magnesium do not affect troponin.

A patient asks the nurse what cardiac glycosides do to improve his condition. What is the nurse's best response? A) "They decrease the force of myocardial contractions." B) "They help renal blood flow and increase urine output." C) "They increase heart rate." D) "They increase conduction velocity."

Ans: B Feedback: Cardiac glycosides increase intracellular calcium and allow more calcium to enter myocardial cells. This action causes an increased force of myocardial contraction, an increased cardiac output, and renal perfusion that increases urine output. Cardiac glycosides also serve to slow the heart rate and decrease conduction velocity.

A triage nurse in the emergency department is assessing a 78-year-old man. It is determined that the patient is experiencing severe digoxin toxicity. What drug will the nurse administer immediately? A) Inamrinone (Inocor) B) Digoxin immune Fab (Digibind) C) Verapamil hydrochloride (Calan) D) Quinidine sulfate

Ans: B Feedback: Digoxin immune Fab is an antigen-binding fragment derived from specific antidigoxin antibodies. The drug is used for the treatment of life-threatening digoxin intoxication when serum levels are greater than 10 ng/mL. Inamrinone is a phosphodiesterase inhibitor that acts as a cardiotonic agent. Verapamil hydrochloride is a calcium channel blocker. Quinidine is an antiarrhythmic agent that when taken with digoxin increases both the therapeutic and toxic effects of digoxin.

The nurse is providing patient teaching to a patient who has been prescribed digoxin. The patient tells the nurse that she occasionally use herbals and other alternative therapies. What herb would the nurse warn the patient to avoid taking with digoxin? A) Black cohosh B) Ginseng C) Saw palmetto D) Valerian

Ans: B Feedback: Digoxin toxicity can occur if the drug is taken concurrently with licorice, ginseng, or hawthorn. St. John's wort and psyllium have been shown to decrease the effectiveness of digoxin, so that combination should be avoided. There is no drug-to-drug interaction with black cohosh, saw palmetto, or valerian.

What order for a digitalizing dose of digoxin (Lanoxin) for a 62-year-old man would the nurse consider appropriate and safe to administer? A) 1.25 mg IV now B) 0.75 mg orally now C) 0.25 mg orally every day D) 1 mg intramuscularly every 4 hours × 24 hours

Ans: B Feedback: Digoxin: Adult: loading dose 0.75 to 1.25 mg orally or 0.125 to 0.25 mg IV, then oral maintenance dose of 0.125 to 0.25 mg/d; decrease dose with renal impairment.

The nurse reviews the patient's lab results and recognizes the patient is at risk for digoxin toxicity due to what electrolyte imbalance? A) Hyperkalemia B) Hypokalemia C) Hypernatremia D) Hyponatremia

Ans: B Feedback: Electrolyte abnormalities (e.g., increased calcium, decreased potassium, decreased magnesium) could alter the action potential and change the effects of the drug. Hypokalemia and hypomagnesemia increase cardiac excitability and ectopic pacemaker activity leading to dysrhythmias.

An elderly patient has been taking zolpidem (Ambien) as a sleep aid for the past 2 months. On admission to the assisted-living facility, it is determined that the drug is no longer needed. What is an important nursing consideration concerning this drug? A) Hallucinations are common. B) The drug needs to be withdrawn gradually. C) Another anxiolytic will need to be substituted. D) Sundowning is common with withdrawal from this drug.

Ans: B Feedback: It is important for the nurse to understand that zolpidem must be withdrawn gradually over a 2-week period after prolonged use. If chloral hydrate is stopped suddenly, it will result in serious adverse effects. Hallucinations and sundowning are not common with withdrawal of the drug. The prescriber and the patient would determine the need for chloral hydrate to be substituted for another anxiolytic.

The nurse is caring for a 55-year-old patient receiving metoprolol (Lopressor). What statement by the patient would lead the nurse to believe that he needs additional instruction? A) If I have side effects from the medication, I will contact my physician before I stop taking it. B) I can take over-the-counter (OTC) cold medication while on metoprolol. C) I will take the medication on an empty stomach. D) I will report a weight gain of 2 pounds or more in 1 week.

Ans: B Feedback: OTC medications can interact to increase or decrease the effects of antiadrenergic drugs. Antacids decrease the effects of beta-adrenergic blocking drugs. Decreased antihypertensive effects result when taken with ibuprofen. Other options reflect correct statements and would not indicate that the patient would need further instruction.

A 5-year-old African American patient has been admitted to the pediatric intensive care unit with pheochromocytoma. The physician has ordered phentolamine. The nurse knows that the other indication for phentolamine is what? A) Migraine headaches B) Extravasation of IV norepinephrine or dopamine C) Life-threatening arrhythmias D) Heart failure

Ans: B Feedback: Phentolamine (Regitine) is used for the prevention of cell death and tissue sloughing after extravasation of intravenous norepinephrine or dopamine, and severe hypertension reactions caused by manipulation of the pheochromocytoma before and during surgery; diagnosis of pheochromocytoma. Phentolamine would not be indicated for treatment of migraine headaches, life-threatening arrhythmias, or heart failure.

The nurse prepares to administer a phosphodiesterase inhibitor by what route? A) Oral B) IV C) Subcutaneous D) Intramuscular

Ans: B Feedback: Phosphodiesterase inhibitors are only given IV. They cannot be given orally, intramuscularly, or subcutaneously.

The nurse is caring for a resident in a long-term care facility who is African American with a history of an anxiety disorder. The patient is receiving oral lorazepam (Ativan) 2 mg t.i.d. When developing this patient's plan of care, what priority assessment will the nurse include? A) Depression B) Extreme sedation C) Phlebitis D) Nightmares

Ans: B Feedback: Special care should be taken when anxiolytic or hypnotic drugs are given to African Americans. About 15% to 20% of African Americans are genetically predisposed to delayed metabolism of benzodiazepines. As a result, they may develop high serum levels of these drugs, with increased sedation and an increased incidence of adverse effects. Depression is not a common adverse effect. Phlebitis can occur at injection sites but this patient is taking the medication orally. Nightmares occur during drug withdrawal.

When a drug is said to increase the force of contraction of the heart muscle, the nurse appropriately uses what term? A) Positive chronotropic B) Positive inotropic C) Negative inotropic D) Negative dromotropic

Ans: B Feedback: Sympathetic stimulation causes an increase in heart rate, blood pressure, and rate and depth of respirations, as well as a positive inotropic effect (increased force of contraction) on the heart and an increase in blood volume (through the release of aldosterone). A negative inotropic effect would be one that decreased the force of a contraction. A negative dromotropic effect is one that slows the conduction of the impulse through the atrioventricular node. A positive chronotropic effect is one that increases the heart rate.

The nurse frequently administers propranolol (Inderal) as treatment for what condition? A) Hypotension B) Angina C) Prevent first myocardial infarction (MI) D) Cluster headaches

Ans: B Feedback: The beta-adrenergic blocking agents are used to treat cardiovascular problems (hypertension, angina, migraine headaches) and to prevent reinfarction after MI. The prototype drug, propranolol, was in fact the most prescribed drug in the country in the 1980s and is still considered a first-line drug. Propranolol does not prevent first MIs and it is not used for hypotension or cluster headaches.

After administering an IV dose of digoxin, the nurse would expect to see effects within what period of time? A) 30 to 120 minutes B) 5 to 30 minutes C) 1 hour D) 2 hours

Ans: B Feedback: The drug has a rapid onset of action and rapid absorption (30 to 120 minutes when taken orally, 5 to 30 minutes when given IV).

The nurse is caring for a patient with coronary artery disease and hypertension. The nurse explains to the patient that coronary artery disease disables what process that controls blood pressure? A) Capacitance system B) Ability of the arterioles to increase or decrease resistance C) Oncotic pressure D) Ability of the venules to return blood to the veins

Ans: B Feedback: The entire arterial system contains muscles in the walls of the vessels all the way to the terminal branches or arterioles, which consist of fragments of muscle and endothelial cells. These muscles offer resistance to the blood that is sent pumping into the arterial system by the left ventricle, generating pressure. The arterial system is referred to as a resistance system. Its vessels can either constrict or dilate, thus increasing or decreasing resistance, respectively, based on the needs of the body. The capacitance system is the venous system and does not contribute to regulation of arterial blood pressure. Oncotic pressure acts to pull the fluid back into the vessel. The venules have no impact on regulation of arterial blood pressure.

The nurse is preparing digoxin for an infant. What is the nurse's priority intervention? A) To perform hand hygiene B) To have another nurse check dosage calculations C) To check the child's apical pulse D) To identify the patient by checking the ID bracelet

Ans: B Feedback: The margin of safety for the dosage of this drug is very narrow in children. The dosage needs to be very carefully calculated and should be double-checked by another nurse before administration. The other options are important and the nurse should implement all three. However, these actions are of lower priority.

The 96-year-old patient is receiving digoxin (Lanoxin) and furosemide (Lasix). In the morning, the patient complains of a headache and nausea. What will the nurse do first? A) Contact the patient's physician immediately. B) Check her laboratory values and vital signs. C) Administer acetaminophen and Maalox. D) Give her clear liquids and have her lie down.

Ans: B Feedback: The nurse will check the patient's digoxin level and electrolytes. Assessing vital signs is important because the risk of cardiac arrhythmias could increase due to the patient's receiving furosemide, which is a potassium-losing diuretic. The adverse effects most frequently seen with the cardiac glycosides include headache, weakness, drowsiness, and vision changes (a yellow halo around objects is often reported). Gastrointestinal (GI) upset and anorexia also commonly occur. Only after checking lab values and assessing vital signs might the nurse call the physician. Acetaminophen and Maalox would not be indicated. Having her lie down and restricting her diet to clear liquids would be appropriate but not the first actions.

The nurse is caring for a patient in intensive care unit receiving IV lorazepam (Ativan) to reduce anxiety related to mechanical ventilation. While injecting the medication the nurse notes a decrease in blood pressure and bradycardia. What is the nurse's priority action? A) Discontinue drug administration. B) Give the IV drug more slowly. C) Notify the patient's health care provider. D) Document the reaction to the drug.

Ans: B Feedback: The nurse's priority action is to slow the rate of injection because rapid injection of benzodiazepines can result in hypotension and bradycardia and can lead to cardiac arrest.

A nurse is discussing the use of alprazolam (Xanax) with a 68-year-old patient. What statement indicates that the patient has an understanding of the drug? A) "When I stop having panic attacks, I can stop taking the drug." B) "This drug will calm me down in about 30 minutes after I take it." C) "One dose will keep me calm for about 24 hours." D) "I am taking an increased dose because of my age."

Ans: B Feedback: The onset of alprazolam is about 30 minutes. The drug must be tapered after long-term use and the duration is approximately 4 to 6 hours. Elderly patients usually have a reduced dosage.

The nurse evaluates teaching as effective when a patient taking a benzodiazepine states, A) "I should always take the medication with meals." B) "I should not stop taking this drug without talking to my health care provider first." C) "I cannot take aspirin with this medication." D) "I will have to take this medication for the rest of my life."

Ans: B Feedback: The patient makes a correct statement when saying the drug should not be stopped without talking to the health care provider first because withdrawal of benzodiazepines require careful monitoring and should be gradually withdrawn. Medications do not have to be taken with food, aspirin is not Test Bank - Focus on Nursing Pharmacology (7th Edition by Amy Karch) 332 contraindicated, and the medication need only be taken while the condition being treated continues. Patients with anxiety may only need the medication for a few weeks whereas those with a seizure disorder may take it for longer periods of time.

What common action do both cardiac glycosides and phosphodiesterase inhibitors have in common related to therapeutic action? A) Blocking the enzyme phosphodiesterase B) Increasing cellular calcium C) Developing ventricular arrhythmias D) Metabolizing in the liver and excreted in the urine

Ans: B Feedback: The phosphodiesterase inhibitors block the enzyme phosphodiesterase. This blocking effect leads to an increase in myocardial cell cyclic adenosine monophosphate (cAMP), which increases calcium levels in the cell (Figure 44.4). Increased cellular calcium causes a stronger contraction and prolongs the effects of sympathetic stimulation, which can lead to vasodilation, increased oxygen consumption, and arrhythmias. Digoxin also increases intracellular calcium and allows more calcium to enter myocardial cells during depolarization.

The student nurse is studying for a pharmacology exam and notices that many of the adrenergic blocking antagonists drugs studied in class have what suffix? A) -aine B) -lol C) -azole D) -triptan

Ans: B Feedback: The suffix -lol is seen in many of the drug names for adrenergic blocking antagonists. The suffix -aine would indicate a topical anesthetic, whereas -azole indicates an antifungal, and the suffix -triptan relates to the triptans.

What assessment finding indicates to the nurse that timolol (Timoptic) has been effective? A) The patient's blood pressure increases. B) The patient's intraocular pressure is reduced. C) The patient's pulse is reduced. D) The patient's angina is reduced.

Ans: B Feedback: Timolol and carteolol are available in an ophthalmic form of the drug for reduction of intraocular pressure in patients with open-angle glaucoma. A decrease in intraocular pressure would indicate it has been effective. Timolol can also be used to treat hypertension but an increase in blood pressure would indicate the drug was not effective. Reduced pulse rate and reduced angina would not be related to this drug, especially if it was given in ophthalmic form when very little of the drug is absorbed systemically.

The nurse is caring for a patient who is taking a benzodiazepine. The nurse knows that caution should be used when administering a benzodiazepine to the elderly because of what possible adverse effect? A) Acute renal failure B) Unpredictable reactions C) Paranoia D) Hallucinations

Ans: B Feedback: Use benzodiazepines with caution in elderly or debilitated patients because of the possibility of unpredictable reactions and in patients with renal or hepatic dysfunction, which may alter the metabolism and excretion of these drugs, resulting in direct toxicity. Dosage adjustments usually are needed for such patients. Acute renal failure, paranoia, and hallucinations are not commonly related to therapy with these medications in the elderly.

Why would the nurse expect the patient with liver disease to receive a smaller dose of benzodiazepines? A) Excretion of the drug relies on liver function. B) The drugs are metabolized extensively in the liver. C) They are lipid soluble and well distributed throughout the body. D) The drugs are well absorbed from the gastrointestinal tract.

Ans: B Test Bank - Focus on Nursing Pharmacology (7th Edition by Amy Karch) 324 Feedback: The benzodiazepines are metabolized extensively in the liver. Patients with liver disease must receive a smaller dose and be monitored closely. Excretion is primarily through the urine. All of the answer options are true, but only the fact that the benzodiazepines are metabolized in the liver explains why a patient with liver disease would require smaller dosages.

Antidepressants carry a black box warning about the increased risk of suicide in what age group? A) Infants and children B) Children and adolescents C) Adolescents and adults D) Adults and older adults

Ans: B Feedback: A black box warning was added to all antidepressants bringing attention to the increase in suicidality, especially in children and adolescents, when these drugs were used. Therefore, the other age groups are incorrect.

The mental health nursing instructor is talking with the class about depression. What deficiency does the instructor explain will result in depression? A) Epinephrine, norepinephrine, and acetylcholine B) Norepinephrine, dopamine, and serotonin C) Acetylcholine, gamma-aminobutyric acid, and serotonin D) Gamma-aminobutyric acid, dopamine, and epinephrine

Ans: B Feedback: A current hypothesis regarding the cause of depression is a deficiency of norepinephrine, dopamine, or serotonin, which are all biogenic amines, in key areas of the brain. Acetylcholine is a neurotransmitter that communicates between nerves and muscles. Epinephrine is a catecholamine that serves as a neurotransmitter that is released in the sympathetic branch of the autonomic nervous system and can be hormones when released from cells in the adrenal medulla. Gamma-aminobutyric acid is a neurotransmitter that inhibits nerve activity and prevents over excitability or stimulation.

The patient has been severely depressed since her father died 6 months ago. The physician has prescribed amitriptyline. The nurse reviews the patient's chart before administering the medication. What preexisting condition would require cautious use of this drug? A) Osteosarcoma B) Cardiovascular disorders C) Closed head injury D) Bleeding ulcer

Ans: B Feedback: Caution should be used with tricyclic antidepressants in patients with preexisting cardiovascular (CV) disorders because of the cardiac stimulatory effects of the drug and with any condition that would be exacerbated by the anticholinergic effects (e.g., angle-closure glaucoma, urinary retention, prostate hypertrophy, GI or genitourinary surgery). There is no indication that caution is needed with patients diagnosed with osteosarcoma, closed head injury, or bleeding ulcer.

A 10-year-old patient is being seen by a physician for severe depression. The physician prescribes an antidepressant. What antidepressant could the nurse safely administer to a child? A) Trazodone B) Nortriptyline C) Fluvoxamine D) Phenelzine

Ans: B Feedback: Nortriptyline has established pediatric doses and can be used in children older than 6 years but such children should be monitored closely for adverse effects. Phenelzine is a monoamine oxidase inhibitor (MAOI), a class of drugs that should be avoided in children if at all possible because of the potential for drugfood interactions and the serious adverse effects. Trazodone can be used with children but is not a first-line drug because it has many adverse effects on the central nervous system associated with its use. Luvox is an SSRI that can be used in children to treat obsessive-compulsive disorder but selective serotonin reuptake inhibitors can cause serious adverse effects in children

A patient comes to the clinic because she is feeling depressed and has gained some weight. The physician prescribes oral tranylcypromine (Parnate) 10 mg twice daily for an atypical depression. When prescribed in this manner, when would the nurse tell the patient to expect the drug to reach peak levels in the body? A) 1 to 2 hours B) 2 to 3 hours C) 3 to 4 days D) 4 to 5 weeks

Ans: B Feedback: The monoamine oxidase inhibitors are well absorbed from the GI tract, reaching peak levels in 2 to 3 hours. The other time frames are incorrect.

What drug, if prescribed for the patient, would indicate the need to assess the patient for depression characterized by anxiety and addictive behaviors? A) Imipramine (Tofranil) B) Venlafaxine (Effexor) C) Fluvoxamine (Luvox) D) Tranylcypromine (Parnate)

Ans: B Feedback: Venlafaxine is used to treat and prevent depression in generalized anxiety disorder, social anxiety disorder; it also diminishes addictive behavior. Fluvoxamine is a selective serotonin reuptake inhibitor, tranylcypromine is a monoamine oxidase inhibitor, and imipramine is a tricyclic antidepressant that are not indicated for treatment of anxiety disorder and addictive behavior.

A 31-year-old male patient has been prescribed propranolol to reduce and prevent angina. What will the nurse assess this patient for related to the medication? (Select all that apply.) A) Sleep disturbance B) Impotence C) Bronchospasm D) Gastric pain E) Tachycardia

Ans: B, C, D Feedback: Adverse effects of propranolol that the nurse would assess for include allergic reaction, bradycardia, heart failure, cardiac arrhythmias, cerebrovascular accident, pulmonary edema, gastric pain, flatulence, impotence, decreased exercise tolerance, and bronchospasm.

A patient with severe depression has been hospitalized and the physician has ordered amitriptyline. What common adverse effect will the nurse monitor and assess the patient for? (Select all that apply.) A) Fever B) Myocardial Infarction C) Stroke D) Dry mouth E) Gynecomastia

Ans: B, C, D Feedback: Use of tricyclic antidepressants may lead to GI anticholinergic effects, such as dry mouth, constipation, nausea, vomiting, anorexia, increased salivation, cramps, and diarrhea. Cardiovascular effects (e.g., orthostatic hypotension, hypertension, arrhythmias, myocardial infarction, angina, palpitations, stroke) may occur. Fever and gynecomastia are not normally attributed to amitriptyline therapy.

The nurse is caring for a patient with severe coronary artery disease (CAD) who is experiencing chest pain because the oxygen demand exceeds supply. What forces could potentially be lowered to reduce oxygen consumption? (Select all that apply.) A) Pulse pressure B) Afterload C) Stretch on the ventricles D) Heart rate E) Preload

Ans: B, C, D, E Feedback: The primary forces that determine the heart's use of oxygen or oxygen consumption include: Heart rate (the more the heart has to pump, the more oxygen it requires), preload (the more blood that is returned to the heart, the harder it will have to work to pump the blood around), afterload (the higher the resistance in the system, the harder the heart will have to contract to force open the valves and pump the blood along), and stretch on the ventricles (if the ventricular muscle is stretched before it is stimulated to contract, more actomyosin bridges will be formed, which will take more energy). Pulse pressure does not impact oxygen consumption.

31. The nurse is caring for a child who has been newly diagnosed with asthma. What environmental modifications should the nurse encourage the parents to make to help their child avoid future attacks? (Select all that apply.) A) Make sure the child begins herbal therapy as soon as possible. B) Avoid crowded areas as much as possible. C) Keep the child away from any known allergens. D) Encourage the child to use a broom to sweep the bedroom every day. E) Keep the child away from areas that are filled with cigarette smoke.

Ans: B, C, E Feedback: Parents need to be encouraged to take measures to prevent acute attacks, including avoidance of known allergens, smoke-filled rooms, and crowded or dusty areas. OTC drugs and herbal remedies should be avoided if possible. The child should not be sweeping the bedroom because this will produce a lot of dust.

What priority teaching point does the nurse include in the teaching plan for a patient on a monoamine oxidase inhibitor (MAOI)? (Select all that apply.) A) Take medication at bedtime. B) Monitor blood pressure. C) Do not take over-the-counter (OTC) drugs without talking to physician. D) Report double vision right away E) Reduce tyramine intake

Ans: B, C, E Feedback: MAOIs can cause drugdrug and drugfood interactions, which can precipitate cardiovascular effects that include orthostatic hypotension, arrhythmias, palpitations, angina, and the potentially fatal hypertensive crisis. Priority teaching points include monitoring blood pressure which will elevate with tyramine ingestion and the importance of not taking any OTC without physician or pharmacist consultation due to multiple drugdrug interactions. When taking an MAOI, you would not necessarily take the drug at bedtime or drink lots of fluid. Blurred, but not double, vision is an adverse effect of an MAOI.

35. The nursing instructor is discussing the use of sympathomimetics in patients who have acute bronchospasm. The instructor shares with the students that this classification of drugs is contraindicated or only used with great caution in patients with what disorders? (Select all that apply.) A) Hypothyroidism B) Cardiac disease C) Kidney disease D) Diabetes mellitus E) Peripheral vascular disease

Ans: B, D, E Feedback: Before administering a sympathomimetic the nurse should assess for possible contraindications or cautions that include any known allergies to any drug in this class, cigarette use, cardiac disease, vascular disease, arrhythmias, diabetes, and hyperthyroidism. Sympathomimetics have no known adverse effects in hypothyroidism or kidney disease.

The nurse teaches a class on the systems that control blood pressure and includes what systems in the discussion? (Select all that apply.) A) The capacitance system B) Aldosterone-antidiuretic hormone (ADH) system C) Difference between the systolic and the diastolic readings D) Stimulus from the sympathetic system E) Reflex control of blood volume

Ans: B, D, E Feedback: Blood pressure is maintained by stimulus from the sympathetic system and reflex control of blood volume and pressure by the renin-angiotensin system and the aldosterone-ADH system. The difference between the systolic and the diastolic readings does nothing to control blood pressure. The capacitance system does not regulate blood pressure.

10. A patient presents at the emergency department in acute respiratory distress. A quick assessment by the triage nurse indicates that the patient experienced difficulty breathing immediately after taking Combivent for the first time. The nurse suspects that the patient may be allergic to what? A) Aspirin B) Penicillin C) Peanuts D) Ragweed pollen

Ans: C Feedback: Combivent is a combination drug of ipratropium and albuterol. The propellant used to make ipratropium has a cross-sensitivity to the antigen that causes peanut allergies. Aspirin, penicillin, and ragweed pollen are not associated with this drug.

2. A 6-year-old child weighing 52 pounds has had a loading dose of aminophylline (Truphylline). The nurse is ready to administer the regular prescribed dose of 4 mg/kg every 4 hours for three doses. How many milligram will the patient receive in 12 hours? A) 44 mg B) 94 mg C) 284 mg D) 344 mg

Ans: C Feedback: First, using the formula: 2.2 pounds and 52 pounds: times kg, determine the child's weight in kg (52/2.2 = 23.64). Next, using the formula of amount of drug prescribed times weight in kg, determine the amount the child will receive in one dose (4 times 23.64 = 94.56). To determine the mg in a 12-hour period multiply 94.56 times 3 = 283.68. Round to 284 mg. Options A, B, and D are not correct.

7. A patient who has chronic bronchial asthma has had a mast cell stabilizer prescribed. What drug would the physician prescribe? A) Ipratropium or budesonide B) Isoetharine or montelukast C) Nedocromil or cromolyn D) Aminophylline or caffeine

Ans: C Feedback: Nedocromil and cromolyn are mast cell stabilizers used in the treatment of asthma. Aminophylline and caffeine are xanthines. Ipratropium is an anti-cholinergic drug and budesonide is a corticosteroid. Isoetharine is a sympathomimetic drug and montelukast is a leukotriene receptor antagonist.

22. The nurse is writing a plan of care for a patient newly admitted to the floor with an asthma attack that occurred while exercising. What would be the most appropriate intervention for this patient? A) Assist patients with moderate to severe asthma in obtaining a home nebulizer unit B) Try to prevent or reduce panic, which may initiate bronchospasm C) Teach patient to use an inhaler before exercising D) Monitor peak flow rates, especially in children

Ans: C Feedback: Teach patient who use one of these drugs for exercise-induced asthma to use it 30 to 60 minutes before exercising to ensure peak therapeutic effects when they are needed. The most important intervention would be to use a bronchodilator as prophylaxis for a patient with exercise-induced asthma. It would not be monitoring peak flow rates, trying to prevent or reduce panic, or assisting patient in obtaining a home nebulizer unit.

5. An inhaled sympathomimetic drug has been ordered for a teenage athlete who has exercise-induced asthma. What should the patient be instructed to do? A) Use the inhaler every day at the same time each day. B) Use the inhaler as soon as the symptoms start. C) Use the inhaler 30 to 60 minutes before exercising to ensure peak therapeutic levels when needed. D) Use the inhaler 2 to 3 hours before exercising to ensure peak effectiveness.

Ans: C Feedback: Teaching a patient about using an inhaled sympathomimetic for management of exercise-induced asthma should include instructions to use the inhaler 30 to 60 minutes before exercising to ensure therapeutic levels when needed. The inhaler would not be used daily and waiting until symptoms occur will be too late for prevention. Options B and D are not correct.

A nurse is providing discharge instructions to a patient who is taking atenolol (Tenormin) to treat hypertension. What would the nurse teach the patient regarding a possible drugdrug interaction? A) Antibiotics B) Oral contraceptives C) Nonsteroidal anti-inflammatory drugs (NSAIDs) D) Antifungal agents

Ans: C Feedback: A decreased hypertensive effect can occur if a beta-selective adrenergic blocking agent is used in combination with NSAIDs. If this combination is used, the patient should be monitored closely and dosage adjustments made. Antibiotics, oral contraceptives, and antifungal agents are not known to have a drugdrug interaction.

The nurse provides patient teaching for a patient who has a new order for nadolol (Corgard) to treat hypertension. What statement by the patient concerning nadolol (Corgard) would indicate that the teaching has been effective? A) I should cover my head at all times while I am outdoors. B) Since I am taking this drug, I no longer need to worry about diet and exercise. C) I will not stop taking this drug abruptly and will talk to my doctor before discontinuing. D) I may have a very dry mouth while taking this drug.

Ans: C Feedback: A patient receiving an adrenergic blocker must be aware that abruptly stopping the medication may result in a serious reaction. When changing medications or discontinuing their use, these drugs need to be tapered off gradually. This drug is not associated with photophobia or the anticholinergic effect of dry mouth. If the teaching were effective, the patient would be aware that he would need to continue lifestyle modifications, including diet and exercise.

What would the nurse assess for when benzodiazepines are abruptly stopped? A) Urinary retention and change in sexual functioning B) Dry mouth, constipation, nausea, and vomiting C) Nausea, headache, vertigo, malaise, and nightmares D) In most cases nothing significant

Ans: C Feedback: Abrupt cessation of these drugs may lead to a withdrawal syndrome characterized by nausea, headache, vertigo, malaise, and nightmares. When benzodiazepines are stopped abruptly the likelihood of withdrawal symptoms increases with the length of time the patient took the medication. Urinary Test Bank - Focus on Nursing Pharmacology (7th Edition by Amy Karch) 325 retention, change in sexual functioning, dry mouth, constipation, nausea, and vomiting are all common adverse effects of the medications classified as benzodiazepines.

A patient with benign prostatic hypertrophy (BPH) has been prescribed prazosin (Minipress) and asks the nurse what this is going to do for him. The nurse's response will include what action to explain the purpose of taking this medication? A) Decreasing vascular tone and vasodilation B) Reducing the size of the prostate to reduce pressure on the urethra C) Relaxing the bladder and prostate and improving urine flow D) Lowering blood pressure

Ans: C Feedback: Alpha1 -selective adrenergic blocking agents block smooth muscle receptors in the prostate, prostatic capsule, prostatic urethra, and urinary bladder neck, which leads to a relaxation of the bladder and prostate and improved flow of urine in male patients. Although they also block the postsynaptic alpha1-receptor sites, causing a decrease in vascular tone and vasodilation that leads to a fall in blood pressure without the reflex tachycardia that occurs when the presynaptic alpha2-receptor sites are blocked, this is not the purpose for administering the drug to a patient with BPH. They do not reduce the size of the prostate.

The nurse is caring for an older adult patient who is displaying alterations in the conduction of impulses in the SA node which the nurse assesses as what? A) Cardiac gallop B) Tachycardia C) Cardiac dysrhythmias D) Heart murmurs

Ans: C Feedback: Alterations in the generation of conduction of impulses in the heart cause arrhythmias (dysrhythmias), which can upset the normal balance in the cardiovascular system and lead to a decrease in cardiac output, affecting all of the cells of the body. Tachycardia is only one of the possible arrhythmias that may result. Cardiac gallop and murmurs in the older adult with no history of congenital anomalies is usually caused by a poorly functioning heart valve and not by an alteration in conduction.

A patient presents at the free clinic complaining of nervousness, worrying about everything, and feeling very tense. What diagnose would the nurse suspect? A) Neurosis B) Psychosis C) Anxiety D) Depression

Ans: C Feedback: Anxiety is a common disorder that may be referred to as nervousness, tension, worry, or using other terms that denote an unpleasant feeling. The other options would not be described by these symptoms.

A nurse is working with a patient who is taking an adrenergic blocking agent. While assessing the patient's medication history, the nurse discovers that the patient takes several alternative therapies. What herb is the nurse concerned may interact with the adrenergic blocking agent and affect the patient's blood glucose level? A) Ginseng B) Nightshade C) Di huang D) Saw Palmetto

Ans: C Feedback: Di huang is an alternative therapy that can lower blood glucose when used in combination with adrenergic blocking agents. Ginseng increases antihypertensive effects; nightshade slows the heart rate; and saw palmetto increases the risk of urinary tract complications when used in combination with adrenergic blocking agents.

What is the priority nursing assessment for a patient who is about to begin digoxin therapy? A) Blood glucose levels B) Neurological function C) Kidney function D) Liver function

Ans: C Feedback: Digoxin is primarily excreted unchanged in the urine, so caution should be exercised if renal impairment is present. Blood glucose levels and neurological and liver function would not be a priority assessment related to digoxin therapy.

What assessment finding would indicate the patient's left-sided heart failure is worsening? A) Increased jugular venous pressure B) Liver enlargement C) Increased crackles in lung fields D) Increased pulse rate

Ans: C Feedback: Fluid may accumulate in the lungs due to left sided heart failure. Patients may evidence dyspnea, tachypnea, and orthopnea. Right-sided failure would include increased jugular venous pressure and liver enlargement. Pulse rate could increase or decrease depending on medications administered.

A patient arrives at the emergency room after attempting suicide by taking an entire bottle of diazepam. What antidote will the nurse most likely administer? A) Phenobarbital (Luminal) B) Dexmedetomidine (Precedex) C) Flumazenil (Romazicon) D) Ramelteon (Rozerem)

Ans: C Feedback: Flumazenil is an antidote to benzodiazepine overdose and is administered to reverse the effects of benzodiazepines when used for anesthesia. Phenobarbital, a barbiturate, would further depress the body functions of this patient. Dexmedetomidine is a new hypnotic drug used in the intensive care unit for mechanically ventilated patients. Ramelteon is also new; it is used as a hypnotic. Adverse effects of this drug include depression and suicidal ideation.

The nurse administers an IV phosphodiesterase inhibitor. What drug will result in forming a precipitate if given via the same IV line without adequate flushing? A) Albuterol (Proventil) B) Nifedipine (Procardia) C) Furosemide (Lasix) D) Lovastatin (Mevacor)

Ans: C Feedback: Furosemide, when given with a phosphodiesterase inhibitor, forms precipitates; therefore, this combination should be avoided. Use alternate lines if both of these drugs are being given IV. There are no known drug-to-drug interactions with albuterol, nifedipine, or lovastatin.

The nurse administers a human B-type natriuretic peptide with the expectation it will have what action? A) Decrease blood volume B) Increase force of cardiac contraction C) Reduce venous return D) Lighten the heart's workload

Ans: C Feedback: Human B-type natriuretic peptides are normally produced by myocardial cells as a compensatory response to increased cardiac workload and increased stimulation by the stress hormones. They bind to endothelial cells, leading to dilation and resulting in decreased venous return, peripheral resistance, and cardiac workload. They also suppress the body's response to the stress hormones, leading to increased fluid loss and further decrease in cardiac workload. Diuretics decrease blood volume, cardiac glycosides increase force of contraction, and vasodilators lighten the heart's workload.

Hypnotic drugs are used to aid people in falling asleep. What physiological system does a hypnotic act on to be effective in helping a patient to sleep? A) Limbic system B) Sympathetic nervous system C) Reticular activating system D) Lymph system

Ans: C Feedback: Hypnotics are used to help people fall asleep by causing sedation. Drugs that are effective hypnotics act on the reticular activating system and block the brain's response to incoming stimuli. Hypnosis, therefore, is the extreme state of sedation, in which the person no longer senses or reacts to incoming stimuli. The other options are incorrect.

A patient presents at the clinic complaining of shortness of breath, fatigue, and difficulty performing activities of daily living (ADLs). The nurse notes the bluish color around the patient's mouth and fingernail beds. What would the nurse expect the patient has? A) A dysrhythmia B) Hypotension C) Heart failure D) Hypertension

Ans: C Feedback: If the heart muscle fails to do its job of pumping blood through the system effectively, blood backs up and the system becomes congested. This is called heart failure (HF). In the case described, the patient does not present with symptoms characteristic of a dysrhythmia, hypotension, or hypertension.

The nurse is caring for a patient who has digoxin toxicity. As the nurse assesses the changes in the patient's daily activities, what finding could indicate the cause of the toxic level? A) The patient has been sleeping more lately. B) The patient took nitroglycerin for chest pain twice yesterday. C) The patient's daughter brought her a bag of licorice that she has been enjoying. D) The patient's intake of sodium increased lately because she's been eating seafood.

Ans: C Feedback: Increased digoxin toxicity has been reported with ginseng, hawthorn, and licorice. Patients should be advised to avoid these combinations. Increased sodium intake will exacerbate the patient's heart failure, which might explain why she is sleeping more and requiring nitroglycerin but these actions did not contribute to the digoxin toxicity.

The nurse assesses the patient before administering digoxin (Lanoxin) and withholds the drug and notifies the physician with what finding? A) Respiratory rate falls below 14 B) History reveals liver failure C) Pulse is 44 beats/min D) Blood pressure is 72/40 mm Hg

Ans: C Feedback: Monitor apical pulse for 1 full minute before administering the drug to assess for adverse effects. Hold the dose if the pulse is less than 60 beats/min in an adult or less than 90 beats/min in an infant; retake pulse in 1 hour. If pulse remains low, document pulse, withhold the drug, and notify the prescriber.

A patient is scheduled to have a pacemaker implanted. The nurse knows pacemakers can be inserted to correct what problem? A) Increased blood pressure B) Increased pulse pressure C) Malfunction of the sinuatrial (SA) node D) Onset of oncotic pressure

Ans: C Feedback: Pacemaker cells are found in the SA node of the heart, which controls cardiac contraction and relaxation, therefore controlling the overall heart rate. If the SA node malfunctions, a pacemaker would be indicated. Hypertension, increased pulse pressure, and onset of oncotic pressure would not be an indication for pacemaker insertion.

23. The nurse assesses the patient receiving phentolamine (Regitine) and suspects what finding is an adverse effect of the medication? A) Hypertension B) Wheezing C) Tachycardia D) Depressed respirations

Ans: C Feedback: Patients receiving phentolamine often experience extensions of the therapeutic effects, including hypotension, orthostatic hypotension, angina, myocardial infarction, cerebrovascular accident, flushing, tachycardia, and arrhythmiaall of which are related to vasodilation and decreased blood pressure. Headache, weakness, and dizziness often occur in response to hypotension. Nausea, vomiting, and diarrhea may also occur. Hypertension, wheezing, and depressed respiration would not be associated with phentolamine.

The nurse is caring for a 36-year-old man who experienced a seizure 30 minutes before coming into the emergency room, where he begins to have another. What barbiturate has the fastest onset and would be most appropriate to give to the patient to quickly stop the seizure? A) Amobarbital (Amytal Sodium) B) Mephobarbital (Mebaral) C) Phenobarbital (Luminal) D) Secobarbital (Seconal)

Ans: C Feedback: Phenobarbital's onset is between 10 and 60 minutes, depending on the route administered, and most likely this would be given to the patient. Amobarbital is given for convulsions and the onset is between 15 and 60 minutes. Mephobarbital's onset is between 30 and 60 minutes. Secobarbital is given for convulsive seizures of tetanus and has an onset of 1 to 4 hours.

30. Beta-adrenergic blocking drugs are used in children for disorders similar to those in adults. What adrenergic blocking agent is used during surgery for pheochromocytoma? A) Propranolol B) Prazosin C) Phentolamine D) Guanethidine

Ans: C Feedback: Phentolamine is used during surgery for pheochromocytoma in children. Prazosin is used to treat hypertension in children. Propranolol and guanethidine are not indicated for use in children with a pheochromocytoma.

The nurse admits a patient to the constant care unit with a digoxin level of 11 ng/mL and a serum potassium level of 5.2 mEq/L. Digoxin immune Fab is administered. The next day, the patient's digoxin level remains elevated. What action does the nurse anticipate? A) Administer digoxin immune Fab again. B) Administer a reduced dosage of digoxin. C) Continue to monitor the patient's digoxin level daily. D) Notify the health care provider of the elevated level.

Ans: C Feedback: Serum digoxin levels will be very high and unreliable for about 3 days after the digoxin immune Fab infusion because of the high levels of digoxin in the blood. The patient should not be redigitalized for several days to 1 week after digoxin immune Fab has been used, because of the potential of fragments remaining in the blood. There is no need to notify the health care provider or to administer digoxin immune Fab again.

When a muscle acts as a pump what is a necessary property for the muscle to function efficiently? A) Antagonistic contraction B) Antagonistic relaxation C) Simultaneous contraction D) Simultaneous relaxation

Ans: C Feedback: Simultaneous contraction is a necessary property for a muscle that acts as a pump. A hollow pumping mechanism must also pause long enough in the pumping cycle to allow the chambers to fill with fluid. The heart muscle relaxes long enough to ensure adequate filling; the more completely it fills, the stronger the subsequent contraction is. This occurs because the muscle fibers of the heart, stretched by the increased volume of blood that has returned to them, spring back to normal size. This property is defined through Starling's law of the heart. Simultaneous relaxation or antagonistic contraction or relaxation is not necessary.

When administering milrinone (Primacor), the nurse will assess the patient for what common adverse effect? A) Hypoglycemia B) Confusion C) Hypotension D) Seizures

Ans: C Feedback: The adverse effects most frequently seen with these drugs are ventricular arrhythmias (which can progress to fatal ventricular fibrillation), hypotension, and chest pain. Hypoglycemia, confusion, and seizures are not generally adverse effects of milrinone.

What description of an artery, made by the nurse, is accurate? A) Connective tubes composed of simple endothelial cells B) Floppy capacitance tubes C) Rigid resistance tubes D) Tubes carrying blood into the heart

Ans: C Feedback: The arteries are muscular, rigid, resistance tubes of the cardiovascular system that carry blood away from the heart. The veins are capacitance tubes that carry blood to the heart. Capillaries connect arteries and veins.

A nurse is caring for a 4-year-old child who is receiving a barbiturate. What common adverse effect would the nurse assess for? A) Decrease in respiration B) Vomiting C) Excitability D) Dry mucous membranes

Ans: C Feedback: The barbiturates, being older drugs, have established pediatric dosages. These drugs must be used with caution because of the often unexpected responses. Children must be monitored very closely for central nervous system (CNS) depression and excitability. The most common adverse effects are related to general CNS depression. Other CNS effects may include drowsiness, somnolence, lethargy, ataxia, vertigo, a feeling of a ​hangover,​ thinking abnormalities, paradoxical excitement, anxiety, and hallucinations. Alteration in respirations and dried mucous membranes are adverse effects of Test Bank - Focus on Nursing Pharmacology (7th Edition by Amy Karch) 320 antihistamines, which can be given to calm children or induce sleep. Vomiting could occur with the use of paraldehyde due to the unpleasant taste and odor of the drug.

The patient has a diagnosis of atherosclerosis. When a branch of the coronary artery becomes completely blocked, what event will the nurse expect to occur? A) Heart failure B) Arrhythmia C) Myocardial infarction D) Hypertension

Ans: C Feedback: The blood supply is most frequently altered, however, when the coronary vessels narrow and do not respond to stimuli to dilate and deliver more blood. This happens in atherosclerosis or coronary artery disease. The end result of this narrowing can be total blockage of a coronary artery, leading to hypoxia and eventual death of the cells that depend on that vessel for oxygen. This is called myocardial infarction (MI); as of 2010, it is the leading cause of death in the United States. An arrhythmia is an alteration in the conduction system. Heart failure is caused by ineffective pumping action of the heart muscle. Hypertension is an increase in blood pressure.

The patient has had a myocardial infarction. The nurse realizes the significance of this injury is damage to what layer of the heart? A) Endocardium B) Pericardium C) Myocardium D) Epicardium

Ans: C Feedback: The fibers of the cardiac muscle, or myocardium, form two intertwining networks called the atrial and ventricular syncytia. A myocardial infarction impacts this layer, which results in reduced pumping ability. The endocardium is the inner lining of the heart that comes in contact with the blood. The pericardium is the sac around the heart. The epicardium is the outermost layer of the heart.

The nurse is caring for a patient who experiences anxiety and insomnia and is prescribed benzodiazepines. When developing the plan of care, what would be an appropriate nursing diagnosis related to potential adverse effects of the drug? A) Provide patient teaching about drug therapy. B) Anxiety related to drug therapy. C) Risk for injury related to central nervous system (CNS) effects. D) Avoid preventable adverse effects, including abuse and dependence.

Ans: C Feedback: The most appropriate nursing diagnosis related to adverse effects of the drug is risk for injury related to CNS effects because benzodiazepines can have many CNS adverse effects. Anxiety is the condition for which drug therapy is prescribed not related to drug therapy. Patient teaching and avoiding adverse effects are interventions and not nursing diagnoses.

How would the nurse calculate the patient's pulse pressure? A) The ejection pressure minus the filling pressure B) The end-artery pressure minus the resting pressure C) The difference between the ejection and the resting pressures D) The filling pressure of the coronary artery minus the end-artery pressure

Ans: C Feedback: The pressure that fills the coronary arteries is the difference between the systolic (ejection) pressure and the diastolic (resting) pressure. This is called the pulse pressure (systolic minus diastolic blood pressure readings). Options A, B, and D are not correct.

A nurse is about to administer a parenteral benzodiazepine to a female patient in the hospital before the performance of a procedure. What is the priority nursing action before administration of the drug? A) Make sure that the side rails are up and the bed is in the lowest position. B) Close the blinds and ensure appropriate room temperature for the patient. C) Help the patient out of bed to the bathroom and encourage her to void. D) Ask all visitors to leave the room and remain in the waiting area.

Ans: C Feedback: The priority action would be to help the patient up to void. After the medication is administered the patient should not get out of bed because of possibly injury due to drowsiness. Safety should always be the priority concern. After administration of the drug the nurse would ask visitors to leave before beginning the procedure, make the room conducive to rest and sleep, and make sure that both side rails are up and the bed is in the lowest position.

The nurse is caring for a patient whose blood pressure is 120/78. What is the pulse pressure? A) 22 B) 32 C) 42 D) 52

Ans: C Feedback: The pulse pressure is the difference between systolic and diastolic pressure. The systolic pressure is usually 40 points greater than the diastolic pressure. A pulse pressure of over 50 points or less than 30 points is considered abnormal. This patient's pulse pressure is 42 (120 minus 78).

The nurse is caring for a patient with chronic renal failure. The patient asks the nurse how his kidney disease causes hypertension. What is the nurse's best response? A) When blood flow to the kidney is under high pressure it causes release of antidiuretic hormone (ADH). B) When too much blood flows to the kidney, it causes vasodilation. C) When blood flow to the kidney declines, cells in the kidney release renin. D) Renin production converts ADH to angiotensin I in the liver.

Ans: C Feedback: When blood flow to the kidneys is decreased, cells in the kidney release an enzyme called renin. Renin is transported to the liver, where it converts angiotensinogen (produced in the liver) to angiotensin I. Angiotensin I travels to the lungs, where it is converted by angiotensin-converting enzyme (ACE) to angiotensin II. Angiotensin II travels through the body and reacts with angiotensin II receptor sites on blood vessels to cause a severe vasoconstriction. This increases blood pressure and should increase blood flow to the kidneys to decrease the release of renin. Angiotensin II also causes the release of aldosterone from the cortex of the suprarenal glands, which causes retention of sodium and water, leading to the release of ADH to retain water and increase blood volume.

The nurse is caring for a patient who is taking amitriptyline for depression. What teaching will the nurse include in the teaching plan to help the patient monitor for adverse effects? A) If chest pain occurs an over-the-counter pain reliever will help. B) Nasal congestion indicates a respiratory virus is beginning. C) Measure and record your blood pressure daily. D) Adverse effects will subside as you adjust to the medication.

Ans: C Feedback: Cardiovascular effects of amitriptyline include orthostatic hypotension, hypertension, arrhythmias, myocardial infarction, angina, palpitations, and stroke. Miscellaneous reported effects include alopecia, weight gain or loss, flushing, chills, and nasal congestion. Teaching the patient how and when to monitor blood pressure would be an important teaching point. Chest pain could be a serious finding and patients should be taught to call 911. Nasal congestion is a possible adverse effect of the drug and not an indication of a respiratory virus infection. Adverse effects often will not subside and may continue so long as the drug is taken.

A nurse is working with a 9-year-old child who exhibits signs and symptoms of obsessive-compulsive disorder (OCD). What drug will the nurse anticipate may be prescribed for the child? A) Phenelzine (Nardil) B) Amitriptyline (Elavil) C) Fluvoxamine (Prozac) D) Isocarboxazid (Marplan)

Ans: C Feedback: Fluvoxamine is a selective serotonin reuptake inhibitor that has established pediatric dosage guidelines for the treatment of obsessive-compulsive disorder. Isocarboxazid and phenelzine are monoamine oxidase inhibitors and should be avoided in pediatric use because of the potential drugfood interactions and other serious adverse effects. Amitriptyline is also a tricyclic antidepressant not recommended for pediatric use.

The nurse is creating a care plan for a patient taking a selective serotonin reuptake inhibitor (SSRI). What would be an appropriate nursing diagnosis for this patient? A) Risk for infection B) Establish suicide precautions C) Disturbed thought processes related to central nervous system (CNS) effects of medication D) Dysfunctional Family Processes

Ans: C Feedback: Nursing diagnoses related to SSRI therapy might include: Disturbed thought processes related to central nervous system effects because adverse effects of SSRIs include headache, drowsiness, dizziness, insomnia, anxiety, tremor, agitation, and seizures. There would be no reason to expect the patient is at increased risk for infection, there is not enough information known about the patient's family to know if processes are dysfunctional. Establishing suicide precautions is an intervention and not a nursing diagnosis.

The biogenic amine theory of depression states that depression results from a deficiency of biogenic amines in key areas of the brain. Why might a deficiency of these biogenic amines exist? A) A slowing of the action of the neurons may lead to their depletion. B) Monoamine oxidase (MAO) strengthens the impact of biogenic amines. C) The number or sensitivity of postsynaptic receptors may increase. D) Norepinephrine may be depleted because biogenic amines feed off of loose particles of the neurotransmitter.

Ans: C Feedback: The three reasons for depression according to the biogenic amine theory include (1) MAO may break down biogenic amine to be recycled or restored in the neuron. (2) Rapid fire of the neurons may lead to their depletion. (3) The number or sensitivity of postsynaptic receptors may increase, thus depleting neurotransmitter levels.

12. A patient, diagnosed with asthma, has been prescribed tiotropium (Spiriva). What should the nurse teach the patient about this drug? (Select all that apply.) A) It makes you fatigued. B) You need to stay out of direct sunlight. C) It is an anticholinergic. D) You only need to take it once a day. E) It has a rapid onset of action and a long duration.

Ans: C, D, E Feedback: Tiotropium is the first drug approved for once-daily maintenance treatment of bronchospasm associated with chronic obstructive pulmonary disease (COPD). Patients who cannot tolerate the sympathetic effects of the sympathomimetics might respond to the anticholinergic drugs ipratropium (Atrovent) or tiotropium. Tiotropium has a rapid onset of action and a long duration, with a half-life of 5 to 6 days. Tiotropium is not associated with fatigue or photosensitivity.

16. The nurse is caring for a patient who is taking an adrenergic bronchodilator. In what disease process should adrenergic bronchodilators be used cautiously? A) Liver failure B) Renal failure C) Respiratory failure D) Heart failure

Ans: D Feedback: Adrenergic drugs cause cardiac stimulation. Patients with liver failure, renal failure, or respiratory failure do not need to use adrenergic bronchodilators cautiously.

27. The pharmacology instructor is explaining the difference between bronchodilators and anti-inflammatory drugs. How does an anti-inflammatory drug reduce bronchoconstriction? A) Increases ability to metabolize medication B) Decreases formation of mucus secretions C) Increases reactivity to medication D) By decreasing airway hyperreactivity to various stimuli

Ans: D Feedback: Bronchodilators, or antiasthmatics, are medications used to facilitate respirations by dilating the airways. They are helpful in symptomatic relief or prevention of bronchial asthma and for bronchospasm associated with chronic obstructive pulmonary disease (COPD). Reducing inflammation prevents and reduces bronchoconstriction by decreasing airway hyperreactivity to various stimuli that decreases mucosal edema and formation of mucus secretions that narrow airways. Anti-inflammatory drugs do not increase the ability to metabolize medication or increases reactivity to medication.

15. A patient, newly diagnosed with chronic obstructive pulmonary disease (COPD), calls the clinic and asks the nurse to explain what the newly prescribed medications are for. What would be the most appropriate response by the nurse? A) "The medications that have been ordered for you are what the physician thinks will help you the most." B) "The medications that have been ordered for you are to help you breathe easier." C) "The medications that have been ordered for you are designed to work together to help you feel better." D) "The medications that have been ordered for you are to help relieve the inflammation and promote dilation of the bronchi."

Ans: D Feedback: Drug treatment of asthma and COPD aims to relieve inflammation and promote bronchial dilation. The other options do not give the patient information about the use of these new medications.

4. A patient presents to the emergency department (ED) having an acute asthma attack. An ED physician has ordered a sympathomimetic (epinephrine). The nurse expects what as the therapeutic effect of this drug? A) Decrease the inflammatory response in the airways B) Reduce the surface tension within the alveoli allowing for gas exchange C) Inhibit the release of histamine and slow-reacting substance of anaphylaxis (SRSA) to prevent the allergic asthmatic response D) Cause dilation of the bronchi with increased rate and depth of respiration

Ans: D Feedback: Epinephrine will cause the bronchi to dilate and also cause the rate and depth of respiration to increase. Inhaled steroids decrease the inflammatory response and lung surfactants reduce the surface tension within the alveoli. Mast cell stabilizers inhibit the release of histamine and SRSA to prevent the allergic response. Options A, B, and C are not correct.

6. A premature newborn is being treated for respiratory distress syndrome. The nurse teaches the parents about what adverse effect that can occur with the use of lung surfactants? A) Kidney dysfunction B) Cardiac arrhythmias C) High fever D) Collapsed lung

Ans: D Feedback: Lung surfactants used therapeutically can cause many adverse effects including pneumothorax (collapsed lung), hypotension, pulmonary leak, hyperbilirubinemia, and sepsis. Other adverse effects may occur in the infant related to the degree of immaturity of the child's system and may not be related to the drug therapy. Options A, B, and C are not correct.

13. The nurse is providing health teaching to a newly diagnosed asthmatic patient. The patient has been prescribed theophylline. What is contraindicated with the use of this drug? A) Using insulin B) Taking anti-inflammatory drugs C) Exercising D) Smoking cigarettes

Ans: D Feedback: Nicotine increases the metabolism of xanthines in the liver so that xanthine dosage must be increased in patients who continue to smoke while using xanthines. In addition, extreme caution must be used if the patient decides to decrease or discontinue smoking because severe xanthine toxicity can occur. Bronchoconstriction is not caused by using insulin and anti-inflammatory drugs do not cause bronchoconstriction. Exercise with a physician's supervision is encouraged in individuals with asthma.

1. A 70-year-old man is being treated for chronic obstructive pulmonary disease (COPD) with theophylline (Theo-Dur). What will be a priority assessment by the nurse? A) Ingestion of fatty foods B) Weight C) Activity level D) Use of nicotine

Ans: D Feedback: Nutritional status, weight, and activity level would be important for a nurse to know about a COPD patient. However, it would be most important for the nurse to know whether the patient smokes or uses tobacco in other ways or smoking cessation methods that involve nicotine. Nicotine increases the metabolism of theophyllines; the dosage may need to be increased to produce a therapeutic effect.

20. A patient with chronic obstructive pulmonary disease (COPD) presents at the emergency department in acute respiratory distress. The patient's family tells the nurse that the patient's problems began right after the patient took his or her first dose of ipratropium (Atrovent). What would the nurse suspect is the problem? A) An allergy to milk B) Overexertion by the patient C) Patient not taking the medications correctly D) An allergy to soy products

Ans: D Feedback: The use of ipratropium or tiotropium is contraindicated in the presence of known allergy to the drug or to soy products or peanuts (the vehicle used to make ipratropium an aerosol contains a protein associated with peanut allergies) to prevent hypersensitivity reactions. An allergy to milk is not associated with sensitivity to ipratropium. Overexertion would not cause the patient to develop respiratory distress after using ipratropium for the first time. Misuse of the inhaler would not cause respiratory distress.

23. A patient tells the nurse that a friend has recommended the use of caffeine to treat the patient's asthma. The nurse counsels the patient to begin treatment immediately with the prescribed medication for what reason? A) Caffeine can aggravate the drugs used to treat asthma. B) Most natural products are less toxic or more potent than traditional asthma medications. C) Natural products decrease the adverse effects associated with adrenergic bronchodilators. D) Delays in appropriate treatment can have serious, even fatal, consequences.

Ans: D Feedback: The xanthines, including caffeine and theophylline, come from a variety of naturally occurring sources. These drugs were formerly the main treatment choices for asthma and bronchospasm. However, because they have a relatively narrow margin of safety, and they interact with many other drugs, they are no longer considered the first-choice bronchodilators. Delays in appropriate treatment can have serious, even fatal, consequences. Natural products do not decrease the adverse effects associated with adrenergic bronchodilators. Natural products have not been proven to be less toxic or more potent than prescribed asthma medications. Caffeine does not aggravate drugs used to treat asthma, but it can have an additive effect.

The nurse provides teaching about digoxin to the 62-year-old patient. The nurse evaluates patient understanding and determines further teaching is needed when the patient says she will do what? A) "Take the medication daily in the morning." B) "Take her pulse before taking her dose." C) "Weigh herself daily at the same time." D) "Take the medication with a meal."

Ans: D Feedback: Avoid administering the oral drug with food or antacids to avoid delays in absorption. The other answers are appropriate actions for the patient to take when self-administering digoxin.

What would be the teaching priority for a diabetic patient being treated with a nonselective beta-blocker? A) To take his own pulse B) To weigh himself once a week at the same time of day C) To avoid smoke-filled rooms D) To understand signs and symptoms of hypo- or hyperglycemic reaction

Ans: D Feedback: Because the beta-blockers stop the signs and symptoms of a sympathetic stress reaction, the signs and symptoms associated with hypo- or hyperglycemia, the diabetic patient taking a beta-blocker will need to understand this and learn new indicators of these reactions. Taking his pulse, weekly weighing, and avoiding smoke-filled rooms are good health practices and should be done, but not specifically needed by a diabetic patient taking a beta-blocker.

The 64-year-old patient has smoked since age 15 and has been diagnosed with chronic obstructive pulmonary disease. What classification of adrenergic blocking antagonist would be safest for this patient to treat angina? A) Nonselective adrenergic blocking agents B) Nonselective alpha-adrenergic blocking agents C) Alpha1 -selective adrenergic blocking agents D) Beta1 -selective adrenergic blocking agents

Ans: D Feedback: Beta1 -selective adrenergic blocking agents have an advantage over the nonselective beta-blockers in some cases. Because they do not usually block beta2 -receptor sites, they do not block the sympathetic bronchodilation that is so important for patients with lung diseases or allergic rhinitis. Consequently, these drugs are preferred for patients who smoke or who have asthma, any other obstructive pulmonary disease, or seasonal or allergic rhinitis. Nonselective adrenergic blocking agents block both alpha- and beta-adrenergic receptors exacerbating respiratory conditions by the loss of norepinephrine's effect of bronchodilation. Nonselective alpha-adrenergic blocking agents are not used to treat angina. Alpha1 - selective adrenergic blocking agents are not used to treat angina.

Bisoprolol (Zebeta) would be the drug of choice for which patient with a diagnosis of hypertension? A) A 7-year-old patient B) A 15-year-old patient C) A 37-year-old patient D) A 69-year-old patient

Ans: D Feedback: Bisoprolol is the drug of choice for older adults. It is not associated with as many adverse effects in the elderly and regular dosing profiles can be used. This drug does not have an established pediatric dosage. Although the 37-year-old patient is an adult, there are additional choices for this patient, with a more favorable adverse effect profile.

The nurse expects the patient's heart failure (HF) is caused by what diagnosis that is responsible for 95% of the cases diagnosed? A) Cardiomyopathy B) Hypertension C) Congenital anomaly D) Coronary artery disease (CAD)

Ans: D Feedback: CAD is the leading cause of HF, accounting for approximately 95% of the cases diagnosed. CAD results in an insufficient supply of blood to meet the oxygen demands of the myocardium. Consequently, the muscles become hypoxic and can no longer function efficiently. When CAD evolves into a myocardial infarction, muscle cells die or are damaged, leading to an inefficient pumping effort. Cardiomyopathy, hypertension, and congenital anomaly are rarely associated with heart failure.

The nurse administers a cardiac glycoside for what therapeutic effect? A) To decrease cardiac output B) To decrease afterload C) To increase ventricular rate D) To increase the force of the contraction of the heart

Ans: D Feedback: Cardiac glycosides exert a negative cardiotonic and positive inotropic effect. They do not decrease cardiac output, decrease afterload, or increase the ventricular rate of the heart.

A 6-year-old child weighing 60 pounds has been prescribed oral digoxin (Lanoxin) 30 mcg/kg as a loading dose. How many milligram will she be given? A) 0.218 mg B) 0.418 mg C) 0.618 mg D) 0.818 mg

Ans: D Feedback: First, using the formula: 2.2 pounds and 60 pounds: multiplied by kg, determine the child's weight in kg (60/2.2 = 27.27 kg). Next, using the formula: amount of drug prescribed times weight in kg, determine the dose in mcg the child should receive (30 multiplied by 27.27 = 0. 818 mcg). Then to determine the amount of mg the child should receive, use the formula: 1 mg: 1,000 mcg = X mg: 818 mcg (818/1,000 = 0.818 mg).

The patient has edema resulting from heart failure (HF) in both ankles. The nurse interprets this to mean the hydrostatic pressure is higher than what opposing pressure? A) Pulse pressure B) Edema pressure C) Fluid pressure D) Oncotic pressure

Ans: D Feedback: Hydrostatic pressure pushing fluid out of the capillary is soon higher than the oncotic pressure that is trying to pull the fluid back into the vessel, causing fluid to be lost into the tissues. This shift of fluid accounts for the edema seen in association with HF. This transfer of fluid in the capillaries, called the capillary fluid shift, is carefully regulated by a balance between hydrostatic (fluid pressure) forces on the arterial end of the capillary and oncotic pressure (the pulling pressure of the large, vascular proteins) on the venous end of the capillary. Pulse pressure has nothing to do with edema. Edema pressure and fluid pressure do not exist.

For what purpose would the nurse choose to administer a hypnotic instead of another classification of antianxiety drug? A) Treating insomnia B) Treating seizure disorder C) Treating panic attach D) Treating confusion and agitation

Ans: D Feedback: Hypnotics are used to help people fall asleep by causing sedation. Drugs that are effective hypnotics act on the reticular activating system (RAS) and block the brain's response to incoming stimuli. Hypnotics would not be the most effective drugs to treat seizure disorders, panic attack, or confusion with agitation.

A patient presents to the emergency department with rales, wheezing, and blood-tinged sputum. What does the nurse recognize that these symptoms indicate? A) Cardiomyopathy B) Cardiomegaly C) Valvular heart disease D) Pulmonary edema

Ans: D Feedback: In left-sided heart failure, the left ventricle pumps inefficiently resulting in a backup of blood into the lungs causing pulmonary vessel congestion and fluid leaks into the alveoli and lung tissue. As more fluid continues to collect in the alveoli, pulmonary edema develops. The patient will present with rales, wheezes, blood-tinged sputum, low oxygenation, and development of a third heart sound. Cardiomyopathy can occur as a result of a viral infection, alcoholism, anabolic steroid abuse, or a collagen disorder. It causes muscle alterations and ineffective contraction and pumping. Cardiomegaly is an enlargement of the heart due to compensatory mechanisms in congestive heart failure (CHF) and leads to ineffective pumping and eventually exacerbated CHF. Valvular heart disease leads to an overload of the ventricles because the valves do not close adequately causing blood to leak backward. This causes muscle stretching and increased demand for oxygen and energy.

The patient has been prescribed inamrinone (Inocor). Before administering the drug the nurse needs to know the drug has what pharmacokinetic effect? A) Decrease in cyclic adenosine monophosphate (cAMP) B) Decrease in cardiac output C) Increase in cardiac preload D) Increase in cAMP

Ans: D Feedback: Inamrinone is a phosphodiesterase inhibitor that blocks the enzyme phosphodiesterase. This blocking effect leads to an increase in myocardial cell cAMP, which increases calcium levels in the cell. These drugs do not decrease cardiac output or increase cardiac preload.

A busy patient with many responsibilities is to have a medication ordered to treat her hypertension. To increase compliance with drug therapy, what drug would be a good choice for this patient? A) Acebutolol (Sectral) B) Atenolol (Tenormin) C) Bisoprolol (Zebeta) D) Metoprolol (Lopressor)

Ans: D Feedback: Metoprolol would be the best choice because it has an extended release form that only needs to be taken once a day, which should increase patient compliance. Acebutolol, atenolol, and bisoprolol do not come in extended release forms.

Which drug is in the class of drugs called human B-type natriuretic peptides? A) Bosentan (Tracleer) B) Milrinone (Primacor) C) Digoxin (Lanoxin) D) Nesiritide (Natrecor)

Ans: D Feedback: Nesiritide is the only drug currently available in a class of drugs called human B-type natriuretic peptides. Digoxin is a cardiac glycoside. Milrinone is a phosphodiesterase inhibitor. Bosentan is an endothelin receptor antagonist.

Which nonselective alpha-adrenergic blocking agent is still used? A) Metoprolol B) Propranolol C) Timolol D) Phentolamine

Ans: D Feedback: Of the nonselective alpha-adrenergic blocking agents, only phentolamine is still used today. Metoprolol is a beta1-selective adrenergic blocking agent. Timolol and propranolol are nonselective beta-adrenergic blocking agents

The nurse is caring for a well-known stage actor who has suddenly developed severe stage fright that is preventing him from working. What drug does the nurse suspect will be prescribed for this patient? A) Carteolol (Cartrol) B) Nebivolol (Bystolic) C) Nadolol (Corgard) D) Propranolol (Inderal)

Ans: D Feedback: One of the indications for use of propranolol is prevention of stage fright, which is a sympathetic stress reaction to a particular situation. None of the other options are indicated for this use.

A 62-year-old African American man diagnosed with congestive heart failure and hypertension has BiDil included in his drug therapy. What nursing assessment finding would indicate the patient is developing a complication from this drug? A) Alopecia B) Photosensitivity C) Anorexia D) Orthostatic hypotension

Ans: D Feedback: Orthostatic hypotension is an adverse effect of a combination of isosorbide dinitrate and hydralazine called BiDil. This could lead to safety concerns and should be addressed in drug teaching for this patient. Alopecia, photosensitivity, and anorexia are not adverse effects related to this drug.

A nurse is caring for a neonate born with a congenital heart anomaly. To better help the parents understand the impact of this disorder, the nurse begins by describing the usual flow of blood through the heart which takes what course? A) Deoxygenated blood from the lungs enters the left atrium through the pulmonary artery. B) Oxygenated blood from the lungs enters the right atrium through the pulmonary veins. C) Deoxygenated blood from the lungs enters the right atrium through pulmonary veins. D) Oxygenated blood from the lungs enters the left atrium through the pulmonary veins.

Ans: D Feedback: Oxygenated blood from the lungs enters the left atrium through the pulmonary veins and passes through the mitral valve into the left ventricle, which contracts and ejects blood through the aortic valve into the aorta and out to the systemic circulation.

The patient has just been prescribed milrinone (Primacor). The nurse recognizes the drug is contraindicated due to the patient's allergy to what? A) Penicillins B) Salicylates C) Opioids D) Bisulfites

Ans: D Feedback: Phosphodiesterase inhibitors are contraindicated in the presence of allergy to the drug or to bisulfites. Penicillins, salicylates, and opioids have no contraindications when used with milrinone.

A 54-year-old man has a myocardial infarction, resulting in left-sided heart failure. The nurse caring for the man is most concerned that he will develop edema in what area of the body? A) Abdominal B) Liver C) Peripheral D) Pulmonary

Ans: D Feedback: Pulmonary edema can occur when the heart is damaged and the left side of the heart is unable to effectively pump blood returning from the right side of the heart into systemic circulation. This lack of blood movement out of the lungs results in an increased hydrostatic pressure in the capillary beds of the lungs and the result can be pulmonary edema. Abdominal, liver, and peripheral edemas occur when the right side of the heart is failing and the returning venous blood pools in the organs and periphery.

The nurse, caring for a patient who is receiving cardiac glycosides to treat heart failure, will teach the patient to follow what diet? A) High sodium, low potassium, high fat B) Low sodium, low potassium, low fat C) High iron, high calcium, high potassium D) Low sodium, high potassium, low fat

Ans: D Feedback: Restrict dietary sodium to reduce edema in patients receiving cardiac glycosides. If the patient is hyponatremic or using a potassium-losing diuretic, increase potassium in diet, as well as limit fat intake to reduce weight and atherogenic activity.

An older adult African American patient comes to the clinic and is diagnosed with generalized anxiety disorder (GAD). The physician orders oral flurazepam 30 mg. What is the nurse's priority action? A) Teach the patient about the prescribed medication. B) Administer the first dose of medication. C) Tell patient to take first dosage after driving home. D) Talk to the physician about the dosage.

Ans: D Feedback: Test Bank - Focus on Nursing Pharmacology (7th Edition by Amy Karch) 331 If an anxiolytic or hypnotic agent is the drug of choice for an African American patient, the smallest possible dose should be used, and the patient should be monitored very closely during the first week of treatment. Dosage adjustments are necessary to achieve the most effective dose with the fewest adverse effects. In addition, older adults also require careful titration of dosage. Older patients may be more susceptible to the adverse effects of these drugs, from unanticipated central nervous system (CNS) adverse effects including increased sedation, dizziness, and even hallucinations. Dosages of all of these drugs should be reduced and the patient should be monitored very closely for toxic effects and to provide safety measures if CNS effects do occur. As a result, the priority action is to talk to the physician about the dosage. The other actions may be appropriate after a proper dosage is ordered

The nurse takes the patient's pulse and finds a regular rate at 44 beats/min. What area of the heart is controlling this patient's heart rate? A) The atrioventricular (AV) node B) The bundle of His C) The Purkinje fibers D) The sinuatrial (SA) node

Ans: D Feedback: The SA node generates an impulse about 90 to 100 times a minute, the AV node about 40 to 50 times a minute, and the complex ventricular muscle cells only about 10 to 20 times a minute.

The nurse is explaining the normal conduction pattern of the heart to an adolescent with a newly diagnosed congenital heart anomaly. The nurse tells the patient that what serves as the pacemaker for the heart? A) The atrioventricular (AV) node B) The bundle of His C) The Purkinje fibers D) The sinuatrial (SA) node

Ans: D Feedback: The SA node, which is located near the top of the right atrium, acts as the pacemaker of the heart. The SA node sends the signal to the AV node, and then to the bundle of His, and into the Purkinje fibers, but it is the SA node that sets the rate.

A nurse is caring for a 9-year-old patient and has received an order for diazepam (Valium) 10 mg given orally q.i.d. What is the nurse's priority action? A) Perform hand hygiene and prepare the drug. B) Send the order to the hospital pharmacy. C) Determine when to administer the first dose. D) Call the physician and question the order.

Ans: D Feedback: The first action of the nurse would be to call the physician and question the order. The normal oral dosage for a pediatric patient is 1 to 2.5 mg t.i.d. or q.i.d. The ordered dose would be unsafe for this patient. If the dosage was changed and the correct amount administered, the nurse would order the medication from the pharmacy if necessary and determine what time to start the medication. She would then wash her hands in preparation for administering the medication, but not until obtaining an appropriate dosage of medication.

A patient who is on a ventilator has been declared brain dead. A family member asks the nurse how the heart can still function if his father's brain is dead. What statement is an appropriate response by the nurse? A) "Death of the heart will follow brain death in a few hours." B) "The heart is controlled by the brainstem, not the upper levels of the brain." C) "The cardiovascular center in the medulla would not be reflected in any tests." D) "The heart is self-controlled and does not depend on the brain to beat."

Ans: D Feedback: The heart can generate action potentials on its own and can function without connection to the rest of the body. The heart will continue to function as long as oxygen and glucose are supplied to it. The brain does not initiate heart functioning. The cardiovascular center in the medulla helps to regulate blood pressure, not heart function.

A patient is being discharged home from the hospital after receiving treatment for pneumonia. The patient is going home and continuing to take the same drugs he or she was taking before he or she was hospitalized. These drugs include an antianxiety medication and a medication for insomnia. The home care nurse is following this patient. On the initial visit what is the nurse's priority teaching point? A) The names and purposes of medications prescribed B) How to contact the provider if needed C) The importance of taking medications for insomnia only occasionally D) Warning signs that may indicate serious adverse effects

Ans: D Feedback: The home care nurse should provide thorough patient teaching, with a priority teaching point being the warning signs the patient may experience that indicate a serious adverse effect. Although this may have been discussed by the discharging nurse in the hospital, this is essential information for the patient to thoroughly understand. By the time the home care nurse visits, the patient should already have filled the prescriptions and know the names and purposes of the medications prescribed from the hospital nurse but it is a good idea to review this information, although it is not a priority. Medications for insomnia should be taken as prescribed. The patient should have received the provider's contact information when leaving the hospital but the home care nurse may need to review this, even though it is not the priority teaching point.

The nurse is caring for a patient who has not been able to sleep. The physician orders a barbiturate medication for this patient. What adverse effect should the nurse teach the patient about? A) Double vision B) Paranoia C) Tinnitus D) Thinking abnormalities

Ans: D Feedback: The most common adverse effects are related to general central nervous system (CNS) depression. CNS effects may include drowsiness, somnolence, lethargy, ataxia, vertigo, a resembling a ​hangover,​ thinking abnormalities, paradoxical excitement, anxiety, and hallucinations. Barbiturate drugs generally do not cause double vision, paranoia, or tinnitus.

A priority nursing assessment for a patient who is to receive an alpha- or beta-adrenergic blocking agent would be what? A) Monitoring respiratory rate B) Checking blood glucose level C) Measuring urine output D) Assessing heart rate

Ans: D Feedback: The most serious adverse effect would be severe bradycardia, so the nurse's priority would be assessing the heart rate. If the patient were identified as having diabetes, then monitoring blood glucose levels would become important because these drugs can aggravate diabetes by blocking sympathetic response including masking the usual signs and symptoms of hypoglycemia and hyperglycemia. Respiratory rate could be impacted if the patient was identified as having a condition causing bronchospasm and diabetes because the combination could worsen both conditions. Measuring urine output should be part of the patient's care, but it is not the priority assessment.

The nurse explains the normal conduction pathway to the patient and evaluates the patient understood when they provide what description of the conduction pathway in the heart? A) Sinuatrial (SA) node to bundle of His to atrioventricular (AV) node to Purkinje fibers B) SA node to AV node to Purkinje fibers to bundle of His C) SA node to bundle of His to Purkinje fibers to AV node D) SA node to AV node to bundle of His to Purkinje fibers

Ans: D Feedback: The normal conduction route is SA node to AV node to bundle of His to Purkinje fibers. Therefore, options A, B, and C are incorrect.

The nurse is caring for a patient in a state of shock. The family asks the nurse why the patient is so sick. What is the nurse's best response? A) The patient probably has an infection that is making him look so sick. B) The coronary arteries are not supplying adequate blood flow to the myocardium. C) The pumping action of the ventricles is diminished, resulting in blood backing up in his lungs. D) Low blood pressure means inadequate supply of oxygen to the body's tissues.

Ans: D Feedback: The pressure of the blood in the arteries needs to remain relatively high to ensure that blood is delivered to every cell in the body and to keep the blood flowing from high-pressure to low-pressure areas. The pressure can fall dramatically—this is called hypotension—from loss of blood volume or from failure of the heart muscle to pump effectively. Severe hypotension can progress to shock and even death as cells are cut off from their oxygen supply. An infection would result in septic shock. Many types of shock exist. Inadequate supply of oxygen via the coronary arteries results in a myocardial infarction. Reduced pumping action of the heart is called heart failure.

Several nursing students are creating a poster on the mechanism of the heart. What structure would they label as separating the right half of the heart from the left? A) Auricle B) Bundle of His C) Syncytia D) Septum

Ans: D Feedback: The septum is a partition that separates the right and left halves of the heart. The right half receives deoxygenated blood from everywhere in the body and the left half receives oxygenated blood from the lungs. The auricle is an appendage attached to each atrium, which collects blood that is pumped into the ventricles by atrial contractions. Impulses are sent from the atria into the ventricles by way of the bundle of His, which then enters the septum and subdivides into three bundle branches that become a network of fibers that delivers the electrical impulse to the ventricular cells. The myocardium forms two intertwining networks, atrial and ventricular syncytia, which enable first the atria and then the ventricles to contract synchronously when excited by the same stimulus.

A patient presents at the emergency department with respiratory depression and excessive sedation. The family tells the nurse that the patient has been taking medication throughout the evening and gives the nurse an almost empty bottle of benzodiazepines. What other adverse effects would the nurse assess this patient for? A) Seizures B) Tachycardia C) Headache D) Coma

Ans: D Feedback: Toxic effects of benzodiazepines include excessive sedation, respiratory depression, and coma. Flumazenil (Anexate) is a specific antidote that competes with benzodiazepines for benzodiazepine receptors and reverses toxicity. Seizures, tachycardia, and headache would not normally be associated with benzodiazepine toxicity.

A 72-year-old patient presents at the emergency department with respiratory depression and excessive sedation. The family tells the nurse that the patient has been taking medication throughout the evening. The nurse suspects benzodiazepine overdose and would expect what drug to be ordered? A) Valium B) Phenergan C) Hydroxyzine D) Flumazenil

Ans: D Feedback: Toxic effects of benzodiazepines include excessive sedation, respiratory depression, and coma. Flumazenil is an antidote for the benzodiazepines. Hydroxyzine is an antihistamine with anticholinergic effects and would not be appropriate for this patient. Valium would enhance the effects of benzodiazepines. Phenergan is not indicated for this patient; it is similar in actions to hydroxyzine.

The nurse is assigned to watch the cardiac monitors in the constant care unit and notes four different patients displaying arrhythmias. Which arrhythmia is the nurse's highest immediate priority? A) Sinus bradycardia at a rate of 54 beats/min B) Atrial flutter C) Sinus tachycardia at a rate of 108 beats/min D) Ventricular fibrillation

Ans: D Feedback: Ventricular fibrillation is seen as a bizarre, irregular, distorted wave. It is potentially fatal because it reflects a lack of any coordinated stimulation of the ventricles. The ventricles' inability to contract in a coordinated fashion results in no blood being pumped to the body or the brain. Thus, cardiac output is totally lost. Sinus bradycardia, sinus tachycardia, and atrial flutter may all require attention but are not as life threatening as ventricular fibrillation.

A patient comes to the mental health clinic for a regular appointment. The patient tells the nurse he has been taking oral fluoxetine (Prozac) 20 mg daily for the past 3 weeks and that he has lost 3 pounds during that time due to a loss of appetite. What action should the nurse take? A) Teaching the patient about healthy eating to maintain weight B) Congratulating the patient on his weight loss and commenting how well he looks C) Encouraging the patient to increase fluid intake to avoid further weight loss D) Reassuring the patient that a decrease in weight is a common adverse effect with this medication

Ans: D Feedback: Adverse effects of fluoxetine include anorexia and weight loss. Although teaching about healthy eating is a good idea, it is more important to teach the patient how to take the medication in a way that will reduce adverse effects as well as how to optimize healthy calories to maintain weight. The patient should increase caloric intake, not just fluid intake. The patient should continue the medication to see whether therapeutic effects are obtained and adjust nutritional intake if necessary. More information about the patient's baseline weight is needed before congratulating the patient because a patient who is already too thin would not appreciate the nurse's comment.

A patient receives a new prescription for fluvoxamine (Luvox). What will the nurse instruct the patient about taking the medication? A) Take medication after eating breakfast. B) Take medication with at least 8 ounces of liquid. C) The dosage may need to be increased if the patient is not feeling better in 2 weeks. D) The medication should be taken once a day before bedtime.

Ans: D Feedback: Fluvoxamine is a selective serotonin reuptake inhibitor that should be taken once a day before bedtime. The medication does not require 8 ounces of fluid for absorption. It should be taken for at least 4 weeks before a therapeutic effect is noted.

The nurse is teaching a patient taking a monoamine oxidase inhibitor (MAOI) about dietary changes required to minimize adverse effects of the drug. The nurse determines the patient understands a low tyramine diet when what meal is chosen? A) A chop salad with blue cheese, sardines, and pepperoni B) A sandwich with turkey, avocado, and Swiss cheese C) Corned beef hash, eggs, and hash browns D) A hamburger, French fries, and a strawberry milkshake

Ans: D Feedback: Hamburger, French fries, and a strawberry milkshake do not contain tyramine and, although high in fat, it would not be contraindicated for a patient taking an MAOI. Blue cheese, sardines, pepperoni, Swiss cheese, and corned beef are all high in tyramine and would indicate further teaching was needed.

A patient is admitted with a presumed diagnosis of colon cancer who takes a monoamine oxidase inhibitor for depression. What drug will the nurse keep on hand for this patient in case of the onset of an adverse reaction? A) Epinephrine B) Injectable naloxone (Narcan) C) Phenylalanine D) Phentolamine

Ans: D Feedback: Have phentolamine or another adrenergic blocker on standby as treatment in case of hypertensive crisis. The other options are distracters for this question.

The nurse is caring for a patient who was referred to a psychiatrist for treatment of a severe anxiety disorder. What medication does the nurse consider appropriate for this patient? A) Chlorpromazine (Thorazine) 25 mg three times daily orally B) Benztropine (Cogentin) 2 mg twice daily orally C) Clozapine (Clozaril) 200 mg twice daily orally D) Paroxetine (Paxil) 10 mg once daily orally

Ans: D Feedback: Paroxetine is a selective serotonin reuptake inhibitor indicated for the treatment of depression, obsessive compulsive disorder, panic attacks, bulimia, premenstrual dysphoria disorder, posttraumatic stress disorders, social phobias, and social anxiety disorders. Chlorpromazine and clozapine are antipsychotic medications whereas benztropine is a drug used to treat Parkinson's disease. None of these would be appropriate options to treat anxiety disorders.

A 12-year-old patient is hospitalized with severe depression. The patient has been taking a selective serotonin reuptake inhibitor (SSRI). What is the priority nursing action for the patient? A) Monitor food intake for levels of tyramine. B) Assess for weight loss and difficulty sleeping. C) Monitor the patient for severe headaches. D) Implement suicide precautions.

Ans: D Feedback: Recent studies have linked the incidence of suicide attempts to the use of SSRIs in pediatric patients (see box 21.3 Focus on the Evidence). The priority concern for the nurse would be safety for the patient. Severe headache and reactions to tyramine- containing foods are associated with monoamine oxidase therapy. Weight loss and difficulty sleeping are of a lower priority concern than the patient's safety.

What is the physiological action of tricyclic antidepressants (TCAs)? A) Inhibiting monoamine oxidase inhibitors that break down norepinephrine B) Inhibiting nerve activity, which prevents over excitability or stimulation C) Blocking the reuptake of serotonin, which increases the levels of norepinephrine D) Inhibiting reuptake of norepinephrine and serotonin

Ans: D Feedback: TCAs inhibit presynaptic reuptake of norepinephrine and serotonin, which cause an accumulation of the neurotransmitters that is thought to create the antidepressant effect. Monoamine oxidase inhibitors irreversibly inhibit monoamine oxidase that breaks down norepinephrine and serotonin. Selective serotonin reuptake inhibitors block the reuptake of serotonin; gamma-aminobutyric acid inhibits nerve activity.

What reason might the nurse give for why venlafaxine (Effexor) has become more popular with adults in treating their depression? A) It is taken orally. B) It does not have adverse effects. C) It can be taken during pregnancy. D) An extended release form is available.

Ans: D Feedback: Venlafaxine mildly blocks reuptake of norepinephrine, 5-hydroxytriptamine, and dopamine. It has fewer adverse central nervous system effects than trazodone. Its popularity has increased with the introduction of an extended-release form that does away with the multiple daily doses that are required with the regular form. Venlafaxine is readily absorbed from the gastrointestinal (GI) tract, extensively metabolized in the liver, and excreted in urine. Adequate studies have not been done in pregnancy and lactation, so that it should be used during those times only if the benefit to the mother clearly outweighs the potential risk to the neonate. It is taken orally, which is the case with most antidepressants

The patient is diagnosed with myasthenia gravis, a condition in which antibodies lock, alter, or destroyed the receptors for acetylcholine. What symptom would the nurse expect this patient to display? A) muscle dysfunction B) Seizures C) uncoordinated movement D) sleep all the time

Answer: A Feedback: acetylcholine communicates Between nerves and muscles so inability of this neurotransmitter to function properly, whether blocking, altering, or destroying the receptors, would result and muscle dysfunction. And adequate gamma-aminobutyric acid would result in seizures. Inadequate dopamine would result in uncoordinated movements. Serotonin is important in arousal and sleep

which neurotransmitter communicates between nerves and muscles? A) Acetylcholine B) Dopamine C) gamma-aminobutyric acid (GABA) D) Serotonin

Answer: A Feedback: acetylcholine, which communicates between nerves and muscles, is also important as the preganglionic neurotransmitter throughout the autonomic nervous system and ask the postganglionic neurotransmitter in the parasympathetic nervous system and in several Pathways to the brain. dopamine is involved in the coordination of impulses and responses, both motor and intellectual. Gaba inhibits nerve activity. Serotonin is important in arousal and sleep

the nurse is caring for a patient who has recent ultrasound of the Carotids diagnose to 90% occlusion of the right carotid artery and a 92% occlusion of the left carotid artery. The patient asks the nurse, if one of these arteries becomes completely occluded will I have a stroke? What is the nurses best response? A) Common vessel receiving all Blood to the Head called The Circle of Willis will distribute blood from other arteries to the brain is needed. B) A stroke is caused by a lack of blood supply to a part of the brain so if you're right artery becomes blocked you'll have a stroke on the right side of your brain C) it's hard to predict exactly what will happen so you'll have to wait until your provider sees you because only the provider can answer that question D) with only 10% of the blood needed getting through to your right artery at 8% through your left, you could have a stroke now

Answer: A Feedback: all the arteries that Supply Blood to the Head deliver blood to a common vessel at the bottom of the brain called The Circle of Willis, which distributes the blood to the brain when it is needed. The role of the Circle of Willis becomes a parent when someone has an occluded carotid artery. Although the passage of blood through one of the carotid arteries maybe negligible, the areas of the brain on that side will still have a full blood supply because of the blood center those areas Through The Circle of Willis

the student nurses are learning about the nervous system. What would students learn are capable of conducting along the entire membrane of the nerve? A) Action potentials B) Engrams C) chemical synapses D) sodium potassium channels

Answer: A Feedback: nerve membranes, which are capable of conducting action potentials along the entire membrane, send messages to nearby neurons or effector cells that may be located inches to feet away the others electrical communication system.

The physiology instructor explains to the students that some substances increase actual learning. What is one of these substances? A) Oxytocin B) Acetylcholine C) Serotonin D) Dopamine

Answer: A Feedback: oxytocin and Mild stress act to increase actual learning. Childbirth is the only time that oxytocin levels increase and this phenomenon is that clearly understood. Acetylcholine, serotonin, and dopamine do not increase actual learning; they function as neurotransmitters

the nurse is caring for a patient whose emotions often swing from one extreme to the other. The patient's spouse tells the nurse these mood swings started when the patient awoke from a coma following a serious head trauma. What area of the brain does the nurse suspect was damaged? A) the limbic system B) Forebrain C) the hindbrain D) the cerebellum

Answer: A Feedback: stimulation of the limbic system, which appears to be responsible for the expression of emotions, may lead to anger, pleasure, motivation, and stress. The forebrain is were thinking and coordination of sensory and motor activity occur. The hindbrain controls vital functions and arousal. Cerebellum controls motor function that regulates balance

The nurse is caring for a patient with a malignant brain tumor. The patient asks the nurse why the tumor is being treated with radiation instead of chemotherapy. The nurses explanation involves what important information? A) medications have difficulty crossing the blood-brain barrier B) neurons in the brain are easily damaged by chemotherapy C) tumors arising from nervous tissue are not impacted by chemotherapy D) chemotherapy reduces nerve transmission and cannot be used

Answer: A Feedback: the blood-brain barrier is a functioning boundary that plays a defensive role by keeping toxins, proteins, and other large structures out of the brain and preventing their contact with the sensitive and fragile neuron. as a result, medications like chemotherapy can have difficulty Crossing this barrier to reach the tumor. the other answers are neither true nor the correct option

Well diagram in the brain for their anatomy and physiology class, the nursing students would place the swallowing Center where? A) hindbrain B) right hemisphere C) Forebrain D) left hemisphere

Answer: A Feedback: the pons and medulla oblongata are in the hindbrain and control basic, vital functions, such as the respiratory Center, which controls breathing; the cardiovascular centers, which regulate blood pressure; the chemoreceptor trigger Zone and the emetic Zone, which controls vomiting; and the swallowing Center, which coordinates of complex swallowing reflex

the sensory nerves enter the brain and react with related nerves to cause a reaction. What mediates this reaction? A) muscles or glands B) the limbic system C) The cerebral cortex D) Neurotransmitters

Answer: A Feedback: the sensory nerves that enter the brain react with related motor nerves to cause a reaction mediated by muscles or glands. The motor impulses that leave the cortex are further regulated or coordinated by the pyramidal system, which coordinates voluntary movement, and the extrapyramidal system, which coordinates unconscious motor activity that regulates control of position and posture. Therefore, the other options are incorrect

do nursing students are giving an oral presentation on the forebrain. What information will they include about this area? ( Select all that apply.) A) coordinates speech and communication B) area where learning takes place C) houses the extrapyramidal motor system D) cranial nerves emerge from here E) the swallowing Center is here

Answer: A, B, C Feedback: the forebrain is made up of two cerebral hemispheres join together by the corpus callosum. The two hemispheres contain the sensory and motor neurons. It also contains areas that coordinate speech and communication and is thought to be where learning takes place. Cranial nerves emerge from the hindbrain, which is where the swallowing Center is located as well

1) What part of the neuron carries information into the neuron from other neurons? A) Axon B)Dendrite C) Nucleus D) Soma

Answer: B Feedback: dendrites carry information to the nerve and axons; they also carry information from the nerve to be transmitted to effector cells, which are found in muscles, glands, or other nerve. Soma Refers to the cell body. The nucleus is the central part of a cell, which is responsible for the cell's growth, reproduction, and metabolism.

What is the purpose of the myelin sheath? A) protects owner from damage B) speeds electrical conduction C) produces Schwann cells D) secretes neurotransmitters

Answer: B Feedback: long nerves are myelinated: they have a myelin sheath that speeds electrical conduction and protects the nerves from the fatigue that results from frequent formation of action potentials, not from damage. Although myelin sheath have Schwann cells, they do not produce the cells and the myelin sheath does not secrete neurotransmitters

As the nursing students learn about the functioning of the nervous system, they learned that the energy required by the nerves is provided by what? A) Dopamine and electricity B) oxygen and glucose C) sodium and potassium D) acetylcholine and serotonin

Answer: B Feedback: nerves require energy (e.g., oxygen and glucose) and the correct balance of the electrolytes sodium and potassium to maintain normal action potentials and transmit information into and out of the nervous system. Energy required by the nerves is not provided by dopamine, electricity, acetylcholine, or serotonin

The anatomy and physiology instructor is discussing neurotransmitters with the pre-nursing anatomy and physiology class. What neurotransmitter would the instructor tell the students is a catecholamine classified as a hormone when it is released from the Adrenal medulla? A) Ephedrine B) Norepinephrine C) Dopamine D) Acetylcholine

Answer: B Feedback: norepinephrine and epinephrine are catecholamines, which are released by nerves in the sympathetic branch of the autonomic nervous system and are classified as hormones when they are released from cells in the Adrenal medulla. Therefore, options a, c, and d are incorrect

The nurse is caring for a patient with meningitis who is not responding to the prescribed antibiotic and whose condition continues to deteriorate. What rationale will the nurse give the family to explain why the antibiotic is not as affected as it was hoped? A) The meninges do not have a blood supply B)the blood-brain barrier prevents the antibiotics from Crossing into the brain C) The Circle of Willis redirects the antibiotic elsewhere D) the pressure in the hindbrain prevents entry into the skull

Answer: B Feedback: the blood-brain barrier works to keep large molecules out of the brain and away from the nerves. Most antibiotics are protein bound and cannot pass through the blood-brain barrier. When the infection becomes severe, the blood-brain barrier will stop being effective and the antibiotics can pass into the brain. The brain has a unique blood supply to protect the neurons from lack of oxygen and glucose. After the blood-brain barrier allows the antibiotic to pass through, the Circle of Willis distributes the blood to the areas of need. If someone has an occluded carotid artery, which could build pressure up in the area, The Circle of Willis can redirect the blood supply and provide full blood supply to the affected areas

the nurse is caring for a patient who has an injured hindbrain. What would the nurse expect to find altered when assessing this patient? A) Arousal and awareness B) basic vital functions C) coordination and motor activity D) Learning and motivation

Answer: B Feedback: the hindbrain contains centers that control basic vital functions (e.g., blood pressure, respirations, vomiting). the reticular activating system in the medulla controls arousal and awareness. Learning and motivation occur in the cerebral cortex. Coordination and motor activity are controlled through the cerebellum and basal ganglia.

Neurotransmission is important to the function of the central nervous system. For neurotransmission to occur, how do neurons communicate with other cells? A) Selectively B) Chemical C) Excitably D) Acessibly

Answer: B Feedback: the transmission of information between two neurons or between a nerve and a gland or muscle is chemical. Options a, c, and d are incorrect

When a neuron is stimulated and causes depolarization of the nerve, what occurs? A)Calcium rushes into the cell B) sodium rushes into the cell C) potassium rushes into the cell D) sodium and potassium are actively pumped out to the cell

Answer: B Feedback: when depolarization occurs, sodium rushes into the South. During repolarization, potassium is pumped out of the cell and the resting membrane potential is re-established. Calcium ions decrease the cell membrane permeability to sodium and increase the threshold needed to depolarize the cell

while discussing the central nervous system, the nursing instructor tells students that the major inhibitory neurotransmitter in the CNS is what? A) Acetylcholine B) Dopamine C) gamma-aminobutyric acid (GABA) D) Serotonin

Answer: C Feedback: GABA, which is found in the brain, inhibits nerve activity and is important in preventing overexcitability or stimulation such a seizure activity. Acetylcholine, which communicates between nerves and muscles, is also important as the preganglionic neurotransmitter throughout the autonomic nervous system and as the postganglionic neurotransmitter in the parasympathetic nervous system and in several Pathways in the brain. Dopamine is involved in the coordination of impulses and responses, both motor and intellectual. Acetylcholine, dopamine, and serotonin or not the major inhibitory neurotransmitter in the CNS. Serotonin is important in arousal and sleep

what neurotransmitter inhibits overexcitability and is important in preventing seizure activity in a patient? A) Acetylcholine B) Dopamine C) gamma-aminobutyric acid (GABA) D) Serotonin

Answer: C Feedback: Gaba is found in the brain and inhibits nerve activity in. Is important in preventing overexcitability or stimulation such a seizure activity. Acetylcholine communicates between nerves and muscles. Dopamine is involved in the coordination of impulses and responses, but with motor and intellectual. Serotonin is found in the limbic system and is important in arousal and sleep as well as in preventing depression and promoting motivation.

and 87 year old woman undergoes extensive surgery for an acoustic neuroma ( a benign tumor of the inner ear), and 6 hours after surgery. She hemorrhages and goes into a coma. After awakening and two months of therapy, she is transferred to a long-term care facility. Do the damage in the midbrain, the nurse caring for the patient will expect the patient to exhibit what? A) Difficulty in sleeping B) difficulty in hearing C) difficulty in distinguishing hot and cold D) difficulty in speaking

Answer: C Feedback: Salamence, located in the midbrain, is responsible for temperature control. The patient will have difficulty distinguishing hot and cold. Centers of control for sleep and hearing are found in the hindbrain and area that controls speech and communication are found in the forebrain.

A patient who nearly drowned is brought to the emergency department. The paramedics told the nurse that the patient was anoxic for approximately 5 minutes because of this anoxia, what might happen to the nerve cells? A) The nerves might not be able to repolarize B) the nerves might not be able to depolarize C) the nerves might not be able to maintain the sodium potassium pump D) the nerves might not be able to maintain their action potential

Answer: C Feedback: if a person has anoxia or hypoglycemia, the nerves might not be able to maintain the sodium potassium pump, and with continued lack of oxygen and or glucose, the nerve cell will die.

A nurse is working on a surgical unit has several patients who require preoperative teaching. Which patient demonstrates Behavior indicating that this is an appropriate time to begin teaching? A) a patient who is wide-eyed and extremely frightened about being put to sleep B) a patient who appears to be unconcerned about what is happening and wants to watch his favorite TV show C) patient who is clearing her throat several times while asking the nurse questions during their conversation and who appears to be slightly stressed D) patient who is getting up and down from the bed, talking very fast, and appears to be extremely anxious

Answer: C Feedback: several substances appear to be affecting learning. Antidiuretic hormone (ADH), which is released during reaction to stress, is one such substance. Although too much stress prevents learning, feeling slightly stressed may increase a person's ability to learn. The patient was a little nervous about upcoming surgery, for example, seems to display a better Mastery of facts about the surgery and post-operative procedures than a patient who is very stressed and scared or one who appears to show no interest or concern

the nursing instructor is talking about neurotransmitters, including which of these chemicals? A) Calcium ion B) Acetylcholinesterase C) Acetylcholine D) monoamine oxidase

Answer: C Feedback: the cholinergic system uses acetylcholine as its neurotransmitter. A calcium ion is an electrolyte circulating in the serum. Acetylcholinesterase is an enzyme that breaks down acetylcholine. Monoamine oxidase is an enzyme that breaks down the neurotransmitter norepinephrine

the nursing instructor is teaching the new nursing students about patient teaching. where in the brain would the instructor tell the student nurses is the area we're learning takes place? A) area that coordinates sensation B) the area that coordinates movement C) the areas that coordinates speech and communicate D) the areas that communicate between motor and sensory neurons

Answer: C Feedback: the forebrain is made up of two cerebral hemispheres that contain areas that coordinate speech and communication and are thought to be the area where learning takes place

the physiology instructor is discussing the limbic system. What would the instructor say occurs with stimulation of this area? A) Intelligence B) heart rate C) Mood D) Reflexes

Answer: C Feedback: the limbic system is an area of the brain that contains high levels of three neurotransmitters: epinephrine, norepinephrine, and serotonin. Stimulation of this area appears to be responsible for the expression of emotions (e.g., anger, pleasure, motivation, stress).

a female patient has experienced a stroke affecting the right side of her brain. What will the nurse expect to assess in this patient? A) Inability to recall the name of her best friend B) inability to State her telephone number C) inability to distinguish a spoon from a fork D) inability to recall how to apply her makeup

Answer: C Feedback: the right side of the brain is an artistic side and is concerned with forms and shapes. This patient could have difficulty distinguishing the roundness of the spoon with the straight line of the top of the fork. The left side of the brain is more analytical and is concerned with names, numbers, and process.

the anatomy and physiology instructor discusses the thalamus with the nursing class. The instructor tells the students that the thalamus does what? A) Relays motor impulses from the cortex to the spinal cord B) is responsible for voluntary movement C) send information into the cerebrum to transfer sensation D) helps maintain red blood cell production

Answer: C Feedback: the thalamus sends direct information into the cerebrum to transfer Sunsations, such a , heat, pain, touch, and muscle sense. Motor fibers from the cortex cross to the other side of the spinal cord before emerging to interact with peripheral effectors. In this way, motor stimuli coming from the right side of the brain affect motor activity on the left side of the brain. The limbic system is not responsible for voluntary movement or red blood cell production.

a 46 year old male patient sustained a closed head injury 4 hours ago. He presents to the emergency department with difficulty breathing. What areas of the brain does the nurse suspect is injured based on the patient's symptoms? A) Thalamus B) Cerebrum C) Pituitary D) medulla oblongata

Answer: D Feedback: behind brain, which runs from the top of the spinal cord into the midbrain, is the most primitive area of the brain and contains the brain stem, where the pons and the medulla oblongata are located. This area of the brain controls basic vital functions such as the respiratory centers, which control breathing; the cardiovascular centers, which regulate blood pressure; the chemoreceptor trigger Zone and emetic Zone, which control vomiting; the swallowing Center, which coordinates the complex swallowing reflex; and the reticular activating system, which controls arousal and awareness of stimuli and contains Sleep Center. The midbrain contains the thalamus and hypothalamus and the limbic system that transfer Sensations into the cerebrum and control temperature. The pituitary gland is known as the master gland, controlling other glands with hormones secreted here

when a person learns, this action begins as an electrical circuit called what? A) impulse B) Synapse C) Memory D) Engram

Answer: D Feedback: learning begins as an electrical circuit called an engram, a reverberating Circuit of action potentials that eventually become a long-term, permanent memory in the presence of proper neurotransmitters and hormones

the nursing instructor explains the limbic system contains what neurotransmitters? A) Acetylcholine, epinephrine, and serotonin B) gamma-aminobutyric acid, dopamine, and serotonin C) epinephrine, dopamine, and Gamma Aminobutyric acid. D) Epinephrine, norepinephrine, and serotonin

Answer: D Feedback: limbic system contains high levels of epinephrine, norepinephrine, and serotonin. Dopamine, acetylcholine, and Gaba are found in the brain but not primarily in the limbic system. Options A, B, and C are incorrect

a student ask the anatomy and physiology and structure to explain the two hemispheres of the brain and what they regulate. What statement, if made by the instructor, is accurate? A) the hemispheres regulate the electrical conduction system of the brain B) the hemispheres regulate the afferent conduction system C) the hemispheres regulate the efferent conduction system D) hemispheres regulate communication between sensory and motor neurons

Answer: D Feedback: the cerebral cortex consists of two hemispheres, which regulate the communication between sensory and motor neurons and are the sites of thinking and learning.

what will the nurse describe to A peer as a factor in increasing synaptic transmission? A) Enzymes B) electrical impulses C) calcium reaction D) Neurotransmitter

Answer: D Feedback: the nerve axon, called the presynaptic nerve, releases a chemical called neurotransmitter into the synaptic cleft; the neurotransmitter reacts with a very specific receptor site on the postsynaptic south to cause a reaction that increases synaptic transmission. Enzymes break down the neurotransmitter. The synaptic transmission is an electrical impulse. Calcium is in electrolytes but does not increase an object transmission

what brings information from the central nervous system to the peripheral nervous system? A) motor nerves B) Synapses C) afferent neurons D) sensory nerves

Answer: D Feedback: the pns is composed of sensory receptors that bring information into the CNS and motor nerves that carry information away from the CNS to facilitate response to stimuli. Synapses are the gaps between neurons. afferent fibers are nerve axons that run from the peripheral receptors into the CNS

A nurse caring for a patient who is having adverse drug reaction. The patient is experiencing Tremors, is unable to hold his or her head up, and is having difficulty sitting up in bed. The nurse suspects that this is due to what? A) An interference with extrapyramidal system. B) a faulty engram C) an alteration in the reticular activating system D) an interference with a neurotransmitter

Answer; A Feedback: the extrapyramidal system coordinates unconscious motor activity that regulates control of possession imposter. And engram is a reverberating Circuit of action potentials that become a long-term, permanent memory in the presence of the proper neurotransmitters and hormones. The reticular activating system, which is located in the hindbrain to, controls arousal and awareness of stimuli and contains a Sleep Center. A neurotransmitter is a chemical that stimulates postsynaptic cells either by exciting or inhibiting them.

A patient has orders to receive 2 L of IV fluid over a 24-hour period with ½ this amount to be infused in the first 10 hours of treatment. How many milliliters per hour will the nurse administer during the first 10 hours of the infusion? A) 50 mL/h B) 100 mL/h C) 83 mL/h D) 200 mL/h

B

A patient is experiencing pain, so the physician orders codeine ½ grain every 4 hours. How many milligrams of codeine would the nurse administer? A) 15 mg B) 30 mg C) 60 mg D) 120 mg

B

An 80-year-old patient with internal bleeding is admitted through the emergency room after a motor vehicle accident. The physician has ordered 2 units of packed red blood cells (1 unit is 250 mL) to infuse over 1 hour each. The drip rate on the blood administration set is10 gtt/mL. The nurse administers how many drops per minute to infuse the blood as ordered? A) 47 B) 42 C) 37 D) 32

B

An adolescent is admitted to the intensive care unit with diabetic ketoacidosis. The nurse prepares a continuous insulin infusion of 100 units (U) regular insulin in 500 milligram normal saline. When documenting this medication, how many units of regular insulin will this patient receive per milligram of IV solution? A) 0.175 U/milligram B) 0.2 U/milligram C) 0.25 U/milligram D) 0.5 U/milligram

B

The patient returns from the postanesthesia care unit (PACU) with the following order: morphine 3 mg IV every 2 hours as needed for relief of pain. The vial reads morphine, 4 mg/mL. How many milliliters of morphine will the nurse administer? A) 1 mL B) 0.75 mL C) 0.5 mL D) 0.25 mL

B

A patient who works on road construction has been diagnosed with hypertension. After attempting to decrease his blood pressure with lifestyle changes and a mild diuretic, it is determined that he will need to be placed on an angiotensin-converting enzyme (ACE) inhibitor. Based on his occupation, what is the nurse's priority assessment? A) Chronic constipation B) Excessive sweating on the job C) Three large meals a day D) One beer every night

B A patient taking an ACE inhibitor should be sure to maintain fluid intake, so excessive sweating on the job places him at risk for a drop in fluid volume. Excessive sweating, vomiting, diarrhea, or dehydration need to be monitored and treated if they occur while taking an ACE inhibitor. Six smaller meals rather than three larger ones would be better and should be encouraged. However, this could be a problem with his job. The best treatment for constipation would be to increase fluid and fiber and one beer a night would be within reason when considering alcohol intake.

A patient with a migraine took a dose of a prescribed triptan, eletriptan (Relpax), and 1 hour later the headache is still intense. The patient's husband calls the clinic and asks the nurse what they should do. What is an appropriate nursing response? A) "Tell her to lie down in a quiet cool room and just wait it out. It will subside." B) "She can take another dose of the drug 2 hours after the initial dose if the headache continues." C) "Give her a dose of an ergot drug if you have it. It will decrease the intensity of the pain." D) "Ibuprofen may increase the action of the triptan."

B A patient taking eletriptan to relieve a migraine can take another dose in 2 hours if the headache is not relieved. The combination of ergot drugs with triptans is not indicated because of the vasoconstriction caused by both. The patient will not get relief by "waiting it out." Ibuprofen is an anti-inflammatory that does not affect the mechanism associated with migraines.

The nurse administers a narcotic analgesic to the postoperative patient. What is the best way for the nurse to evaluate response to the medication? A) Observe the patient without her awareness. B) Use a pain assessment tool before and 30 minutes after administration. C) Assess vital signs. D) Measure oxygen saturation.

B A standard pain assessment tool should be used both pre- and post-analgesia. Observing the patient when she is not aware you are watching, assessing vital signs, and measuring oxygen saturation may all contribute useful data but it would not be the best means of determining pain response following analgesic administration.

The pharmacology instructor is discussing diuretic drugs with the nursing class. What would the instructor cite as an adverse effect of loop diuretics? A) Hyperkalemia B) Alkalosis C) Hypertension D) Hypercalcemia

B Alkalosis is a drop in serum pH to an alkaline state due to bicarbonate loss in urine. Hypokalemia, hypocalcemia, and hypotension are also adverse effect of these drugs. Therefore, the other options are not correct.

The nurse on the coronary unit is caring for a patient with known coronary artery disease who is being treated with cholestyramine (Questran) and hydrochlorothiazide (HydroDIURIL). What action will the nurse take? A) Call the physician and refuse to give the drugs without further orders. B) Make sure that the drugs are given at least 2 hours apart. C) Give the patient an antacid with the drugs. D) Check the patient's blood glucose level before giving the drugs.

B Because of its effects in the GI tract, cholestyramine should be taken at least 2 hours earlier or later than hydrochlorothiazide to ensure the absorption of hydrochlorothiazide. This combination of drugs can be used effectively. An antacid would further aggravate drug absorption and would not be recommended. Blood glucose levels would not be affected by either of these drugs.

The nurse is providing drug teaching for a patient who is prescribed enalapril (Vasotec). What drug specific adverse effect will the nurse include in the drug teaching? A) Sedation B) Persistent cough C) Tachycardia D) Rash

B Benazepril, enalapril, and fosinopril are generally well tolerated but cause an unrelenting cough, possibly related to adverse effects in the lungs, where the angiotensin-converting enzyme is inhibited, which may lead patients to discontinue the drug. This persistent cough develops in approximately 10% to 20% of patients.

The nurse provides drug teaching to the patient prescribed captopril (Capoten). What statement made by the patient does the nurse interpret to mean teaching has been effective? A) "I will limit my fluid intake to 1,200 mL daily." B) "I will call my doctor if I bruise easily or become extremely tired." C) "I will move from a reclining to a standing position slowly." D) "I will increase my intake of foods high in potassium."

B Captopril has been associated with a sometimes-fatal pancytopenia, cough, and unpleasant gastrointestinal (GI) distress so the patient should be alert to symptoms related to anemia, reduction in platelets, or infection. There is no need to increase potassium intake because a slight rise in potassium level is associated with this drug. Orthostatic hypotension is not a listed adverse effect. There is no need to limit fluid intake for most patients unless they have a comorbid disorder.

When nonsteroidal anti-inflammatory drugs (NSAIDs) are combined with loop diuretics, there is a potential for what? A) Decreased antihypertensive effect B) Decreased diuretic effect C) Lithium toxicity D) Anaphylactoid reactions

B Diuretic effect is often decreased when NSAIDs are taken with loop diuretics. There is a potential for decreased antihypertensive effect of beta-blockers if NSAIDs are combined and there have also been reports of lithium toxicity, especially when lithium is combined with ibuprofen.

What is the term for the action of a diuretic in a patient with glaucoma? A) Intraocular pressure (IOP) B) Osmotic pull C) Diuresis D) Potassium sparing

B Glaucoma is an eye disease characterized by increased pressure in the eye—known as intraocular pressure (IOP)—which can cause optic nerve atrophy and blindness. Diuretics are used to provide osmotic pull to remove some of the fluid from the eye, which decreases IOP, or as adjunctive therapy to reduce fluid volume and pressure in the cardiovascular system, which also decreases pressure in the eye somewhat. Potassium sparing refers to a class of diuretics that help to retain potassium.

The nurse works with the patient, diagnosed with hypertension, and the patient's family to determine the goal of drug therapy for the patient taking an antihypertensive medication is what? A) Maintaining compliance B) Maintaining the blood pressure within normal limits C) Maintaining a fluid volume balance D) Maintaining homeostasis

B Helping the patient to maintain the blood pressure within normal limits is the goal of drug therapy. How blood pressure is maintained within normal limits may involve balancing fluid volume and patient compliance with the plan of care, but these are interventions and not the goal of therapy. Returning the patient to homeostasis comes before maintaining homeostasis.

A nurse is presenting an educational event for a group of new parents. One topic that the nurse addresses is the overuse of acetaminophen, which can cause liver toxicity. What would the nurse tell the parents it is important to do? A) "Do not give acetaminophen (Tylenol) unless you receive a doctor's order." B) "Check the label of over-the-counter (OTC) medications carefully to watch for inclusions of acetaminophen in the ingredients." C) "Monitor their child's temperature carefully and regulate the Tylenol dose based on the fever." D) "Mix OTC children's medications to get the best coverage for their child's symptoms."

B Inadvertent overdose with acetaminophen frequently occurs because of the combining of OTC drugs that contain the same ingredients. Parents should be taught to carefully check the labels of OTC products and follow the dosage guidelines. A prescription is not required for acetaminophen. Dosage guidelines are the best guide to follow to prevent overdose.

What medication used to treat rheumatic arthritis not only has anti-inflammatory effects but is also used in premature infants to close a patent ductus arteriosus? A) Penicillamine B) Indomethacin C) Antimalarials D) Prednisone

B Indomethacin given IV is used in premature infants to close a patent ductus arteriosus and avoid a surgical procedure. Penicillamine, antimalarials, and prednisone are not used for this purpose.

The nurse is caring for an obese child with hypertension and slightly elevated serum glucose levels. What would the nurse anticipate will be ordered if lifestyle changes do not return blood pressure to an acceptable limit? A) Lifestyle changes B) Mild diuretic C) Calcium channel blocker. D) Beta-blocker

B Lifestyle changes should be instituted before drug therapy if at all possible. If drug therapy is used, a mild diuretic may be tried first, with monitoring of blood glucose and electrolyte levels on a regular basis. Calcium channel blockers have been used to treat hypertension in children and may be among the first considerations if drug therapy other than mild diuretics is needed. Beta-blockers have been used with success in some children; adverse effects may limit their usefulness in others. The safety and efficacy of the angiotensin-converting-enzyme (ACE) inhibitors and the angiotensin-receptor blockers (ARBs) have not been established in children.

A nurse is caring for a patient in the early stage of rheumatoid arthritis. The nurse would expect what medication classification to be used in the treatment of this patient? A) Antimalarial agents B) Nonsteroidal anti-inflammatory drugs (NSAIDs) C) Xanthine oxidase inhibitors D) Uricosuric agents

B NSAIDs are indicated for relief of the signs and symptoms of rheumatoid arthritis and osteoarthritis, for relief of mild to moderate pain, for treatment of primary dysmenorrhea, and for fever reduction. Antimalarial agents are used in the treatment of systemic lupus erythematosus. Xanthine oxidase inhibitors and uricosuric agents are used in the treatment of gout.

The pediatric patient has a fever and the nurse is preparing to administer an antipyretic. What drug would be the best choice for this patient? A) Balsalazide (Colazal) B) Naproxen (Naprosyn) C) Indomethacin (Indocin) D) Aspirin

B Naproxen is approved for pediatric use and has antipyretic properties. Balsalazide is used to treat ulcerative colitis and would not be appropriate for treating a fever. Indomethacin has anti-inflammatory effects but does not have antipyretic effects. Aspirin would not be appropriate for treating a child with a fever of unknown origin due to risk of Reye's syndrome.

The nurse is providing discharge instructions to a 72-year-old patient who has been discharged home on a diuretic. What would the patient's instructions regarding the use of a diuretic at home include? A) Measuring intake and output of urine B) To weigh themselves on the same scale, at the same time of day, in the same clothing C) Restrict fluids to 500 mL/d to limit the need to urinate D) Decrease exercise to conserve energy

B Patients taking a diuretic at home need to learn to weigh themselves every day, at the same time, and in the same clothes to monitor for loss or retention of fluid. They should not be asked to measure urine output or to decrease activity. Restricting fluids can lead to a rebound fluid retention when compensatory mechanisms are activated.The emergency department (ED) nurse is caring for a patient who is experiencing pulmonary edema. The patient is treated with furosemide (Lasix). What will the nurse monitor? A) Sodium levels B) Bone narrow function C) Calcium levels D) Potassium levels

The nurse administers pentazocine cautiously to what population? A) Patients with known GI disease B) Patients with known heart disease C) Patients with known urinary disease D) Patients with known respiratory disease

B Pentazocine must be administered cautiously to patients with known heart disease because the drug may cause cardiac stimulation including arrhythmias, hypertension, and increased myocardial oxygen consumption, which could lead to angina, myocardial infarction, or congestive heart failure. No indication exists that it must be given cautiously to patients with gastrointestinal, urinary, or respiratory diseases.

The nurse is providing discharge instruction to a patient who has just begun using diuretics. The nurse counsels the patient that it is most important to monitor the intake of foods that contain which element? A) Calcium B) Potassium C) Glucose D) Magnesium

B Potassium is the most important element to monitor in the diet because diuretics are most likely to lead to hyper- or hypokalemia depending on the diuretic prescribed. Calcium, glucose, and magnesium may need to be monitored in the diet but potassium would be the most important.

By what route will the nurse administer methylnaltrexone (Relistor)? A) IV B) Subcutaneously C) Intranasally D) Orally

B Relistor is only given by subcutaneous injection once each day.

An older adult patient is taking a sustained-release antihypertensive drug. What is the nurse's priority teaching point about this medication? A) "Take your blood pressure only at night." B) "Swallow the drug whole and do not to cut, crush, or chew it." C) "Take the drug before bedtime." D) "Use over-the-counter (OTC) drugs to control headache or cold symptoms."

B Sustained-release drugs are suspended in a matrix system that allows a steady release of the drug over time. Sustained-release drugs cannot be cut, crushed, or chewed; it destroys the matrix system and allows absorption of the complete dose all at once. Older patients should be especially cautioned about sustained-release antihypertensives that cannot be cut, crushed, or chewed to avoid the potential for excessive dosing if these drugs are inappropriately cut. Many OTC drugs contain ingredients that increase blood pressure and so are not recommended for patients with hypertension. The patient can take his or her blood pressure any time during the day but should take the drug in the morning.

A nurse is assessing a patient who has been taking nonsteroidal anti-inflammatory drugs (NSAID). What statement by the patient indicates to the nurse that the patient has a good understanding of the use of this therapy? A) "I drink a glass of wine just about every night." B) "I asked my doctor to check for blood in my stool regularly." C) "I do not like to swallow tablets so I crush them." D) "I drink as little water as possible when I take my medication."

B Taking certain anti-inflammatory drugs can irritate the gastric mucosa and increase the risk of bleeding; therefore, by asking his or her doctor to check his or her stool for bleeding, the nurse knows that the patient is aware of this. Alcohol and crushing the tablets can interfere with anti-inflammatory metabolism. A full glass of water should be taken with this medication to increase absorption.

What drug is a safe and effective calcium channel blocker only if the nurse administers them as sustained-release or extended-release preparations to treat hypertension? A) Aliskiren (Tekturna) B) Diltiazem (Cardizem) C) Atenolol (Tenormin) D) Metoprolol (Lopressor)

B The calcium channel blockers available in immediate-release and sustained-release forms that are used in treating hypertension include amlodipine (Norvasc), felodipine (Plendil), isradipine (DynaCirc, DynaCirc CR), and nicardipine (Cardene, Cardene SR). Other calcium channel blockers are safe and effective for this use only if they are given as sustained-release or extended-release preparations. These include diltiazem (Cardizem, Dilacor CR), nifedipine (Procardia XL), nisoldipine (Sular), and verapamil (Calan SR). Aliskiren (Tekturna) is a renin inhibitor. Atenolol (Tenormin) and Metoprolol (Lopressor) are beta-blockers, not calcium channel blockers.

Before administering an ergot drug to the patient for the first time, the nurse would assess the patient's currently prescribed medications for what drug? A) Antidiabetic agents B) Beta adrenergic blockers C) Oral contraceptives D) Selective serotonin reuptake inhibitors (SSRIs)

B The concurrent use of beta blockers and ergot preparations increases the patient's risk for peripheral ischemia and gangrene. This combination should be avoided. There is no indication for concern with the use of antidiabetic agents, SSRIs, and oral contraceptives with these drugs.

A 91-year-old patient is being discharged on the diuretic spironolactone (Aldactone). What is the major adverse effect of this type of medication? A) Hypokalemia B) Hyperkalemia C) Gastric irritation D) Hypertension

B The most common adverse effect of potassium-sparing diuretics is hyperkalemia, which can cause lethargy, confusion, ataxia, muscle cramps, and cardiac arrhythmias. Hypokalemia, gastric irritation, and hypertension are not recognized as adverse effects of spironolactone.

The nurse has just administered 150 g of mannitol IV to a patient with increased intracranial pressure. What is most important for the nurse to monitor in the hour after administration? A) Weight of patient B) Blood pressure of patient C) Pulse of patient D) Respiratory rate of patient

B The most common and potentially dangerous adverse effect related to an osmotic diuretic is the sudden drop in fluid levels. Mannitol peaks 1 hour after administration, therefore, it would be most important to monitor blood pressure. Weight is the best indicator over time but would not be as effective in indicating a dangerous fluid drop as the blood pressure. Respiratory and pulse rates would also not be as effective as blood pressure in evaluating dangerous fluid drops.

A patient newly diagnosed with hypertension has just been given a prescription for medication. Along with promoting safety, what is the other goal of the nurse's teaching plan? A) A blood pressure of 120/80 B) Medication compliance C) A discussion with his insurance company about the cost of the drug D) Verbalization of why drugs should be kept out of the reach of children

B The nurse provides thorough patient teaching, including the name of the drug, dosage prescribed, measures to avoid adverse effects, warning signs of problems, and the need for periodic monitoring and evaluation, to enhance patient knowledge about drug therapy and to promote compliance. All of the options are realistic outcomes for this patient. However, compliance is a great concern for people who are in need of hypertensive agents. It would be most important for his or her health care provider to know that he or she has filled the prescription and is taking his or her medication as prescribed. Keeping the drug out of the reach of children would be a safety measure.

The nurse is caring for a patient who is receiving an opioid analgesic. What are the nurse's priority assessments? A) Pain intensity and blood glucose level B) Level of consciousness and respiratory rate C) Respiratory rate and electrolytes D) Urine output and pain intensity

B The nurse should assess respiratory rate and level of consciousness because respiratory depression and sedation are adverse effects of opioid analgesics. Blood glucose levels, electrolytes, and urine output are not priority assessments with opioid ingestion.

A nurse has admitted a 10-year-old child to the short-stay unit. The child has complained of chronic headaches and his or her mother reports that he or she gives him or her acetaminophen (Tylenol) at least twice a day. What will the nurse evaluate? A) Renal function B) Hepatic function C) Respiratory function D) Cardiac function

B The nurse should evaluate the patient's hepatic function. Severe hepatotoxicity can occur from overuse of acetaminophen. Significant interferences do not occur in the kidney, heart, or lung with acetaminophen.

The staff educator in the ICU is talking with a group of new nurses about osmotic diuretics. The educator would tell the new nurses that osmotic diuretics act on which site in the nephron? A) Proximal tubule B) Glomerulus C) Ascending limb of loop of Henle D) Collecting tubule

B These drugs are freely filtered at the renal glomerulus, poorly reabsorbed by the renal tubule, not secreted by the tubule, and resistant to metabolism. Therefore, options A, C, and D are not correct.

When the nurse learns that the patient with rheumatic arthritis is complaining of stomatitis, the nurse should further assess the patient for the adverse effects of what medication? A) Corticosteroids B) Gold-containing compounds C) Antimalarials D) Salicylate therapy

B Various adverse effects are common with the use of gold salts and are probably related to their deposition in the tissues and effects at that local level: stomatitis, glossitis, gingivitis, pharyngitis, laryngitis, colitis, diarrhea, and other GI inflammation; gold-related bronchitis and interstitial pneumonitis; bone marrow depression; vaginitis and nephrotic syndrome; dermatitis, pruritus, and exfoliative dermatitis; and allergic reactions ranging from flushing, fainting, and dizziness to anaphylactic shock. The disease-modifying antirheumatic drug (DMARD) category of antimalarials may cause visual changes, GI upset, rash, headaches, photosensitivity, and bleaching of hair. Tinnitus is associated with salicylate therapy. Hirsutism is associated with corticosteroid therapy.

A patient has been started on losartan (Cozaar), an angiotensin II-receptor blocker (ARB), for hypertension. After 6 weeks of therapy, it is decided that the losartan alone is not controlling the patient's hypertension. What does the nurse anticipate will be added to the losartan regimen for better control of this patient's hypertension? A) Candesartan (Atacand) B) Hydrochlorothiazide (Hyzaar) C) Captopril (Capoten) D) Antidiuretic hormone (ADH)

B When losartan therapy is started, maximal effects on blood pressure usually occur within 3 to 6 weeks. If losartan alone does not control blood pressure, a low dose of a diuretic may be added. A combination product of losartan and hydrochlorothiazide (Hyzaar) is available. Adding a second ARB such as candesartan or captopril would risk causing toxic adverse effects. ADH causes retention of water in the nephrons, which would further increase blood pressure.

The nurse teaches a young mother the importance of administering appropriate dosages of acetaminophen (Tylenol) and determines further teaching is needed when the mother makes what statement? A) "The children's dosage will change with time as they grow." B) "My baby's dose of Tylenol is about 1 half an adult dose." C) "It is important to give the right dose to prevent toxic effects of the medication." D) "My children's dose of Tylenol should be based on their weight or age."

B A child's dose is never based on an adult's dose. A child's dosage is based on weight and age and will change with age as they grow. Larger than directed dosages can result in toxic effects of this medication.

A newly admitted patient has orders to receive 1,000 mL of normal saline IV over 8 hours. If the IV infusion set is a microdrip set that delivers 60 drops per mL, how many drops per minute should the nurse administer to the patient? A) 60 drops/min B) 125 drops/min C) 240 drops/min D) 480 drops/min

B If a patient was to receive 1,000 mL in 8 hours, dividing 1000 by 8 would mean that the patient would receive 125 mL in 1 hour, or 60 minutes. Setting up the equation, 60 drops/mL ÷ X = 125 mL/60 minutes; cross-multiplying, the answer is 125 drops/min.

The nurse is teaching a diabetic patient to self-administer Humulin insulin, supplied in a vial labeled 100 units/mL. The provider has ordered 32 units of Humulin insulin to be taken each morning. How many mL of insulin would the patient prepare for one dose? A) 0.032 mL B) 0.32 mL C) 3.2 mL D) 0.64 mL

B There are 100 units in each mL. Divide that amount by 32 units for the answer (0.32 mL)

The nurse in the clinic is caring for a patient who has seasonal rhinitis and the patient wants to know what causes this to occur. What is the nurse's best response? A) "You are experiencing symptoms because bacteria have entered the nose and caused an infection." B) "Your upper airways are experiencing an inflammatory response to an inhaled antigen that is causing you to have sneezing and watery eyes." C) "Your sympathetic nervous system is responding to an acute amount of stress in your life causing you to have nasal congestion." D) "Your runny nose and sneezing have occurred because a portion of your lung has collapsed."

B) "Your upper airways are experiencing an inflammatory response to an inhaled antigen that is causing you to have sneezing and watery eyes." Feedback: Seasonal rhinitis usually occurs when the upper airways become inflamed because of the bodys response to an inhaled antigen. The lungs do not collapse and the sympathetic systems response to stress usually opens the airways and does not cause inflammation. Bacteria entering the nose do not cause inflammation of the rest of the upper airways.

A nurse in the postanesthesia care unit is caring for a 77-year-old male patient after hip replacement surgery. While assessing the patient, the nurse notes crackles, dyspnea, cough, and changes in movement of the chest wall. The nurse would suspect the patient has developed what? A) Pneumonia B) Atelectasis C) Bronchitis D) Emphysema

B) Atelectasis Feedback: Patients may present with crackles, dyspnea, fever, cough, hypoxia, and changes in chest wall movement. Treatment may involve clearing the airways, delivering oxygen, and assisting ventilation. In the case of pneumothorax, treatment would also involve insertion of a chest tube to restore the negative pressure to the space between the pleura. These signs and symptoms are not the presentation of pneumonia, bronchitis, or emphysema.

The nursing instructor is discussing cystic fibrosis (CF) with his clinical group. What would the instructor cite as the hallmark pathology of CF? A) Alveolar mucous plugging, infection, and eventual bronchiectasis B) Bronchial mucous plugging, inflammation, and tissue damage C) Atelectasis, infection, and eventual chronic obstructive pulmonary disease (COPD) D) Bronchial mucous plugging, infection, and eventual chronic obstructive pulmonary disease (COPD)

B) Bronchial mucous plugging, inflammation, and tissue damage Feedback: CF results in the accumulation of copious amounts of very thick secretions in the lungs. Eventually, the secretions obstruct the airways, leading to destruction of the lung tissue. Mucous plugs occur in CF, but it is at the bronchial level, not the alveolar level. COPD is not an eventual outcome in this disease.

The nurse is developing the teaching portion of a care plan for a patient with asthma. What would be an important component for the nurse to emphasize? A) Smoking a half a pack of cigarettes weekly is allowable. B) Chronic inhalation of nonallergic inhaled irritants can trigger an attack. C) Minor respiratory infections are not treated. D) Activities of daily living (ADLs) should be completed in the waking hours.

B) Chronic inhalation of nonallergic inhaled irritants can trigger an attack. Feedback: Asthma is characterized by reversible bronchospasm, inflammation, and hyperactive airways. The hyperactivity is triggered by allergens or nonallergic inhaled irritants or by factors such as exercise and emotions. The trigger causes an immediate release of histamine, which results in bronchospasm in about 10 minutes. The later response (i.e., 3 to 5 hours) is cytokine-mediated inflammation, mucus production, and edema contributing to obstruction. Patients with asthma should not smoke at all and even minor respiratory infections should be treated to prevent an exacerbation of asthma and ADLs should be completed whenever the patient feels able.

A nurse is discussing ways the body protects itself against infection. What are the microscopic, hair-like projections of the cell membrane found in the nasal cavity, which transport foreign substances toward the throat where it can be swallowed and kept away from the respiratory system? A) Goblet cells B) Cilia C) Alveolar sacs D) Sinuses

B) Cilia Feedback: Cilia are found in the epithelial cells of the lining of the nasal cavity and are constantly in motion directing mucus and trapped substances down toward the throat. Goblet cells are found in the epithelial lining and produce mucus, which traps foreign substances. Alveolar sacs are located in the lower respiratory tract and are considered the functional units of the lung. Sinuses are air-filled passages through the skull, which open into the nasal cavity.

The nurse is admitting a patient with an obstructive respiratory disorder. The nurse knows this includes which disorders? (Select all that apply.) A) Atelectasis B) Cystic fibrosis C) Asthma D) Pneumonia E) Bronchiectasis

B) Cystic fibrosis C) Asthma Feedback: Obstructive disorders of the lower respiratory tract include asthma, chronic obstructive pulmonary disease (COPD), cystic fibrosis, and respiratory distress syndrome (RDS). Atelectasis is a collapse of once-expanded alveoli. Pneumonia is an infection of the lower respiratory tract. Bronchiectasis is a disorder of chronic infection and inflammation of the bronchial passages.

While discussing gas exchange, the instructor would tell the students that oxygen and carbon dioxide enter and leave the body by what method? A) Osmosis B) Diffusion C) Passive transport D) Active transport

B) Diffusion Feedback: The alveolar sac holds the gas, allowing needed oxygen to diffuse across the respiratory membrane into the the capillary, whereas carbon dioxide, which is more abundant in the capillary blood, diffuses across the membrane and enters the alveolar sac to be expired.

A patient is admitted with an asthma attack caused by an allergic reaction to a medication. The nurse is aware that this severe allergic response is triggered by the immediate release of what? A) Antihistamine B) Histamine C) Epinephrine D) Surfactant

B) Histamine Feedback: Asthma is characterized by reversible bronchospasm, inflammation, and hyperactive airways. The hyperactivity is triggered by allergens or nonallergic inhaled irritants or by factors such as exercise and emotions. The trigger causes an immediate release of histamine, which results in bronchospasm in about 10 minutes. An antihistamine is used to treat allergic responses because it counteracts the effects of histamine. Surfactant is a lubricating substance that is necessary to keep the alveoli open. Epinephrine is a medication used to treat acute allergic responses.

A nurse is caring for an 80-year-old patient with pneumonia. The most appropriate nursing diagnosis for this patient would be what? A) Dysfunctional ventilatory weaning response B) Impaired gas exchange C) Ineffective health maintenance D) Risk for delayed development

B) Impaired gas exchange Feedback: Pneumonia causes swelling, engorgement, and exudation of protective sera in the lower respiratory tract. The respiratory membrane is affected, resulting in decreased gas exchange. There is no indication that the patient has been on a ventilator so a diagnosis that concerns weaning is not appropriate. There is also no indication that this patient has pneumonia because of ineffective health maintenance and because the patient is 80 years old, a diagnosis of risk for delayed development is also inappropriate.

A patient is complaining of an inability to breathe nasally because of severe rhinitis. The nurse is aware that the nose plays what important role in breathing that is disrupted when the nasal passages are blocked? A) It decreases the number of pathogens inhaled. B) Inspired air is warmed and humidified. C) Carbon dioxide will not be inhaled. D) It simulates surfactant release from the alveoli.

B) Inspired air is warmed and humidified. Feedback: Air usually moves into the body through the nose and into the nasal cavity. The nasal hairs catch and filter foreign substances that may be present in the inhaled air. The air is warmed and humidified as it passes by blood vessels close to the surface of the epithelial lining in the nasal cavity. The epithelial lining contains goblet cells that produce mucus. This mucus traps dust, microorganisms, pollen, and any other foreign substances. The epithelial cells of the lining also contain ciliamicroscopic, hair-like projections of the cell membranewhich are constantly moving and directing the mucus and any trapped substances down toward the throat (Figure 53.2). The action of the goblet cells and cilia is commonly called the mucociliary escalator. It does not affect carbon dioxide or pathogen inhalation and does not stimulate surfactant release.

A 39-year-old teacher with bronchitis has been up all night with intense coughing spasms and asks the nurse "How is all this coughing related to my bronchitis?" The nurse tells the patient that a cough is initiated by what? A) Irritation to receptors in the nasal cavity B) Irritation to receptors in the bronchi C) Irritation to receptors in the pharynx D) Irritation to receptors in the sinus cavities

B) Irritation to receptors in the bronchi Feedback: Irritation to bronchial receptors will initiate a cough, which causes air to be pushed through the bronchial tree. Irritation to receptors in the nasal cavity, pharynx, and sinuses are more likely to initiate the sneeze reflex.

Why do COX-2 inhibitors increase the risk for cardiovascular problems? (Select all that apply.) A) Vasoconstriction is blocked. B) Vasodilation is blocked. C) Platelet clumping is blocked. D) Water and sodium balance is altered. E) Gastrointestinal (GI) integrity is altered.

B, C Recent studies suggest that COX-2 inhibitors may block some protective responses in the body, such as vasodilation and inhibited platelet clumping, which is protective if vessel narrowing or blockage occurs. Blocking this effect could lead to cardiovascular problems. Vasoconstriction is not blocked, water and sodium balance is not altered, and GI integrity is not impacted by COX-2 inhibitors but can be impacted by COX-1 inhibitors.

A nursing instructor is teaching a group of students about loop diuretics. Which would be included in this classification? (Select all that apply.) A) Acetazolamide B) Torsemide C) Ethacrynic acid D) Mannitol E) Bumetanide

B, C, E Torsemide, ethacrynic acid, and bumetanide are examples of loop diuretics. Acetazolamide is a carbonic anhydrous inhibitor and mannitol is an osmotic diuretic.

A patient has been prescribed hydrochlorothiazide (HydroDIURIL) and the nurse is preparing to give the patient discharge instructions. Which adverse effects may this patient experience while taking this medication? (Select all that apply.) A) Constipation B) Dizziness C) Polyphagia D) Nocturia E) Muscle cramps

B, D, E The adverse effects associated with hydrochlorothiazide are dizziness, vertigo, orthostatic hypotension, nausea, anorexia, vomiting, dry mouth, diarrhea, polyuria, nocturia, muscle cramps, and spasms. The patient would not experience polyphagia (great hunger) and constipation.

A 79-year-old female patient presents at the clinic complaining of constipation for 1 week. The nurse practitioner prescribes Milk of Magnesia 2 teaspoons by mouth as needed for relief of constipation. How many milliliter will the nurse administer? A) 30 mL B) 7.5 mL C) 10 mL D) 15 mL

C

A patient is going to have bowel surgery in the morning. The physician orders 500 mL of GoLytely PO to be administered at 5 PM this evening. How many liters will the nurse administer? A) 1 B) ¾ C) ½ D) ¼

C

A patient with an acute myocardial infarction is admitted to the coronary care unit. The physician has ordered heparin 25,000 units in 250 mL normal saline to infuse at a rate of 600 units/h. The nurse sets the infusion pump to deliver how many milliliters in an hour? A) 8 B) 7 C) 6 D) 5

C

A patient with diabetic ketoacidosis is to receive a continuous infusion of regular insulin. The physician orders 1 L of 5% dextrose and water solution to run at 150 mL/h once the patient's blood glucose has reached 250 mg/dL. The drip factor of the tubing is 15 gtt/mL. How many drops per minute will the nurse deliver? A) 0.25 gtt/min B) 62 gtt/min C) 37 gtt/min D) 250 gtt/min

C

The patient drinks 18 ounces of fluid at lunchtime. How many milliliters of intake will the nurse document? A) 1.7 mL B) 0.6 mL C) 540 mL D) 54 mL

C

The physician has ordered 30 mg of Demerol IM for relief of a severe migraine headache. The package insert reads meperidine hydrochloride (Demerol) 50 mg/mL. How many milliliters would the nurse administer? A) 1.6 B) 1 C) 0.6 D) 0.5

C

The physician prescribes 250 mg of a drug. The information on the drug vial says the concentration is 500 mg/mL. How much of the drug will the nurse prepare? A) 0.25 mL B) 0.33 mL C) 0.5 mL D) 0.75 mL

C

What order for naloxone would be appropriate for the nurse to administer for reversal of opioid effects? A) 1 mg IV repeat every 2 to 3 minutes B) 5 mg IV repeat every 5 minutes C) 0.1 mg IV repeat every 2 to 3 minutes D) 0.4 mg IV repeat every 3 minutes

C 0.1 to 0.2 mg is given IV and then repeated every 2 to 3 minutes for reversal of opioid effects. If the patient has overdosed on opioids the dose would be 0.4 to 2 mg every 2 to 3 minutes. The other options are incorrect.

According to the Gate Control Theory, what interventions by the nurse could help to block pain impulses? A) Administration of opioid medications B) Administration of narcotic agonist-antagonists C) Back massage D) Acupuncture

C According to the gate control theory, the transmission of these impulses can be modulated or adjusted all along these tracts. All along the spinal cord, interneurons can act as "gates" by blocking the ascending transmission of pain impulses. It is thought that the gates can be closed by stimulation of the larger A fibers and by descending impulses coming down the spinal cord from higher levels in such areas as the cerebral cortex, the limbic system, and the reticular activating system. Administration of medications does not use the Gate Control Theory. Acupuncture uses the Gate Control Theory but is not performed by the nurse.

A mother asks the nurse how acetaminophen works. What statement best describes the therapeutic action of acetaminophen? A) Acetaminophen (Tylenol) works by blocking the increase of interleukin-1. B) Acetaminophen reacts with free-floating tumor necrosis (TNF) factor released by active leukocytes. C) Acetaminophen acts directly on the hypothalamus to cause vasodilation and sweating. D) Acetaminophen is taken up by macrophages, thus inhibiting phagocytosis and release of lysosomal enzymes.

C Acetaminophen acts on the hypothalamus to cause vasodilation and sweating to reduce fever. The mechanism of action as an analgesic is not understood. Anakinra (Kineret) blocks the increased interleukin-1, which is responsible for the degradation of cartilage in rheumatoid arthritis. Etanercept (Enbrel) reacts with free-floating TNF released by active leukocytes in autoimmune inflammatory disease to prevent the damage caused by TNF. Gold compounds are taken up by macrophages, which, in turn, inhibits phagocytosis and releases lysosomal enzymes, which causes damage associated with inflammation.

A mother has brought her 6-year-old child to the clinic. The child has a fever of 102.8ºF and is diagnosed with the flu. What medication will the nurse suggest for this child? A) Etanercept (Enbrel) B) Penicillamine (Depen) C) Acetaminophen (Tylenol) D) Aspirin (Bayer)

C Acetaminophen would be the suggested medication. It is prescribed for relief of pain and fever for influenza in children. Aspirin would be contraindicated because it increases the risk for Reye's syndrome. Etanercept and penicillamine are given for severe rheumatoid arthritis therapy.

A mother brings her 3-year-old child to the emergency department telling the nurse the child has eaten a bottle of baby aspirin. The mother cannot tell the nurse how many tablets were in the bottle. What dose of salicylate would be toxic in a child? A) 2 g B) 3 g C) 4 g D) 5 g

C Acute salicylate toxicity may occur at doses of 20 to 25 g in adults or 4 g in children. Therefore, options A, B, and D are incorrect.

The nurse administers ambrisentan (Letairis) to the patient diagnosed with pulmonary arterial hypertension. What single indicator would the nurse use to evaluate the effectiveness of this medication? A) Oxygen saturation B) Resting respiratory rate C) Exercise tolerance D) Breath sounds

C Although it is certainly important to assess all aspects of oxygenation and ventilation, the single best indicator of drug effectiveness is improved exercise tolerance. Many patients can oxygenate at rest and maintain a normal resting respiration, but activity increases oxygen demand, which is when signs of pulmonary hypertension are best seen.

The nurse is caring for a patient who is taking acetazolamide (Diamox) for treatment of glaucoma. What drug, if taken with acetazolamide (Diamox), would cause the nurse to contact the physician? A) Indomethacin (Indocin) B) Colestipol (Colestid) C) Lithium (Eskalith) D) Ibuprofen (Motrin)

C An increase in the excretion of lithium can occur when taken with acetazolamide, so that special monitoring or a dosage adjustment may be necessary. Indomethacin, colestipol, and ibuprofen do not produce drug-to-drug interactions when given with acetazolamide.

The clinic nurse assesses a patient taking benazepril (Lotensin) to control hypertension. What is the priority nursing assessment related to this drug? A) Mental illness B) Hepatic disease C) Renal disease D) Peptic ulcer disease

C Benazepril is an angiotensin-converting enzyme inhibitor; drugs in this class are contraindicated in the presence of impaired renal function. Mental illness, hepatic disease, or peptic ulcer disease is not a contraindication with this drug.

A patient taking diltiazem (Cardizem) for hypertension has come to the clinic for a follow-up appointment. What adverse effects would the nurse assess the patient for? A) Chest pain and pale skin B) Shortness of breath and wheezing C) Peripheral edema and bradycardia D) Tachycardia and increased energy level

C Cardiovascular adverse effects of diltiazem include bradycardia, peripheral edema, and hypotension. Skin flushing and rash may occur. There should be no effect on the lungs and usually this drug causes fatigue rather than increased energy levels.

A patient, 6 days postoperative, is being weaned off an opioid analgesic. The patient reports he is getting no relief from the pain with the new non-opioid medication he is receiving. What might the nurse suspect is causing this patient's pain? A) The patient needs a higher dose of the opioid analgesic. B) The patient has become addicted to the opioid medication. C) The patient has developed withdrawal syndrome. D) The patient has developed a cross-hypersensitive reaction.

C Caution should be used in cases of physical dependence on a narcotic because a withdrawal syndrome may be precipitated, the narcotic antagonistic properties can block the analgesic effect, and so intensify the pain. It is important to differentiate between addiction and dependence because addiction generally does not occur in patients receiving narcotics for medical reasons. There is no indication of a hypersensitivity reaction. Giving a higher dose of the opioid would eliminate the progress made to date on weaning the patient from the narcotic, so attempts should be made to avoid this intervention.

Before administering an opiate medication, what will the nurse assess? A) The patient's weight B) The patient's heart rate C) The patient's respiratory rate D) The patient's drug tolerance

C Check the rate, depth, and rhythm of respirations before each dose. If the patient's heart rate is slower than 12 beats per minute, delay or omit the dose and report to the physician. Weight would be assessed before determining dosage. Heart rate would not be an essential assessment before administration. Drug tolerance is assessed by monitoring patient's response to the medication and could not be assessed before administration.

The nurse provides dietary teaching to the patient with hypertension and determines the patient understood the information when what meal is selected? A) Hot dogs, baked beans, and cole slaw B) French fries, grilled hamburger, and cola drink C) Grilled chicken, green salad with dressing, and baked apple D) Bologna sandwich with mayonnaise, potato chips, and a chocolate-chip cookie

C Chicken, salad, and fruit are all low in sodium. The other meal options all contain foods high in sodium (i.e., hot dogs, French fries, processed meats like bologna, and potato chips).

The nurse is caring for a patient with hypertension who is preparing to be discharged from the hospital after suffering a myocardial infarction. What drug might the nurse administer that will both treat his hypertension and reduce myocardial oxygen consumption? A) Captopril B) Losartan C) Diltiazem D) Nitroprusside

C Diltiazem inhibits the movement of calcium ions across the membranes of cardiac and arterial muscle cells, depressing the impulse and leading to slowed conduction, decreased myocardial contractility, and dilation of arterioles, which lowers blood pressure and decreases myocardial oxygen consumption. Captopril, losartan, and nitroprusside do not have actions to reduce myocardial oxygen consumption.

The nurse is teaching the patient, who has been newly prescribed etanercept (Enbrel), how to administer the medication. What statement is accurate? A) "Be sure to drink a whole glass of water when swallowing the pill." B) "Do not take this medication for at least 1 hour after taking an antacid." C) "You can use each of the subcutaneous injection sites to avoid tissue damage." D) "Inject this medication deeply into the muscle to promote absorption."

C Etanercept is given by injecting it into the subcutaneous tissues. The injection sites should be rotated to avoid tissue damage. Because it is not taken orally, there is no requirement related to amount of water to be taken or waiting an hour after taking an antacid. Etanercept is not injected into the muscle but rather into the subcutaneous tissue.

The nurse is caring for a 66-pound child with orders for choline magnesium trisalicylate (Tricosal). The orders read 50 mg/kg/d PO in two divided doses. How many milligram will the patient receive per dose? A) 250 mg B) 500 mg C) 750 mg D) 1,000 mg

C First, the nurse must determine the child's weight in kilogram. One kg is equal to 2.2 pounds. Divide 2.2 into 66 to equal 30 kg. Multiply 50 mg times 30 kg to equal 1,500 mg. Divide 1,500 by 2 for the divided doses, which will equal 750 mg per dose.

What medication would the nurse administer to the patient in severe pain? A) Codeine B) Hydrocodone C) Hydromorphone D) Opium

C Hydromorphone is indicated for moderate-to-severe pain. Codeine is indicated for mild-to-moderate pain, hydrocodone is indicated for moderate pain, and opium is indicated for treatment of diarrhea and relief of moderate pain.

What is the nurse's priority to assess before giving a female patient her prescription for an angiotensin II-receptor blocker (ARB)? A) "Do you eat something when you take your medications?" B) "How much physical exercise do you get?" C) "When was your last menstrual period (LMP)?" D) "Have you always weighed 130 pounds since you grew up?"

C It would be important to know when the patient's LMP occurred and that the patient was not pregnant. These drugs can cause fetal abnormalities and fetal death. The other questions are appropriate and would help the nurse plan care for the patient; however, it would not be as important as assessing for the possibility of pregnancy before beginning of therapy. The nurse should teach the patient the need to avoid pregnancy using a barrier contraceptive.

The nurse is caring for a patient with a severe head injury. An osmotic diuretic is ordered. The nurse understands which drug is an osmotic diuretic? A) Spironolactone (Aldactone) B) Bumetanide (Bumex) C) Mannitol (Osmitrol) D) Ethacrynic (Edecrin)

C Mannitol is an osmotic diuretic. Spironolactone is a potassium sparing diuretic. Bumetanide and ethacrynic are loop diuretics.

A patient has just been prescribed furosemide (Lasix). After reviewing the patient's medication history, what drug would cause the nurse concern when taken with furosemide (Lasix)? A) Acetaminophen B) Ferrous sulfate (Feosol) C) Naproxen sodium (Naprosyn) D) Ampicillin

C Naproxen sodium is a nonsteroidal antiinflammatory drug. There may also be a decreased loss of sodium and decreased antihypertensive effects if these drugs are combined with indomethacin, ibuprofen, salicylates, or other nonsteroidal antiinflammatory drugs. The patient receiving this combination should be monitored closely and appropriate dosage adjustments should be made. There is no contraindication to the intercurrent use of acetaminophen, ferrous sulfate, or ampicillin.

A nurse is caring for a patient with severe rheumatoid arthritis who takes anti-inflammatory agents on a regular basis. What medication should the nurse question if ordered by the physician to be taken in addition to the anti-inflammatory agent? A) Oral antidiabetic agent B) Calcium channel blocker C) Beta-blocker D) Antibiotic

C Nonsteroidal anti-inflammatory drugs have the potential to decrease antihypertensive effects from beta blockers if these drugs are taken at the same time. Patients who receive these combinations should be monitored closely and appropriate dosage adjustments made if needed. Drug interactions do not usually occur with oral antidiabetic agents, calcium channel blocking medications, or antibiotics.

The nurse is providing patient teaching about a prescribed opioid analgesic. What is an important teaching point related to a possible adverse effect of this drug? A) Ataxia B) Blurred vision C) Hypotension D) Dysrhythmias

C Orthostatic hypotension is commonly seen in association with some narcotics. Ataxia, blurred vision, and dysrhythmias are not commonly seen adverse effects of an opioid analgesic.

A patient presents at the emergency department complaining of dizziness, mental confusion, and difficulty hearing. What should the nurse suspect is wrong with the patient? A) Anakinra toxicity B) Ibuprofen toxicity C) Salicylism D) Acetaminophen toxicity

C Salicylism can occur with high dosage of aspirin. Dizziness, ringing in the ears, difficulty hearing, nausea, vomiting, diarrhea, mental confusion, and lassitude can occur. This combination of adverse effects is not associated with anakinra toxicity, ibuprofen toxicity, or acetaminophen toxicity.

A patient taking a calcium channel blocker is seen in the clinic and diagnosed with drug toxicity. When collecting the nursing history, what finding would indicate the likely cause of this drug toxicity? A) Intake of alcohol B) The use of eggs in the diet C) The ingestion of grapefruit juice D) Intake of aged cheese

C The calcium channel blockers are a class of drugs that interact with grapefruit juice. When grapefruit juice is present in the body, the concentrations of calcium channel blockers increase, sometimes to toxic levels. Advise patients to avoid drinking grapefruit juice taking a calcium channel blocker. If a patient on a calcium channel blocker reports toxic effects, ask whether he or she is drinking grapefruit juice. Use of alcohol could be important if the patient was ingesting large amounts, but that would not be the most likely cause of drug toxicity. Eggs and cheese should not exert any food-drug interaction.

The nurse is caring for a patient who has just been diagnosed with essential hypertension. The nurse is aware that the health care provider will begin therapy with which classification of diuretics? A) Loop diuretics B) Carbonic anhydrous inhibitors C) Thiazide and thiazide-like diuretics D) potassium-sparing diuretics

C Thiazides are considered to be mild diuretics compared with the more potent loop diuretics. These agents are the first-line drugs used to manage essential hypertension when drug therapy is needed. Loop and potassium-sparing diuretics and carbonic anhydrous inhibitors would be used in combination with or after the thiazide diuretics are no longer effective.

A patient with migraine headaches is changed from an ergot to a prescription for a triptan. The nurse has completed teaching related to the drug. What statement would indicate she has a clear understanding of the new drug? A) "My life is over. I can't function not knowing when I'm going to have a headache." B) "I will not have to avoid driving because this medication isn't sedating." C) "I should not experience as many adverse effects from my new medication." D) "I take my medication every hour when I have a headache."

C Triptans are a new class of selective serotonin receptor blockers that cause vasoconstriction; they are not associated with as many systemic adverse effects experienced in ergot therapy. Triptan therapy will enable her to live a near normal life even during headaches. Although adverse effects are fewer than those associated with ergot therapy, triptans can still cause dizziness, feelings of strangeness, and vertigo, so the patient should not drive while taking the drug. Medications are often only taken once due to prolonged half-lives, but some may be repeated in 2 to 4 hours if the headache does not subside.

A nurse calculates the pediatric patient's medication dosage using Clark's rule and uses what formula? A) Infant's age in months/150 months times the average adult dose B) Child's age in years/child's age in years plus 12 times the average adult dose C) Weight of child in pounds/150 pounds times the average adult dose D) Surface area in square meters/1.73 times the average adult dose

C Clark's rule uses the child's weight to calculate the dose and assumes the adult dose is based on a 150-pound person. Fried's rule applies to a child younger than 1 year of age and assumes that an adult dose would be appropriate for a child who is 12.5 years (150 months) old. Young's rule applies to children 1 to 12 years of age. Surface area calculation of a child's dose is determined with the use of a nomogram including the child's height and weight.

The nurse is preparing to administer cefadroxil 1 g PO. The medication is supplied in 500-mg tablets. How many tablets will the nurse administer? A) 0.5 tablet B) 1 tablet C) 2 tablets D) 3 tablets

C Convert 1 g to mg by multiplying 1 g times 1,000 mg. There are 500-mg in each tablet. Dividing the 1000 mg prescribed dosage by 500-mg available dosage, the answer is two tablets

A student asks the pharmacy instructor where air exchange takes place in the human body. What would be the instructor's best response? A) "Air exchange in the human body takes place in the trachea." B) "Air exchange in the human body takes place in the bronchioles." C) "Air exchange in the human body takes place in the alveoli." D) "Air exchange in the human body takes place in the nares."

C) "Air exchange in the human body takes place in the alveoli." Feedback: Gas exchange occurs across the respiratory membrane in the alveolar sac. It does not occur in the bronchioles, the trachea, or the nares.

A patient arrives in the emergency room with an attack of acute bronchiectasis. The nurse knows that the principal pathologic finding in the diagnosis of bronchiectasis includes what? A) Increase in the red blood cell concentration in the blood B) Leakage of fluid into the alveolar interstitial spaces C) Chronic, irreversible dilation of the bronchi and bronchioles D) Obstruction of the pulmonary vasculature by a clot

C) Chronic, irreversible dilation of the bronchi and bronchioles Feedback: Bronchiectasis is a chronic disease that involves the bronchi and bronchioles. It is characterized by dilation of the bronchial tree, chronic infection, and inflammation of the bronchial passages. It is not caused by increased red blood cell concentration in the blood, leakage of fluid into the alveolar interstitial spaces, or the obstruction of the pulmonary vasculature by a clot.

A nurse is discussing cystic fibrosis (CF) with a couple who have just given birth to an infant with this disorder. The nurse explains that the respiratory component of this disease is caused by what? A) Bronchospasm B) Infection C) Excessive respiratory tract secretions D) Chemical irritation of the respiratory tract

C) Excessive respiratory tract secretions Feedback: CF is a hereditary disease involving the exocrine glands of the respiratory, gastrointestinal, and reproductive tracts. CF results in the accumulation of copious amounts of very thick secretions in the lungs. CF is not caused by bronchospasm or chemical irritation. Patients with cystic fibrosis develop many lung infections but that is secondary to the disease, not the underlying disease process itself.

The nursing instructor is discussing acute respiratory distress syndrome (ARDS) with her clinical group. What signs and symptoms would the instructor present to the students as characteristic of ARDS? A) Anoxia B) Hypocapnia C) Hypoxia D) Hypercapnia

C) Hypoxia Feedback: Acute respiratory distress syndrome (ARDS) is characterized by progressive loss of lung compliance and increasing hypoxia. Anoxia is a lack of oxygen to the body. ARDS will cause decreased oxygen (hypoxia). ARDS does not cause hypocapnia or hypercapnia, which is too little or too much carbon dioxide, respectively.

The nurse is caring for a patient who has just been admitted with atelectasis and anticipates which possible treatments for this patient? (Select all that apply.) A) Chest tube B) Surgical removal of the affected lung C) Oxygen delivery D) Assisted ventilation E) Antianxiety medication

C) Oxygen Delivery D) Assisted ventilation Feedback: Treatments for atelectasis include airway clearance (e.g., postural drainage and suctioning), oxygen delivery, and assisting ventilation. Patients with atelectasis may feel some anxiety if the patients oxygen level has lowered, but it is not a treatment for atelectasis. Removal of the portion of the lung that is affected is not an effective treatment.

The nurse is teaching a class about common upper respiratory infections to parents of preschoolers. The parents correctly respond that which are examples of upper respiratory infections? (Select all that apply.) A) Bronchitis B) Asthma C) Pharyngitis D) Sinusitis E) Laryngitis

C) Pharyngitis D) Sinusitis E) Laryngitis Feedback: Upper respiratory infections include pharyngitis, sinusitis, and laryngitis. Asthma and bronchitis are examples of lower respiratory disorders.

A nurse is caring for a 45-year-old patient with acute respiratory distress syndrome (ARDS). The nurse is aware that this disease is characterized by what? A) Accumulation of copious amounts of very thick secretions in the lungs B) Loss of elastic tissue of the lungs and destruction of alveolar walls C) Progressive loss of lung compliance and increasing hypoxia D) Reversible bronchospasm, inflammation, and hyperactive airways

C) Progressive loss of lung compliance and increasing hypoxia Feedback: ARDS is characterized by progressive loss of lung compliance and increasing hypoxia and occurs as a result of a severe insult to the body. Accumulation of copious amounts of thick secretions in the lungs is associated with cystic fibrosis. Chronic obstructive pulmonary disease (COPD) is characterized by loss of the elastic tissue of the lung, destruction of alveolar walls, and hyperinflation with tendency to collapse with expiration. Reversible bronchospasm, inflammation, and hyperactive airways are characteristics of asthma.

The nursing instructor is discussing the need for lubrication of the alveoli for effective gas exchange and is produced by type II cells of the alveoli. The students know that what substance is produced by type II cells of the alveoli? A) Erythrocytes B) Lymphatic fluid C) Surfactant D) Pleural fluid

C) Surfactant Feedback: Type II cells produce surfactant. Erythrocytes are made in the bone marrow. Lymphatic fluid is produced by lymph glands; pleural fluid is secreted by cells in the pleural cavity.

A patient asks the nurse what causes a cold. The nurse would tell the patient that the common cold is most often caused by which type of microorganism? A) Gram-positive bacteria B) Gram-negative bacteria C) Virus D) Fungus

C) Virus Feedback: Various viruses cause the common cold. These viruses invade the tissues of the upper respiratory tract, initiating the release of histamine and prostaglandins and causing an inflammatory response. Bacteria and fungi can cause a respiratory infection but the disorder commonly known as a cold is caused by a virus.

Which of these anti-inflammatory drugs have geriatric warnings? (Select all that apply.) A) Sulindac (Clinoril) B) Indomethacin (Indocin) C) Ketorolac (Toradol) D) Naproxen (Naprosyn) E) Ketoprofen (Orudis)

C, D, E Geriatric warnings have been associated with naproxen, ketorolac, and ketoprofen because of reports of increased toxicity when they are used by older patients. These nonsteroidal anti-inflammatory drugs should be avoided if possible. Sulindac and indomethacin are not associated with toxicity in older patients.

Which narcotic analgesics can the nurse administer to a child because she has an established pediatric dose? (Select all that apply.) A) Transdermal fentanyl B) Methadone C) Morphine D) Meperidine E) Hydrocodone

C, D, E Narcotics that have an established pediatric dose include codeine, fentanyl (but not the transdermal form), hydrocodone, meperidine, and morphine. Methadone is not recommended as an analgesic in children.

A patient has orders to receive 3,000 mL of IV fluid at a rate of 150 mL/h. If the infusion starts at 08:00, when would it be finished? A) 20:00 B) 23:00 C) 01:00 D) 04:00

D

An 81-year-old patient with congestive heart failure has been sent to a cardiologist who prescribes digoxin (Lanoxin) 0.125 mg PO every morning. The pharmacy dispenses pills that contain 0.25 mg of Lanoxin. How many pills should the nurse teach the patient to take every morning? A) 2 B) 1.5 C) 1 D) 0.5

D

The pediatric nurse is caring for a child who weighs 44 pounds. The physician has ordered methylprednisolone sodium succinate (Solu-Medrol), 0.03 mg/kg/d IV in normal saline. How many milligrams of medication will the nurse prepare? A) 6.5 B) 6 C) 0.65 D) 0.6

D

A 72-year-old patient is admitted to the hospital for surgery. After the patient returns to the floor, the patient's daughter tells the nurse she is concerned that her mother will overdose on morphine because she keeps pressing the button on her patient-controlled anesthesia (PCA) pump. What is the nurse's best response? A) "You should control how often she presses the button." B) "If she will follow the directions she was given, that will not happen." C) "The PCA device always provides the correct amount, so pressing the button is just for placebo effect." D) "The device is preset, so your mother cannot get more than a specific amount."

D A PCA system using morphine provides a baseline, constant infusion of morphine and gives the patient control of the system to add bolus doses of morphine if the patient believes that pain is not being controlled. The system prevents overdose by locking out extra doses until a specific period of time has elapsed. The PCA is for the patient to control the analgesia, not for a family member to control it because the patient will fall asleep when adequate pain control is reached. If the family keeps pushing the button while the patient's level of consciousness continues to decline, serious overdosage could occur. Not following directions could result in inadequate pain management but not overdosage. The button delivers small bolus dosages so it is not a placebo effect.

No antihypertensive medication is safe during pregnancy but if the benefit for the mother outweighs the risk to the fetus, what classification of medication can the nurse administer to the hypertensive pregnant woman? A) Angiotensin-converting enzyme (ACE) inhibitor B) Angiotensin-receptor blocker (ARB) C) Renin inhibitor D) Calcium channel blocker

D ACE inhibitors, ARBs, and renin inhibitors should not be used during pregnancy, and women of childbearing age should be advised to use barrier contraceptives to prevent pregnancy while taking these drugs. Calcium channel blockers and vasodilators should not be used in pregnancy unless the benefit to the mother clearly outweighs the potential risk to the fetus.

A patient arrives at the emergency department brought by his or her friends. The friends tell the nurse that the patient has taken a whole bottle of aspirin. Blood work for salicylate toxicity is run. What does the nurse expect the results to be? A) >5 g B) >10 g C) >15 g D) >20 g

D Acute salicylate toxicity may occur at doses of 20 to 25 g in adults or 4 g in children. Options A, B, and C would not be high enough to indicate salicylate toxicity.

What does the clinic nurse anticipate as initial drug therapy for a 39-year-old African American man who is 25 pounds overweight and newly diagnosed with hypertension? A) An angiotensin-converting enzyme (ACE) inhibitor B) A beta-blocker C) A calcium channel blocker D) A diuretic

D African Americans are at highest risk for developing hypertension with men more likely than women to develop the disease. African Americans have documented differences in response to antihypertensive therapy. They are most responsive to single-drug therapy and diuretics. The first line use of a diuretic is in combination with diet and other lifestyle changes. The use of a calcium channel blocker and/or alpha-adrenergic blocker should follow. African Americans are less responsive to ACE inhibitors and beta-blocker.

A patient is admitted to the intensive care unit in shock with hypotension. What is an appropriate nursing diagnosis for this patient? A) Impaired gas exchange B) Deficient fluid volume C) Risk for shock D) Ineffective peripheral tissue perfusion

D An appropriate nursing diagnosis would be ineffective peripheral tissue perfusion. If blood pressure becomes too low, the vital centers in the brain, as well as the rest of the tissues of the body, may not receive enough oxygenated blood to continue functioning. Because the patient is already in shock it would not be a risk diagnosis. There is no indication of altered gas exchange or deficient fluid volume.

The nurse is caring for a patient who receives anakinra (Kineret) for arthritis. By what route will the nurse administer this medication? A) Into the affected joint directly B) Oral C) Intramuscular D) Subcutaneous

D Anakinra is administered subcutaneously every day and is often used in combination with other antiarthritis drugs. No other route is appropriate.

A salicylate has been prescribed for a 15-year-old patient who has been diagnosed with arthritis. The mother is concerned about giving her child a salicylate. What salicylates could the nurse tell this mother are recommended for use in children? A) Salsalate (Argesic) B) Olsalazine (Dipentum) C) Sodium thiosalicylate (generic) D) Choline magnesium trisalicylate (Tricosal)

D Aspirin and choline magnesium trisalicylate are the only salicylates recommended for use in children. They should not be used when any risk of Reye's syndrome exists. Salsalate (Argesic), olsalazine (Dipentum), and sodium thiosalicylate (generic) have not been approved for pediatric use and do not provide pediatric dosing guidelines as a result.

A patient has just begun to take a prescribed diuretic. Why would the nurse tell the patient to drink 8 to 10 glasses of water daily (unless it is counterindicated)? A) To decrease the action of the renin-angiotensin cycle B) To make more concentrated plasma C) To dilute the urine D) To avoid rebound edema

D Care must be taken when using diuretics to avoid fluid rebound, which is associated with fluid loss. If a patient stops taking in water and takes the diuretic, the result will be a concentrated plasma of smaller volume. The decreased volume is sensed by the nephrons, which activate the renin-angiotensin cycle. When concentrated blood is sensed by the osmotic center in the brain, antidiuretic hormone (ADH) is released to retain water and dilute the blood. The result can be "rebound" edema as fluid is retained. Drinking 8 to 10 glasses of water will not decrease the action of the renin-angiotensin cycle, or make plasma more concentrated. It may produce urine that is dilute but that is not the reason it is recommended.

What is chrysotherapy? A) Treatment with antimalarials B) Treatment with salicylates C) Treatment with disease-modifying antirheumatic drugs (DMARDs) D) Treatment with gold salts

D Chrysotherapy is the clinical name for treatment with gold salts in which gold is taken up by macrophages, which then inhibit phagocytosis. It is reserved for use in patients who are unresponsive to conventional therapy and can be very toxic. Options A, B, and C are incorrect.

The nurse is administering morphine to a trauma patient for acute pain. Before administering the morphine, what common adverse effect should the nurse inform the patient about? A) Paresthesia in lower extremities B) Occipital headache C) Increased intracranial pressure D) Drowsiness

D Common adverse effects include dizziness, drowsiness, and visual changes. Morphine does not commonly cause paresthesia in the lower extremities, an occipital headache, or increased intracranial pressure.

The nurse evaluates the patient's lifestyle for factors that are contributing to his or her diagnosis of hypertension and then addresses the need to change what factor? A) Working outdoors in the sun B) Weight lifting at the gym C) Married with two children D) Exposure to high-frequency noise

D Factors that are known to increase blood pressure in some people include high levels of psychological stress, exposure to high-frequency noise, a high-salt diet, lack of rest, and genetic predisposition. Working outdoors in the sun may increase risk for skin cancer but does not contribute to hypertension. Weight lifting is good exercise, especially if he uses low weight and frequent repetition. Being married with two children is not a contributing factor for hypertension.

The nurse admits to the clinic a 7-month-old infant whose mother reports that the baby has not had a bowel movement in 6 days. What drug would be appropriate to treat this patient? A) Cascara (generic) B) Magnesium hydroxide (Milk of Magnesia) C) Polyethylene glycol-electrolyte solution (GoLYTELY) D) Glycerin (Sani-Supp)

D Feedback: Glycerin suppositories are the best choice for infants and young children when constipation is a problem. The other choices are not recommended for infants or children.

The emergency department (ED) nurse is caring for a patient who is experiencing pulmonary edema. The patient is treated with furosemide (Lasix). What will the nurse monitor? A) Sodium levels B) Bone narrow function C) Calcium levels D) Potassium levels

D Furosemide is associated with loss of potassium, so that the patient will need to be monitored carefully for low potassium levels, which could cause cardiac arrhythmias and further aggravate pulmonary edema. The nurse would not monitor sodium or calcium levels or bone marrow function because of the effects of the drug during the acute treatment of pulmonary edema.

A patient comes to the clinic for a 1-month follow-up appointment. The patient tells the nurse he or she has been taking chlorothiazide (Diruil) for a month and now has leg cramps and "feels tired all the time." What will the nurse consider as the cause of the patient's symptoms? A) Hypercalcemia B) Hypocalcemia C) Hyperkalemia D) Hypokalemia

D Hypokalemia results from the loss of potassium in the distal tubule and causes muscle weakness, fatigue, and arrhythmias. Hyperkalemia presents with cardiac arrhythmias and occasionally muscle weakness. Hypercalcemia is characterized by fatigue, depression, mental confusion, nausea, vomiting, and constipation. Hypocalcemia presents with muscle spasms, facial grimacing, possible convulsions, irritability, and depression.

A stepped care management approach to treating hypertension includes weight loss, smoking cessation, decreased use of alcohol, reducing salt in the diet, and increased physical exercise. In which step of a stepped-care management approach will the nurse teach about these changes in lifestyle? A) Step 1 only B) Steps 1 and 2 C) Steps 1, 2, and 3 D) Steps 1, 2, 3, and 4

D Lifestyle changes are encouraged in all four steps and should be advocated for the maintenance of good health. These changes are stressed in steps 1 and 2 in the hope that drug therapy will not be necessary. However, these changes should occur in steps 3 and 4 as well.

A patient who is experiencing severe pain is administered a narcotic. What would the nurse write in the plan of care as a desirable and measurable outcome for this patient? A) A shorter period of time between requests for medication B) Reduced complaints about limited movement C) Lack of restlessness and ability to sustain one position D) Increased autonomy in providing AM care

D Monitor patient response to the drug (e.g., relief of pain, sedation).When pain is being adequately managed with opioid therapy, a desirable and measurable outcome would be that the patient is able to be more autonomous in providing care in the morning. Shorter periods between requests for medication would not be a desirable outcome because it is not an indicator of pain control given that some patients are reluctant to ask for medicine even though they are in pain. Patients in pain tend not to move for fear of exacerbating the pain, so lack of movement can be an indication the patient is in pain. Just because the patient does not complain of pain doesn't mean he isn't experiencing pain.

You are caring for a patient taking pentazocine (Talwin). What would be an appropriate nursing diagnosis for this patients care plan? A) Fluid volume deficit related to diarrhea caused by medication B) Risk for pain related to administration of medication C) Monitor timing of analgesic doses. D) Impaired gas exchange related to respiratory depression

D Nursing diagnosis may include impaired gas exchange related to respiratory depression. The drug is more likely to cause constipation due to slowing of the GI tract instead of diarrhea, so that fluid volume deficit would not be appropriate. Monitoring timing of analgesic doses is an intervention and not a nursing diagnosis. If the patient is receiving pentazocine that would indicate he is experiencing pain and is not just at risk for pain, and that the pain is not caused by the drug. The drug is given to reduce the pain so this diagnosis is incorrect.

A geriatric patient received a narcotic analgesic before leaving the post-anesthesia care unit to return to the regular unit. What is the priority nursing action for the nurse receiving the patient on the regular unit? A) Administer a non-steroidal anti-inflammatory drug. B) Encourage fluids. C) Create a restful, dark, quiet environment. D) Put side rails up and place bed in low position.

D Older patients are more susceptible to the central nervous system effects of narcotics; it is important to ensure their safety by using side rails and placing the bed in the low position in case the patient tries to get up unaided. Postoperative patients are allowed nothing by mouth until bowel function returns so an oral medication or encouraging fluids would not be appropriate. This patient will require careful observation for respiratory depression, so a dark room would be unsafe.

Antipyretic drugs (e.g., aspirin, ibuprofen, acetaminophen) often are used to alleviate the discomforts of fever and to protect vulnerable organs, such as the brain, from extreme elevations in body temperature. However, the use of aspirin in children is limited due to the possibility of what disease? A) Munchausen's syndrome B) Guillain-Barré syndrome C) Angelman's syndrome D) Reye's syndrome

D Salicylates like aspirin are contraindicated for the treatment of childhood fevers because of the risk of Reye's syndrome in children and teenagers. Munchausen's syndrome is an unusual condition characterized by habitual pleas for treatment and hospitalization for a symptomatic but imaginary acute illness. Guillain-Barré syndrome is an idiopathic, peripheral polyneuritis that occurs 1 to 3 weeks after a mild episode of fever associated with a viral infection or with immunization. Angelman's syndrome is an autosomal recessive syndrome characterized by jerky puppet-like movements, frequent laughter, mental and motor retardation, a peculiar open-mouthed facial expression, and seizures. Salicylates like aspirin are not contraindicated for patients with Munchausen's syndrome, Guillain-Barré syndrome, or Angelman's syndrome.

The nurse receives an order for a triptan for a patient diagnosed with cluster headaches. What drug would be indicated for this purpose? A) Almotriptan (Axert) B) Frovatriptan (Frova) C) Naratriptan (Amerge) D) Sumatriptan (Imitrex)

D Sumatriptan, the first drug of this class, is used for the treatment of acute migraine attacks and for the treatment of cluster headaches in adults. No other triptans are approved for treatment of cluster headaches.

The health care provider orders oral (PO) codeine as an adjunctive therapy to pain control medication. What order would be appropriate for the nurse to administer? A) Codeine 5 mg PO every 6 hour B) Codeine 10 mg PO every 4 hour C) Codeine 15 mg PO every 2 hour D) Codeine 20 mg PO every 4 hour

D The correct dosage for codeine administered for pain by mouth is 15 to 60 mg q 4 to 6 hour. The other options are incorrect oral dosages because they are too low a dose or give an incorrect dosing frequency.

A 10-year-old child has edema caused by a heart defect. The patient is taking furosemide (Lasix). The dosage is 6 mg/kg/d. The child weighs 76 pounds. How many mg does the child receive in each dose? A) 20 mg B) 50 mg C) 105 mg D) 210 mg

D The nurse will administer 210 mg/dose of the drug (2.2 pounds: 1 kg = 76 pounds: × kg, 76 divided by 2.2 = 34.5, 35 times 6 = 210 mg).

The nurse is talking with a group of nursing students. What drug would the nurse tell them, when combined with furosemide (Lasix), is likely to cause hearing loss? A) Codeine B) Ciprofloxacin (Cipro) C) Digoxin (Lanoxin) D) Gentamicin (Garamycin)

D The risk of ototoxicity increases if loop diuretics are combined with aminoglycoside antibiotics (gentamicin) or cisplatin. No known increased risk of ototoxicity exists when furosemide is taken with codeine, ciprofloxacin, or digoxin.

The class of diuretics that act to block the chloride pump in the distal convoluted tubules and leads to a loss of sodium and potassium and a minor loss of water is what? A) Carbonic anhydrase inhibitors B) Osmotic diuretics C) Potassium-sparing diuretics D) Thiazide diuretics

D Thiazide diuretics work to block the chloride pump, which leads to a loss of sodium, potassium, and some water. They are considered mild diuretics. Carbonic anhydrase inhibitors work to block the formation of carbonic acid and bicarbonate in the renal tubules. Osmotic diuretics use hypertonic pull to remove fluid from the intravascular spaces and to deliver large amounts of water into the renal tubules. Potassium-sparing diuretics are mild and act to spare potassium in exchange for the loss of sodium and water.

A nurse is caring for a 6-year-old patient after surgery. The child has an order for meperidine (Demerol) 1.8 mg/kg IM every 3 to 4 hour as needed for pain. The child weighs 30 kg and the meperidine is available as 50 mg/mL. How many mL will the nurse administer per dose? A) 1 mL B) 1.8 mL C) 0.8 mL D) 1.08 mL

D To calculate the correct amount to be administered, first multiply 1.8 mg times 30 kg (54 mg). Next determine the volume in mL that 54 mg is equal to (50 mg: 1 mL as 54 mg: × mL). Solve for x (50x is equal to 54 mg; 54 divided by 50 is equal to 1.08 mL).

The home care nurse administers oral morphine to the patient with cancer pain. When will the nurse expect this medication to reach peak activity? A) 10 minutes B) 30 minutes C) 45 minutes D) 60 minutes

D With oral administration, peak activity occurs in about 60 minutes. The duration of action is 5 to 7 hours.

The physician writes an order for oxazepam for a 6-year-old child. The nurse verifies that there is no established dosage for children 6 to 12 years of age for oxazepam. The nurse knows that the usual adult dose is 10 mg tid. What would the nurse calculate the appropriate dose to be? A) 0.03 mg tid B) 0.3 mg tid C) 1.8 mg tid D) 3.3 mg tid

D Because the nurse knows only the child's age, the nurse would need to use Young's rule to determine the appropriate dosage. The formula for Young's rule is: Child's dose = child's age in years ÷ (child's age + 12) × average adult dose. Using the information provided in the question: Dose = 6 ÷ (6 + 12) × 10 mg = 6 ÷ 18 × 10 = 0.33 × 10 = 3.3.9.

The clinic nurse is caring for a patient who has just been diagnosed with chronic obstructive pulmonary disease (COPD). The patient asks the nurse what they could have done to minimize the risk of contracting this disease. What would be the nurse's best answer? A) "The most important risk factor for COPD is inadequate nutrition." B) "The most important risk factor for COPD is regular exercise." C) "The most important risk factor for COPD is exposure to dust and pollen." D) "The most important risk factor for COPD is cigarette smoking."

D) "The most important risk factor for COPD is cigarette smoking." Feedback: COPD is a permanent, chronic obstruction of airways, often related to cigarette smoking. Inadequate nutrition, regular exercise, and exposure to dust and pollen are not risk factors for COPD.

A patient presents to the clinic with a temperature of 101.5°F; malaise, myalgia, arthralgia, and a purulent, productive cough. The patient states "The fever just started today but I have had this cough for many, many months." What would the nurse suspect the patient has? A) Acute pneumonia B) Bronchitis C) Chronic obstructive pulmonary disease (COPD) D) Bronchiectasis

D) Bronchiectasis Feedback: Bronchiectasis is a chronic disease that involves the bronchi and bronchioles. It is characterized by dilation of the bronchial tree and chronic infection and inflammation of the bronchial passages. With chronic inflammation, the bronchial epithelial cells are replaced by a fibrous scar tissue. The loss of the protective mucus and ciliary movement of the epithelial cell membranes, combined with the dilation of the bronchial tree, leads to chronic infections in the now unprotected lower areas of lung tissue. Patients with bronchiectasis often have an underlying medical condition that makes them more susceptible to infections (e.g., immune suppression, acquired immune deficiency syndrome, chronic inflammatory conditions). Patients present with the signs and symptoms of acute infection, including fever, malaise, myalgia, arthralgia, and a purulent, productive cough. Patients who have pneumonia and bronchitis may present with the above symptoms, but it is not a chronic disorder. COPD is a chronic disorder but the patient with this disorder has more symptoms related to shortness of breath.

While assessing a new patient on the unit, the nurse notes the following: productive cough, respiratory rate of 22, oxygen saturation of 90%, and increased secretions. The patient has a 20-year history of smoking 1.5 packs of cigarettes daily. What chronic condition might this patient have? A) Pneumonia B) Cystic fibrosis C) Pleural effusion D) Chronic obstructive pulmonary disease (COPD)

D) Chronic obstructive pulmonary disease (COPD) Feedback: Chronic obstructive pulmonary disease (COPD) is a permanent, chronic obstruction of airways, often related to cigarette smoking. It is caused by two related disorders, emphysema and chronic bronchitis, both of which result in airflow obstruction on expiration, as well as overinflation of the lungs and poor gas exchange. Emphysema is characterized by loss of the elastic tissue of the lungs, destruction of alveolar walls, and a resultant alveolar hyperinflation with a tendency to collapse with expiration. Chronic bronchitis is a permanent inflammation of the airways with mucus secretion, edema, and poor inflammatory defenses. Characteristics of both disorders often are present in the person with COPD. Pneumonia and pleural effusion are not chronic conditions. Cystic fibrosis is a genetic disease of excessive pulmonary tract secretions and GI tract involvement.

The nurse is caring for a patient who suffered a head injury and is now having difficulty breathing. The nurse knows that this patient may have injured what part of the central nervous system? A) Cerebral cortex B) Cerebellum C) Hypothalamus D) Medulla oblongata

D) Medulla oblongata Feedback: The act of breathing is controlled by the medulla, which depends on a functioning muscular system and a balance between the sympathetic and parasympathetic systems. The cerebral cortex, cerebellum, and hypothalamus are not involved with this process.

A patient returns to the unit after thoracic surgery with a water-sealed chest drainage system. What should the nurse instruct the patient and the family that this drainage system is used for? A) Maintaining positive chest wall pressure B) Monitoring pleural fluid C) Providing positive intrathoracic pressure D) Re-expanding the lung and restoring the negative pressure to the space between the pleura

D) Re-expanding the lung and restoring the negative pressure to the space between the pleura Feedback: In the case of a pneumothorax, treatment would involve insertion of a chest tube to restore the negative pressure to the space between the pleura. A water-sealed chest drainage system does not maintain positive chest wall pressure, monitor pleural fluid, or provide positive intrathoracic pressure.

The nursing instructor is teaching a class on respiratory disorders and asks the students "What condition is a result of the upper airways response to pollen, mold, or dust?" A) Cystic fibrosis B) Adult respiratory distress syndrome (ARDS) C) Atelectasis D) Seasonal rhinitis

D) Seasonal rhinitis Feedback: Seasonal rhinitis is an inflammation of the nasal cavity; it occurs when the upper airways respond to a specific antigen such as pollen, mold, or dust. Cystic fibrosis, atelectasis, and ARDS are not associated with an allergic response.

A woman has just given birth to a premature infant. The mother asks why the infant is having such a hard time breathing. The nurse explains that the infant's alveoli are unable to stay open. What is the infant lacking that is necessary for correct functioning of the alveoli? A) Adenosine triphosphate (ATP) B) Histamine C) Serotonin D) Surfactant

D) Surfactant Feedback: Surfactant is a lipoprotein that decreases the surface tension in the sac and prevents alveolar collapse. ATP, histamine, and serotonin are released from mast cells throughout the airway to ensure a quick and intense inflammatory reaction to any cell injury, which may lead to a respiratory infection.

The charge nurse, working on a pediatric unit, sees an order was written to administer chloramphenicol (Chloromycetin) to one of the children assigned to a new graduate nurse. The charge nurse would make sure the new graduate was familiar with what possible adverse effects of this medication? (Select all that apply.) a. Gray syndrome b. Bone marrow depression c. Aplastic anemia d. Liver failure e. Hearing loss

a, b, c Chloramphenicol (Chloromycetin), an older antibiotic, prevents bacterial cell division in susceptible bacteria. Because of the potential toxic effects of this drug, its use is limited to serious infections for which no other antibiotic is effective. Chloramphenicol produces a "gray syndrome" in neonates and premature babies, which is characterized by abdominal distention, pallid cyanosis, vasomotor collapse, irregular respirations, and even death. In addition, the drug may cause bone marrow depression, including aplastic anemia that can result in death. Liver failure and hearing loss are not usually associated with this drug.

A group of nursing students are giving a report on the emergence of drug-resistant microbial agents. What could the students cite as a good way to minimize the emergence of drug-resistant microbial agents? (Select all that apply.) a. Avoid the use of broad-spectrum antibacterial drugs when treating trivial or viral infections. b. Use narrow-spectrum agents if they are thought to be effective. c. Do not use vancomycin unnecessarily. d. Antibiotics are best started before the culture and sensitivity report returns. e. Administer the smallest effective dosage available.

a, b, c Exposure to an antimicrobial agent leads to the development of resistance, so it is important to limit the use of antimicrobial agents to the treatment of specific pathogens known to be sensitive to the drug being used. Drug dosage is important in preventing the development of resistance. Doses should be high enough and the duration of drug therapy should be long enough to eradicate even slightly resistant microorganisms. It is best to wait until cultures return before initiating antibiotics when possible, but patients with severe infections may be started on broad -spectrum antibiotics while waiting for culture results.

The pharmacology instructor is discussing antimicrobials with the nursing class. What would the instructor tell the students about the mechanism of action of antimicrobials? (Select all that apply). a. Preventing cell division b. Causing cell death c. Inhibiting cell wall synthesis d. Causing leakage of cell wall allowing fluid to leak in e. Inhibiting synthesis of ribonucleic acid (RNA)

a, b, c Sites of cellular action of carbapenems, ketolides, lincosamides, aztreonam, penicillins, sulfonamides, tetracyclines, and antimycobacterials. Carbapenems, ketolides, and lincosamides change protein function and prevent cell division or cause cell death. Aztreonam alters cell membranes to allow leakage of intracellular substances and causes cell death; it does not cause leakage of fluid into the cell. Penicillins prevent bacteria from building their cells during division. Sulfonamides inhibit folic acid synthesis for RNA and deoxyribonucleic acid production but does not inhibit RNA synthesis

When the nurse cares for a patient receiving an antibiotic, what instructions will the nurse provide no matter what medication is prescribed? (Select all that apply.) a. Drink plenty of fluids to avoid kidney damage. b. Take all medications as prescribed until all of the medication is gone. c. Report difficulty breathing, severe headache, or changes in urine output. d. Take antibiotic with food to avoid gastrointestinal (GI) upset. e. Take safety precautions such as changing position slowly.

a, b, c The patient taking any antibiotic needs to drink plenty of fluids to avoid kidney damage and improve excretion of the metabolized drug; take all medications as prescribed until all of the medication is gone to avoid developing a resistant strain of bacteria; and report any difficulty breathing, severe headache, or changes in urine output because these are primary manifestations of serious adverse effects. Although some antibiotics need to be taken with food, others may be best taken on an empty stomach so this does not apply to all antibiotics. Not all antibiotics are associated with central nervous system (CNS) toxicity so taking safety precautions need only be included in patient teaching if they are taking a drug associated with CNS adverse effects.

A nursing student asks the pharmacology instructor for ways to minimize the emergence of drug-resistant microbial agents. What would be an appropriate response by the instructor? (Select all that apply.) A) Avoid the use of broad-spectrum antibacterial drugs when treating trivial or viral infections. B) Use narrow-spectrum agents if they are thought to be effective. C) Do not use vancomycin unnecessarily. D) Prescribe antibiotics when the patient believes they are warranted. E) Start the antibiotics, do culture and sensitivity tests, and provide patient education.

a, b, c To prevent or contain the growing threat of drug-resistant strains of bacteria, it is very important to use antibiotics cautiously, to complete the full course of an antibiotic prescription, and to avoid saving antibiotics for self-medication in the future. You would not give antibiotics every time the patient wants them, nor would you do a culture and sensitivity test after starting antibiotics. Therefore, Options D and E are incorrect.

The mother brings her 18-month-old toddler to the pediatrician because the child has a fever and has been tugging on his or her left ear. Examination of the tympanic membrane confirms an ear infection and the toddler also has a cold with nasal congestion, rhinorrhea, and a cough. The provider tells the mother to apply heat and gives her a prescription for an otic anesthetic to make the ear more comfortable until the infection resolves. The mother is not happy and says she wants a prescription for an antibiotic. What important teaching points will the nurse include in the teaching plan? (Select all that apply.) a. Ear infections that accompany viral respiratory infections do not respond to antibiotics. b. Habitual use of antibiotics for viral infections contribute to development of resistant strains. c. Adverse effects from antibiotics in children can cause diarrhea and dehydration. d. Antibiotics will only be prescribed if a culture indicates the presence of bacteria in the ear. e. The pediatrician knows more than the mother and she should trust what she is being told.

a, b, c When the child has a viral respiratory infection, the organism involved in ear infections is usually viral as well. As a result, antibiotics will have no effect on the infection that will resolve independently and only comfort care is indicated. Habitual use of antibiotics for viral infections contributes to the development of resistant strains of bacteria and the adverse effects can make the child more uncomfortable causing diarrhea and dehydration. Cultures of ear fluid are almost never done because it would be an invasive procedure to remove fluid from the middle ear. It is never right for the nurse to patronize the mother, who has every right to advocate for her child, and it is more important she understand why the antibiotic is not being prescribed than telling her the pediatrician knows more.

The nurse is caring for a child who weighs 30 kg. The physician orders gentamicin (Garamycin) tid. The recommended dosage range is 6 to 7.5 mg/kg/day. Why is it important to give a dosage within this recommended range? (Select all that apply.) a. To avoid toxic effects b. To protect other patients c. To reduce the risk of drug-resistant organisms d. To eradicate the bacteria e. To promote lactic acid removal

a, b, c, d By administering the correct dosage, you avoid overdosage and reduce the risk of toxic effects. The correct dosage reduces the risk of creating drug-resistant organisms; it also protects both the patient and the other patients who might be susceptible to the drug-resistant organisms as well. The proper dosage is needed to eradicate the bacteria. Lactic acid removal is not related to the proper dosage and is a distracter for this question.

The nurse promotes optimal drug effectiveness by doing what? (Select all that apply.) a. Incorporate basic history and physical assessment factors into the plan of care. b. Evaluate the effectiveness of drugs after they have been administered. c. Modify the drug regimen to modify adverse or intolerable effects. d. Minimize the number of medications administered to patients. e. Examine factors known to influence specific drugs if they are to be effective.

a, b, c, e Incorporate basic history and physical assessment factors into any plan of care so that obvious problems can be identified and handled promptly. If a drug simply does not do what it is expected to do, further examine the factors that are known to influence drug effects. Frequently, the drug regimen can be modified to deal with that influence. Minimizing the number of medications administered is usually not an option because each drug is ordered for a reason of necessity for the patient.

The nurse administers polymyxin B to a patient with a gram-negative bacterial infection. What symptoms would cause the nurse to suspect drug fever, hold the medication, and call the health care provider immediately? (Select all that apply.) a. Fever b. Dizziness c. Ataxia d. Increased activity e. Reduced urine output

a, b, c, e The actions of polymyxin B on cell membranes means it can be toxic to the human host, leading to nephrotoxicity, neurotoxicity (e.g., facial flushing, dizziness, ataxia, paresthesias, drowsiness), and drug-related fever and rash. This drug is reserved for infections that do not respond to less toxic drugs; the nurse needs to be alert for serious reactions and hold the drug until notifying the provider.

The nurse administers antipsychotic medications to the patient who has taken these same drugs for many years. What signs and symptoms would the nurse attribute to secondary actions of the drug? (Select all that apply.) a. Muscular tremors b. Drooling c. Changes in gait d. Yellow discoloration of skin and sclera e. Fine red rash on the trunk

a, b, d Drugs that affect the dopamine levels in the brain (e.g., antipsychotic drugs), cause a syndrome that resembles Parkinson's disease including lack of activity, akinesia, muscular tremors, drooling, changes in gait, rigidity, extreme restlessness or "jitters" (akathisia), or spasms (dyskinesia). Yellow discoloration of the skin and sclera indicate jaundice and would suggest liver damage. A fine red rash on the trunk would be a dermatologic reaction unrelated to an antipsychotic agent's secondary effects.

Several processes enable a drug to reach a specific concentration in the body. Together they are called dynamic equilibrium. What are these processes? (Select all that apply.) a. Distribution to the active site b. Biotransformation c. Absorption from the muscle d. Excretion e. Interaction with other drugs

a, b, d The actual concentration that a drug reaches in the body results from a dynamic equilibrium involving several processes: Absorption from the site of entry (can be from the muscle, the gastrointestinal (GI) tract if taken orally, of the subcutaneous tissue if given by that route); Distribution to the active site; biotransformation (metabolism) in the liver; excretion from the body. Interaction with other drugs is not part of the dynamic equilibrium.

The nurse needs to ask what specific questions when collecting a drug history? (Select all that apply.) a. "Do you take any over-the-counter medications?" b. "Do you take any herbal supplements?" c. "Do you use any alternative therapies?" d. "Do you take any natural supplements or vitamins?" e. "What unusual therapies do you take?"

a, b, d The nurse needs to specifically question the patient's use of over-the-counter drugs, herbal supplements, natural supplements, and vitamins. Use of terms like "alternative therapies" or "unusual therapies" is too vague and may not elicit the kind of information needed.

When a drug is ordered off-label, what must the nurse be clear about before administering the drug? (Select all that apply.) a. Why the drug is being given b. Its potential for problems c. The research that has been done d. The age group it was pretested on e. The intended use

a, b, e Liability issues surrounding many of these uses are very unclear, and the nurse should be clear about the intended use, why the drug is being given, and its potential for problems. Knowing the age group it was pretested on and knowing the research that has been done are not factors the nurse needs to know before administering the drug.

A patient is admitted to the unit and the nurse assesses whether he or she is at increased risk for infection when what factors are determined? (Select all that apply.) a. Malnutrition b. Hypertension c. Suppression of immune system d. Advanced age e. Decreased amylase levels

a, c, d Factors that suppress the host defense mechanisms include malnutrition, suppression of immune system, and advanced age. Hypertension does not predispose a person to infection neither does a decreased amylase level.

What concerns might the nurse legitimately have related to the use of alternative therapies? (Select all that apply.) a. The Food and Drug Administration (FDA) does not test or regulate active ingredients. b. The incidental ingredients are clearly marked on the label. c. The dosage contained in each tablet may vary greatly. d. No alternative therapies have been found to be effective. e. Advertising of alternative products is not as restrictive or accurate.

a, c, e Alternative products are not controlled or tested by the FDA and advertising is not as restrictive or accurate as with classic drugs. Incidental ingredients are often unknown and strength of tablets may vary within the bottle depending on the conditions under which they were grown. While some alternative therapies have been found to be effective, there are others who have not been studied.

The nurse collects the past medical history of a patient new to the clinic. The patient states he or she is allergic to penicillin. What would the nurse question next? (Select all that apply.) a. What signs and symptoms were displayed with the reaction? b. What treatment was required to control the allergic reaction? c. How was the medication administered? d. How many dosages were administered before the reaction occurred? e. Had the medication ever been prescribed before the time when the reaction occurred?

a, d, e It is important to determine what the allergic reaction was and when the patient experienced it (e.g., after first use of drug, after years of use). If she had been prescribed this medication before with no reaction and then had a reaction the next time it was prescribed, this would be important information to know. Some patients report having a drug allergy, but closer investigation indicates that their reaction actually constituted an anticipated effect or a known adverse effect to the drug. It would not necessarily be important to find out what was done to stop the reaction or who the caregiver was at the time of the reaction or what type of allergic reaction it was.

A client reports that he thinks he is taking an antidepressant, but he is not sure. In reviewing his medication history, which of the following drugs would be considered antidepressants a. tetracyclic drugs b. cholinergics c. SSRIs d. MAOIs e. angiotensin II receptor blockers f. benzodiazepines

a,c,d

Depression is a very common affective disorder that strikes many people. In assessing a client who might be suffering from depression, the nurse would expect to find which of the following a. lack of energy b. hyperactivity c. sleep disturbances d. libido problems e. confusion f. decreased reflexes

a,c,d

The nurse uses what term to describe the drug level required to have a therapeutic effect? a. Critical concentration b. Dynamic equilibrium c. Selective toxicity d. Active transport

a. A critical concentration of a drug must be present before a reaction occurs within the cells to bring about the desired therapeutic effect. A dynamic equilibrium is obtained from absorption of a drug from the site of drug entry, distribution to the active site, metabolism in the liver, and excretion from the body to have a critical concentration. Selective toxicity is the ability of a drug to attach only to those systems found in foreign cells. Active transport is the process that uses energy to actively move a molecule across a cell membrane and is often involved in drug excretion in the kidney.

The nurse is caring for a 62-year-old patient who is receiving IV gentamicin (Garamycin). The patient complains of difficulty hearing. What should the nurse do? a. Hold the dose and notify the physician immediately. b. Administer the dose and speak in a louder voice when talking to the patient. c. Administer the dose and report this information to the oncoming nurse. d. Administer the dose and document the finding in the nurse's notes.

a. Aminoglycosides are contraindicated in the following conditions: known allergy to any of the aminoglycosides; renal or hepatic disease that could be exacerbated by toxic aminoglycoside effects and that could interfere with drug metabolism and excretion, leading to higher toxicity; preexisting hearing loss, which could be intensified by toxic drug-related adverse effects on the auditory nerve. Ototoxicity should be reported and the drug should be stopped. You would not administer the dose and then call the physician, administer the dose and report information to oncoming nurse, or administer the dose and document the finding in the nurse's notes because each additional dose administered could potentially worsen hearing loss.

The patient is admitted to the acute care facility with acute septicemia and has orders to receive gentamicin and ampicillin IV. The nurse is performing an admission assessment that includes a complete nursing history. What information provided by the patient would indicate the need to consult the health care provider before administering the ordered medication? a. Takes furosemide (Lasix), a potent diuretic, daily b. Had prostate surgery 3 months ago c. History of hypothyroidism d. Allergic to peanuts and peanut products

a. Aminoglycosides should be avoided if the patient takes a potent diuretic because of the increased risk of ototoxicity, nephrotoxicity, and neurotoxicity. Learning the patient takes a potent diuretic would indicate the need to consult with the health care provider before administering gentamicin. Prostate surgery, hypothyroidism, and an allergy to peanuts would not preclude administration of these medications and would not indicate a need to consult with the provider.

A 42-year-old male patient is brought to the emergency department by ambulance. The patient is in distress. The nurse suspects an anaphylactic reaction resulting from taking oral penicillin. What assessment findings are important in making this diagnosis? a. Blood pressure (BP): 186/100, difficulty breathing b. Hematocrit (Hct): 32%, decreased urine output c. Temperature: 102º, swollen joints d. Profuse sweating, Blood Pressure: 92/58

a. An anaphylactic reaction is an immune reaction that causes a massive release of histamine, which results in edema and swelling that can lead to respiratory distress and increased blood pressure. A decreased hematocrit and decreased urine output suggests a cytotoxic reaction. An increased temperature and swollen joints could suggest serum sickness. Profuse sweating and decreased blood pressure may indicate cardiac-related issues.

The nurse administers an anticholinergic medication to the patient. When assessing this patient, what finding will the nurse consider a secondary effect of the drug? a. Nasal congestion b. Tachycardia c. Hyperthermia d. Profuse sweating

a. Anticholinergic secondary effects include dry mouth, altered taste perception, dysphagia, heartburn, constipation, bloating, paralytic ileus, urinary hesitancy and retention, impotence, blurred vision, cycloplegia, photophobia, headache, mental confusion, nasal congestion, palpitations, decreased sweating, and dry skin. Tachycardia, hyperthermia, and profuse sweating would not be expected findings or consistent with anticholinergic effects and would require further assessment.

A 68-year-old patient who must take antihistamines for severe allergies is planning a vacation to Mexico. The nurse will encourage the patient to do what? a. Avoid sightseeing during the hottest part of the day. b. Discontinue the antihistamines if he becomes extremely restless. c. Decrease the dosage of the drugs if he experiences excessive thirst. d. Continue taking the antihistamines even if he begins to hallucinate.

a. Antihistamines can cause anticholinergic effects, which would result in decreased sweating and place the patient at high risk for heat stroke. Avoiding the hottest part of the day will help prevent dehydration and heat prostration. Extreme restlessness could indicate Parkinson-like syndrome not usually associated with antihistamines. Excessive thirst is characteristic of hyperglycemia. Hallucinations are associated with drugs that affect neurologic functioning. Further, nurses should never tell patients to decrease or discontinue a drug unless the prescriber has instructed them to do so.

The nursing instructor is discussing the off-label use of drugs. What group of drugs would the instructor tell the nursing students is often used for off-label indications? a. Drugs used to treat psychiatric problems b. Drugs used to treat gastrointestinal (GI) problems c. Drugs used to treat cardiovascular problems d. Drugs used to treat musculoskeletal problems

a. Drugs often used for off-label indications include the drugs used to treat various psychiatric problems. Drugs used to treat GI, cardiovascular, or musculoskeletal problems do not fall in the category of frequent off-label uses.

The student nurse asks the instructor why a patient with a central nervous system infection is receiving antibiotics that will not cross the blood-brain barrier. What is the instructor's most correct response? a. A severe infection alters the blood-brain barrier to allow the drug to cross. b. A medication that is water soluble is more likely to cross the blood-brain barrier. c. Antibiotics are the exception to the blood-brain barrier and cross easily. d. An infection that spreads outside the central nervous system helps drugs cross the barrier.

a. Effective antibiotic treatment can occur only when the infection is severe enough to alter the blood-brain barrier and allow antibiotics to cross. Lipid-soluble, not water-soluble, medications cross the blood-brain barrier more easily and most antibiotics are lipid soluble, so they are not the exception. No matter where the infection originates, drugs must cross the blood-brain barrier to treat central nervous system infections.

What cardiac effect would the nurse be prepared to see in the patient with an extremely high potassium level? a. Arrhythmia b. Tachycardia c. Sudden death d. Bradycardia

a. Elevated potassium levels irritate cardiac cells and increase the likelihood of a cardiac arrhythmia. Tachycardia, sudden death, and bradycardia would be the result of an arrhythmia if they were to occur.

The clinic nurse is providing health teaching to a patient who has been prescribed doxycycline (Doxycin). What is a priority teaching point for this patient? a. Stay out of the sun. b. Avoid sexual activity. c. Take an antacid with the drug if nausea occurs. d. Chew the tablets completely before swallowing.

a. Encourage the patient to apply sunscreen and wear protective clothing if sun exposure cannot be avoided to protect exposed skin from rashes and sunburn associated with photosensitivity reactions. If the patient is a woman the nurse may advise the patient to use barrier methods of contraceptives (if she is taking oral contraceptives) due to the drug-drug interaction but the patient would not be told to avoid sexual activity. Antacid therapy and chewing the tablets would be inaccurate information.

After administering an antibiotic, the nurse assesses the patient for what common, potentially serious, adverse effect? a. Rash b. Pain c. Constipation d. Hypopnea

a. Examine skin for any rash or lesions, examine injection sites for abscess formation, and note respiratory status—including rate, depth, and adventitious sounds to provide a baseline for indications of an allergic or adverse response to the drug. Report nausea, vomiting, diarrhea, rash, recurrence of symptoms for which the antibiotic drug was prescribed, or signs of new infection (e.g., fever, cough, sore mouth, drainage). These problems may indicate adverse effects of the drug, lack of therapeutic response to the drug, or another infection. Pain, constipation, and hypopnea are not common adverse effects of antibiotic drugs.

The nurse is caring for a patient with a gram-positive infection. What antibiotic would be most effective in treating this infection? a. Cefaclor (Ceclor) b. Cefoxitin (generic) c. Cefotaxime (Claforan) d. Cefazolin (Zolicef)

a. First-generation cephalosporins are largely effective against gram-positive bacteria and include cefadroxil (generic), cefazolin (Zolicef), and cephalexin (Keflex). Second-and third-generation cephalosporins are less effective against gram-positive bacteria. Cefoxitin (generic) is a second-generation cephalosporin and cefotaxime (Claforan) and cefazolin (Zolicef) are third-generation cephalosporins.

The nurse provides teaching to the patient using herbal therapies and includes what important information related to the effects of the herbal therapy? a. They can interact with prescription drugs. b. They always contain known ingredients. c. They are natural so they are effective and safe. d. The ingredients are natural, meaning toxicity is not a concern.

a. Herbal therapies can produce unexpected adverse effects and toxic reactions, can interact with prescription drugs, and can contain various unknown ingredients that alter the therapies' effectiveness and toxicity.

The nursing instructor is teaching the class about how prescription drugs become over-the-counter (OTC) drugs and lists what factor as preventing a drug from becoming classified as OTC? a. If the patient cannot reliably self-diagnose the condition the drug is intended to treat b. If it would mask signs and symptoms of an underlying problem, the drug remains available by prescription only. c. If the drug would cause toxic effects if not taken as directed, it remains a prescription drug. d. OTC drugs must not have any adverse effects that could harm the patient.

a. If a diagnosis requires medical intervention, such as hyperlipidemia, which can only be diagnosed through laboratory studies, there is no point in making the drug an OTC medication. Most, if not all, OTC drugs have the capacity to mask signs and symptoms of an underlying disease so this is not a factor in deciding if a drug can be sold OTC. All drugs have the potential for toxic effects if not taken as directed and virtually all drugs have the potential for adverse effects.

The nurse administers a specific medication to an older adult patient every 4 hours. The patient has a history of chronic renal failure. Why would this patient be at risk for toxic drug levels? a. Cumulative effect b. First-pass effect c. Drug interactions d. Cross-tolerance effect

a. If a drug is taken in successive doses at intervals that are shorter than recommended, or if the body is unable to eliminate a drug properly, the drug can accumulate in the body, leading to toxic levels and adverse effects. This is a cumulative effect. First-pass effect addresses the reduction of available drug when taken orally due to metabolism in the liver before the drug reaches the bloodstream. Drug interactions occur when taken with other drugs, food, or complementary alternative therapies. Cross-tolerance is resistance to drugs within the same class.

The triage nurse in the emergency department sees a patient suspected of abusing amphetamines brought in by friends. While assessing this patient, what would the nurse be likely to find if steroids are being abused? a. Hypertension b. Bradycardia c. Drowsiness d. Elated mood

a. Increases in blood pressure, tachycardia, and insomnia are symptoms of amphetamine abuse. Elation can indicate abuse of cannabis.

Ipecac, formerly used as the drug of choice by parents for treatment of suspected poisoning in children, was tested by the Food and Drug Administration (FDA) in 2003. What was the finding of this testing? a. Ipecac is ineffective for its intended use. b. Ipecac is the safest treatment for poisoning in children. c. Ipecac was "grandfathered" in as an ineffective drug. d. Ipecac induces vomiting.

a. Ipecac, a formerly standard over-the-counter drug, was used for many years by parents to induce vomiting in children in cases of suspected poisoning or suspected drug overdose. The drug was finally tested and in 2003, the FDA announced that it was not found to be effective for its intended use. Although it was grandfathered in as an effective drug, this was not what the study researched. Ipecac is not effective and does not consistently induce vomiting.

When evaluating information accessed over the Internet, an important question the nurse should teach the patient to ask is what? a. Is the information anecdotal? b. Where has this information been obtained? c. Is this information paid for by the drug company? d. How many patients have had input into the information?

a. Many people do not know how to evaluate the drug-related information that they can access over the Internet. Is it accurate or anecdotal is an important concept for the nurse to teach the patient to assess to verify the accuracy of the information. Where the information came from is unimportant. It would be expected that all drug advertising is paid for by the drug company and this is not an important concern. Number of patients with input into the information is most likely none because information is gathered from health care professionals.

The mother of a 5-year-old asks the nurse why it seems amoxicillin is always prescribed when her child needs an antibiotic. What is the priority rationale the nurse should give the mother? a. It is better absorbed. b. It is less costly. c. It has a less frequent dosing schedule. d. It tastes better in oral form.

a. Most penicillins are rapidly absorbed from the GI tract, reaching peak levels in 1 hour. Although amoxicillin is less expensive, that fact has far less impact on choosing the proper antibiotic than the effectiveness of the drug. Most oral antibiotics for children are available in pleasant tasting syrups so taste would not be a factor. Ampicillin is often given up to 4 times a day so it actually has a frequent dosing schedule.

The nurse is teaching the patient about amoxicillin prior to discharge and includes what important teaching point? a. Blackening of the tongue may occur but will subside when the drug is discontinued. b. Even if it seems like the infection is not improving, the drug is still working. c. Yeast infections are unlikely to occur with this medication because it is narrow spectrum. d. Appearance of a rash is common and does not indicate an allergic reaction.

a. One of the adverse effects of ampicillin is blackening of the tongue but the discoloration goes away after stopping the drug. If it is accompanied by swelling, the patient should be instructed to call the prescribing health care provider immediately. Many penicillin-resistant pathogens exist, so if the infection does not seem to be responding to the drug, the patient should notify the health care provider because a different antibiotic may be required. Yeast infections are very likely after taking ampicillin because it is a broad-spectrum antibiotic. Appearance of a rash should be evaluated by a health care professional because allergic reactions to this class of antibiotic are very common.

A nurse is providing teaching to a group of patients who are beginning drug therapy for acquired immunodeficiency syndrome (AIDS). What should be included in her instructions to the group? a. "Take your medications as directed. Poisoning occurs with overdosage causing damage to more than one body system." b. "Renal injury results from first-pass effect when the drug is excreted from the system." c. "A blood dyscrasia due to drug therapy can be serious. Call us if your skin looks yellowish or you experience itching." d. "Most drugs are metabolized in the liver and the first indication of damage is dark red papules, which should be reported immediately."

a. Poisoning resulting from overdosage can lead to the potential for fatal reactions when more than one body system is affected. Liver, not kidney, injury can be caused by the first-pass effect and can cause the skin to have a yellow appearance. Most drugs are metabolized in the liver but liver damage causes jaundice, manifested as a yellow tinge to the skin and sclera. Dark red papules appearing on limbs are characteristic of Stevens-Johnson syndrome, a potentially fatal erythema multiforme exudativum, which should be reported but is not due to liver damage.

Bactericidal agents do not prevent compounds fight infection and destroy microorganisms by inhibiting what? a. Protein synthesis b. Deoxyribonucleic acid (DNA) replication c. Cell wall synthesis d. Leukocytes

a. Some anti-infectives are so active against the infective microorganisms that they actually cause the death of the cells they affect. These drugs are said to be bactericidal. Bactericidal action inhibits protein synthesis. Bacteriocidal agents do not prevent DNA replication , do not inhibit cell wall synthesis, and do not impact leukocytes.

The pharmacology instructor is explaining combination drugs to the nursing class. The instructor tells the students that a combination of anti-infective agents may be used for several reasons. What is one of them? a. Some drugs are synergistic. b. Increased likelihood of killing the microorganisms c. Requires larger doses of the drugs d. Absorption of drugs increased

a. Some drugs are synergistic, which means that they are more powerful when given in combination. The health care provider may be encouraged to use a smaller dosage of each drug, leading to fewer adverse effects, but still having a therapeutic impact on the pathogen. Many microbial infections are caused by more than one organism; each pathogen may react to a different anti-infective agent. Combination drugs do not have a better chance at killing the microorganism and they do not increase the absorption of the drugs.

What can make a nurse or any health care provider lose credibility with the patient? a. Being unprepared to deal with "the disease of the week" b. Refusing to write prescriptions for the drug the patient requests c. Not being knowledgeable about diseases described on House d. Being prepared to discuss the role of "concierge" doctor

a. Some health care providers have learned to deal with the "disease of the week" as seen on talk shows; others can be unprepared to deal with what was presented and may lose credibility with the patient.

The nurse is caring for a female patient whose tests confirm she is 10 weeks pregnant and has contracted tuberculosis. The health care provider orders a combination of antimycobacterials. What combination of drugs would the nurse identify as safest for this pregnant patient? a. Isoniazid, ethambutol, and rifampin b. Rifabutin, streptomycin, and rifampin c. Capreomycin, cycloserine, and ethionamide d. Dapsone, ethambutol, and cycloserine

a. The antituberculosis drugs are always used in combination to affect the bacteria at various cellular stages and first-line drugs are always the first choice, using second-line drugs only when the patient is unable to take the first-line medications. Because this patient is pregnant, the safest choices would be isoniazid, ethambutol, and rifampin but no drug is administered during pregnancy unless the benefit outweighs the risk. The other drug choices would be less safe and would not be used unless the safer drugs were contraindicated.

The patient, diagnosed with cancer, is receiving morphine sulfate (a potent narcotic pain reliever) to relieve cancer pain. Approximately every 7 days the medication is no longer effective in controlling the patient's pain and a larger dose is needed to have the same effect. How might the nurse explain why this is happening? a. Tolerance b. Cumulation c. Interactions d. Addiction

a. The body may develop a tolerance to some drugs over time. Tolerance may arise because of increased biotransformation of the drug, increased resistance to its effects, or other pharmacokinetic factors. When tolerance occurs, the amount of the drug no longer causes the same reaction. Therefore, increasingly larger doses are needed to achieve a therapeutic effect. Cumulative effect occurs when the drug is not properly eliminated and more of the drug is administered, resulting in toxic levels accumulating. Interactions occur when the drug reacts badly with another substance such as food, another drug, or an alternative or complementary therapy. Addiction is the psychological need for a substance.

What drug administered by the nurse belongs to the group of Carbapenems? a. Primaxin b. Gemifloxacin c. Demeclocycline d. Cefuroxime

a. The group consists of three drugs: imipenem-cilastatin (Primaxin), meropenem (Merrem), and ertapenem (Invanz). Gemifloxacin is a Fluoroquinolones, Cefuroxime is a second-generation cephalosporin, and demeclocycline is a tetracycline.

The local news has been discussing a specific rare disorder that killed a child in the community this week, describing the symptoms of the disease as including nasal congestion, ear pain, and a cough. The pediatrician's office is receiving numerous calls asking to make appointments to rule out this rare disease. What is the nurse's best action? a. Prepare a handout that describes the disorder discussed in the news in greater detail. b. Tell parents their child is experiencing the common cold and do not need to be seen. c. Direct all calls to the local news agency to answer questions and provide details. d. Become familiar with the disorder and screen each call for more specific symptoms.

a. The nurse needs to not only become more familiar with the disorder in the news, but also needs to be prepared to teach parents about the "disease of the week" to allay fears so a handout with detailed information would allow the parents to have something to consult after leaving the provider's office. Turning parents away without seeing their child will increase fears and the office will lose credibility for lack of interest in their child's well-being. Directing calls to the news agencies will not provide parents with essential information. Screening calls without seeing the child could be potentially very dangerous.

The nurse is admitting a 12-year-old girl to the acute care facility and notices discolored secondary teeth. The mother says she doesn't know why the teeth are discolored because the child is very good about brushing and flossing and sees the dentist regularly. What question would the nurse ask? a. "Has she ever received tetracycline?" b. "Has she ever received gentamicin?" c. "Has she ever received ampicillin?" d. "Has she ever received cephalexin?"

a. The nurse would question whether the child was ever given tetracycline because this drug is commonly associated with discoloration of secondary teeth when it is administered to children who still have their primary teeth. Gentamicin, ampicillin, and cephalexin are not associated with discoloration of the teeth.

The nurse is teaching the patient how to safely use the Internet for health information and includes what information in the teaching plan? a. The Web site where information is obtained needs to be evaluated for credibility. b. Most information found on the Internet is accurate. c. Information on the Internet is most reliable when people give their reviews of the drug. d. Only a health care professional can tell whether a Web site is reliable.

a. There are excellent sites for reliable drug information, but each site must be evaluated for credibility and the nurse can teach the patient things to look for to increase confidence in the site. However, a lot of information on the Internet is not accurate; the patient needs to learn how to recognize unreliable information when he or she comes across these sites. Just because a person reviews a drug and gives it multiple stars or a thumbs up does not mean the drug is any more effective or useful in the patient's care.

The processes involved in dynamic equilibrium are key elements in the nurse's ability to determine what? a. Dosage scheduling b. Amount of solution for mixing parenteral drugs c. Timing of other drugs the patient is taking d. How long the patient has to take the drug

a. These processes are key elements in determining the amount of drug (dose) and the frequency of dose repetition (scheduling) required to achieve the critical concentration for the desired length of time. The processes in dynamic equilibrium are not key elements in determining the amount of diluents for intramuscular (IM) drugs; they do not aid in the timing of the other drugs the patient is taking or how long the patient has to take the drug.

A patient is receiving meropenem (Merrem IV). What drug-related reaction will the nurse assess for? a. Gastrointestinal toxicity b. Hepatic toxicity c. Nephrotoxicity d. Neurotoxicity

a. This drug has been associated with potentially fatal pseudomembranous colitis, which affects the gastrointestinal tract. This drug is not associated with liver, kidney, or nerve toxicity.

A nurse is caring for a patient taking multiple drugs and is concerned about a possible drug-drug interaction. What is the nurse's first and best means of avoiding this problem? a. Consult a drug guide. b. Call the pharmacist. c. Contact the provider. d. Ask another nurse.

a. Whenever two or more drugs are being given together, first consult a drug guide for a listing of clinically significant drug-drug interactions. Sometimes problems can be avoided by staggering the administration of the drugs or adjusting their dosages. Consulting the pharmacist is not wrong, but it would not be the first action to take. The nurse holds responsibility for his or her own practice so asking a health care provider or another nurse is based on the assumption that that professional is knowledgeable about all drug-drug interactions, which is likely not the case.

A nurse collects a culture sample of infected tissue. What does the result of testing the culture contribute to the patient's care? a. Identifies the specific organism causing the infection b. Pinpoints the exact site of the infection c. Identifies individualized patient factors contributing to infection d. Describes the length of time the patient has experienced infection

a. A culture is collected to identify the causative organism of an infection. It can help with determining the site of infection in some cases if the infection is limited only to the site where the culture is collected. It does not individualize patient factors contributing to infection. These must be determined through assessment. It cannot indicate how long the patient has had the infection, which is often determined by the white blood cell count and differential.

The nurse is caring for a patient who is receiving an aminoglycoside. What would be a priority assessment on this patient? a. Respiratory function b. Vision c. Cardiac function d. Liver function

a. Aminoglycosides come with a black box warning alerting health care professionals to the serious risk of ototoxicity and nephrotoxicity. Central nervous system effects include ototoxicity, possibly leading to irreversible deafness; vestibular paralysis resulting from drug effects on the auditory nerve; confusion; depression; disorientation; and numbness, tingling, and weakness related to drug-related adverse effects on other nerves. Visual alterations are not usually reported in relation to this drug. Respiratory function and liver function are not usually impacted by this drug.

The nurse is assessing a patient new to the clinic. The patient says she is allergic to penicillin. What would be the nurse's appropriate next action? a. Ascertain the exact nature of the patient's response to the drug. b. Document the patient is allergic to penicillin. c. Mark the patient's chart in red that she has a penicillin allergy. d. Continue to assess the patient for other allergies.

a. Ask additional questions of patients who state that they have a drug "allergy" to ascertain the exact nature of the response and whether it is a true drug allergy. Patients may confuse secondary actions of the drug with an allergy. Only after it was determined the action was truly an allergy would the nurse document the allergy, mark the patient's chart, and continue to assess for other allergies.

A local bioterrorism medical team is learning about germ warfare. The team is instructed that a fluoroquinolone may be used to prevent an outbreak of anthrax infection. What fluoroquinolone would the nurse be most likely to administer for this purpose? a. Ciprofloxacin (Cipro) b. Gemifloxacin (Factive) c. Norfloxacin (Noroxin) d. Sparfloxacin (Zagam)

a. Ciprofloxacin (Cipro) is the most widely used fluoroquinolone and is indicated for the prevention of anthrax infection. Gemifloxacin and sparfloxacin are most useful in treating acute episodes of chronic bronchitis and community-acquired pneumonia. Norfloxacin is recommended only for certain types of urinary tract infections.

The patient is taking a drug that affects the body by increasing cellular activity. Where does this drug work on the cell? a. Receptor sites b. Cell membrane c. Golgi body d. Endoplasmic reticulum

a. Many drugs are thought to act at specific areas on cell membranes called receptor sites. After the receptor site is activated, this in turn activates the enzyme systems to produce certain effects, such as increased or decreased cellular activity, changes in cell membrane permeability, or alterations in cellular metabolism. Receptor sites are generally located on the outside of cells and allow the drug to bypass the cell membrane. The Golgi body and endoplasmic reticulum are not involved in this process.

The post-anesthesia care unit nurse is serving a patient after a right knee arthroscopy. As the patient begins to wake up from anesthesia, the nurse assesses rigidity, involuntary movements, hyperthermia, and tachycardia. What would the nurse suspect is causing these effects? a. Neuroleptic malignant syndrome b. Parkinson-like syndrome c. Malignant tachycardia d. Anaphylactic shock

a. Neuroleptic malignant syndrome is a generalized syndrome that includes extrapyramidal symptoms such as slowed reflexes, rigidity, involuntary movements; hyperthermia; autonomic disturbances (e.g., hypertension, fast heart rate); fever may be noted as well. This is most often seen after administering general anesthesia or drugs with central nervous system (CNS) effects. This syndrome was once known as malignant hyperthermia. These symptoms are not consistent with Parkinson-like syndrome or anaphylactic shock. Malignant tachycardia is a distracter.

The nurse provides discharge teaching for a patient who will receive a prescription for cefaclor (Ceclor). What important information will the nurse provide this patient? a. Avoid alcohol until 72 hours after stopping this medication. b. Genital itching will go away after the drug is discontinued. c. Monitor for yellowing of the skin or eyes and call the doctor if it occurs. d. Avoid grapefruit juice when taking this medication to prevent adverse effects.

a. Patients should be taught to avoid alcohol for up to 72 hours after discontinuing cefaclor (Ceclor) to prevent a disulfiram-like reaction that results in unpleasant symptoms such as flushing, throbbing headache, nausea and vomiting, chest pain, palpitations, dyspnea, syncope, vertigo, blurred vision, and in extreme reactions, cardiovascular collapse, convulsions, or even death. Genital itching in women indicates the possibility of a superinfection and the patient should see her health care provider. Liver damage, indicated by jaundice, is not a likely adverse effect with this drug. There is no need to avoid grapefruit juice.

With the need to protect our environment, what is it now important for the nurse to teach patients to do? a. Dispose of drugs no longer used on an annual basis. b. Flush drugs down the toilet. c Bury unused in the yard. d. Throw unused pill bottles in the trash in original containers.

a. Patients should go through their medicine cabinet annually and dispose of drugs no longer used. Unused drugs should not be flushed down the toilet or buried in the yard because they seep into the community water supply. Pills should be removed from their bottle and mixed with an undesirable substance to prevent someone from using the medication if found.

The pharmacology instructor explains to students that adverse effects can be extensions of what? a. Primary action of a drug b. Anaphylaxis c. Secondary action of a drug d. Anticholinergic responses to the drug

a. Primary action adverse effects are extensions of the therapeutic action and are usually the result of overdosage, essentially too much of the therapeutic effect. Anaphylaxis is not an extension of the therapeutic action of the drug but a histamine reaction to an allergen. Secondary actions of a drug are negative effects of the drug that occur even when the drug is in the therapeutic range. Anticholinergic responses occur in response to drugs that block the parasympathetic nervous system.

The nursing students are learning about the half-life of drugs. A student asks the instructor to explain half-life. What is the instructor's best response? a. Half-life of a drug is the time it takes for the amount of drug in the body to decrease to half of the peak level it previously achieved. b. Half-life is the amount of time it takes for the drug to be metabolized by the body. c. Half-life is the amount of time it takes for half of the drug to reach peak level in the body. d. Half-life of a drug is the time it takes for the drug to reach half its potential peak level in the body.

a. The half-life of a drug is the time it takes for the amount of drug in the body to decrease to half the peak level it previously achieved. Therefore Options B, C, and D are not correct.

The parent of a 2-year-old child is visiting his or her pediatric health care provider and shows the nurse the advertisement for allergy medication found in a magazine in the waiting room saying, "This drug sounds like it would be far more effective to treat my son's asthma and I'd only have to give it once a day." What is the nurse's best response? a. "Talk with your health care provider about this drug, but be aware that advertisements do not always provide all the important information you need to know." b. "Oh, I need to throw that magazine away because so many people show me that ad and it is all complete nonsense with no truth to it at all." c. "I've been seeing amazingly positive results from that medication so you are absolutely right to want to give it to your child." d. "That drug is dangerous and should not be given to children under the age of 5 unless there are no other good options."

a. The health care provider should make the decision about what medications are to be prescribed, not the nurse. However, the nurse can make the mother aware of the fact that there is often more that goes into choosing the correct drug than the bit of information disclosed in the advertisement. Becoming upset with the mother, agreeing with the mother, or frightening the mother about the medication is the wrong approach for the nurse to use.

The nurse admits a patient with septicemia (i.e., infection in the bloodstream). The patient denies any allergies and the doctor has ordered cefuroxime based on blood culture results that report the active pathogen is susceptible to this drug. The patient asks what antibiotic was ordered, and when the nurse says cefuroxime, the patient says, "Call my doctor and tell him I want vancomycin because I've been reading about drug-resistant bacteria and I don't want to take any chances." What is the nurse's best response? a. "Vancomycin is a powerful drug with many adverse effects and it is reserved for when no other drug will work against the infection." b. "There are some resistant infections that require vancomycin so you are right to prefer a stronger antibiotic." c. "I appreciate your concern but your doctor ordered the right medication for you so don't worry about it." d. "You can't believe anything you read on the Internet because most of it is just someone's opinion and not fact."

a. The patient is right in saying that vancomycin is effective against drug-resistant bacteria but needs help to understand that he or she does not have a resistant infection as indicated by the culture and sensitivity and that use of such a powerful drug when it is not needed increases risk of developing a vancomycin-resistant infection. It is never right to tell a patient "not to worry" because they have every right to participate in his or her own care and should not be patronized. Although some information on the Internet may not be accurate, it would be incorrect to say it is all just someone's opinion and not fact, especially given that the patient's information is accurate.

The nursing instructor teaches the students about selective toxicity when one of the students asks, "What happens when a drug doesn't have selective toxicity?" What is the instructor's best response? a. Healthy cells are damaged. b. All pathogens are destroyed in the body. c. Reduced enzymes are produced. d. Protein malnutrition

a. When a drug does not display selective toxicity, healthy cells are damaged because the drug does not specifically target only the pathogen. Anti-infectives work by a variety of different means so one drug is not likely to kill every type of pathogen in the body. Selective toxicity does not impact enzyme production or cause protein malnutrition.

The nurse administers a medication to the patient that induces the secondary action of hypoglycemia. What organ will be most acutely impacted by inadequate circulating glucose? a. Brain b. Heart c. Lungs d. Skin

a. While all cells require glucose to function, the brain uses the greatest amount. As a result, hypoglycemia has the greatest impact on the brain, which explains why hypoglycemia has so many neurological signs and symptoms including fatigue, drowsiness, anxiety, headache, shaking, lack of coordination, numbness and tingling of the mouth, tongue, and /or lips; confusion, and in severe cases, seizures or coma may occur because the brain cannot function without adequate supplies of glucose.

A patient has come to the clinic and been diagnosed with Lyme disease. The physician has ordered oral tetracycline. What is important for the nurse to include in the teaching plan about tetracycline? (Select all that apply.) a. Do not take the drug with anything high in sodium content to keep from producing a state of hypernatremia in the body. b. Do not take the drug with foods or other drugs that contain calcium. c. Do not take the drug at the same time you take an iron supplement or with foods that are high in iron content. d. Avoid exposure to the sun when taking this drug as it can turn your skin purple. e. Avoid eating bananas at the same time you take this drug as the potassium content of the tetracycline can produce hyperkalemia in the body.

b, c The antibiotic tetracycline is not absorbed from the gastrointestinal (GI) tract if calcium or calcium products (e.g., milk) are present in the stomach. It cannot be taken with iron products because a chemical reaction occurs preventing absorption. Although tetracycline can increase sun sensitivity, it does not turn the skin purple. Patients who take tetracycline do not need to avoid eating bananas or foods that are high in potassium.

Federal guidelines state that when advertising a drug, if the company states what the drug is used for, what other information must also be included in the advertisement? (Select all that apply.) a. Symptoms b. Contraindications c. Adverse effects d. Precautions e. Cost

b, c, d If a drug advertisement states what the drug is used for, it must also state contraindications, adverse effects, and precautions. The advertisement does not have to state symptoms or cost.

The nurse administers an intravenous medication with a half-life of 24 hours but recognizes what factors in this patient could extend the drug's half-life? (Select all that apply.) a. Gastrointestinal disease b. Kidney disease c. Liver disease d. Cardiovascular disease e. Route of administration

b, c, d Kidney disease could slow excretion and extend the drug's half-life. Liver disease could slow metabolism resulting in an extended half-life. Cardiovascular disease could slow distribution resulting in a longer half-life. Gastrointestinal disease would not impact half-life because the medication was injected directly into the bloodstream. Route of administration would not extend half-life because IV injection eliminates the absorption step in the process.

Because of the amount of care now being done in the home care setting, it is imperative that the nurse teach the patients what? (Select all that apply.) a. Care givers' educational level. b. Generic names of medication c. Over-the-counter (OTC) drugs that need to be avoided d. Alleviation of adverse effects e. How to calculate safe dosages

b, c, d The responsibility of meeting the tremendous increase in teaching needs of patients frequently resides with the nurse. Patients need to know exactly what medications they are taking (generic and brand names), the dose of each medication, and what each is supposed to do. Patients also need to know what they can do to alleviate some of the adverse effects that are expected with each drug (e.g., small meals if gastrointestinal upset is common, use of a humidifier if secretions will be dried and make breathing difficult), which OTC drugs or alternative therapies they need to avoid while taking their prescribed drugs, and what to watch for that would indicate a need to call the health care provider.

A student nurse asks her study group how to define a drug allergy. What would be the peer group's best response? a. A second effect of the body to a specific drug b. The formation of antibodies to a drug protein causing an immune response when the person is next exposed to that drug. c. A serum sickness caused by a reaction to a drug d. Immediate systemic reaction to the drug when exposed to the drug the first time.

b. A drug allergy is the formation of antibodies to a drug or drug protein; causes an immune response when the person is next exposed to that drug. A drug allergy does not occur at the first exposure to a drug. A second action of a specific drug is an adverse response that the drug causes in addition to the therapeutic effect. Serum sickness is one type of allergic reaction but does not define allergic reaction. An immediate systemic reaction to a drug, usually not on first exposure, is an anaphylactic reaction.

A patient with seasonal allergies is taking an antihistamine to relieve itchy, watery eyes, and a runny nose. When planning teaching for this patient, the nurse would include what teaching point? a. Advise the patient to limit fluid intake to dry out mucous membranes. b. Advise the patient to avoid driving or operating machinery. c. Advise the patient to report strange dreams or nightmares. d. Advise the patient to decrease dietary fat.

b. An adverse effect of antihistamines is drowsiness, so that injury to the patient or others can occur if driving or operating machinery. An increase in fluids would be indicated to help keep nasal membranes moist. It is common for dreams to occur when taking medication and it is not necessary to report them. Dietary fat should not interfere with the drug metabolism of antihistamines.

The nurse is preparing to contact the physician for an antibiotic order for the patient's infection. What information will the nurse be prepared to provide for the physician to choose the proper antibiotic? a. First day of infection symptoms b. Culture and sensitivity test results c. The patient's intake and output for past 2 days d. Results of complete blood count with differential

b. Antibiotics are best selected based on culture results that identify the type of organism causing the infection and sensitivity testing that shows what antibiotics are most effective in eliminating the bacteria. First day of symptoms of infection is likely already known if culture and sensitivity testing has been performed. Although measurement of intake and output is one indicator of renal function, a blood-urea-nitrogen test and assessment of creatinine levels would be better ways of assessing renal function, which will be used to determine dose of medication but not for selection of the correct antibiotic. The white blood cell count and differential would indicate the possibility of an infection but are not needed in choosing the proper antibiotic.

The nurse is administering an anti-infective to a pediatric patient. What will the nurse assess for related to adverse effects in this patient? a. Cardiovascular function and perfusion b. Hydration and nutritional status c. Liver and pancreatic function d. Rest and sleep status

b. Because children can have increased susceptibility to the gastrointestinal and nervous system effects of anti-infectives, monitor hydration and nutritional status carefully. Patients should be encouraged to drink fluids. Cardiovascular, hepatic, and pancreatic function are not at greater risk in children. Rest and sleep status are important but are not impacted by anti-infectives.

A 28-year-old patient has been prescribed penicillin for the first time. What nursing diagnosis would be most appropriate for this patient? a. Acute pain related to gastrointestinal (GI) effects of the drug b. Deficient knowledge regarding drug therapy c. Imbalance nutrition: less than body requirements related to multiple GI effects of the drug d. Constipation

b. Because this is the first time the patient has taken penicillin, she is likely to have limited knowledge about the drug. She may not understand the importance of taking the medication as ordered to increase effectiveness of the drug or to report adverse effects. because the patient has not started the drug yet, there is no way to know what adverse effects, if any, she will experience. Only if she develops acute pain related to GI effects of the drug would this be appropriate. If GI symptoms develop it may lead to imbalanced nutrition, but that remains to be seen. No indication about constipation exists.

What factor influences drug absorption? a. Kidney function b. Route of administration c. Liver function d. Cardiovascular function

b. Drug absorption is influenced by the route of administration. IV administration is the fastest method; drug absorption is slower when given orally. Kidney function impacts excretion, liver function impacts metabolism, and cardiovascular function impacts distribution.

The pharmacology instructor is discussing the adrenergic drug ephedrine with the nursing students and lists an adverse reaction of this drug as what? a. Bronchoconstriction b. Hyperglycemia c. Cardiac arrhythmias d. Severe constipation

b. Ephedrine (generic), a drug used as a bronchodilator to treat asthma and relieve nasal congestion, can break down stored glycogen and cause an elevation of blood glucose by its effects on the sympathetic nervous system. Ephedrine does not cause bronchoconstriction, cardiac arrhythmias, or severe constipation.

A 78-year-old woman, who lives alone and is forgetful, is being seen by her home health nurse. In reviewing the patient's medication, the nurse discovers that the patient is taking Azithromycin (Zithromax) for urethritis. Why would this be a good choice of antibiotics for this patient? a. The half-life of the drug is 3 to 7 hours. b. It is taken only once a day. c. It has very few adverse effects. d. It can be given without consideration to drug-drug interactions.

b. Given that the patient is "forgetful" and lives alone, a daily dose would likely promote improved compliance. Azithromycin can be administered once daily because the half-life is 68 hours. Azithromycin is associated with GI adverse effects and can cause pseudomembranous colitis; neurological symptoms can occur as well. Azithromycin (Zithromax) may adversely interact with cardiac glycosides, oral anticoagulants, theophyllines, carbamazepine, and corticosteroids to name a few agents.

A patient presents to the emergency department with a drug level of 50 units/mL. The half-life of this drug is 1 hour. With this drug, concentrations above 25 units/mL are considered toxic and no more drug is given. How long will it take for the blood level to reach the non-toxic range? a. 30 minutes b. 1 hour c. 2 hours d. 3 hours

b. Half-life is the time required for the serum concentration of a drug to decrease by 50%. After 1 hour, the serum concentration would be 25 units/mL (50/2) if the body can properly metabolize and excrete the drug. After 2 hours, the serum concentration would be 12.5 units/mL (25/2) and reach the nontoxic range. In 30 minutes the drug level would be 37.5 units/mL, whereas in 3 hours the drug level would be 6.25.

A nurse is planning patient teaching about a newly prescribed drug. What is a priority teaching point included by the nurse to improve compliance and safety? a. List of pharmacies where the drug can be obtained b. Measures to alleviate any discomfort associated with adverse effects c. The cost of the brand name drug compared with the generic form d. Statistics related to Phase III testing for the prescribed drug

b. If a patient is aware of certain adverse effects and how to alleviate or decrease the discomfort, he or she is more likely to continue taking the medication. A list of pharmacies can be useful information but will not improve safety or compliance. Knowing the cost of the brand name versus the generic form could also be helpful to the patient. However, a substitution may not be allowable and the cost of a drug does not improve patient safety. Most patients are not concerned with the statistics related to drug testing and it would not improve compliance or safety even if the patient was interested in the information.

A nurse is preparing a drug for administration to a patient. The drug does not have an indicated use for the patient's medical diagnosis. What should the nurse do? a. Administer the drug as ordered. b. Question the prescriber concerning the ordered drug. c. Ask a coworker his or her thoughts about the ordered drug for the patient. d. Ask the patient why the drug has been prescribed for him or her.

b. If the nurse is not sure about giving a drug, the order should be questioned. The nurse should never give a medication that is not clear. Mistakes do happen and the drug ordered, if not approved for the condition that the patient has, could be an error on someone's part. The person who wrote the order should be questioned, not a co worker, who probably does not know why an off-label drug is being used. It would be unprofessional and inappropriate to ask the patient about the drug.

A patient calls the clinic and asks to speak to a nurse. The patient questions the nurse about the use of a drug that was advertised on TV. The patient tells the nurse he or she is sure that the drug will make him or her feel the same way as described in the commercial. What response is most appropriate for the nurse to make? a. "I'm glad that you want to be involved in treatment decisions but you are not qualified to decide what medications are best for your condition." b. "It's important to remember that drug advertisements emphasize the positive effects of drug therapy and not the adverse effects or contraindications." c. "You need to remember that the drugs being advertised are much more expensive than other drugs that have the same effect." d. "I've seen those advertisements and I would want to take that medication too if I had the condition it was designed to treat."

b. It would be important for the nurse to remind the patient that advertisements always emphasize the positive effects of drug therapy. The patient should not be discouraged from contributing to the plan of care by being told she is not adequately qualified to make decisions because no one is more qualified to make decisions about her own body. Although the drug may be more expensive, this is not a reason to choose or avoid a medication that could be more effective. Agreeing with the patient is not meeting the nurse's obligation to teach and inform.

Before administering a macrolide antibiotic the nurse would question the order for what patient? a. An 82-year-old woman with hypertension b. A 12-year-old boy with hearing loss c. A 30-year-old pregnant patient d. A 51-year-old man after myocardial infarction

b. Macrolide antibiotics can cause severe auditory nerve damage so the nurse would question administration of this drug to the child with hearing loss because another antibiotic may be indicated to preserve remaining hearing. This drug is not contraindicated in older adults, although a lower dosage may be indicated. It may safely be given in pregnancy and after myocardial infarction.

When patients do not understand the information provided with their medication, whose responsibility is it to help them sort through and comprehend the meaning? a. Care giver b. Nurse c. Patient d. Physician

b. Many pharmacies provide written information with each drug that is dispensed, but trying to organize these sheets of information into a usable and understandable form is difficult for many patients. The nurse is often the one who needs to sort through the provided information to organize, simplify, and make sense of it for the patient.

A patient is admitted to the intensive care unit with hyperkalemia. The nurse caring for the patient knows that the most severe adverse effect of hyperkalemia is what? a. Renal failure b. Cardiac emergency c. Liver failure d. Respiratory distress

b. Monitor for cardiac irregularities because potassium is an important electrolyte in the action potential, needed for cell membrane stability. When potassium levels are too high, the cells of the heart become very irritable and rhythm disturbances can occur. Be prepared for a possible cardiac emergency. Hyperkalemia is often found in the patient with renal failure. Liver failure and respiratory distress are not generally caused by hyperkalemia.

The nurse is acting as the triage nurse in the emergency department when a 16-year-old boy is brought in by his friends. The patient is in respiratory distress, he is vomiting, and blood is noted in the vomitus. He is somnolent and his electrocardiogram shows an arrhythmia. The boy's friends tell you he was taking a "bunch of little green pills" he got from the cupboard at his grandparent's house. The nurse suspects what? a. Overdose of sleeping pills b. Poisoning c. Anaphylactic shock d. Allergic reaction to Dyazide

b. Poisoning occurs when an overdose of a drug damages multiple body systems, leading to the potential for fatal reactions. The situation described does not indicate what was contained in the little green pills so it is not possible to say whether the drugs were sedatives or triamterene and hydrochlorothiazide (Dyazide). The symptoms do not indicate an anaphylactic reaction, which would not normally include bloody vomitus.

What classification of drug allergy would be described as an immune system reaction to injected proteins used to treat immune conditions? a. A cytotoxic reaction b. Serum sickness c. A delayed reaction d. An anaphylactic reaction

b. Serum sickness is an immune system reaction to certain medications, injected proteins used to treat immune conditions, or antiserum, the liquid part of blood that contains antibodies that help protect against infectious or poisonous substances. An allergic reaction can occur with any drug, not just those containing protein. Anaphylaxis is an acute, systemic, life-threatening allergic reaction. A cytotoxic reaction is one in which antibodies circulate and attack antigens on cell sites, causing death of that cell.

The nursing instructor is talking with the students about anti-infective medication and explains that drugs that are very selective in their actions are said to be what? a. Broad spectrum b. Narrow spectrum c. Bactericidal d. Bacteriostatic

b. Some anti-infectives are so selective in their action that they are effective against only a few, or possibly only one, microorganism with a very specific metabolic pathway or enzyme. These drugs are said to have a narrow spectrum of activity. They are not called broad spectrum, which applies to a drug with little selectivity; bactericidal, which is a substance that causes death of bacteria; or bacteriostatic, which prevents replication of a bacterium.

A patient comes to the clinic to talk with the nurse about planned overseas travel. The patient tells the nurse that he or she is planning a trip to an area of the world where malaria is common. He wants to know how to prevent contracting the disease. What should the nurse respond? a. "We can ask the physician to give you some anti-infectives in case you get malaria." b. "We can ask the physician for some anti-infectives for you to take prophylactically." c. "Don't worry, if you get malaria they have some good doctors where you are going." d. "If you get malaria, you can always be treated on the way home."

b. Some anti-infectives are used as a means of prophylaxis when patients expect to be in situations that will expose them to a known pathogen, such as travel to an area where malaria is endemic, or undergoing oral or invasive gastrointestinal surgery in a person who is susceptible to subacute bacterial endocarditis. After the patient contracts malaria, it is much harder to treat so he would not start the medication or obtain treatment after being infected.

What severe reaction would the nurse assess for if it were necessary to administer trimethoprim/sulfamethoxazole (TMP/SMX) to an older adult? a. Diarrhea b. Bone marrow depression c. Vomiting d. Decreased gastrointestinal (GI) motility

b. TMP/SMX is associated with an increased risk of severe adverse effects in patients with reduced liver and kidney function. Because kidney function is known to decline as a natural part of aging, older adults would be at more increased risk of severe reactions and would require more careful monitoring. Severe skin reactions and bone marrow depression are the most frequently reported severe reactions. Diarrhea and vomiting are possible adverse effects of most medications but are not examples of severe reactions, although they would require proper intervention to prevent dehydration. GI motility is more likely to increase than to decrease.

A patient calls the clinic to talk to the nurse. The patient states that he or she saw the physician "last week" and was prescribed penicillin for a strep throat. The patient goes on to say that they feel so much better they stopped taking the drug "today," even though there are "a few pills left." What is the nurse's best response? a. "Okay, thank you for letting me know. I will document in your medical record that the treatment was effective." b. "It is important that you take all the medication so all the germs are killed. Otherwise they could come right back and be even stronger." c. "What you have described is the halo effect of the drug, making you feel better when you are still infected. You'll feel sick again when the drug is out of your system." d. "You will need to come to the clinic and be evaluated by your physician to make sure the infection is really gone."

b. The duration of drug use is critical to ensure that the microbes are completely, not partially, eliminated and are not given the chance to grow and develop resistant strains. The nurse must explain the importance of taking all of the prescribed medication and should not agree with the patient. This is not related to a halo effect and the patient may feel well until drug levels decrease rather than being completely eliminated from the body. The patient does not need to be seen if the infection is responding to treatment, but they must take the rest of the antibiotic.

The patient is taking a 2-mg dose of ropinerol XR. The drug has a half-life of 12 hours. How long will it be before only 0.25 mg of this drug remains in the patient's system? a. 24 hours b. 36 hours c. 48 hours d. 60 hours

b. The half-life of a drug is the time it takes for the amount of drug in the body to decrease to half of the peak level it previously achieved. At 12 hours there will be 1 mg of the drug available to the body. At 24 hours there will be 0.5 mg; at 36 hours there will be 0.25 mg; at 48 hours there will be 0.125 mg, and at 60 hours there will be 0.0625 mg.

Today, an abundance of information is available in the health care arena for consumers, resulting in the nurse encountering patients who have a much greater use of what? a. Over-the-counter (OTC) therapies b. Alternative therapies c. Prescription drugs d. Off-label drugs

b. The patient now comes into the health care system burdened with the influence of advertising, the Internet, and a growing alternative therapy industry. Many patients no longer calmly accept whatever medication is selected for them. Indeed, an increasing number of patients are turning to alternative therapies with the belief that they will treat their disorder and reduce risk of adverse effects. Although more prescription drugs are used today, that is not related to abundant information. No indication exists of an increase in use of OTC or off-label drugs.

The nurse, writing a care plan for a patient on an aminoglycoside, includes what intervention to reduce the accumulation of the drug in the kidney? a. Avoid caffeine intake. b. Increase fluids. c. Decrease activity. d. Increase consumption of fruits and vegetables.

b. To prevent the accumulation of anti-infective drugs in the kidneys, which can damage the kidney, patients taking anti-infective drugs should be well hydrated. Decreasing the dosage will likely reduce the therapeutic action and increase risk of resistance. There is no evidence of association between caffeine intake and drug accumulation in the kidney. Decreasing activity and increasing fruits and vegetables in the diet would not be effective in decreasing drug accumulation.

A nurse is administering digoxin to a patient. To administer medications so that the drug is as effective as possible, the nurse needs to consider what? a. Pharmacotherapeutics b. Pharmacokinetics c. Pharmacoeconomics d. Pharmacogenomics

b. When administering a drug, the nurse needs to consider the phases of pharmacokinetics so that the drug regimen can be made as effective as possible. Pharmacogenomics is the area of study that includes mapping of the human genome. Pharmacotherapeutics is the branch of pharmacology that deals with the uses of drugs to treat, prevent, and diagnose disease. Pharmacoeconomics includes all costs involved in drug therapy.

The nurse administers a drug to treat Neisseria gonorrhoeae that works on no other bacteria. How would the nurse describe this drug? a. Broad spectrum b. Narrow spectrum c. Bactericidal d. Bacteriostatic

b. Without knowing the name of the antibiotic and how it works to treat N. gonorrhoeae, the only thing that can be said is that it is a narrow-spectrum anti-infective because it only treats one specific organism. Broad-spectrum anti-infectives treat multiple organisms. The name of the drug and how it works would need to be known to determine whether it is bacteriocidal or bacteriostatic.

A nurse is caring for a patient who is supposed to receive two drugs at the same time. What is the nurse's priority action? a. Wash her hands before handling the medications. b. Consult a drug guide for compatibility. c. Question the patient concerning drug allergies. d. Identify the patient by checking the armband and asking the patient to state his name.

b. A nurse should first consult a drug guide for compatibility when two or more drugs are being given at the same time. After compatibility is determined the medication can be administered. The nurse will perform hand hygiene, check for patient allergies, and ensure the right patient receives the medication by using two identifiers.

A patient is taking chloroquine (Aralen) for rheumatoid arthritis. What problem reported by the patient would the nurse suspect may be an adverse reaction of the medication? a. "I have to urinate all the time." b. "Sometimes I have blurred vision." c. "I have tingling in my arms and legs." d. "Sometimes I feel like I am off balance."

b. Chloroquine (Aralen) can cause ocular toxicity with blurring of vision, color vision changes, corneal damage, and blindness. Increased urination, tingling, and numbness are signs of hyperkalemia and hypokalemia. Loss of balance can be caused by auditory damage due to drug toxicity.

The patient is taking low dose aspirin daily for his heart. The nurse knows only a portion of the medication taken actually reaches the tissue due to what process? a. Distribution b. First-pass effect c. Reduced absorption d. Gastrointestinal circulation

b. Drugs that are taken orally are usually absorbed from the small intestine directly into the portal venous system and then delivers these absorbed molecules into the liver, which immediately break the drug into metabolites, some of which are active and cause effects in the body, and some of which are deactivated and can be readily excreted from the body. As a result, a large percentage of the oral dose is destroyed at this point and never reaches the tissues. This process is not caused by distribution, absorption, or gastrointestinal circulation.

The nurse receives a call from a frantic mother saying, "My child swallowed some of my birth control pills. Should I give Ipecac?" What is the nurse's best response? a. "Yes, give Ipecac and follow the dosage directions on the bottle." b. "Ipecac is not effective for this use so you should not give it to your child." c. "Give the Ipecac only if you are absolutely sure your child swallowed the pills." d. "No, don't give Ipecac because it will cause your child to vomit and make a mess."

b. Ipecac is a drug that the Food and Drug Administration tested in 2003 and found, despite its use for many years, that it was not effective in inducing vomiting in children suspected of poisoning. As a result, it is no longer used. The mother should be instructed not to give it and to call poison control to get up-to-date instructions on how to deal with this emergency. Whether the pills were swallowed, this child requires appropriate intervention because it is better to err on the side of caution. Making a mess is not a concern.

The nurse is providing patient teaching before discharging a patient home. The patient is taking ciprofloxacin (Cipro). What would the nurse teach this patient is the best way to prevent crystalluria caused by ciprofloxacin (Cipro)? a. Eliminate red meat and seafood from the diet. b. Encourage at least 2 liters of fluid per day. c. Avoid caffeine and alcohol. d. dSpend time in the sun each day to optimize vitamin D levels.

b. Provide the following patient teaching: Avoid driving or operating dangerous machinery because dizziness, lethargy, and ataxia may occur; try to drink a lot of fluids and maintain nutrition (very important), even though nausea, vomiting, and diarrhea may occur. There is no need to eliminate red meat, seafood, caffeine, or alcohol from the diet, although alcohol may increase the risk of GI irritation. Patients should be taught to avoid the sun due to possible photosensitivity.

The nurse is caring for a patient receiving an antineoplastic medication who reports fever, chills, sore throat, weakness, and back pain. What type of adverse effect does the nurse suspect this patient is experiencing? a. Dermatologic reaction b. Blood dyscrasia c. Electrolyte imbalance d. Superinfection

b. Symptoms of blood dyscrasias include fever, chills, sore throat, weakness, back pain, dark urine, decreased hematocrit (anemia), low platelet count (thrombocytopenia), low white blood cell count (leukopenia), and a reduction of all cellular elements of the complete blood count (pancytopenia). Dermatologic reactions would be reflected in skin alterations, electrolyte imbalances would result in differing symptoms depending on the electrolyte involved but would not cause chills and fever, and a superinfection could cause a fever but would not cause a sore throat, weakness, or back pain unless the infection involved those body parts.

The nurse is caring for a patient receiving an antimycobacterial who reports dizziness, headache, and drowsiness. What is the priority nursing diagnosis? a. Imbalanced nutrition: less than body requirements b. Disturbed sensory perception (kinesthetic) related to central nervous system (CNS) effects of the drug c. Acute pain related to gastrointestinal (GI) effects of the drug d. Deficient knowledge regarding drug therapy

b. The priority concern for this patient right now is the disturbed sensory perception related to the CNS effects of the drug. Acute Pain could also be used but it would be related to CNS effects, not GI effects. There is no indication of imbalanced nutrition or deficient knowledge in the question.

You might question an order for a monoamine oxidase inhibitor (MAOI) as a first step in the tx of depression, remembering that these drugs are reserved for use in cases in which there has been no response to other agents because a. MAOIs can cause hair loss b. MAOIs are associated with potentially serious drug-food interactions c. MAOIs are mostly recommended for use in surgical pts d. MAOIs are more expensive than other agents

b. MAOIs are associated with potentially serious drug-food interactions

The drug of choice for a pt with a documented obsessive-compulsive disorder who is also suffering from depression and occasional panic disorder would be a. Celexa b. Paxil c. Luvox d. Prozac

b. Paxil

While administering a medication that the nurse has researched and found to have limited effectiveness, the patient tells the nurse, "I have read all about this drug and it is such a wonder drug. I'm so lucky my doctor prescribed it because I just know it will treat my problem." The nurse suspects this drug will be more effective than usual for this patient because of what effect? a. Cumulative effect b. First-pass effect c. Placebo effect d. Cross-tolerance effect

c. A drug is more likely to be effective if the patient thinks it will work than if the patient believes it will not work. This is called the placebo effect. If a drug is taken in successive doses at intervals that are shorter than recommended, or if the body is unable to eliminate a drug properly, the drug can accumulate in the body, leading to toxic levels and adverse effects. This is a cumulative effect. First-pass effect addresses the reduction of available drug when taken orally due to metabolism in the liver before the drug reaches the bloodstream. Cross-tolerance is resistance to drugs within the same class.

How has the patient's access to drug information changed the way the patient interacts with the nurse and other health care providers? a. Patients share information from research reports with health care providers. b. Patients are contacting drug companies to see what their latest reports say. c. Patients are more likely to challenge the health care provider with their own research. d. Patients are more likely to self-prescribe and not obtain prescriptions from their health care provider.

c. Access to consumer advertising, mass media health reports, and the Internet influence some patients to request specific treatments, to question therapy, and to challenge the health care provider. Consumers do not generally read research reports from medical facilities and contact drug companies to see what their reports say, and they cannot self-medicate because many of these drugs require a prescription to obtain them.

A pharmacology student asks the instructor what an accurate description of a drug agonist is. What is the instructor's best response? a. A drug that reacts with a receptor site on a cell preventing a reaction with another chemical on a different receptor site b. A drug that interferes with the enzyme systems that act as catalyst for different chemical reactions c. A drug that interacts directly with receptor sites to cause the same activity that a natural chemical would cause at that site d. A drug that reacts with receptor sites to block normal stimulation, producing no effect

c. Agonists are drugs that produce effects similar to those produced by naturally occurring neurotransmitters, hormones, or other substances found in the body. Noncompetitive antagonists are drugs that react with some receptor sites preventing the reaction of another chemical with a different receptor site. Drug-enzyme interactions interfere with the enzyme systems that stimulate various chemical reactions.

When administering anti-infectives to patients, the nurse is aware of the risk for what potentially fatal adverse effect? a. Gastrointestinal toxicity b. Eighth cranial nerve damage c. Anaphylaxis d. Toxic effects on the kidney

c. Anaphylaxis is an acute, systemic allergic response to a substance that can be fatal if medical intervention does not occur almost immediately because the airway closes due to tissue edema making it impossible to breathe. Gastrointestinal toxicity, hearing loss due to eighth cranial nerve damage and, toxic effects to the kidney are all adverse effects that may be seen with some anti-infectives. Although these adverse effects can be serious, they are not usually fatal.

A patient has recently moved from Vermont to Southern Florida. The patient presents to the clinic complaining of "dizzy spells and weakness." While conducting the admission assessment, the patient tells the nurse that he have been on the same antihypertensive drug for 6 years and had stable blood pressures and no adverse effects. Since his move, he has been having problems and he feels that the drug is no longer effective. The clinic nurse knows that one possible reason for the change in the effectiveness of the drug could be what? a. The impact of the placebo effect on the patient's response. b. The accumulative effect of the drug if it has been taken for many years. c. The impact of the warmer environment on the patient's physical status. d. Problems with patient compliance with the drug regimen while on vacation.

c. Antihypertensive drugs work to decrease the blood pressure. When a patient goes to a climate that is much warmer than usual, blood vessels dilate and the blood pressure falls. If a patient is taking an antihypertensive drug and moves to a warmer climate, there is a chance that the patient's blood pressure will drop too low, resulting in dizziness and feelings of weakness. Even mild dehydration could exacerbate these effects. Most antihypertensives are metabolized and excreted and do not accumulate in the body. Patients must be very compliant with their drug regimen on vacation. After several years on an antihypertensive drug, the effects of that drug are known; therefore, the placebo effect should not be an issue.

It is important for the nurse to be aware of what related to the way drugs are marketed? a. The adverse effects the advertisements do not mention b. What magazines and Web sites contain the advertisement c. What patients are seeing in the advertisements about these drugs d. The name of the cheerful, happy models who are advertising these drugs

c. As the marketing power for prescription drugs continues to grow, the nurse must be constantly aware of what patients are seeing, what the ads are claiming, and the real data behind the indications and contraindications for these "hot" drugs. The Food and Drug Administration regulates the information that needs to be contained within medication ads. Where the patient saw the ad and the actors in the ads are unimportant.

The nurse attends a class on preventing resistance to anti-infectives and learns that the critical concept in preventing the development of resistant strains of microbes is what? a. Exposure of pathogens to an antimicrobial agent without cellular death b. Drug dosages that are below a therapeutic level c. The duration of drug use d. Frequency of drug ingestion

c. Exposure of pathogens to an antimicrobial agent without cellular death leads to the development of resistance so it is important to limit the use of these agents to treat pathogens with a known sensitivity to the drug being used. Drug dosages are also important in preventing the development of resistance. However, the duration of drug use is critical to ensure that microbes are completely eliminated and not given the chance to grow and develop resistant strains. It is hard to convince patients that they must always complete the entire course of antimicrobial agents when they begin to "feel better," because stopping early favors the emergence of drug-resistant strains.

The nurse is providing an inservice on alternative therapies for peers and explains that the term "alternative therapies" includes what? a. Holistic drug therapy b. Hospice care c. Nondrug measures d. Home care

c. Herbal medicines and alternative therapies are found in ancient records and have often been the basis for discovery of an active ingredient that is later developed into a regulated medication. Today, alternative therapies can also include non-drug measures, such as imaging and relaxation. Options A, B, and D are not included in alternative therapies.

A patient is told that he or she will have to undergo extensive dental surgery. The dentist prescribes a course of antibiotic therapy before beginning the procedures and continuing for 5 days after the procedure. What is this is an example of? a. Chemotherapy b. Curative treatment c. Prophylaxis d. Synergism

c. In a situation where an infection is likely to occur, antibiotics can be used to prevent it. This is called prophylaxis. Synergism is using two antibiotics at the same time to improve their effectiveness. Chemotherapy is the use of drugs to destroy abnormal cells, usually cancer cells. Curative treatment involves treating an actual infection to promote a cure.

An important concept taught by the nurse when providing medication teaching is the need to provide a complete list of medications taken to health care providers to avoid what? a. Spending large amounts of money on medications b. Allergic reactions to medications c. Drug-drug interactions d. Critical concentrations of medications in the body

c. It is important that all health care providers have a complete list of the patient's medications to avoid drug-drug interactions caused by one provider ordering a medication, unaware of another medication the patient is taking that could interact with the new prescription. Using the same pharmacist for all prescriptions will also help to prevent this from happening. Informing the provider of all medications taken will not reduce costs of medications, which is best accomplished by requesting generic medications. Allergies should be disclosed to all health care providers as well, but this is not why it is important to provide a complete list of medications taken. Critical concentrations are desirable because that is the amount of drug needed to cause a therapeutic effect, or, in other words, to have the effect the drug is prescribed for.

An 80-year-old patient presents at the clinic for a follow-up appointment. She is taking a macrolide antibiotic and is experiencing tinnitus. The nurse is talking with family members about home care for the patient. What should the nurse include in her instructions regarding home care? a. Keep the patient in a prone position when in bed. b. Eliminate salt from the patient's diet. c. Provide protective measures to prevent falling or injury. d. Monitor exposure to sunlight.

c. Macrolide antibiotics can cause severe auditory nerve damage, which can cause dizziness, ringing in the ears (tinnitus), and loss of balance and hearing. The patient would be at high risk for injury due to falls. Usually a person who is dizzy is unable to lie flat and needs to recline with the head elevated. Salt and sunlight are not a component of this patient's presenting complaint.

A 32-year-old female patient is admitted to the floor with a superinfection. Her orders read tigecycline (Tygacil) 100 mg IV followed by 50 mg IV every 12 hours infused over 30 to 60 minutes for 5 days. What would be important for the nurse to educate this patient about? a. Analgesics b. Antihistamines c. Contraceptives d. Decongestants

c. Many antibiotics interfere with the effectiveness of oral contraceptives and unplanned pregnancies can occur. Women should be advised to use a barrier form of contraceptives when taking this drug. No known serious drug-drug interactions involve analgesics, antihistamines, or decongestants.

The home care nurse is taking care of a patient on IV vancomycin for cellulitis of the left calf. How would the nurse explain how microorganisms develop resistance to anti-infective medications? a. Microorganisms can alter the blood supply to the infection. b. Microorganisms can stop the cell from reproducing. c. Microorganisms produce a chemical that acts as an antagonist to the drug. d. Microorganisms change their cell membrane to make it look like the drug.

c. Microorganisms develop resistance in a number of ways, including the following: changing cellular permeability to prevent the drug from entering the cell or altering transport systems to exclude the drug from active transport into the cell; altering binding sites on the membranes or ribosomes, which then no longer accept the drug; and producing a chemical that acts as an antagonist to the drug. Microorganisms do not alter the blood supply to the infection, stop a cell from reproducing, or change the appearance of the cell membrane.

The Kardex record of a male patient who is prescribed antihistamines for treating an allergy reads as follows: Age: 32; Profession: Carpenter; Lifestyle & diet: Lives alone, average smoker, nonalcoholic, no food preferences, practices yoga; Medical history: Suffers from hay fever, recent urinary tract infection that has been treated successfully. What information from the Kardex is likely to have the greatest implication in educating the patient about antihistamine administration? a. The patient's age b. The patient's smoking habit c. The patient's profession d. The patient's medical history

c. Most antihistamines cause drowsiness, so the nurse should advise the patient not to operate machinery or perform tasks that require alertness when taking antihistamines (e.g., climbing ladders, working on rooftops, standing on iron supports at the top of a building). Because the patient is not an older adult, his age has no implications on the therapy. Although encouraging the patient to make better lifestyle choices is an important part of the patient's plan of care, this information is not related to administration of antihistamines. There is nothing in the documented medical history that is significant to antihistamine use.

What needs to happen to the protein-drug complex for the drugs to reach the cells where the drug can act? a. The protein-drug complex must break itself into smaller pieces to enter the capillaries. b. The binding site on the protein picks up a chemical to make it soluble in the serum. c. The drug must break away from the protein-binding site and float freely. d. The drug must be dissolved in the plasma so it can enter the capillaries and then the tissues.

c. Most drugs are bound, to some extent, to proteins in the blood to be carried into circulation. The protein-drug complex is relatively large and cannot enter into capillaries and then into tissues to react. The drug must be freed from the protein's binding site at the tissues. This occurs without the introduction of another chemical or by dissolving in it plasma.

According to Center for Disease Control and Prevention (CDC) recommendations, what is the role of the nurse in preparing for the possibility of bioterrorism? a. Post updated information on signs and symptoms of infections caused by biological agents b. Provide guidelines for treating patients exposed to, or potentially exposed to, biological agents c. Remain current on recognition and treatment of infections caused by biological weapons d. Advocate for increased funding for research involving bioterrorism and patient treatment

c. Nurses need to remain current about recognition of and treatment for those exposed to biological weapons because nurses are often called upon to answer questions, reassure the public, offer educational programs, and serve on emergency preparedness committees. The CDC posts updated information on signs and symptoms of infections caused by biological agents that nurses would read. The CDC also provides guidelines for how to treat patients exposed to biological agents and the nurse must remain current on this information. Although nurses could advocate for funding, this is not usually the role of the nurse.

The nurse is caring for a patient who is receiving a broad-spectrum anti-infective agents. The nurse would assess the patient for what common adverse effect of broad spectrum anti-infective agents? a. Destruction of pathogens b. Decrease in infection c. Destruction of the normal flora d. Decrease in inflammation

c. One offshooof the use of anti-infectives, especially broad-spectrum anti-infectives, is destruction of the normal flora resulting in superinfections. Destruction of pathogens is the therapeutic effect and not an adverse effect resulting in a decrease in infection. Inflammation is reduced by resolution of infection.

A patient with a gram-negative infection is being treated with an aminoglycoside. What system should the nurse expect to monitor closely while the patient is taking this medication? a. Respiratory system b. Ophthalmic system c. Renal system d. Musculoskeletal system

c. Renal function should be tested daily because aminoglycosides depend on the kidney for excretion and if the glomerular filtration rate (GFR) is abnormal it may be toxic to the kidney. The results of the renal function testing could change the daily dosage. Aminoglycosides do not usually adversely affect respiratory, hepatic, or musculoskeletal function, although baseline data concerning these systems is always needed.

The nurse is caring for a patient receiving penicillin. The nurse knows this type of antibiotic works by what mechanism? a. Inhibiting growth and development of the organism b. Inhibiting protein synthesis c. Inhibiting cell wall biosynthesis d. Stimulating bacterial reproduction

c. Some anti-infectives interfere with biosynthesis of the bacterial cell wall. Because bacterial cells have a slightly different composition than human cells, this is an effective way to destroy the bacteria without interfering with the host (see Box 8.2). The penicillins work in this way. The sulfonamides inhibit growth and development of the organism's cells. Aminoglycosides, macrolides, and chloramphenicol interfere with protein synthesis. Fluoroquinolones interfere with synthesis of deoxyribonucleic acid, resulting in the inability to reproduce.

The nurse is providing discharge teaching to a patient who is being sent home on oral tetracycline (Sumycin). What instructions should the nurse include? a. Take the medication only once a day. b. Check pulse rate and hold the drug if lower than 60 beats per minute (bpm). c. Take the drug on an empty stomach. d. Take the medication with 2 ounces of water.

c. Tetracycline should be taken on an empty stomach 1 hour before or 2 hours after meals with a full 8 ounces of water to ensure full absorption. Tetracycline is usually taken at least once every 12 hours. Checking the pulse and holding the dose if below 60 bpm is an action specific to the use of cardiac glycosides.

The patient tells the nurse that he or she has begun ordering his or her medications over the Internet because it is cheaper. What statement made by the nurse in response to this information is accurate? a. All drugs are manufactured with the same quality controls. b. Any drug that is shipped into this country is safe to use. c. Foreign drugs may have the same name as domestic drugs, but they are not the same drug. d. If you order from Canada or Mexico, the drugs are safe because they undergo testing.

c. The Food and Drug Administration has begun checking these drugs when they arrive in this country and have found many discrepancies between what was ordered and what is in the product, as well as problems in the storage of these products. Some foreign brand names are the same as brand names in this country but are associated with different generic drugs. Options A, B, and D are incorrect because not all drugs are manufactured the same and they are not always safe coming from another country.

When discussing cephalosporins with the nursing class, the pharmacology instructor explains that this classification of drug is primarily excreted through which organ? a. Lung b. Liver c. Kidney d. Skin

c. The cephalosporins are primarily metabolized in the liver and excreted in urine. These drugs cross the placenta and enter breast milk. They are not excreted through the lungs, liver, or skin.

The patient has a diagnosis of multiple sclerosis and is taking the drug interferon beta-1a (Rebif). The patient takes this drug by subcutaneous injection three times a week. The dosage is 44 mcg per injection. If the patient takes an injection on Monday, how much of the drug would still be in the patient's system when she takes her next injection on Wednesday, assuming the half-life of the drug is 24 hours? a. 22 mcg b. 16.5 mcg c. 11 mcg d. 5.5 mcg

c. The half-life of a drug is the time it takes for the amount of drug in the body to decrease to 1 half the peak level it previously achieved. On Tuesday, there would be 22 mcg remaining in the body, so option A is incorrect. On Wednesday 11 mcg would remain, so option C is the correct answer. At 12 hours before taking the next dose on Wednesday, there would be 16.5 mcg remaining. If the injection were not taken on Wednesday, 12 hours after the dose was due, there would be 5.5 mcg remaining.

The patient is a 6-year-old child who is taking 125 mg of amoxicillin every 6 hours. Assuming that the half-life of Amoxicillin is 3 hours, how much Amoxicillin would be in the child's body at the time of the next administration of the drug? a. 62.5 mg b. 46.875 mg c. 31.25 mg d. 15.625 mg

c. The half-life of a drug is the time it takes for the amount of drug in the body to decrease to 1 half the peak level it previously achieved. Option A would occur at 3 hours after the original dose of amoxicillin. Option B would occur 4 1/2 hours after the original dose. Option C would occur at 6 hours after the original dose. Option D would occur at 7 1/2 hours after the original dose.

The nurse administers amoxicillin 500 mg. The half-life of this drug is approximately 1 hour. At what point would the drug level in the body be 62.5 mg if the drug was not administered again? a. 1 hours after the original dose b. 2 hours after the original dose c. 3 hours after the original dose d. 4 hours after the original dose

c. The half-life of a drug is the time it takes for the amount of drug in the body to decrease to one-half of the peak level it previously achieved. At a dose of 500 mg the drug level would be 250 mg in 1 hour, 125 mg in 2 hours, 62.5 mg in 3 hours, and 31.25 mg in 4 hours so the correct answer is 3 hours.

The pharmacology instructor is talking to the nursing students about potassium-sparing diuretics and how they can lead to hyperkalemia, indicated by what assessment finding? a. Urine output of 1,500 mL/24 hours b. Blood pressure of 98/60 c. Potassium level of 5.9 mEq/L d. Calcium level of 11.4 mg/dL

c. The normal range of serum potassium for an adult is 3.5 to 5.0 mEq/L. A level higher than 5.0 mEq/L can indicate hyperkalemia. Normal urinary output is between 1,500 and 2,000 cc per day. Urinary output below 1,000 mL per day would include oliguria and would indicate hyperkalemia. A decrease in blood pressure and pulse can indicate hypokalemia. Hyperkalemia refers to an elevated potassium level and not an elevated calcium level.

What is the priority reason for the nurse to consider questioning an order for tetracycline in a child younger than 8 years of age? a. Children younger than 8 years of age cannot take tetracyclines. b. Weight-bearing joints have been impaired in young animals given the drugs. c. Tetracyclines can damage developing teeth and bone in children younger than 8 years of age. d. Liver and kidney function may be damaged when it is given to children under 8 years of age.

c. Use tetracyclines with caution in children younger than 8 years of age because they can potentially damage developing bones and teeth. Although the drug does not cause damage to liver and kidneys, it may be contraindicated in patients with hepatic or renal dysfunction because it is concentrated in the bile and excreted in the urine. Fluoroquinolones, not tetracyclines, are generally contraindicated for use in children (i.e., those younger than 18 years of age) because weight-bearing joints have been impaired in young animals given the drugs. Clindamycin (Dalacin C) warrants monitoring hepatic and renal function when it is given to neonates and infants. Trimethoprim-sulfamethoxazole (Nu-Cotrimox) is used in children, although children younger than 2 months of age have not been evaluated. Children under 8 years of age can take tetracycline, but it should be used with caution.

The nurse is talking with a group of nursing students who are doing clinical hours on the unit. A student asks if all intramuscular (IM) drugs are absorbed the same. What factor would the floor nurse tell the students to affect absorption of the IM administration of drugs? a. Perfusion of blood to the subcutaneous tissue b. Integrity of the mucous membranes c. Environmental temperature d. Blood flow to the gastrointestinal tract

c. A cold environmental temperature can cause blood vessels to vasoconstrict and decreases absorption or in a hot environment vasodilate and increase absorption of IM medications. Blood flow to the subcutaneous tissues interferes with subcutaneous injection and blood flow to the gastrointestinal (GI) tract causes alterations in absorption for oral medications. The condition of mucous membranes can interfere with sublingual (under the tongue) and buccal (in the cheek) administration of drugs.

Why does the nurse need to be alert for any indication of an allergic reaction in patients? a. To obtain early warning of noncompliance in drug therapy b. To increase the effectiveness of a specific medication c. To maintain the patient's safety during drug therapy d. To reduce the risk of adverse effects during drug therapy

c. Being alert to adverse effects—what to assess and how to intervene appropriately—can increase the effectiveness of a drug regimen, provide for patient safety, and improve patient compliance. Indications of allergic reactions would not indicate noncompliance or improve effectiveness of a specific medication. Indications of allergic reaction would indicate an adverse effect and would not reduce the risk.

The patient with diabetes is also taking ephedrine to treat asthma that causes her blood sugar to increase. The patient asks the nurse, "Why does this medication make my blood sugar go up?" What is the nurse's best response? a. "The active ingredient in ephedrine is mixed with sugar." b. "Ephedrine is a placebo containing only sugar." c. "Stored glycogen is broken down by ephedrine, which is causing higher blood sugar levels." d. "Insulin is inactivated by ephedrine so it cannot work to control sugar levels."

c. Ephedrine breaks down stored glycogen, which then enters the bloodstream as glucose and causes an increase in serum blood glucose, or blood sugar, levels. Ephedrine is not mixed with sugar and is not a placebo. Ephedrine has no effect on insulin.

A clinic nurse is caring for a 66-pound child who has acute otitis media. The physician has ordered ceftibuten (Cedax) 9 mg/kg per day PO for 10 days. The drug comes in an oral suspension of 90 mg/5 mL. How many mL will the nurse administer? a. 5 mL b. 10 mL c. 15 mL d. 20 mL

c. First, using the formula: 2.2 lb/1 kg = 66 lb/X kg, determine the child's weight in kg (66/2.2 = 30 kg). Next, determine the desired dose by using the formula: amount of prescribed drug times weight in kg (9 mg/kg times 30 kg = 270 mg). To determine the volume of medication to administer, use the formula: amount of drug available/volume available = amount of drug prescribed/volume to administer (90 mg/5 mL = 270 mg/X mL, 90mg/(X) = 1,350 mg/mL, X = 15 mL).

The nurse administers a loop diuretic to the patient. In addition to sodium and water, what other electrolyte would the nurse expect to be excreted in significant amounts? a. Calcium b. Magnesium c. Potassium d. Zinc

c. Loop diuretics increase excretion of sodium, water, and potassium most significantly. Although other electrolytes may be excreted, loss of magnesium, calcium, and zinc are usually not significant.

The nurse is reviewing the results of the patient's laboratory tests. What must the nurse keep in mind when reviewing these results related to medication administration? a. The patient's emotional response to the disease process b. The timing of the last dose of medication relative to when blood was drawn c. The possibility of a drug-laboratory test interaction d. A change in the body's responses or actions related to the drug

c. The body works through a series of chemical reactions. Because of this, administration of a particular drug may alter results of tests that are done on various chemical levels or reactions as part of a diagnostic study. This drug-laboratory test interaction is caused by the drug being given and not necessarily by a change in the body's responses or actions. The patient's emotional response or timing of the last dose is not important in drug-laboratory interactions.

The nurse is caring for a child weighing 30 kg. The physician orders gentamicin (Garamycin) 100 mg tid for the patient. The recommended dosage range is 6 to 7.5 mg/kg/day. What action should the nurse take? a. Administer the medication and assess hearing frequently. b. Question the physician about the frequency of administration. c. Question the physician about the dosage of the medication. d. Administer the medication and assess renal function frequently.

c. The dosage is outside the recommended dosage range at 10 mg/kg/day so the nurse should question the dosage before administering the medication. It is appropriate to administer gentamicin tid to pediatric patients so there would be no need to question frequency of dosage. The drug should not be administered until the correct dosage is ordered so there is no need to assess hearing or renal function.

Drugs do not metabolize the same way in all people. For what patient would a nurse expect to assess for an alteration in drug metabolism? a. A 35-year-old woman with cervical cancer b. A 41-year-old man with kidney stones c. A 50-year-old man with cirrhosis of the liver d. A 62-year-old woman in acute renal failure

c. The liver is the most important site of drug metabolism. If the liver is not functioning effectively, as in patients with cirrhosis, drugs will not metabolize normally so that toxic levels could develop unless dosage is reduced. A patient with cervical cancer or kidney stones would not be expected to have altered ability to metabolize drugs so long as no liver damage existed. The patient with renal failure would have altered excretion of the drugs through the renal system but metabolism would not be impacted.

The nurse is caring for a patient who experienced a severe headache. When the prescribed number of over-the-counter pain relievers did not work the patient said she took double the dosage an hour later. The nurse recognizes this patient is at greatest risk for what? a. An allergic reaction b. Anaphylactic reaction c. Poisoning d. Sedative effects

c. This patient has taken an overdosage of the medication. Poisoning occurs when an overdose of a drug damages multiple body systems, leading to the potential for fatal reactions. Allergic and anaphylactic reactions can occur with any drug administration but this is not the patient's greatest risk. More information about the exact type of medication would be needed to determine whether sedative effects are likely.

A parasitic infection is suspected. What type of culture is the nurse likely to collect? a. Blood b. Urine c. Stool d. Sputum

c. When investigators search for parasitic sources of infection, the stool is examined for ova and parasites. Blood, urine, and sputum are unlikely to reflect signs of parasitic infection.

Depression is an affective disorder that is a. always precipitated by a specific event b. most common in pts with head injuries c. characterized by overwhelming sadness, despair, and hopelessness d. very evident and easy to diagnose in the clinical setting

c. characterized by overwhelming sadness, despair, and hopelessness

The biogenic amine theory of depression states that depression is a result of a. an unpleasant childhood b. GABA inhibition c. deficiency of norepinephrine, dopamine, or 5HT in key areas of the brain d. blockages within the limbic system, which controls emotions and affect

c. deficiency of norepinephrine, dopamine, or 5HT in key areas of the brain

When teaching a pt receiving tricyclic anidepressants (TCAs), it is important to remember that TCAs are associated with many anticholinergic adverse effects. Teaching about these drugs should include anticipation of a. increased libido and increased appetite b. polyuria and polydipsia c. urinary retention, arrhythmias, and constipation d. hearing changes, cataracts, and nightmares

c. urinary retention, arrhythmias, and constipation

The home health nurse is caring for an elderly patient with benign prostatic hypertrophy. An anticholinergic drug has been prescribed. What would be the nurse's priority teaching point for this patient? a. Urinary incontinence may develop. b. Bladder hypertonia may develop. c. An increased dosage may be required. d. Empty the bladder before taking the drug.

d. A patient with an enlarged prostate who takes an anticholinergic drug may develop urinary retention or even bladder paralysis when the drug's effects block the urinary sphincters, so anticholinergic drugs are avoided whenever possible. However, if the medication is needed, the patient must be taught to empty the bladder before taking the drug. A reduced dosage also may be required to avoid potentially serious effects on the urinary system but this would not be a teaching point for the patient because the provider will make that decision. Hypotonia, not hypertonia, is more likely to occur. Urinary incontinence is not a likely effect in this case.

Selective toxicity, or the ability to affect certain proteins or enzyme systems in the infecting organism, is a much sought-after quality in an anti-infective agent. How many anti-infective agents have this quality? a. 75% b. 50% c. 25% d. 0%

d. Although anti-infective agents target foreign organisms infecting the body of a human host, they do not possess selective toxicity, which is the ability to affect certain proteins or enzyme systems used by the infecting organism but not by human cells. Because all living cells are somewhat similar, however, no anti-infective drug has yet been developed that does not affect the host. Therefore Options A, B, and C are incorrect.

Oral antidiabetic drugs can cause alterations in glucose metabolism. Patients who are taking these drugs would need to be observed for what? a. Increased urination b. Deep Kussmaul's respirations c. Thirst and hot or flushed skin d. Confusion and lack of coordination

d. Antidiabetic medications decrease blood glucose levels. If levels fall too low, symptoms of hypoglycemia would include confusion and lack of coordination. Elevated blood glucose levels can occur when the patient does not take the medications. With inadequate dosage, hyperglycemia can occur, resulting in increased urination in an attempt to eliminate serum glucose, deep Kussmaul's respirations to reduce blood pH by eliminating carbon dioxide, thirst, and hot or flushed skin.

The increasing number of patients who go to their health care provider and request a drug they have seen advertised on television or in a magazine has created what continuing challenge to health care providers? a. Treating infections appropriately b. Treating sicker patients c. Prescribing cost-effectively d. Staying knowledgeable about drug therapy

d. As the marketing power for prescription drugs continues to grow, the health care provider must be constantly aware of what patients are seeing (or reading), what the commercials and ads are promising, and the real data behind the indications and contraindications for these "hot" drugs. It is a continuing challenge to stay up-to-date and knowledgeable about drug therapy.

Overuse of anti-infective agents is known to contribute to the onset of superinfections in the body. What is a causative agent of a superinfection? a. Escherichia coli b. Probenecid c. Protozoans d. Pseudomonas

d. Common superinfections include vaginal or gastrointestinal yeast infections, which are associated with antibiotic therapy, and infections caused by Proteus and Pseudomonas throughout the body, which are a result of broad-spectrum antibiotic use. Probenicid is a medication, not a causative organism. Protozoa and E. coli do not usually cause superinfections.

The nurse has provided patient teaching for a patient who will be discharged to home on an anti-infective. What statement made by the patient indicates the nurse needs to provide additional teaching concerning the use of anti-infectives? a. "Antibiotics will not help me when I have a viral infection." b. "A bacterial culture will be done before antibiotics are prescribed for me." c. "I could develop diarrhea as a result of taking an antibiotic." d. "I will stop taking the antibiotic as soon as I feel better."

d. Compliance with anti-infective therapy is a concern. Patients tend to stop taking the drugs when they begin to "feel better." A nurse should instruct the patient to take the entire course of prescribed drug to ensure a sufficient period to rid the body of pathogens and to help prevent the development of resistance. Antibiotics are not prescribed for viral infections. It is important that cultures be performed before antibiotics are prescribed to determine what organism is causing the infection so that the correct drug is prescribed. Diarrhea is the most common adverse effect from anti-infectives.

What patient populations would the nurse expect is most likely to be prescribed a drug for an off-label use? a. Adolescent and middle-aged adult patients b. Patients with diabetes or heart disease c. Obstetric and neonatal patients d. Pediatric and geriatric patients

d. Drugs being used for an off-label purpose are commonly prescribed for pediatric and geriatric populations due to the lack of drug trial information and minimal premarket testing. Often a trial-and-error method is used in treating both the pediatric and geriatric populations when only adult information is known. The geriatric population responds to medication more like children because of their decreased ability to metabolize medications. Adolescents, especially later adolescents, use medications similarly to young adults as do middle-aged adults. Patients with different diagnoses are often involved in drug testing including those with diabetes and heart disease. Drugs are discouraged for use in obstetric patients.

The nurse is assessing a diabetic patient who has presented at the clinic reporting several hypoglycemic episodes during the past 3 weeks. The nurse questions the patient about the use of herbal or alternative therapies, suspecting what herbal remedy could cause the hypoglycemic episodes? a. St. John's wort b. Kava c. Fish oil d. Ginseng

d. Ginseng is known to decrease blood sugar levels. If the patient used this in combination with his or her oral antidiabetic agent, diet, and exercise, his or her blood sugar could drop below therapeutic levels. St. John's wort interacts with many drugs, but not with antidiabetic agents. Kava is associated with liver toxicity. Fish oil has been associated with decreased coronary artery disease.

The nurse is explaining how medications work to a group of peers and explains that disruption of a single step in any enzyme system disrupts what? a. Cell life b. Cell membrane c. Cell receptor sites d. Cell function

d. If a single step in one of the many enzyme systems is blocked, normal cell function is disrupted. Cell life and cell membrane may be impacted by disruption of some enzymes but not all enzymes. Receptor sites would not be disrupted by disruption in a single step in the enzyme system.

The pathophysiology class is learning how microorganisms develop resistance to anti-infective drugs. What is one way the nursing students would learn that microorganisms develop resistance to anti-infective drugs? a. By rearranging their deoxyribonucleic acid (DNA) to produce membranes that are permeable to the drug b. By producing an enzyme that stimulates the drug c. By changing the cellular membrane to allow the drug entry into the cell d. By altering binding sites on the membrane or ribosomes so that the drug cannot enter the cell

d. Microorganisms have developed resistance by changing cellular permeability to prevent the drug from entering the cell by altering binding sites on the membranes or on ribosomes so the drug can no longer be accepted and by producing enzymes that deactivate the drug. Microorganisms have not been found to be able to rearrange their DNA to change their membrane structure.

A patient tells the clinic nurse that he or she has been taking over-the-counter (OTC) Pepcid to relieve acid indigestion for several years. This is the first time the patient has ever reported this issue to a health care provider. As part of the teaching plan for this patient, the nurse explains what risk associated with not sharing OTC drug use with the provider? a. The OTC drug could be more expensive than seeking health care advice. b. The drug could mask symptoms of a serious problem that is undiagnosed. c. Use of the drug could cause a rebound effect of Pepcid. d. The drug could interact with several cold medicines.

d. OTC drugs allow patients to self-diagnose and treat routine signs and symptoms without seeing a health care provider. This self-prescribed treatment, however, could mask a more serious underlying medical problem and result in a poor outcome for the patient. The issues of drug rebound and drug interaction need to be considered, but the safety issue related to self-diagnosis and self-prescription presents the greatest risk to the patient. Patients should always be encouraged to discuss the use of OTC products with their health care provider.

The patient calls the clinic nurse and says, "I looked this medication up on the Internet after it was prescribed yesterday and there is nothing in the literature about this drug being used to treat my disorder. Should I still take it?" What is the nurse's best response? a. "No, stop taking it immediately until I can consult with the doctor because it is obvious a mistake was made." b. "Oh, that's okay. Go ahead and take it because the doctor wouldn't order it if he or she didn't think it would be effective." c. "It is quite common for drugs to be found to have positive effects for a condition not originally intended so it is safe to take." d. "Let me talk with the physician about why this medication was ordered for you and I will call you back."

d. Off-label use is relatively common because new information is gathered when the drug is used by large numbers of people that may indicate another condition for which the drug is effective. However, if the nurse does not know for a fact that the drug prescribed is the right drug for the patient's condition, it is always best to consult with the prescriber to make sure the patient is taking the right drug and to avoid a medication error. The medication may be perfectly safe so the patient should not be told the doctor made a mistake.

The patient in the clinic receives a prescription for an anti-infective to treat a urinary tract infection. The patient asks the nurse, "Would you ask the doctor to give me refills on this prescription? I get a urinary tract infection almost once a year it seems and I'd like to have a refill I can store for the next time so I don't have to come back to the clinic." What is the nurse's priority response? a. "Sure, I'd be glad to ask. How many refills would you like to have?" b. "Most medications, if not used, should be discarded after a year so it is better to get a new prescription next year when you need it." c. "This antibiotic doesn't destroy every pathogen that could cause a urinary tract infection so it is better to get the right antibiotic next time." d. "Saving antibiotics for another time and self-diagnosing when antibiotics are needed lead to resistant organisms that no longer respond to drugs."

d. Option A is incorrect because the patient should not be given refills to use indiscriminately. The remaining options are all important teaching points for this patient, but the priority is teaching this patient about drug-resistant organisms and how they can be prevented, as well as what happens if an infection results from a resistant organism.

A nurse is working as a member of a research team involved in exploring the unique response to drugs each individual displays based on genetic make-up. What is this area of study is called? a. Pharmacotherapeutics b. Pharmacodynamics c. Pharmacoeconomics d. Pharmacogenomics

d. Pharmacogenomics is the area of study that includes mapping of the human genome. In the future, medical care and drug regimens may be personally designed based on a patient's unique genetic make-up. Pharmacotherapeutics is the branch of pharmacology that deals with the uses of drugs to treat, prevent, and diagnose disease. Pharmacodynamics involves how a drug affects the body. Pharmacoeconomics includes the costs involved in drug therapy.

The nurse administers erythromycin, a drug that is known to irritate mucosa in the stomach lining. When the patient reports abdominal discomfort after taking the medication, the nurse would classify this discomfort as what type of adverse effect? a. Primary action b. Secondary action c. Hypersensitivity reaction d. Allergic reaction

d. Secondary actions are those actions that occur as a result of taking a medication but do not fall under the category of therapeutic action and are often negative. This patient is experiencing a secondary action of erythromycin. Primary actions would be extensions of therapeutic action. Hypersensitivity reaction would be an excessive response to either the primary or secondary effects of a drug. An allergic reaction would be an immune response to the drug.

A 22-year-old female is diagnosed with mycobacterial tuberculosis. The physician orders rifampin (Rifadin) 600 mg PO daily. What should the nurse question the patient about? a. Her diet b. Sun exposure c. Type of exercise she does d. Use of contact lenses

d. Some antimycobacterial drugs can cause discoloration of body fluids. The orange tinged discoloration can cause permanent stain to contact lenses. The patient should avoid wearing them while on the antimycobacterial therapy. With antimycobacterial drugs there is not a concern is warranted about photosensitivity or exercise. However, due to the GI adverse effects, the nurse may want to discuss an appropriate diet if the patient experiences GI upset after beginning treatment.

The nurse is caring for a patient who is receiving gentamicin, 250 mg and fluconazole (Diflucan), 500 mg at the same time. The nurse knows that if these two drugs competed with each other for protein-binding sites, what would this do? a. Make the patient gentamicin deficient b. Make the patient fluconazole deficient c. Counteract any positive benefit the drugs would have d. Alter the effectiveness of both drugs

d. Some drugs compete with each other for protein-binding sites, altering effectiveness or causing toxicity when the two drugs are given together. Nothing in the scenario would indicate that the patient would be either Gentamicin or Diflucan deficient, nor does it indicate that these drugs cannot be given together because they would counteract each other.

A 22-year-old patient calls the clinic and tells the nurse that she has been depressed and is thinking about taking St. John's wort but wants to know if it is safe first. The nurse begins by questioning what other medications the patient takes and would be concerned about a drug-alternative drug interaction if the patient is also taking what type of medication? a. Antihistamines b. Analgesics c. Antibiotics d. Oral contraceptives

d. St. John's wort can interact with oral contraceptives that alter drug metabolism, which can decrease the effectiveness of the contraceptive. Analgesics, antibiotics, and antihistamines can be taken in combination with St. John's wort without known adverse effects.

A patient presents at the clinic complaining of vaginal itching and a clear discharge. The patient reports to the nurse that she has been taking an oral antibiotic for 10 days. The nurse is aware that the patient is experiencing what? a. An adverse reaction from the antibiotic b. A drug toxicity effect of the antibiotic c. An overdose of the drug that is damaging to more than one body system d. A superinfection caused by the antibiotic, which has destroyed normal flora

d. Superinfections often occur with antibiotic use because the drug kills normal bacterial flora. This is not a result of toxic levels of the antibiotic, but rather an effect of the medication that has killed normal flora, which it is designed to do. Vaginal itching and a clear discharge are not considered adverse effects of an antibiotic. An overdose of a drug that damages more than one body systems is considered drug poisoning.

When conducting patient teaching about using antibiotic medications, what is it critical for the nurse to include to help stop the development of resistant strains of microorganisms? a. Antibiotics should be used quickly to treat colds and other viral infections before the invading organism has a chance to multiply. b. Antibiotic dosage should be reduced and used for shorter periods of time to reduce unnecessary exposure to the drug. c. Prescriptions for antibiotics should be readily available so they can be filled as soon as patients suspect they have an infection. d. It is very important to take the full course of an antibiotic as prescribed and not save remaining drugs for future infections.

d. Teaching patients to take the full course of their antibiotic as prescribed can help to decrease the number of drug-resistant strains. Antibiotics should only be used to treat bacterial infections that have been cultured to identify the antibiotic sensitivity and then patients should be instructed to use the antibiotic for the prescribed course, which will help to eliminate drug-resistant strains. Reducing dosage and time intervals increases the chance for drug resistance because anti-infectives are most effective when taken exactly as indicated.

The patient calls the clinic and talks to the nurse saying, "I found the same drug the provider prescribed on the Internet and it is much cheaper. Is it safe for me to order my drug from this site?" What is the nurse's best response? a. "It is usually safe to order drugs from Internet Web sites if it is a reliable site." b. "Most drugs ordered online come from another country and are safely used there." c ."The drug you get will be the same chemical prescribed but the dosage may differ." d. "The Food and Drug Administration (FDA) has issued warnings to consumers about the risk of taking unregulated drugs."

d. The FDA has begun checking these drugs when they arrive in this country and have found many discrepancies between what was ordered and what is in the product, as well as problems in the storage of these products. Some foreign brand names are the same as brand names in this country but are associated with different generic drugs. The FDA has issued many warnings to consumers about the risk of taking some of these drugs without medical supervision, reminding consumers that they are not protected by U.S. laws or regulations when they purchase drugs from other countries.

The clinic nurse is talking with a patient about information concerning a drug her or she bought online. What is the nurse's responsibility to the patient concerning this information? a. Encourage the patient to seek information about drugs from a pharmacist. b. Explain that information obtained from the Internet is not always accurate. c. Offer the patient a drug reference guide to read and learn more about the drug. d. Interpret the information and explain it in terms that the patient will understand.

d. The Internet can be a good reference for drug information. However, the amount and reliability of the information can be overwhelming. The nurse should always try to interpret the information and explain it in terms that the patient will understand. A pharmacist is a good resource person but may not be able to teach from a holistic perspective. Drug reference guides may be hard for the patient to understand and he or she would still need someone to interpret the information.

How would the nurse describe selective toxicity? a. Selective toxicity interferes with a biochemical reaction common to many different organisms. b. Selective toxicity will decrease invading bacteria by interfering with the pathogens' ability to reproduce. c. Selective toxicity will eliminate bacteria by interrupting protein synthesis and damaging the pathogen's cell wall. d. Selective toxicity is the ability of the drug to kill foreign cells without causing harm to one's own body cells.

d. The choice of antibiotics in a clinical situation is determined by assessing which drug will affect the causative organism and lead to the fewest adverse effects. Selective toxicity is the ability of the drug to kill foreign cells without causing harm to the human body cells. How the antibiotic works to kill bacteria varies by drug type and may reduce the ability to reproduce, damage the cell wall, or interfere with a biochemical reaction, but this is a description of how the antibiotic works and does not describe selective toxicity

A patient asks the nurse how an anti-infective produces a therapeutic effect. What key point will the nurse explain to this patient? a. Drugs used to treat infections date back to the 17th century. b. All anti-infectives work in the same way to destroy organisms. c. Selective toxicity determines the appropriate drug dosage needed. d. The goal of anti-infectives is to interfere with normal functioning of the organism.

d. The goal of anti-infectives is to interfere with the normal function of the invading organism to prevent it from reproducing and to cause cell death without affecting host cells. Each class of anti-infectives works in a different way, but all have the same goal. Because bacteria cells have a slightly different composition than human cells, the bacteria are destroyed without interfering with the host. The first drugs used to treat systemic infections were developed in the early 20th century. The term selective toxicity refers to the ability to affect certain proteins or enzyme systems that are used by infecting organisms, but not by human cells.

How can the nurse find the most up-to-date information about emergency preparedness related to bioterrorism agents? a. Read textbooks devoted to the topic. b. Ask coworkers to explain current events. c. Read journal articles about bioterrorism agents. d. Visit the Centers for Disease Control and Prevention (CDC) Web site.

d. The most current information will be found on the CDC Web site because new information can be posted immediately whereas textbooks and journal articles take time to print. Coworkers may or may not remain current on emergency preparedness and should not be the primary source of information.

A 12-year-old patient with a complicated skin infection has been admitted to the pediatric unit. The physician has ordered Ertapenem (Invanz). What is the nursing priority? a. Transcribe the order to the medication administration record (MAR). b. Perform hand hygiene before preparing the medication. c. Assess the patient's renal and hepatic functions. d. Question the order by calling the physician who prescribed it.

d. The nurse should call the physician and question the order Because this drug is not recommended for children younger than 18 years of age. Following clarification of the order, the drug would be transcribed and listed in the MAR. The nurse would then wash her hands before preparing the drug for administration. Assessment of renal and hepatic function is good practice before administering any medication but is not the nursing priority.

An intensive care unit nurse is caring for a patient taking kanamycin. What is the nurse's priority action? a. Giving the drug for no longer than 7 days b. Assessing liver function daily c. Contacting the ordering physician d. Monitoring renal function daily

d. The potential for nephrotoxicity and ototoxicity with amikacin is very high, so the drug is used only as long as absolutely necessary and should not be administered for longer than 7 to 10 days because of its potentially toxic adverse effects, which include renal damage, bone marrow depression, and gastrointestinal (GI) complications. The nurse cannot stop administering the drug after 7 days if the doctor orders it to be given longer but the nurse could question the order and promote change to another antibiotic if necessary. Monitoring renal function is the priority action when this drug is administered and the provider should be notified if signs of renal failure occur. Liver function is not usually impacted by this drug, although a patient with preexisting liver alterations may require a change in dosage to prevent toxicity. There is no indication of a need to contact the health care provider.

A student asks the pharmacology instructor if there is a way to increase the benefits and decrease the risks of antibiotic therapy. What would be an appropriate response by the instructor? a. Taking drugs not prescribed for the particular illness tends to maximize risks and minimize benefits. b. Never use antibiotics in combination with other prescriptions or in combination with other antibiotics. c. Maximize antibiotic drug therapy by administering the full dose when the patient has a fever. d. Use antibiotics cautiously and teach patients to complete the full course of an antibiotic prescription.

d. To prevent or contain the growing threat of drug-resistant strains of bacteria, it is very important to use antibiotics cautiously, to complete the full course of an antibiotic prescription, and to avoid saving antibiotics for self-medication in the future. A patient and family teaching program should address these issues, as well as the proper dosing procedure for the drug (even if the patient feels better) and the importance of keeping a record of any reactions to antibiotics. Thus, taking drugs not prescribed for the particular illness tends to maximize risks and minimize benefits. Also, if the infection is viral, antibacterial drugs are ineffective and should not be used.

The nurse is caring for a patient receiving IV aminoglycosides for an intractable infection in his or her leg. What would it be important for the nurse to monitor this patient for? a. Visual disturbances b. Liver dysfunction c. Serum glucose levels d. Renal dysfunction

d. When patients are taking aminoglycosides, it is important they be monitored closely for any sign of renal dysfunction. Aminoglycosides do not generally cause visual disturbances, liver dysfunction, or altered serum glucose levels.

The nurse needs to consider teratogenic effects of medications when caring for what population of patients? a. Older adults b. Patients with a history of cancer c. Children d. Young adult women

d. A teratogen is a drug that can harm the fetus or embryo so the nurse would consider the teratogenic properties of medications when caring for woman of child-bearing age including adolescents and young adult women. Teratogens have no impact on older adults or children. Carcinogens are chemicals that cause cancer.

What does the lipid solubility of the drug influence? a. Absorption of the drug b. Metabolism of the drug c. Excretion of the drug d. Distribution of the drug

d. Factors that can affect distribution include the drug's lipid solubility and ionization and the perfusion of the reactive tissue. The lipid solubility of a drug does not influence absorption, metabolism, or excretion.

What medication would the nurse question if ordered for a pediatric patient? a. Amikacin b. Cefazolin c. Streptomycin d. Levofloxacin

d. Fluoroquinolones are contraindicated in patients who are younger than 18 years of age. Levofloxacin is the only fluoroquinolone among the answer options and is contraindicated for pediatric patients under age 18.

The nurse is writing a plan of care for a patient who is exhibiting Parkinson-like syndrome. What would be an appropriate intervention if, on assessing the patient, the nurse finds the patient is having difficulty swallowing? a. Thicken all liquids. b. Keep the patient NPO (not taking anything orally). c. Give only soft or pureed foods. d. Provide small, frequent meals.

d. Provide small, frequent meals if swallowing becomes difficult. Keeping the patient NPO would be inappropriate because these effects often result from medications that will be taken throughout the patient's life. Soft or pureed foods are often more difficult to swallow than more rigid foods. Thickening liquids would only be necessary if the dysphagia, or difficulty swallowing, continued to progress.

The nursing student learns about anti-infectives in class and demonstrates the need to study more when making what statement about how anti-infectives work? a. Some anti-infectives interfere with biosynthesis of the pathogen's cell wall. b. Some anti-infectives prevent the cells of the organism from using essential substances. c. Many anti-infectives interfere with the steps involved in protein synthesis. d. Some anti-infectives interfere with ribonucleic acid (RNA) synthesis in the cell leading to cell death.

d. Some anti-infectives interfere with deoxyribonucleic acid (DNA) synthesis, not RNA synthesis, in the cell, leading to inability to divide and causing cell death. The fluoroquinolones work in this way. The other three options are correct and would not indicate the need for further study time. Penicillins interfere with biosynthesis of the cell wall, sulfonamides prevent organisms from using substances essential to their growth and development, whereas aminoglycosides, macrolides, and chloramphenicol interfere with protein synthesis.

A patient with Parkinson's disease is taking an anticholinergic drug to decrease the tremors and drooling caused by the disease process. The patient complains that he is having trouble voiding. The nurse would explain that this is what? a. A hypersensitive action of the drug b. A primary action of the drug c. An allergic action of the drug d. A secondary action of the drug

d. Sometimes the drug dosage can be adjusted so that the desired effect is achieved without producing undesired secondary reactions. But sometimes this is not possible, and the adverse effects are almost inevitable. In such cases, the patient needs to be informed that these effects may occur and counseled about ways to cope with the undesired effects. The situation described is not a hypersensitivity reaction that would indicate an allergic reaction, a primary reaction that would be excessive therapeutic response, or an allergic reaction to the drug.

A 77-year-old man is brought to the clinic by his daughter for a routine follow-up appointment. The daughter tells the nurse that her father is only taking half the prescribed dosage of several of his medications. What effect would the nurse explain could result from this behavior? a. Increased risk of primary actions b. Dermatologic reaction c. Superinfection d. Reduced therapeutic effect

d. Taking too little of the medication would mean that therapeutic levels are not being reached and the drugs will be less effective at lower dosages. Primary actions are the result of overdose, which is not the case in this patient who is taking too little of the drug. Dermatologic reactions are not likely if the patient is taking too little of the drug unless the drug is treating a dermatologic problem, which is not indicated by the question. Superinfection would only result if the patient was taking an antibiotic, which is not indicated by the question.

A nurse is caring for a patient who has been receiving a drug by the intramuscular route but will receive the drug orally after discharge. How does the nurse explain the increased dosage prescribed for the oral dose? a. Passive diffusion b. Active transport c. Glomerular filtration d. First-pass effect

d. The first-pass effect involves drugs that are absorbed from the small intestine directly into the portal venous system, which delivers the drug molecules to the liver. After reaching the liver, enzymes break the drug into metabolites, which may become active or may be deactivated and readily excreted from the body. A large percentage of the oral dose is usually destroyed and never reaches tissues. Oral dosages account for the phenomenon to ensure an appropriate amount of the drug in the body to produce a therapeutic action. Passive diffusion is the major process through which drugs are absorbed into the body. Active transport is a process that uses energy to actively move a molecule across a cell membrane and is often involved in drug excretion in the kidney. Glomerular filtration is the passage of water and water-soluble components from the plasma into the renal tubule.

A drug with a half-life of 4 hours is administered at a dosage of 100 mg. How much of the drug will be in the patient's system 8 hours after administration? a. 75 mg b. 50 mg c. 37.5 mg d. 25 mg

d. The half-life of a drug is the time it takes for the amount of drug in the body to decrease to 1 half the peak level it previously achieved. Option A would occur 2 hours after administration of the drug. Option B would occur at 4 hours. Option C would occur at 6 hours. Option D would occur at 8 hours after the original administration of the drug.

Your pt is being treated for depression and is started on a regimen of Prozac (fluoxetine). She calls you 10 days after the drug therapy has started to report that nothing has changed and she wants to try a different drug. You should a. tell her to try sertraline (Zoloft) because some pts respond to one selective serotonin reuptake inhibitor (SSRI) and not another b. ask her to try a few days without the drug to see whether there is any difference c. add an MAOI to her drug regimen to get an increased antidepressant effect d. encourage her to keep taking the drug as prescribed because it usually takes up to 4 weeks to see the full antidepressant effect

d. encourage her to keep taking the drug as prescribed because it usually takes up to 4 weeks to see the full antidepressant effect

Venlafaxine (Effexor) is a relatively new antidepressant that might be very effective for use in pts who a. have proven to be responsive to other antidepressants b. can tolerate multiple side effects c. are reliable at taking multiple daily dosings d. have not responded to other antidepressants and would benefit from once-a-day dosing

d. have not responded to other antidepressants and would benefit from once-a-day dosing

Adverse effects may limit the usefulness of TCAs with some pts. Nursing interventions that could alleviate some of the unpleasant aspects of these adverse effects include a. always administering the drug when the pt has an empty stomach b. reminding the pt not to void before taking the drug c. increasing the dose to override the adverse effects d. taking the major portion of the dose at bedtime to avoid experiencing drowsiness and the unpleasant anticholinergic effects

d. taking the major portion of the dose at bedtime to avoid experiencing drowsiness and the unpleasant anticholinergic effects


Ensembles d'études connexes

Hands-On Ethical Hacking and Network Defense C. 4 Review

View Set

Physics unit 1 lab safety work sheet

View Set

A&P II - Ch. 22 Quiz Questions (Lymphatic System)

View Set

MBE Selected questions - Tx Bar Exam

View Set

ISDS 3115-Chapter 6S "Statistical Process Control"

View Set